Patent Exam Questions

अब Quizwiz के साथ अपने होमवर्क और परीक्षाओं को एस करें!

Which of the following definitions does not accord with proper USPTO rules and the procedures set forth in the MPEP relating to drawings in patent applications? A Original drawings are drawings submitted with the application when filed, and may be either formal or informal. B Formal drawings are stamped "approved" by the Draftsperson. C Drawings may be informal for reasons such as the size of reference elements. D A substitute drawing is usually submitted to replace an original formal drawing. E A drawing may be declared as informal by the applicant when filed.

(B) Correct Answer(s): A B C D E Related MPEP Chapter(s): Answer Reasoning:All answers accepted. (B) is the best answer, however.

In accordance with the MPEP, and USPTO rules and procedure, a patent application may be made by someone other than the inventor in certain situations. In which of the following situations would an application not be properly made by someone other than the inventor? A The inventor is deceased, and the application is made by the legal representative of the deceased inventor. B The inventor is deceased, and the application is made by one who has reason to believe that he or she will be appointed legal representative of the deceased inventor. C The inventor is a minor (under age 18) who understands and is willing to execute the declaration, but the application is made by the minor's legal representative. D The inventor is insane, and the application is made by the legal representative of the insane inventor. E The inventor is legally incapacitated, and the application is made by the legal representative of the legally incapacitated inventor.

(C) Correct Answer(s): A B C D E Related MPEP Chapter(s): Answer Reasoning:All answers accepted.

A non-final Office action dated Friday, November 8, 2000 set a three month shortened statutory period for reply. The practitioner petitioned for a one-month extension of time on Monday, February 10, 2003 and paid the appropriate one-month extension fee. An amendment responsive to the Office action was filed on Tuesday, March 11, 2003. Each independent claim in the application was revised and two dependent claims were cancelled. No claim was added by the amendment. In the Remarks portion of the amendment, the practitioner express his belief that no fees are required by the amendment, but nevertheless authorized charging any necessary fees to the practitioner's deposit account, including fees for any required extension of time. A duplicate copy of the amendment was filed. No fees were submitted with the amendment. Assuming a valid deposit account, which of the following statements is in accord with the USPTO rules and the procedures set forth in the MPEP? A The amendment should be entered with no fees charged to practitioner's deposit account. B The amendment should be entered, but the fee for a second month extension of time should be charged to the practitioner's deposit account. C The amendment should not be entered because it is untimely. D The request to charge any required fees, including fees for any necessary extension of time, is ineffective because it was not made in a separate paper. E Statements (C) and (D) are in accord with the USPTO rules and the procedures set forth in the MPEP.

A B C D E Related MPEP Chapter(s): Answer Reasoning:All answers were accepted. If the initial date was 11/8/2002, Answer (B) would have been correct.

Which one of the following statements is in accord with the patent laws, rules and procedures as related in the MPEP regarding double patenting rejections? A A rejection of application claims for obviousness-type double patenting over the claims of a patent which names a different inventive entity (where one inventor is in common with the inventive entity in the application) and was not commonly owned at the time applicant made his or her invention can be overcome with an affidavit or declaration under 37 CFR 1.131 showing that the applicant made the invention in the United States prior to the effective filing date of the patent. B A rejection for obviousness-type double patenting of application claims over patent claims can properly rely on an embodiment which is disclosed in the patent and provides support for the patent claims on which the rejection is based. C The filing of a terminal disclaimer to overcome an obviousness-type double patenting rejection constitutes a binding admission that the rejection is proper. D Application claims are properly rejected for obviousness-type double patenting over claims of a patent having an effective filing date earlier than the effective filing date of the application only if both of the following two conditions are satisfied: (a) the rejected application claims recite an obvious variation of the subject matter recited in the patent claims on which the rejection is based; and (b) the patent claims on which the rejection is based recite an obvious variation of the subject matter recited in the rejected application claims. E None of statements (A) though (D) is correct.

A B C D E Related MPEP Chapter(s): Answer Reasoning:CREDIT GIVEN FOR ALL ANSWERS. (B) is the best answer, even so.

On May 1, 2012, a complete patent application was filed with the USPTO naming H as the sole inventor. A primary examiner rejected all the claims in the application under 35 USC 102 as being anticipated by a U.S. Patent granted to inventors H and S. The patent was granted on September 25, 2012 on an application filed on December 7, 2011. The claims of the patent application and U.S. patent define the same patentable invention as defined in 37 CFR 1.601(n). The U.S. patent and the application have common ownership. Which of the following, if properly submitted by applicant, would overcome the rejection in accordance with the patent laws, rules and procedures as related in the MPEP? A File a terminal disclaimer in accordance with 37 CFR 1.321(c). B File a declaration under 37 CFR 1.131 to establish the inventor invented the subject matter of the rejected claim prior to the effective date of the reference X. C File a declaration stating that the application and patent are currently owned by the same party, and that the inventor named in the application is the prior inventor under 35 USC 104. D (A) and (C). E All of the above.

A B C D E Related MPEP Chapter(s): MPEP 100 - Secrecy, Access, National Security, and Foreign Filing MPEP 2700 - Patent Terms and Extensions Answer Reasoning:CREDIT GIVEN FOR ALL ANSWERS. Answer (E) should have been "none of the above". Same invention double patenting cannot be overcome.

A patent application is filed having one claim (claim 1). The most relevant prior art uncovered by the primary examiner, after searching the claimed subject matter, is a published abstract summarizing the disclosure of a foreign patent document. The abstract is in English, the foreign document is in German. Both the published abstract and the foreign document are prior art under 35 USC 102. The published abstract provides an adequate basis for concluding that claim 1 is prima facie obvious under 35 USC 103. Which of the following actions is in accord with the patent laws, rules and procedures as related in the MPEP? A Reject claim 1 under 35 USC 103, based on the abstract, because it is a publication in its own right and provides sufficient basis for a prima facie case of obviousness. B Reject claim 1 under 35 USC 103, based on the abstract, because disclosures that are not in English cannot form the basis of a prior art rejection. C Reject claim 1 under 35 USC 103, based on the German-language patent document, as evidenced by the abstract. D Do not reject claim 1 based on the abstract; instead, obtain a translation of the German-language document and determine whether its full disclosure supports a rejection under 35 USC 102 or 103. E Do not reject the claims based on the abstract because an abstract can never provide sufficient disclosure to be enabling.

A D Related MPEP Chapter(s): MPEP 700 - Examination of Applications Answer Reasoning:ANSWER: (A) or (D) is accepted as correct. As to (A), MPEP § 706.02, under the heading "Reliance Upon Abstracts And Foreign Language Documents In Support Of A Rejection," states "[i]n limited circumstances, it may be appropriate for the examiner to make a rejection in a non-final Office action based in whole or in part on the abstract only without relying on the full text document." In the facts, the "published abstract provides an adequate basis for concluding that claim 1 is prima facie obvious under 35 USC 103." As to (D) MPEP § 706.02, under the heading "Reliance Upon Abstracts And Foreign Language Documents In Support Of A Rejection," states "[c]itation of and reliance upon an abstract without citation of and reliance upon the underlying scientific document is generally inappropriate where both the abstract and the underlying document are prior art. See Ex parte Jones, 62 USPQ2d 1206, 1208 (Bd. Pat. App. & Int. 2001) (unpublished)." (B) is incorrect. MPEP § 706.02, under the heading "Reliance Upon Abstracts And Foreign Language Documents In Support Of A Rejection," indicates that documents that are not in English can form the basis of a prior art rejection, although they must be translated first in order to make clear the facts that the examiner is relying on. (C) is incorrect. MPEP § 706.02, under the heading "Reliance Upon Abstracts And Foreign Language Documents In Support Of A Rejection," states "[w]hen an abstract is used to support a rejection, the evidence relied upon is the facts contained in the abstract, not additional facts that may be contained in the underlying full-text document." (E) is incorrect. See, e.g., MPEP § 2121.01. Whether a disclosure is enabling is a fact-dependent determination that must be made on a case-by-case basis. Even pictures may constitute an enabling disclosure in some cases (MPEP § 2121.04).

A claim in a pending patent application for an electric toothbrush is rejected under pre-AIA 35 USC 102 as being anticipated by a U.S. Patent, which was issued to Lancer, the sole name inventor, for a similar electric toothbrush. The Lancer patent was issued one day before the filing date of the application in question. The claim in the pending application contains a limitation specifying the location of an on/off switch. In accordance with USPTO rules and procedures set forth in the MPEP, which of the following arguments, if true, would overcome the rejection? A The Lancer patent discloses and claims an electric toothbrush, but does not mention whether its toothbrush includes a power supply. B Evidence is submitted to show the electric toothbrush claimed in the application is commercially successful. C The Lancer patent teaches away from the bristles of the claimed toothbrush. D Lancer is one of the three named inventors of the claimed toothbrush in the pending application. E The on/off switch in the Lancer patent is on a different side of the body than that recited in the claim for the electric toothbrush in the patent application.

A E Related MPEP Chapter(s): MPEP 2100 - Patentability Answer Reasoning:ANSWER: (A) and (E) are accepted as correct answers. Regarding (E), see MPEP § 2131. To anticipate a claim, the elements of a reference "must be arranged as required by the claim...." See MPEP § 2131, citing In re Bond, 910 F.2d 831, 15 USPQ2d 1566 (Fed. Cir. 1990). In (E), the on/off switch of Lancer's toothbrush is arranged differently than that of the claimed toothbrush. (A) is accepted as correct because the given facts do not specify the location of the power supply as being included within the toothbrush. Though the description of the toothbrush as being electric can imply an inherent source of power, it may also may imply an external power source for the electric toothbrush. Accordingly, (A) is also accepted as a correct answer in the circumstances. (B) is incorrect because evidence of secondary considerations, such as commercial success, is irrelevant to a 35 U.S.C. § 102 rejection. See MPEP § 2131.04. (C) is incorrect. "'Arguments that the alleged anticipatory prior art...'teaches away from the invention'...[are] not 'germane' to a rejection under section 102.'" MPEP § 2131.05 (quoting Twin Disc, Inc. v. United States, 231 USPQ 417, 424 (Cl. Ct. 1986)). (D) is incorrect. "The term 'others' in 35 U.S.C. 102(a) refers to any entity which is different from the inventive entity. The entity need only differ by one person to be 'by others.' This holds true for all types of references eligible as prior art under 35 U.S.C. 102(a) including publications...." MPEP § 2132. Here, because Lancer is only one of three inventors of the claim, the patent is by others.

Applicant properly appealed the primary examiner's final rejection of the claims to the Board of Patent Appeals and Interferences (Board). Claims 1 to 10 were pending in the application. The examiner did not reject the subject matter of claims 7 to 10, but objected to these claims as being dependent on a rejected base claim. Claim 1 was the sole independent claim and the remaining claims, 2 through 10, were either directly or indirectly dependent thereon. After a thorough review of Appellant's brief and the examiner's answer, the Board affirmed the rejection of claims 1 to 6. In accordance with the USPTO rules and the procedures set forth in the MPEP, which of the following is the appropriate action for the examiner to take upon return of the application to his jurisdiction when the time for appellant to take further action has expired? A Abandon the application since the Board affirmed the rejection of independent claim 1. B Convert the dependent claims 7 to 10 into independent form by examiner's amendment, cancel claims 1 to 6, and allow the application. C Mail an Office action to applicant setting a 1-month time limit in which the applicant may rewrite dependent claims 7 to 10 in independent form. If no timely reply is received, the examiner should amend the objected to claims, 7 to 10, and allow the allow the application. D Mail an Office action to applicant with a new rejection of claims 7 to 10 based on the Board's decision. E No action should be taken by the examiner since the Board affirmed the rejection of independent claim 1, the application was abandoned on the date the Board decision was mailed.

A Related MPEP Chapter(s): MPEP 1200 - Appeal Answer Reasoning:ANSWER: (A) is the most correct answer. MPEP § 1214.06, under the heading "Examiner Sustained in Whole or in Part." Under the heading "No Claims Stand Allowed" it states "Claims indicated as allowable prior to appeal except for their dependency from rejected claims will be treated as if they were rejected." (B) and (C) are not the most correct answers. These options would apply to applications where the Board reversed the rejection of the dependent claims and affirmed the rejection of the independent claim. (D) is not correct. The Board does not render a decision on objected to claims. See 37 CFR § Part 41. (E) is not correct because the mailing of a Board decision does not abandoned an application. See 37 CFR § Part 41.

If a reissue application is filed within two years of the original patent grant, the applicant may subsequently broaden the claims during prosecution of the pending reissue prosecution beyond the two year limit, ________________________________. A if the applicant indicates in the oath accompanying the reissue application that the claims will be broadened. B if an intent to broaden is indicated in the reissue application at any time within three years from the patent grant. C if the reissue application is filed on the 2-year anniversary date from the patent grant, even though an intent to broaden the claims was not indicated in the application at that time. D if the reissue application is a continuing reissue application of a parent reissue application, and neither reissue application contained an indication of an intent to broaden the claims until 4 years after the patent grant. E provided, absent any prior indication of intent to broaden, an attempt is made to convert the reissue into a broadening reissue concurrent with the presentation of broadening claims beyond the two year limit.

A Related MPEP Chapter(s): MPEP 1400 - Correction of Patents Answer Reasoning:ANSWER: (A) is correct. MPEP § 1412.03. In re Doll, 164 USPQ 218, 220 (CCPA 1970). (B) is wrong because 35 U.S.C. § 251 prescribes a 2-year limit for filing applications for broadening reissues. (C) is wrong because although Switzer v. Sockman, 142 USPQ 226 (CCPA 1964), holds that while a reissue application filed on the 2-year anniversary date from the patent grant is considered to be filed within 2 years of the patent grant, it is necessary that an intent to broaden be indicated in the reissue application within the two years from the patent grant. MPEP § 1412.03. (D) is wrong because a proposal for broadened claims must be made in the parent reissue application within two years from the grant of the original patent MPEP § 1412.03. In re Graff, 42 USPQ2d 1471, 1473-74 (Fed. Cir. 1997). (E) is wrong because there was no intent to broaden indicated within the two years. MPEP § 1412.03. In re Fotland, 228 USPQ 193 (Fed. Cir. 1985).

The specification of a patent application contains limited disclosure of using antisense technology in regulating three particular genes in E. coli cells. The specification contains three examples, each applying antisense technology to regulating one of the three particular genes in E. coli cells. Despite the limited disclosure, the specification states that the "the practices of this invention are generally applicable with respect to any organism containing genetic material capable of being expressed such as bacteria, yeast, and other cellular organisms." All of the original claims in the application are broadly directed to the application of antisense technology to any cell. No claim is directed to applying antisense technology to regulating any of the three particular genes in E. coli cells. The claims are rejected under 35 USC 112, first paragraph, for lack of enablement. In support of the rejection, a publication is cited that correctly notes antisense technology is highly unpredictable, requiring experimentation to ascertain whether the technology works in each type of cell. The publication cites the inventor's own articles (published after the application was filed) that include examples of the inventor's own failures to control the expressions of other genes in E. coli and other types of cells. In accordance with the patent laws, rules and procedures as related in the MPEP, the rejection is: A appropriate because the claims are not commensurate in scope with the breadth of enablement inasmuch as the working examples in the application are narrow compared to the wide breadth of the claims, the unpredictability of the technology, the high quantity of experimentation needed to practice the technology in cells other than E. coli. B appropriate because the claims are not commensurate in scope with the breadth of the enablement inasmuch no information is provided proving the technology is safe when applied to animal consumption. C inappropriate because the claims are commensurate in scope with the breadth of enablement inasmuch as the specification discloses that the "the practices of this invention are generally applicable with respect to any organism containing genetic material capable of being expressed." D inappropriate because the claims are commensurate in scope with the breadth of enablement inasmuch as the claims are original, and therefore are self-supporting. E inappropriate because the claims are commensurate in scope with the breadth of the enablement inasmuch as the inventor is not required to theorize or explain why the failures reported in the article occurred.

A Related MPEP Chapter(s): MPEP 2100 - Patentability Answer Reasoning:ANSWER: (A) is the most correct answer. 35 U.S.C. § 112, first paragraph; MPEP §§ 2164.01 and 2164.06(b). MPEP § 2164.01 states "[t]he standard for determining whether the specification meets the enablement requirement was cast in the Supreme Court decision of Mineral Separation v. Hyde, 242 U.S. 261, 270 (1916) which postured the question: is the experimentation needed to practice the invention undue or unreasonable? That standard is still the one to be applied. In re Wands , 858 F.2d 731, 737, 8 USPQ2d 1400, 1404 (Fed. Cir. 1988). Accordingly, even though the statute does not use the term 'undue experimentation,' it has been interpreted to require that the claimed invention be enabled so that any person skilled in the art can make and use the invention without undue experimentation." See also the discussion of Enzo Biochem, Inc. v. Calgene, Inc., 52 USPQ2d 1129 (Fed. Cir. 1999) in MPEP § 2164.06(b). (B) is incorrect. MPEP § 2107.01, under the heading "III. Therapeutic Or Pharmacological Utility," states "[t]he Federal Circuit has reiterated that therapeutic utility sufficient under the patent laws is not to be confused with the requirements of the FDA with regard to safety and efficacy of drugs to marketed in the United States. . . In re Brana, 51 F.3d 1560, 34 USPQ2d 1436 (Fed. Cir. 1995). Accordingly, Office personnel should not construe 35 U.S.C. 101, under the logic of 'practical' utility or otherwise, to require that an applicant demonstrate that a therapeutic agent based on a claimed invention is a safe or fully effective drug for humans." (C) is incorrect. 35 U.S.C. § 112, first paragraph; MPEP § 2107.02. MPEP § 2107.02, under the heading "When Is An Asserted Utility Not Credible," states "Rejections under 35 U.S.C. 101 have been rarely sustained by federal courts. Generally speaking, in these rare cases, the 35 .U.S.C. 101 rejection was sustained . . . because . . .[applicant] asserted a utility that . . . was wholly inconsistent with contemporary knowledge in the art. In re Gazave, 379 F.2d 973, 978, 154 USPQ 92, 96 (CCPA 1967)." The disclosure in (C) is inconsistent with published information. (D) is incorrect. MPEP § 2107.01 under the heading "Relationship Between 35 .U.S.C. 112, First Paragraph, and 35 U.S.C. 101," quotes In re Ziegler, 992 F.2d 1197, 12001201, 26 USPQ2d 1600, 1603 (Fed. Cir. 1993) as stating "The how to use prong of section 112 incorporates as a matter of law the requirement of 35 U.S.C. 101 that the specification disclose as a matter of fact a practical utility for the invention. ... If the application fails as a matter of fact to satisfy 35 U.S.C. § 101, then the application also fails as a matter of law to enable one of ordinary skill in the art to use the invention under 35 U.S.C. § 112." Enablement for the claims in a utility application is found in the specification preceding the claims, as opposed to being in the claims. The claims do not provide their own enablement. 35 U.S.C. § 112, first paragraph. (E) is incorrect. MPEP 2107.01 states that the examiner "must treat as true a statement of fact made by an applicant in relation to an asserted utility, unless countervailing evidence can be provided that shows that one of ordinary skill in the art would have a legitimate basis to doubt the credibility of such a statement." Inasmuch as countervailing evidence has been produced, the lack of necessity to theorize or explain the failures does not alleviate the inventor from complying with 35 U.S.C. § 112, first paragraph to provide an enabling disclosure that is commensurate in scope with the claims.

Tribell files a patent application for her aroma therapy kit on November 29, 2015, which issues as a patent on August 7, 2017. She tries to market her kit but all of her prospects are concerned that her patent claims are not sufficiently broad. On September 5, 2017, Tribell asks a registered practitioner for advice on what to do to improve her ability to market her aroma therapy kit. At the consultation the practitioner learns that in the original patent application, Tribell had a number of claims which were subjected to a restriction requirement, but were nonelected and withdrawn from further consideration. The practitioner also determines that the claims in the patent obtained by Tribell were narrower than the broader invention disclosed in the specification, and the cited references may not preclude patentability of the broader invention. Which of the following is the best course of action to pursue in accordance with the patent laws, rules and procedures as related in the MPEP? A Tribell should file a reissue application under 35 USC 251 within two years of the issuance of the patent, broadening the scope of the claims of the issued patent. B Tribell should file a reissue application under 35 USC 251 any time during the period of enforceability of the patent to broaden the scope of the claims of the issued patent, and then file a divisional reissue application of the first reissue application on the nonelected claims that were subjected to a restriction requirement in the nonprovisional application which issued as a patent. C Tribell should simultaneously file two separate reissue applications under 35 USC 251, one including an amendment of broadening the claims in the original patent, and the other including the nonelected claims that were subjected to a restriction requirement in the nonprovisional application which issued as a patent. D Tribell should immediately file a divisional application under 37 CFR 1.53(b) including the nonelected claims that were subjected to a restriction requirement in the original application. E Tribell should immediately file a reissue application under 35 USC 251, including the nonelected claims that were subjected to a restriction requirement in the original application.

A Related MPEP Chapter(s): MPEP 1400 - Correction of Patents Answer Reasoning:ANSWER: (A) is the most correct answer. 35 U.S.C. § 251, MPEP § 1402 (fifth paragraph). MPEP § 1402 states that one of the "most common bases for filing a reissue application [is] (A) the claims are too narrow or too broad." The claims may be broadened in a reissue application filed by the inventor within two years from the patent issue date. (B) is incorrect since the 4th paragraph of 35 U.S.C. § 251 states that no reissued patent shall be granted enlarging the scope of the clams of the original patent unless applied for within two years from the grant of the original patent. (C) and (E) are incorrect. MPEP § 1402, sixteenth paragraph. An applicant's failure to timely file a divisional application while the original application is still pending is not considered to be an error correctable via reissue. See In re Orita, 550 F.2d 1277, 1280, 193 USPQ 145, 148 (CCPA 1977). (D) is incorrect. MPEP § 201.06. In order to claim benefit under 35 U.S.C. § 120 to a parent application, a divisional application must be filed while the parent patent application is still pending.

Mike and Alice, who are not related, are shipwrecked on a heretofore uninhabited and undiscovered island in the middle of the Atlantic Ocean. In order to signal for help, Mike invents a signaling device using bamboo shoots. Alice witnesses but does not assist in any way in the development of the invention. The signaling device works and a helicopter comes and rescues Alice. However, Mike remains on the island due to overcrowding on the helicopter. Unfavorable weather conditions have prevented Mike's rescue to date. Alice comes to you, a registered patent practitioner, to file an application for a patent and offers to pay you in advance. Which of the following, in accordance with the USPTO rules and the procedures set forth in the MPEP, is true? A Since Mike invented the invention, Alice cannot properly file an application for a patent in her name even though Mike is unavailable. B Since Mike is unavailable, you may properly file an application for a patent without his consent. You can accept the money from Alice as payment for the application. C Since Mike is not available and cannot be reached, Alice may properly sign the declaration on his behalf since she has witnessed the invention and knows how to make and use it. D Alice should file an application in her name since she has witnessed the invention and knows how to make and use it. Subsequently, when Mike becomes available, the inventorship may be changed to the correct inventorship. E Even though Mike and Alice are not related, Alice may properly file an application on Mike's behalf.

A Related MPEP Chapter(s): MPEP 200 - Types, Cross-Noting, and Status of Application Answer Reasoning:ANSWER: (A) is the most correct answer. (A) is true since only the inventor may file for a patent. 35 U.S.C. § 101. As to answers (C) and (E), since Alice is not a joint inventor and she does not have sufficient proprietary interest in the invention, she may not file a patent application on Mike's behalf. 35 U.S.C. § 116; 37 CFR § 1.47(b). As to (B), you ordinarily may not accept payment from someone other than your client. 37 CFR § 10.68(a)(1). As to (D), inventorship cannot be changed when there is deceptive intent.

Independent claim 1, fully supported by the specification in a patent application states: Claim 1. An apparatus comprising: a plastic valve; a copper pipe connected to the plastic valve; and an aluminum pipe connected to the plastic valve. Which of the following claims, presented in the application, provide the basis for a proper rejection under 35 USC 112, second paragraph? Claim 2. The apparatus of claim 1, wherein said pipe is statically charged. Claim 3. The apparatus of claim 1, wherein the outer circumference of said copper pipe is statically charged. Claim 4. The apparatus of claim 1, further comprising a thermostat connected to said plastic valve. A Claim 2. B Claim 3. C Claim 4. D Claims 2 and 3. E Claims 3 and 4.

A Related MPEP Chapter(s): MPEP 2100 - Patentability Answer Reasoning:ANSWER: (A) is accepted as the most correct answer. MPEP § 2173.05(e). Claim 2 is indefinite because it is not clear which "said pipe" the claim is referring to since claim 1 recites a copper pipe and an aluminum pipe. Accordingly, claim 2 provides the basis for a proper rejection under 35 U.S.C. § 112, second paragraph. In (B), claim 3 would be construed as definite, inasmuch as "the outer circumference" is an inherent part of the pipe and would not require antecedent recitation. Thus claim 3 does not provide a proper basis for the rejection. Therefore, (B) and (D) would be incorrect. Claim 4 is definite inasmuch as there is antecedent basis for "said plastic valve." Therefore, (C) and (E) are incorrect.

The claimed invention in a patent application is directed to an explosive composition "comprising 60-90% solid ammonium nitrate, and 10-40% water-in-oil in which sufficient aeration is entrapped to enhance sensitivity to a substantial degree." The application discloses that the explosive requires both fuel (the ammonium nitrate), and oxygen to "sensitize the composition." A prior art reference, published more than two years before the effective filing date of the application, discloses explosive compositions containing water-in-oil emulsions having identical ingredients to those claimed, in ranges overlapping with the claimed composition. The only element of the claim not recited in the reference is "sufficient aeration entrapped to enhance sensitivity to a substantial degree." The reference does not recognize that sufficient aeration sensitizes the fuel to a substantial degree. In addition to the prior art reference, a printed publication contains test data demonstrating that "sufficient aeration" is necessarily an inherent element in the prior art blasting composition under the circumstances. In accordance with the patent laws, rules and the procedures as related in the MPEP, the prior art reference: A anticipates the claim because it discloses every limitation of the claim either explicitly or inherently. B does not anticipate the claim because the prior art reference does not recognize an inherent property. C does not anticipate the claim because the prior art reference does not recognize an inherent function of oxygen. D does not anticipate the claim because the prior art reference does not recognize an inherent ingredient, oxygen. E (B), (C) and (D).

A Related MPEP Chapter(s): MPEP 2100 - Patentability Answer Reasoning:ANSWER: (A) is the best answer. 35 U.S.C. § 102; MPEP § 2131.01, under the heading "Extra Reference or Evidence Can Be Used To Show an Inherent Characteristic of the Thing Taught by the Primary Reference," states "that as long as there is evidence of record establishing inherency, failure of those skilled in the art to contemporaneously recognize an inherent property, function or ingredient of a prior art reference does not preclude a finding of anticipation. Atlas Powder Co. v. IRECO, Inc., 190 F.3d 1342, 1349, 51 USPQ2d 1943, 1948 (Fed. Cir. 1999) (Two prior art references disclosed blasting compositions containing water-in-oil emulsions with identical ingredients to those claimed, in overlapping ranges with the claimed composition. The only element of the claims arguably not present in the prior art compositions was "sufficient aeration . . . entrapped to enhance sensitivity to a substantial degree." The Federal Circuit found that the emulsions described in both references would inevitably and inherently have "sufficient aeration" to sensitize the compound in the claimed ranges based on the evidence of record (including test data and expert testimony). (B) is incorrect. The prior art reference, to anticipate the claimed invention, is not required to recognize an inherent property. See MPEP § 2131.01. (C) is incorrect. The prior art reference, to anticipate the claimed invention, is not required to recognize an inherent function of oxygen. See MPEP § 2131.01. (D) is incorrect. The prior art reference, to anticipate the claimed invention, is not required to recognize an inherent ingredient, oxygen. See MPEP § 2131.01. (E) is incorrect because (B), (C), and (D) are incorrect, as explained above.

A patent application filed in the USPTO contains the following dependent claim: Claim 2. The method of Claim 1, further consisting of the step of cooling the mixture to a temperature of 32 degrees F. Following proper USPTO practices and procedures, from which of the following claims does Claim 2 not properly depend? A Claim 1. A method of making liquid compound A consisting of the steps of mixing equal quantities of material C and material D in a beaker and heating the mixture to a temperature of 212Ã F. B Claim 1. A method of making liquid compound A comprising the steps of mixing equal quantities of material C and material D in a beaker and heating the mixture to a temperature of 212Ã F. C Claim 1. A method of making liquid compound A including the steps of mixing equal quantities of material C and material D in a beaker and heating the mixture to a temperature of 212Ã F. D Claim 1. A method of making liquid compound A characterized by the steps of mixing equal quantities of material C and material D in a beaker and heating the mixture to a temperature of 212Ã F. E (C) and (D).

A Related MPEP Chapter(s): MPEP 2100 - Patentability Answer Reasoning:ANSWER: (A) is the correct answer. The phrase "consisting of" excludes any step not specified in the claim. MPEP § 2111.03. Thus, a claim that depends from a claim which "consists of" the recited steps cannot add a step. Id. Here, the dependent claim adds the step of cooling. Answer (B) is incorrect because the transitional term "comprising" is inclusive or open-ended and does not exclude additional, unrecited steps. MPEP § 2111.03. Answers (C) and (D) are incorrect because the terms "including" and "characterized by" are synonymous with the term "comprising." MPEP § 2111.03. Answer (E) is incorrect because Answer (C) and Answer (D) are incorrect.

In connection with the utility of an invention described in a patent application, which of the following conforms to the USPTO rules and the procedure set forth in the MPEP? A A deficiency under 35 USC 101 also creates a deficiency under 35 USC 112, first paragraph. B To overcome a rejection under 35 USC 101, it must be shown that the claimed device is capable of achieving a useful result on all occasions and under all conditions. C A claimed invention is properly rejected under 35 USC 101 as lacking utility if the particular embodiment disclosed in the patent lacks perfection or performs crudely. D To overcome a rejection under 35 USC 101, it is essential to show that the claimed invention accomplishes all its intended functions. E A claimed invention lacks utility if it is not commercially successful.

A Related MPEP Chapter(s): MPEP 2100 - Patentability Answer Reasoning:ANSWER: (A) is the most correct answer. As stated in MPEP § 2107.01 under the heading "IV. Relationship Between 35 U.S.C. 112, First Paragraph, And 35 U.S.C. 101," "A deficiency under 35 U.S.C. § 101 also creates a deficiency under 35 U.S.C. § 112, first paragraph. See In re Brana, 51 F.3d 1560, 34 USPQ2d 1436 (Fed. Cir. 1995); In re Jolles, 628 F.2d 1322, 1326 n.10, 206 USPQ 885, 889 n.11 (CCPA 1980); In re Fouche, 439 F.2d 1237, 1243, 169 USPQ 429, 434 (CCPA 1971) ('If such compositions are in fact useless, appellant's specification cannot have taught how to use them.')." (B) is not correct. MPEP § 2107, under the heading "II. Examination Guidelines For The Utility Requirement," and see Brooktree Corp. v. Advanced Micro Devices, Inc., 977 F.2d 1555, 1571, 24 USPQ2d 1401, 1412 (Fed. Cir. 1992); and E.I. du Pont De Nemours and Co. v. Berkley and Co., 620 F.2d 1247, 1260 n.17, 205 USPQ 1, 10 n.17 (8th Cir. 1980). (C), (D) and (E) are not correct. MPEP § 2107, under the heading "II. Examination Guidelines For The Utility Requirement," and see E.I. du Pont De Nemours and Co. v. Berkley and Co., 620 F.2d 1247, 1260 n.17, 205 USPQ 1, 10 n.17 (8th Cir. 1980).

Evidence that a claim may not comply with 35 USC 112(b) occurs in accordance with the USPTO rules and the procedure set forth in the MPEP where: A Remarks filed by applicant in a reply or brief regarding the scope of the invention differ and do not correspond in scope with the claim. B There is a lack of agreement between the language in the claims and the language set forth in the specification. C The scope of the claimed subject matter is narrowed during pendency of the application by deleting the originally much broader claims, and presenting claims to only the preferred embodiment within the originally much broader claims. D Claims in a continuation application are directed to originally disclosed subject matter (in the parent and continuation applications) which applicants did not regard as part of their invention when the parent application was filed. E All of the above.

A Related MPEP Chapter(s): MPEP 2100 - Patentability Answer Reasoning:ANSWER: (A) is the most correct answer. In accordance with MPEP § 2172, under the heading "II. Evidence To The Contrary," states that evidence that shows a claim does not correspond in scope with that which applicant regards as applicant's invention may be found, for example, in contentions or admissions contained in briefs or remarks filed by applicant. In re Prater, 415 F.2d 1393, 162 USPQ 541 (CCPA 1969). (B) is incorrect. MPEP § 2172, under the heading "II. Evidence To The Contrary," states, "As noted in In re Ehrreich, 590 F.2d 902, 200 USPQ 504 (CCPA 1979) agreement, or lack thereof, between the claims and the specification is properly considered only with respect to 35 U.S.C. 112, first paragraph; it is irrelevant to compliance with the second paragraph of that section." (C) is incorrect. MPEP § 2172, under the heading "III. Shift In Claims Permitted," indicates that the second paragraph of 35 U.S.C. § 112 does not prohibit applicants from changing what they regard as their invention during the pendency of the application. In re Saunders, 444 F.2d 599, 170 USPQ 213 (CCPA 1971) (Applicant was permitted to claim and submit comparative evidence with respect to claimed subject matter which originally was only the preferred embodiment within much broader claims (directed to a method). (D) is incorrect. MPEP § 2172, under the heading "III. Shift In Claims Permitted," indicates that the fact that claims in a continuation application were directed to originally disclosed subject matter which applicants had not regarded as part of their invention when the parent application was filed was held not to prevent the continuation application from receiving benefits of the filing date of the parent application under 35 U.S.C. § 120. In re Brower, 433 F.2d 813, 167 USPQ 684 (CCPA 1970). (E) is incorrect because (B), (C), and (D) are incorrect.

On January 2, 2016, a registered practitioner filed a patent application with the USPTO for inventor Beck. The application includes a specification and a single claim to the invention which reads as follows: 1. Mixture Y made by the process Q1. In the specification, Mr. Beck discloses that mixture Y has a melting point of 150 degrees F. On June 2, 2016, the practitioner received an Office action from the primary examiner rejecting the claim. The claim is rejected under 35 USC 102/103 as being clearly anticipated by or obvious over Patent A. The examiner states "Patent A teaches mixture Y but made by a different process Q2." Beck believes he is entitled to a patent to mixture Y. In accordance with the patent laws, rules and procedures as related in the MPEP, which of the following would be the best reply to the rejection of his claim? A An argument that the claimed product has an unexpectedly low melting point of 150 degrees F, supported by an affidavit showing that the mixture Y made by process Q2 exhibits a melting point of 300 degrees F. B An argument that the processes used by applicant and patent A are different, supported by a third-party declaration stating only that the processes are different. C An argument that the claimed product has an unexpectedly low melting point of 150 degrees F, supported by a third-party declaration stating only that the products are different. D An argument that the processes used by applicant and patent A are different, supported by an affidavit showing that the mixture Y made by process Q2 exhibits a melting point of 300 degrees F. E An argument that the claimed product has an unexpectedly low melting point of 150 degrees F because the claimed mixture Y has a melting point of 150 degrees F and the mixture Y of patent A has a melting point of 300 degrees F.

A Related MPEP Chapter(s): MPEP 2100 - Patentability Answer Reasoning:ANSWER: (A) is the most correct answer. MPEP § 2113, under the heading "Product-By-Process Claims Are Not Limited To The Manipulations Of The Recited Steps, Only The Structure Implied By The Steps," states "'even though product-by-process claims are limited by and defined by the process, determination of patentability is based on the product itself. The patentability of a product does not depend on its method of production. If the product in the .product-by-process claim is the same as or obvious from a product of the prior art, the claim is unpatentable even though the prior product was made by a different process.' In re Thorpe, 777 F.2d 695, 698, 227 USPQ 964, 966 (Fed. Cir. 1985)." The issue is whether the claimed mixture Y is the same as or obvious over the patented mixture Y. MPEP § 2113, under the heading "Once A Product Appearing To Be Substantially Identical Is Found And A 35 U.S.C. 102/103 Rejection Made, The Burden Shifts To The Applicant To Show An Unobvious Difference," states "[o]nce the examiner provides a rationale tending to show that the claimed product appears to be the same or similar to that of the prior art, although produced by a different process, the burden shifts to applicant to come forward with evidence establishing an unobvious difference between the claimed product and the prior art product. In re Marosi, 710 F.2d 798, 802, 218 USPQ 289, 292 (Fed. Cir. 1983)." Evidence that the two processes produce different properties is germane to the issue of patentability of the product-by-process claim. Accordingly, a comparison of the results obtained by conducting the process recited in the claim versus the process used by patent A and which shows that the claimed product exhibits an unexpectedly lower melting point would be a persuasive demonstration that, although the products would appear to be substantially identical, in fact, they are patentably different. Ex parte Gray, 10 USPQ2d 1922 (Bd. Pat. App. & Inter. 1989). Therefore, the best reply to the outstanding rejection would be to argue that the claimed product has an unexpectedly lower melting point and to support that argument with evidence showing that the result of the patent A process is a mixture with higher melting point as compared to the claimed product. (B) is incorrect. The patentability of a product-by-process claim is determined on the basis of product characteristics, not process steps. (C) is incorrect. The declaration is conclusory, as opposed to being factual. Thus, the argument is not supported by facts. As stated in MPEP § 716.02(c), under the heading "Opinion Evidence," "Although an affidavit or declaration which states only conclusions may have some probative value, such an affidavit or declaration may have little weight when considered in light of all the evidence of record in the application. In re Brandstadter, 484 F.2d 1395, 179 USPQ 286 (CCPA 1973)." Thus, the reply in (A) is the most correct answer vis-ˆ-vis (C). (D) like (B), is incorrect for the same reason discussed for (B). (E) is incorrect. Like (C), this reply rightly focuses on product properties. But without the comparative factual evidence to support it, this reply is weaker than one described in (A).

In accordance with USPTO rules and procedures set forth in the MPEP, which of the following statements regarding operability or enablement of a prior art reference is the most correct? A The level of disclosure required for a reference to be enabling prior art is no less if the reference is a United States patent than if it is a foreign patent. B A reference is not presumed to be operable merely because it expressly anticipates or makes obvious all limitations of an applicant's claimed apparatus. C A non-enabling reference may not qualify as prior art for the purpose of determining anticipation or obviousness of the claimed invention. D A reference does not provide an enabling disclosure merely by showing that the public was in possession of the claimed invention before the date of the applicant's invention. E All of the above are correct.

A Related MPEP Chapter(s): MPEP 2100 - Patentability Answer Reasoning:ANSWER: (A) is the most correct answer. MPEP § 2121, under the heading "What Constitutes An 'Enabling Disclosure' Does Not Depend On The Type Of Prior Art The Disclosure Is Contained In," states, in reliance upon In re Moreton, 288 F.2d 708, 711, 129 USPQ 227, 230 (CCPA 1961): "The level of disclosure required within a reference to make it an 'enabling disclosure' is the same no matter what type of prior art is at issue.... There is no basis in the statute (35 U.S.C. 102 or 103) for discriminating either in favor of or against prior art references on the basis of nationality." Answer (B) is incorrect. MPEP § 2121, under the heading "Prior Art Is Presumed To Be Operable/Enabling," states that "[w]hen the reference relied on expressly anticipates or makes obvious all of the elements of the claimed invention, the reference is presumed to be operable." Answer (C) is incorrect. MPEP § 2121.01, under the heading "35 U.S.C. 103 Rejections And Use Of Inoperative Prior Art," quotes Symbol Technologies Inc. v. Opticon Inc., 935 F.2d 1569, 1578, 19 USPQ2d 1241, 1247 (Fed. Cir. 1991) as stating that "a non-enabling reference may qualify as prior art for the purpose of determining obviousness under 35 U.S.C. 103." Answer (D) is incorrect. MPEP § 2121.01 states that "[a] reference contains an 'enabling disclosure' if the public was in possession of the claimed invention before the date of invention." Answer (E) is incorrect because answers (B), (C) and (D) are incorrect.

A registered practitioner files a patent application with the following claim: 1. A plastic insert for the bottom of a shopping cart comprising circular receptacles to receive wine bottles and to maintain them in an upright and stable position even while the shopping cart is moved about a store so that they do not fall and break. Patent A discloses a plastic insert for the bottom of a shopping cart comprising rectangular receptacles to receive cereal boxes and to maintain them in an upright and stable position even while the shopping cart is moved about a store in order to keep them organized in the cart. Patent A also discloses that the receptacles could be any circular diameter to receive complementary shaped bottles or jars such as to securely hold 2-liter soft drink bottles or mayonnaise jars. A primary examiner rejected the claim as being obvious under 35 USC 103 over Patent A reasoning that Patent A suggests to one of ordinary skill in the art an insert for a shopping cart with circular receptacles for the purpose of stably maintaining any bottle, including wine bottles, while pushing the cart about a store so that the cart remains organized. Assume the examiner has made a sufficient prima facie case of obviousness. Following receipt of the rejection, the practitioner filed a timely reply. The practitioner argued that Patent A does not render obvious the claimed subject matter because there is no suggestion of a plastic insert to keep a wine bottle from falling and breaking in a shopping cart. Which of the following best explains why, in accordance with the patent laws, rules and the procedures as related in the MPEP, the examiner should or should not be persuaded by the practitioner's argument? A No, because Patent A suggests circular receptacles for any complementary bottle, albeit for a different purpose. B Yes, because there is no suggestion in Patent A that the plastic insert can hold a wine bottle. C Yes, because the claim uses the insert to keep the bottles from falling and breaking while Patent A uses the insert to keep the cart organized. D Yes, because Patent A is more interested in organizing boxes than holding bottles. E Yes, because the prevention from breakage is an unexpected property of the plastic insert.

A Related MPEP Chapter(s): MPEP 2100 - Patentability Answer Reasoning:ANSWER: (A) is the most correct answer. MPEP § 2144, under the heading "Rationale Different From Applicant's Is Permissible." Patent A suggests an insert with receptacles that are circular and which can be shaped to complement the shape of the object to be received. The purpose for this in Patent A is to keep the cart organized, not as in the claim to prevent the object from falling and breaking. The difference in objectives does not defeat the case for obviousness because, as MPEP § 2144 states, the "reason or motivation to modify the reference may often suggest what the inventor has done, but for a different purpose or to solve a different problem. It is not necessary that the prior art suggest the combination to achieve the same advantage or result discovered by applicant. In re Linter, 458 F.2d 1013, 173 USPQ 560 (CCPA 1972) ...; In re Dillon, 919 F.2d 688, 16 USPQ2d 1897 (Fed. Cir. 1990), cert. denied, 500 U.S. 904 (1991) ... ." In other words, it does not matter that Patent A does not appreciate the claimed purpose of preventing breakage. It suggests an insert with receptacles to hold bottles. That is enough to render the claimed subject matter prima facie obvious. The prima facie case is not rebutted by arguing that the purpose for the claimed insert is different form that specified for the insert described in Patent A. That is why answer (C) is wrong. To rebut the prima facie case, the practitioner must show a difference in structure instead. Answer (B) is wrong because the prima facie case is not rebutted by showing that Patent A does not teach wine bottles. This is not an anticipation rejection where identity of subject matter might be an issue. This is a question of obviousness. Therefore, it is sufficient to point out that Patent A is a generic teaching of shopping cart inserts that hold objects of any size and shape. (D) is not the most correct answer because what Patent A is interested in doing is irrelevant to the question of obviousness. (E) is not the most correct answer inasmuch as it was not the practitioner's argument. However, the question inquires about the merits of the argument that the practitioner made as set forth in the penultimate sentence of the question, not the merits of some hypothetical reply the examiner may communicate.

In accordance with the patent laws, rules and procedures as related in the MPEP, which of the following statements regarding operability or enablement of a prior art reference is the most correct? A The level of disclosure required for a reference to be enabling prior art is no less if the reference is a United States patent than if it is a foreign patent. B A reference is not presumed to be operable merely because it expressly anticipates or makes obvious all limitations of an applicant's claimed apparatus. C A non-enabling reference may not qualify as prior art for the purpose of determining anticipation or obviousness of the claimed invention. D A reference does not provide an enabling disclosure merely by showing that the public was in possession of the claimed invention before the date of the applicant's invention. E All of the above are correct.

A Related MPEP Chapter(s): MPEP 2100 - Patentability Answer Reasoning:ANSWER: (A) is the most correct answer. MPEP § 2121, under the heading "What Constitutes An 'Enabling Disclosure' Does Not Depend On The Type Of Prior Art The Disclosure Is Contained In," states, in reliance upon In re Moreton, 288 F.2d 708, 711, 129 USPQ 227, 230 (CCPA 1961): "The level of disclosure required within a reference to make it an 'enabling disclosure' is the same no matter what type of prior art is at issue.... There is no basis in the statute (35 U.S.C. 102 or 103) for discriminating either in favor of or against prior art references on the basis of nationality." Answer (B) is incorrect. MPEP § 2121, under the heading "Prior Art Is Presumed To Be Operable/Enabling," states that "[w]hen the reference relied on expressly anticipates or makes obvious all of the elements of the claimed invention, the reference is presumed to be operable." Answer (C) is incorrect. MPEP § 2121.01, under the heading "35 U.S.C. 103 Rejections And Use Of Inoperative Prior Art," quotes Symbol Technologies Inc. v. Opticon Inc., 935 F.2d 1569, 1578, 19 USPQ2d 1241, 1247 (Fed. Cir. 1991) as stating that "a non-enabling reference may qualify as prior art for the purpose of determining obviousness under 35 U.S.C. 103." Answer (D) is incorrect. MPEP § 2121.01 states that "[a] reference contains an 'enabling disclosure' if the public was in possession of the claimed invention before the date of invention." Answer (E) is incorrect because answers (B), (C) and (D) are incorrect.

Inventor files an application containing the following original Claim 1: 1. A widget comprising element A, and element B. In a first Office action on the merits, a primary examiner rejects claim 1 under 35 USC 103 as being obvious over reference X. Reference X explicitly discloses a widget having element A, but it does not disclose element B. The examiner, however, takes official notice of the fact that element B is commonly associated with element A in the art and on that basis concludes that it would have been obvious to provide element B in the reference X widget. In reply to the Office action, the registered practitioner representing the applicant makes no amendments, but instead requests reconsideration of the rejection by demanding that examiner show proof that element B is commonly associated with element A in the art. Which of the following actions, if taken by the examiner in the next Office action would be in accord with the USPTO rules and the procedures set forth in the MPEP? I. Vacate the rejection and allow the claim. II. Cite a reference that teaches element B commonly associated with element A in the art and make the rejection final. III. Deny entry of applicant's request for reconsideration on the ground that it is not responsive to the rejection and allow applicant time to submit a responsive amendment. A I and II only. B II only. C II and III only. D I, II, and III. E I and III only.

A Related MPEP Chapter(s): MPEP 2100 - Patentability Answer Reasoning:ANSWER: (A) is the most correct answer. MPEP § 2144.03 provides that when an applicant seasonably traverses an officially noticed fact, the examiner may cite a reference teaching the noticed fact and make the next action final. Here, applicant did seasonably traverse the noticed fact by demanding proof in response to the rejection. II is therefore an appropriate action by the examiner. I is also an appropriate action because the examiner should vacate a rejection based on official notice if no support for the noticed fact can be found in response to a challenge by the applicant. See In re Ahlert, 424 F.2d 1088, 1091 (C.C.P.A. 1970) ("[a]ssertions of technical facts in areas of esoteric technology must always be supported by citation to some reference work" and "[a]llegations concerning specific "knowledge" of the prior art, which might be peculiar to a particular art should also be supported"). (B) is incorrect because (A) is correct. (C), (D), and (E) are incorrect because action III is improper. An applicant is entitled to respond to a rejection by requesting reconsideration, with or without amending the application. 37 CFR § 1.111(a)(1). Applicant is also required to timely challenge a noticed fact in order to preserve the issue for appeal. MPEP § 2144.03.

Inventor files an application containing the following original Claim 1: 1. A widget comprising element A, and element B. In a first Office action on the merits, a primary examiner rejects claim 1 under 35 USC 103 as being obvious over reference X. Reference X explicitly discloses a widget having element A, but it does not disclose element B. The examiner, however, takes official notice of the fact that element B is commonly associated with element A in the art and on that basis concludes that it would have been obvious to provide element B in the reference X widget. In reply to the Office action, the registered practitioner representing the applicant makes no amendments, but instead requests reconsideration of the rejection by demanding that examiner show proof that element B is commonly associated with element A in the art. Which of the following actions, if taken by the examiner in the next Office action would be in accord with the patent laws, rules and procedures as related in the MPEP? I. Vacate the rejection and allow the claim. II. Cite a reference that teaches element B is commonly associated with element A in the art and make the rejection final. III. Deny entry of applicant's request for reconsideration on the ground that it is not responsive to the rejection and allow applicant time to submit a responsive amendment. A I and II only. B II only. C II and III only. D I, II, and III. E I and III only.

A Related MPEP Chapter(s): MPEP 2100 - Patentability Answer Reasoning:ANSWER: (A) is the most correct answer. MPEP § 2144.03 provides that when an applicant seasonably traverses an officially noticed fact, the examiner may cite a reference teaching the noticed fact and make the next action final. Here, applicant did seasonably traverse the noticed fact by demanding proof in response to the rejection. II is therefore an appropriate action by the examiner. I is also an appropriate action because the examiner should vacate a rejection based on official notice if no support for the noticed fact can be found in response to a challenge by the applicant. See In re Ahlert, 424 F.2d 1088, 1091 (C.C.P.A. 1970) ("[a]ssertions of technical facts in areas of esoteric technology must always be supported by citation to some reference work" and "[a]llegations concerning specific "knowledge" of the prior art, which might be peculiar to a particular art should also be supported"). (B) is incorrect because (A) is correct. (C), (D), and (E) are incorrect because action III is improper. An applicant is entitled to respond to a rejection by requesting reconsideration, with or without amending the application. 37 CFR § 1.111(a)(1). Applicant is also required to timely challenge a noticed fact in order to preserve the issue for appeal. MPEP § 2144.03.

Which of the following statements concerning reliance by an examiner on common knowledge in the art, in a rejection under 35 USC 103 is correct? I. An examiner's statement of common knowledge in the art is taken as admitted prior art, if applicant does not seasonably traverse the well known statement during examination. II. Applicant can traverse an examiner's statement of common knowledge in the art, at any time during the prosecution of an application to properly rebut the statement. III. If applicant rebuts an examiner's statement of common knowledge in the art in the next reply after the Office action in which the statement was made, the examiner can never provide a reference to support the statement of common knowledge in the next Office action and make the next Office action final. A I. B II. C III. D I and II. E None of the above.

A Related MPEP Chapter(s): MPEP 2100 - Patentability Answer Reasoning:ANSWER: (A) is the most correct answer. MPEP § 2144.03. II is incorrect because an applicant must seasonably traverse the well-know statement or the object of the well-known statement is taken to be admitted prior art. In re Chevenard, 60 USPQ 239 (CCPA 1943). Therefore (B) and (D) are incorrect. III is incorrect because the action can potentially be made final. Therefore (C) is incorrect. (E) is incorrect because (A) is correct.

15. Able conceived the invention claimed in a patent application. In conceiving the invention, Able used and adopted ideas and materials known in the art and invented by others. Ben, Able's employee, reduced the invention to practice at Able's request and totally pursuant to Able's suggestions. Being unable to afford a patent practitioner's fees to prepare and prosecute the application, Able convinced John to pay for the practitioner's services in return for an interest in the invention. John did nothing more than provide the funds for the practitioner. Which of the following is in accordance with the USPTO rules and the procedures set forth in the MPEP? A Able need not be the one to reduce the invention to practice so long as the reduction to practice occurred on his or her behalf. Able can be properly named as inventor in the application. B To be named an inventor, it is not necessary for John to have contributed to the conception of the invention. Ben, not Able, can be named as inventor in the application. C In conceiving the invention, Able may not consider and adopt ideas and materials derived from any sources, such as ideas of previous inventors. Able cannot be properly named as inventor in the application. D John and Able may be properly named as joint inventors of the invention in the application. E John, Ben, and Able may be properly named as joint inventors of the invention in the application.

A Related MPEP Chapter(s): MPEP 2100 - Patentability Answer Reasoning:ANSWER: (A) is the most correct answer. See MPEP § 2137.01, under the heading "The Inventor Is Not Required To Reduce The Invention To Practice," citing In re DeBaun, 214 USPQ 933, 936 (CCPA 1982). (B) is not correct. MPEP § 2137.01, under the heading "An Inventor Must Contribute To The Conception Of The Invention," citing, Fiers v. Revel, 25 USPQ2d 1601, 1604 - 05 (Fed. Cir. 1993); and In re Hardee, 223 USPQ 1122, 1123 (Dep. Asst. Comm'r Pat. 1984). (C) is not correct. MPEP § 2137.01, under the heading "As Long As The Inventor Maintains Intellectual Domination Over Making The Invention, Ideas, Suggestions, And Materials May Be Adopted From Others," citing Morse v. Porter, 155 USPQ 280, 283 (Bd. Pat. Inter. 1965); and New England Braiding Co., Inc. v. A.W. Chesterton Co., 23 USPQ2d 1622, 1626 (Fed. Cir. 1992). (D) and (E) are not correct. 35 U.S.C. § 116; MPEP § 2137.01, under the heading "Requirements For Joint Inventorship."

An examiner has advanced a reasonable basis for questioning the adequacy of the enabling disclosure in the specification of your client's patent application, and has properly rejected all the claims in the application. The claims in the application are drawn to a computer program system. In accordance with the USPTO rules and the procedures set forth in the MPEP, the rejection should be overcome by submitting _____________ A factual evidence directed to the amount of time and effort and level of knowledge required for the practice of the invention from the disclosure alone. B arguments by you (counsel) alone, inasmuch as they can take the place of evidence in the record. C an affidavit under 37 CFR 1.132 by an affiant, who is more than a routineer in the art, submitting few facts to support his conclusions on the ultimate legal question of sufficiency, i.e., that the system "could be constructed." D opinion evidence directed to the ultimate legal issue of enablement. E patents to show the state of the art for purposes of enablement where these patents have an issue date later than the effective filing date of the application under consideration.

A Related MPEP Chapter(s): MPEP 2100 - Patentability Answer Reasoning:ANSWER: (A) is the most correct answer. See MPEP § 2164.06(c) under the heading "Affidavit Practice (37 CFR 1.132)." Factual evidence directed to the amount of time and effort and level of knowledge required for the practice of the invention from the disclosure alone can rebut a prima facie case of nonenablement. See Hirschfield v. Banner, Commissioner of Patents and Trademarks, 200 USPQ 276, 281 (D.D.C. 1978). (B) is not correct. MPEP § 2164.06(c), under the heading "Arguments of Counsel," and see In re Budnick, 190 USPQ 422, 424 (CCPA 1976); In re Schulze, 145 USPQ 716 (CCPA 1965); and In re Cole, 140 USPQ 230 (CCPA 1964). (C) is not correct. MPEP § 2164.06(c), under the heading "Affidavit Practice (37 CFR 1.132)," and see In re Brandstadter, 179 USPQ 286 (CCPA 1973). (D) is not correct. MPEP § 2164.06(c), under the heading "Affidavit Practice (37 CFR 1.132)," and see Hirschfield v. Banner, Commissioner of Patents and Trademarks, 200 USPQ 276, 281 (D.D.C. 1978). (E) is not correct. MPEP § 2164.06(c), under the heading 'Referencing Prior Art Documents," and see In re Budnick, 190 USPQ 422, 424 (CCPA 1976); and In re Gunn, 190 USPQ 402, 406 (CCPA 1976).

A product-by-process claim is properly rejected over a reference under 35 USC 102. Which of the following statements is incorrect? A There is no anticipation unless each of the process steps recited in the claim is disclosed or inherent in the applied reference. B If the applied reference reasonably indicates that a product disclosed therein is the same or substantially the same as the claimed product, the burden shifts to the applicant to provide evidence to the contrary. C The rejection cannot be overcome by evidence of unexpected results. D The rejection can be overcome by evidence that the product in the reference does not necessarily or inherently possess a characteristic of the applicant's claimed product. E An affidavit or declaration under 37 CFR 1.131 cannot overcome a proper rejection under pre-AIA 35 USC 102(b) over a reference.

A Related MPEP Chapter(s): MPEP 2100 - Patentability Answer Reasoning:ANSWER: (A) is the most correct answer. The patentability of a product-by-process claim is determined based on the product itself, not on the process of making it. See In re Thorpe, 777 F.2d 695, 697, 227 USPQ 964, 966 (Fed. Cir. 1985), and MPEP § 2113. (B) and (D) are not proper choices because when evidence indicates that the applicant's product and that of the prior art are identical or substantially identical, the burden shifts to the applicant to overcome the rejection by providing evidence that the prior art product does not necessarily or inherently possess a relied-upon characteristic of the applicant's claimed product. See In re Fitzgerald, 619 F.2d 67, 70, 205 USPQ 594, 596 (CCPA 1980); In re Best, 562 F.2d 1252, 1255, 195 USPQ 430, 433-34 (CCPA 1977), and MPEP § 2112. (C) is not the proper choice because evidence of unexpected results is not relevant to anticipation. See In re Malagari, 499 F.2d 1297, 1302, 182 USPQ 549, 553 (CCPA 1974), and MPEP § 711.03(c). (E) is not the proper choice because a rejection under 35 U.S.C. § 102(b) is a statutory bar to patentability, and 37 C.F.R. § 1.131(a)(2) states that § 131 cannot be used to establish prior invention when the rejection is based upon a statutory bar.

In accordance with proper USPTO practice and procedure, which of the following statements is true? A Where sole patent applicant Able claims his invention in a Jepson-type claim, and the specification discloses that the subject matter of the preamble was invented by Baker before applicant's invention, the preamble is properly treated as prior art. B Where the sole patent applicant Able claims his invention in a Jepson-type claim, and the specification makes it clear that the claimed invention is an improvement on Able's own prior invention, which Able discovered less than one year before the filing date of the application, the preamble in the claim is properly treated as prior art. C Where the sole patent applicant Able claims his invention in a Jepson-type claim, and the specification makes it clear that the claimed invention is an improvement on an invention that Able discovered and publicly used and commercially sold by Able in Texas for several years before the filing date of the application, the preamble in the claim cannot properly be treated as prior art. D Where the sole applicant, Baker, states that something is prior art, the statement can be taken as being admitted prior art only if corroborated by objective evidence proffered by Baker, or found by the examiner. E No claim, including a Jepson-type claim, carries with it an implied admission that the elements in the preamble are old in the art.

A Related MPEP Chapter(s): MPEP 2100 - Patentability Answer Reasoning:ANSWER: (A) is true, and thus the most correct answer. As stated in MPEP § 2129, and see In re Fout, 675 F.2d 297, 300-01, 213 USPQ 532,535-36 (CCPA 1982). (B) is not true, and thus not correct. MPEP § 2129, and see Reading & Bates Construction Co. v. Baker Energy Resources Corp., 748 F.2d 645, 650, 223 USPQ 1168, 1172 (Fed. Cir. 1984). (C) is not true, and thus not correct because the admitted foundational discovery is a statutory bar. See the reasons discussed for answer (B). (D) is not true, and is thus incorrect. MPEP § 2129, and see In re Nomiya, 184 USPQ 607, 610 (CCPA 1975) (figures in the application labeled "prior art" held to be an admission that what was pictured was prior art relative to applicant's invention.). (E) is not true. MPEP § 2129; and see In re Ehrreich, 590 F.2d 902, 909 - 910, 200 USPQ 504, 510 (CCPA 1979); Sjolund v. Musland, 847 F.2d 1573, 1577, 6 USPQ2d 2020, 2023 (Fed. Cir. 1988); Pentec, Inc. v. Graphic Controls Corp., 776 F.2d 309, 315, 227 USPQ 766, 770 (Fed. Cir. 1985); and Reading & Bates Construction Co. v. Baker Energy Resources Corp., 748 F.2d 645, 650, 223 USPQ 1168, 1172 (Fed. Cir. 1984).

In accordance with the USPTO rules and the procedures set forth in the MPEP, which of the following statements is true? A Where sole patent applicant Able claims his invention in a Jepson-type claim, and the specification discloses that the subject matter of the preamble was invented by Baker before applicant's invention, the preamble is properly treated as prior art. B Where the sole patent applicant Able claims his invention in a Jepson-type claim, and the specification makes it clear that the claimed invention is an improvement on Able's own prior invention, which Able discovered less than one year before the filing date of the application, the preamble in the claim is properly treated as prior art. C Where the sole patent applicant Able claims his invention in a Jepson-type claim, and the specification makes it clear that the claimed invention is an improvement on an invention that Able discovered and publicly used and commercially sold by Able in Texas for several years before the filing date of the application, the preamble in the claim cannot properly be treated as prior art. D Where the sole applicant, Baker, states that something is prior art, the statement can be taken as being admitted prior art only if corroborated by objective evidence proffered by Baker, or found by the examiner. E No claim, including a Jepson-type claim, carries with it an implied admission that the elements in the preamble are old in the art.

A Related MPEP Chapter(s): MPEP 2100 - Patentability Answer Reasoning:ANSWER: (A) is true, and thus the most correct answer. As stated in MPEP § 2129, and see In re Fout, 675 F.2d 297, 300-01, 213 USPQ 532,535-36 (CCPA 1982). (B) is not true, and thus not correct. MPEP § 2129, and see Reading & Bates Construction Co. v. Baker Energy Resources Corp., 748 F.2d 645, 650, 223 USPQ 1168, 1172 (Fed. Cir. 1984). (C) is not true, and thus not correct because the admitted foundational discovery is a statutory bar. See the reasons discussed for answer (B). (D) is not true, and is thus incorrect. MPEP § 2129, and see In re Nomiya, 184 USPQ 607, 610 (CCPA 1975) (figures in the application labeled "prior art" held to be an admission that what was pictured was prior art relative to applicant's invention.). (E) is not true. MPEP § 2129; and see In re Ehrreich, 590 F.2d 902, 909 - 910, 200 USPQ 504, 510 (CCPA 1979); Sjolund v. Musland, 847 F.2d 1573, 1577, 6 USPQ2d 2020, 2023 (Fed. Cir. 1988); Pentec, Inc. v. Graphic Controls Corp., 776 F.2d 309, 315, 227 USPQ 766, 770 (Fed. Cir. 1985); and Reading & Bates Construction Co. v. Baker Energy Resources Corp., 748 F.2d 645, 650, 223 USPQ 1168, 1172 (Fed. Cir. 1984).

In connection with the utility of an invention described in a patent application, which of the following conforms to proper USPTO practice and procedure? A A deficiency under 35 USC 101 also creates a deficiency under 35 USC 112, first paragraph. B To overcome a rejection under 35 USC 101, it must be shown that the claimed device is capable of achieving a useful result on all occasions and under all conditions. C A claimed invention is properly rejected under 35 USC 101 as lacking utility if the particular embodiment disclosed in the patent lacks perfection or performs crudely. D To overcome a rejection under 35 USC 101, it is essential to show that the claimed invention accomplishes all its intended functions. E A claimed invention lacks utility if it is not commercially successful.

A Related MPEP Chapter(s): MPEP 2100 - Patentability Answer Reasoning:ANSWER: The most correct answer is (A). As stated in MPEP § 2107.01 (IV). A deficiency under 35 U.S.C. § 101 also creates a deficiency under 35 U.S.C. § 112, first paragraph. See In re Brana, 51 F.3d 1560, 34 USPQ2d 1436 (Fed. Cir. 1995); In re Jolles, 628 F.2d 1322, 1326 n.10, 206 USPQ 885, 889 n.11 (CCPA 1980); In re Fouche, 439 F.2d 1237, 1243, 169 USPQ 429, 434 (CCPA 1971) ("If such compositions are in fact useless, appellant's specification cannot have taught how to use them."). (B) is not correct. MPEP § 2107 (II), and see Brooktree Corp. v. Advanced Micro Devices, Inc., 977 F.2d 1555, 1571, 24 USPQ2d 1401, 1412 (Fed. Cir. 1992); and E.I. du Pont De Nemours and Co. v. Berkley and Co., 620 F.2d 1247, 1260 n.17, 205 USPQ 1, 10 n.17 (8th Cir. 1980). (C), (D) and (E) are not correct. MPEP § 2107 (II), and see E.I. du Pont De Nemours and Co. v. Berkley and Co., 620 F.2d 1247, 1260 n.17, 205 USPQ 1, 10 n.17 (8th Cir. 1980).

Upon examination of your client's patent application, the patent examiner is considering whether and what rejections to apply to the claims. One or more of the limitations in the claims is indefinite or lacks supporting disclosure. The examiner may not properly take which of the following actions or inactions? A If the claim is subject to plural interpretations due to a limitation being indefinite, the examiner may disregard any possibility of multiple interpretations. B If a claim is subject to more than one interpretation due to a limitation being indefinite, at least one of which would render the claim unpatentable over the prior art, the examiner should reject the claim as indefinite under 35 USC 112, second paragraph, and should reject the claim over the prior art based on the interpretation of the claim that renders the prior art applicable. C If no reasonably definite meaning can be ascribed to certain claim language, the examiner should reject the claim as indefinite under 35 USC 112, second paragraph, and not reject the claim as obvious. D When evaluating claims for obviousness under 35 USC 103, all the limitations of the claims, including new matter lacking supporting disclosure in the originally filed specification, must be considered and given weight. E (C) and (D).

A Related MPEP Chapter(s): MPEP 2100 - Patentability Answer Reasoning:ANSWER: The most correct answer is (A). MPEP § 2143.03 (Indefinite Limitations Must Be Considered). (B) is not correct because it is proper procedure to be followed by an examiner. MPEP § 2143.03, (Indefinite Limitations Must Be Considered), and see Ex parte Ionescu, 222 USPQ 537 (Bd. Pat. App. & Inter. 1984). (C) is not correct because it is proper procedure to be followed by an examiner. MPEP § 2143.03, (Indefinite Limitations Must Be Considered), and see In re Wilson, 165 USPQ 494 (CCPA 1970) (if no reasonably definite meaning can be ascribed to certain claim language, the claim is indefinite, not obvious). (D) is not correct because it is proper procedure to be followed by an examiner. MPEP § 2143.03, (Limitations Which Do Not Find Support In The Original Specification Must Be Considered), and see Ex parte Grasselli, 231 USPQ 393 (Bd. App. 1983), aff'd mem., 738 F.2d 453 (Fed. Cir. 1984). (E) is incorrect because the examiner may properly take the actions set forth in (B), (C), and (D).

An examiner has advanced a reasonable basis for questioning the adequacy of the enabling disclosure in the specification of your client's patent application, and has properly rejected all the claims in the application. The claims in the application are drawn to a computer program system. In accordance with proper USPTO practice and procedure, the rejection should be overcome by submitting _____________ A factual evidence directed to the amount of time and effort and level of knowledge required for the practice of the invention from the disclosure alone. B arguments by you (counsel) alone, inasmuch as they can take the place of evidence in the record. C an affidavit under 37 CFR 1.132 by an affiant, who is more than a routineer in the art, submitting few facts to support his conclusions on the ultimate legal question of sufficiency, i.e., that the system "could be constructed." D opinion evidence directed to the ultimate legal issue of enablement. E patents to show the state of the art for purposes of enablement where these patents have an issue date later than the effective filing date of the application under consideration.

A Related MPEP Chapter(s): MPEP 2100 - Patentability Answer Reasoning:ANSWER: The most correct answer is (A). MPEP § 2164.06(c) (Affidavit Practice (37 C.F.R. § 1.132)). Factual evidence directed to the amount of time and effort and level of knowledge required for the practice of the invention from the disclosure alone can rebut a prima facie case of nonenablement. See Hirschfield v. Banner, Commissioner of Patents and Trademarks, 200 USPQ 276, 281 (D.D.C. 1978). (B) is not correct. MPEP § 2164.06(c)(Arguments of Counsel), and see In re Budnick, 190 USPQ 422, 424 (CCPA 1976); In re Schulze, 145 USPQ 716 (CCPA 1965); and In re Cole, 140 USPQ 230 (CCPA 1964). (C) is not correct. MPEP § 2164.06(c) (Affidavit Practice (37 C.F.R. § 1.132)), and see In re Brandstadter, 179 USPQ 286 (CCPA 1973). (D) is not correct. MPEP § 2164.06(c) (Affidavit Practice (37 C.F.R. § 1.132)), and see Hirschfield v. Banner, Commissioner of Patents and Trademarks, 200 USPQ 276, 281 (D.D.C. 1978). (E) is not correct. MPEP § 2164.06(c), (Referencing Prior Art Documents), and see In re Budnick, 190 USPQ 422, 424 (CCPA 1976); and In re Gunn, 190 USPQ 402, 406 (CCPA 1976).

Regarding a power of attorney or authorization of agent in a patent application, which of the following is in accordance with the USPTO rules and the procedure set forth in the MPEP? A All notices and official letters for the patent owner or owners in a reexamination proceeding will be directed to the attorney or agent of record in the patent file at the address listed on the register of patent attorneys and agents. B Powers of attorney to firms submitted in applications filed in the year 2016 are recognized by the USPTO. C The associate attorney may appoint another attorney. D The filing and recording of an assignment will operate as a revocation of a power or authorization previously given. E Revocation of the power of the principal attorney or agent does not revoke powers granted by him or her to other attorneys or agents.

A Related MPEP Chapter(s): MPEP 400 - Representative of Inventor or Owner Answer Reasoning:ANSWER: (A) is the most correct answer. See 37 CFR § 1.33(c). (B) is incorrect. See MPEP § 403. Powers of attorney to firms filed in executed applications filed after July 2, 1971, are not recognized by the Patent and Trademark Office. However, the firm's address will be considered to be the correspondence address. (C) is incorrect. See MPEP §§ 402.02 and 406. The associate attorney may not appoint another attorney. (D) is incorrect. 37 CFR § 1.36. An assignment will not itself operate as a revocation of a power or authorization previously given. (E) is incorrect. See MPEP § 402.05. Revocation of the power of the principal attorney or agent revokes powers granted by him or her to other attorneys or agents.

In accordance with the MPEP and USPTO rules and procedure, a joint inventor on behalf of himself or herself and a nonsigning joint inventor in certain circumstances may make a patent application. Which of the following is an acceptable reason for filing an application with a declaration signed by a joint inventor, who is not the legal guardian of the other joint inventor, on behalf of himself and the nonsigning joint inventor? A The nonsigning joint inventor refuses to join in the application. B The nonsigning joint inventor is on vacation and is temporarily unavailable to sign the declaration. C The nonsigning joint inventor is hospitalized and is temporarily unavailable to sign the declaration. D The nonsigning joint inventor is out of town and is temporarily unavailable to sign the declaration. E All of the above.

A Related MPEP Chapter(s): MPEP 400 - Representative of Inventor or Owner Answer Reasoning:ANSWER: Choice (A) is the correct answer. MPEP § 409.03; MPEP § 09.03(d); and 37 C.F.R. § 1.47(a). (B) and (D) are each incorrect because MPEP § 409.03, subpart (d) states that "The fact that a nonsigning inventor is on vacation or out of town and is therefore temporarily unavailable to sign the declaration is not an acceptable reason for filing under 37 C.F.R. § 1.47." (C) is incorrect because MPEP § 409.03(d) further states that "the fact that an inventor is hospitalized and/or is not conscious is not an acceptable reason for filing under 37 C.F.R. § 1.47." (E) is incorrect because each of (B), (C), and (D) are incorrect.

With the exception that under 37 CFR 1.53 an application for patent may be assigned a filing date without payment of the basic filing fee, USPTO fees and charges payable to the USPTO requesting any action by the Office for which a fee or charge is payable, are required to be paid, in accordance with the MPEP and USPTO rules and procedure: A in advance, that is, at the time of requesting any action. B upon written notice from the USPTO. C within 20 days of requesting any action. D by the end of the fiscal year. E there are no fees.

A Related MPEP Chapter(s): MPEP 500 - Receipt and Handling of Mail and Papers Answer Reasoning:ANSWER: (A) is the most correct answer. See 37 CFR § 1.22(a); MPEP § 509. Answers (B) through (E) have no factual basis or foundation in the MPEP.

accordance with the patent laws, rules and procedures as related in the MPEP , which of the following paper is precluded from receiving the benefit of a certificate of mailing or transmission under 37 CFR 1.8? A An amendment, replying to an Office action setting a period for reply, transmitted by mail with a certificate of mailing to the USPTO from a foreign country. B An amendment, replying to an Office action setting a period for reply, transmitted by facsimile with a certificate of transmission to the USPTO from a foreign country. C An information disclosure statement (IDS) under 37 CFR 1.97 and 1.98 transmitted after the first Office action. D A request for continued examination (RCE) under 37 CFR 1.114. E An appeal brief.

A Related MPEP Chapter(s): MPEP 500 - Receipt and Handling of Mail and Papers Answer Reasoning:ANSWER: (A) is the most correct answer. See MPEP § 512, which states "The Certificate of Mailing procedure does not apply to papers mailed in a foreign country." (B) is not correct. See MPEP § 512. Certificate of transmission procedure applies to correspondence transmitted to the Office from a foreign country and an amendment is not prohibited from being transmitted by facsimile and is not precluded from receiving the benefits under 37 CFR § 1.8. (C) is not correct. See MPEP § 609, subsection "Time for Filing." An IDS will be considered to have been filed on the date of mailing if accompanied by a properly executed certificate of mailing or facsimile transmission under 37 CFR § 1.8. (D) is not correct. See MPEP § 706.07(h) Comparison Chart. An RCE is entitled to the benefit of a certificate of mailing or transmission under 37 CFR § 1.8. (E) is not correct. See MPEP § 1206. An appeal brief is entitled to the benefit of a certificate of mailing or transmission under 37 CFR § 1.8 because it is required to be filed in the Office within a set time period which is 2 months from the date of appeal.

In accordance with the USPTO rules and the procedures set forth in the MPEP, which of the following papers is precluded from receiving the benefit of a certificate of mailing or transmission under 37 CFR 1.8? A An amendment, replying to an Office action setting a period for reply, transmitted by mail with a certificate of mailing to the USPTO from a foreign country. B An amendment, replying to an Office action setting a period for reply, transmitted by facsimile with a certificate of transmission to the USPTO from a foreign country. C An information disclosure statement (IDS) under 37 CFR 1.97 and 1.98 transmitted after the first Office action. D A request for continued examination (RCE) under 37 CFR 1.114. E An appeal brief.

A Related MPEP Chapter(s): MPEP 500 - Receipt and Handling of Mail and Papers Answer Reasoning:ANSWER: (A) is the most correct answer. See MPEP § 512, which states "The Certificate of Mailing procedure does not apply to papers mailed in a foreign country." (B) is not correct. See MPEP § 512. Certificate of transmission procedure applies to correspondence transmitted to the Office from a foreign country and an amendment is not prohibited from being transmitted by facsimile and is not precluded from receiving the benefits under 37 CFR § 1.8. (C) is not correct. See MPEP § 609, under the heading "Time for Filing." An IDS will be considered to have been filed on the date of mailing if accompanied by a properly executed certificate of mailing or facsimile transmission under 37 CFR § 1.8. (D) is not correct. See MPEP § 706.07(h) Comparison Chart. An RCE is entitled to the benefit of a certificate of mailing or transmission under 37 CFR § 1.8. (E) is not correct. See MPEP § 1206. An appeal brief is entitled to the benefit of a certificate of mailing or transmission under 37 CFR § 1.8 because it is required to be filed in the Office within a set time period which is 2 months from the date of appeal.

Jane files a nonprovisional application with the USPTO containing at least one drawing figure under 35 USC 113 (first sentence) and at least one claim. Subsequently, Jane receives a "Notice of Omitted Items" from the USPTO indicating that the application which Jane filed lacks page 5 of the specification. Assuming that the application without page 5 satisfies 35 USC 112, which of the following statements is true based on proper USPTO practice and procedure? A If Jane is willing to accept the application as filed, she need not respond to the Notice (except to say she is not responding), and the Office will accord the filing date of the original application. Jane will need to file an amendment renumbering the pages consecutively and canceling incomplete sentences caused by the missing page 5. B Jane must promptly submit the omitted page and accept an application filing date as of the date of submission of the omitted page. C Jane must promptly submit the omitted page and will be accorded a filing date as of the date of filing the original application. D Within 3 months of the Notice date, Jane must file an affidavit asserting that page 5 was in fact deposited in the USPTO with the original application. Jane will be accorded the filing date of the original application. E Within 3 months of the Notice date, Jane must file a proper petition asserting that page 5 was in fact deposited in the USPTO with the original application, accompanied by the proper petition fee and evidence that page 5 was in fact deposited as alleged.

A Related MPEP Chapter(s): MPEP 600 - Parts, Form, and Content of Application Answer Reasoning:ANSWER: (A) is correct, however, Jane must inform the office that she is "not responding", and (B), (C), (D) and (E) are wrong. MPEP § 601.01(d).

Which of the following is not in accordance with the provisions of the MPEP Chapter 600? A A request for a refund must be filed within two years from the date the fee was paid or, in the case of a fee paid by mistake, within one year from the time the error was discovered. B A change of purpose after the payment of a fee, such as when a party desires to withdraw a patent filing for which the fee was paid, including an application, an appeal or a request for an oral hearing, will not entitle a party to a refund of such fee. C The Office will not refund amounts of twenty-five dollars or less, unless a refund is specifically requested. D Any refund of a fee paid by credit card will be by a credit to the credit card account to which the fee was charged. E When a fee is paid where no fee is required, this is considered to be a fee paid by mistake.

A Related MPEP Chapter(s): MPEP 600 - Parts, Form, and Content of Application Answer Reasoning:ANSWER: (A) is the best answer as there is no provision regarding one year from discovery in 37 C.F.R. § 1.26. As to (B) through (E) see MPEP § 607.02. The Office will refund amounts of twenty-five dollars or less if requested to do so by the applicant. See MPEP § 607.02 at p. 600-51. As to (A), (B), (D) and (E), see MPEP § 607.02 at p. 600-51.

In accordance with the patent laws, rules and procedures as related in the MPEP, for a nonprovisional application to receive a fixed filing date in the USPTO under 37 CFR 1.53(b), all of the following must be filed except: A An oath or declaration executed by applicant pursuant to 37 CFR 1.63. B A specification as prescribed by the first paragraph of 35 USC 112. C A description pursuant to 37 CFR 1.71. D At least one claim pursuant to 37 CFR 1.75. E A drawing when required by 37 CFR 1.81(a).

A Related MPEP Chapter(s): MPEP 600 - Parts, Form, and Content of Application Answer Reasoning:ANSWER: (A) is the most correct answer. 35 U.S.C. § 111; 37 CFR § 1.53; MPEP § 601.01 As provided in 37 CFR § 1.53(f) and MPEP § 601.01(a), the oath or declaration for an application filed under 37 CFR 1.53(b) can be submitted after the filing date. (B), (C), (D) and (E) are incorrect. 37 CFR § 53(b); MPEP § 601.01. 37 CFR § 1.53(b) provides that a filing date is granted on the date on which a specification as prescribed by 35 U.S.C. § 112 containing a description pursuant to § 1.71 and at least one claim pursuant to § 1.75, and any drawing required by § 1.81(a) are filed in the Office. Thus, (B), (C), (D) and (E) are needed to obtain a filing date.

In accordance with the patent laws, rules and procedures as related in the MPEP, for a nonprovisional application to receive a non-shifting filing date in the USPTO under 37 CFR 1.53(b), all of the following must be filed except: A The basic filing fee required by 37 CFR 1.16(a). B A specification as prescribed by the first paragraph of 35 USC 112. C A description pursuant to 37 CFR 1.71. D At least one claim pursuant to 37 CFR 1.75. E Any drawing required by 37 CFR 1.81(a).

A Related MPEP Chapter(s): MPEP 600 - Parts, Form, and Content of Application Answer Reasoning:ANSWER: (A) is the most correct answer. 35 U.S.C. § 111; 37 CFR § 1.53; MPEP § 601.01. As provided in MPEP § 601.01(a), the filing fee for an application filed under 37 CFR 1.53(b) can be submitted after the filing date. (B), (C), (D) and (E) are incorrect. 37 CFR § 53(b); MPEP § 601.01. 37 CFR § 1.53(b) provides that a filing date is granted on the date on which a specification as prescribed by 35 U.S.C. § 112 containing a description pursuant to 37 CFR § 1.71 and at least one claim pursuant to 37 CFR § 1.75, and any drawing required by 37 CFR § 1.81(a) are filed in the Office. Thus, (B), (C), (D) and (E) are needed to obtain a filing date.

Registered practitioner Joe duly files a non-provisional utility patent application on May 6, 2016. The USPTO sends Joe a notice of allowance dated November 13, 2017. On November 23, 2017, Joe learns about a publication ("Smith reference") which he knows to be material to patentability of the claims presented in the application, but which was not considered by the examiner during prosecution of the application. Joe prepares an information disclosure statement that complies with the provisions of 37 CFR 1.98, listing the Smith reference. In accordance with USPTO rules and procedure which of the following actions, if taken by Joe, will result in the examiner considering the Smith reference during prosecution of the application? A Prior to Wednesday, February 14, 2018, filing a request for continued examination of the application, the information disclosure statement, and the fee for a request for continued examination, but not paying the issue fee. B Timely paying the issue fee, and thereafter filing a request for continued examination of the application together with the information disclosure statement, and the fee for a request for continued examination, but not submitting a petition under 37 CFR 1.313. C After Tuesday, February 13, 2018, filing a request for continued examination of the application together with the information disclosure statement, and the fee for a request for continued examination, but not paying the issue fee. D Timely paying the issue fee, and after the patent issues filing a request for continued examination of the application, the information disclosure statement, the fee for a request for continued examination, and a petition under 37 CFR 1.313. E None of the above.

A Related MPEP Chapter(s): MPEP 600 - Parts, Form, and Content of Application Answer Reasoning:ANSWER: (A) is the most correct answer. 37 CFR 1.114, MPEP § 609, paragraph III. B(1)(b), under the heading "RCE and CPA," and MPEP 706.07(h), under the heading "II. Submission Requirement." In (A), the information disclosure statement, is a submission under 37 CFR § 1.114(c), and the RCE was filed before the payment of the issue fee. 37 CFR § 1.114(a)(1). (B) is incorrect because the request for continued examination was filed after payment of the issue fee, and is filed without a petition under 37 CFR § 1.313 being granted. Therefore (B) does not satisfy the provision of 37 CFR § 1.114(a)(1). (C) is incorrect because the application becomes abandoned on February 14, 2018 for failure to pay the issue fee. Therefore the request for continued examination does not satisfy the provision of 37 CFR § 1.114(a)(2). (D) is incorrect because a petition under 37 CFR § 1.313 will not be effective to withdraw the application from issue unless it is actually received and granted by the appropriate officials before the date of issue. 37 CFR § 1.313(d). Thus, the request for continued examination in (D) does not satisfy the provision of 37 CFR § 1.114(a)(1). (E) is incorrect because (A) is correct.

16. In accordance with the USPTO rules and the procedures set forth in the MPEP, for a nonprovisional application to receive a non-shifting filing date in the USPTO under 37 CFR 1.53(b), all of the following must be filed except: A The basic filing fee required by 37 CFR 1.16(a). B A specification as prescribed by the first paragraph of 35 USC 112. C A description pursuant to 37 CFR 1.71. D At least one claim pursuant to 37 CFR 1.75. E Any drawing required by 37 CFR 1.81(a).

A Related MPEP Chapter(s): MPEP 600 - Parts, Form, and Content of Application Answer Reasoning:ANSWER: (A) is the most correct answer. See 35 U.S.C. § 111; 37 CFR § 1.53; MPEP § 601.01. As provided in MPEP § 601.01(a), the filing fee for an application filed under 37 CFR 1.53(b) can be submitted after the filing date. Answers (B), (C), (D) and (E) are incorrect. See 37 CFR § 53(b); MPEP § 601.01. 37 CFR § 1.53(b) provides that a filing date is granted on the date on which a specification as prescribed by 35 U.S.C. § 112 containing a description pursuant to 37 CFR § 1.71 and at least one claim pursuant to 37 CFR § 1.75, and any drawing required by 37 CFR § 1.81(a) are filed in the Office. Thus, (B), (C), (D) and (E) are needed to obtain a filing date.

In accordance with the USPTO rules and the procedures set forth in the MPEP, for a nonprovisional application to receive a non-shiftable filing date in the USPTO under 37 CFR 1.53(b), all of the following must be filed except: A An oath or declaration under 37 CFR 1.51(b)(2). B A specification as prescribed by the first paragraph of 35 USC 112. C A description pursuant to 37 CFR 1.71. D At least one claim pursuant to 37 CFR 1.75. E Any drawing required by 37 CFR 1.81(a).

A Related MPEP Chapter(s): MPEP 600 - Parts, Form, and Content of Application Answer Reasoning:ANSWER: (A) is the most correct answer. See 35 U.S.C. § 111; 37 CFR § 1.53; MPEP § 601.01. As provided in MPEP § 601.01(a), the oath or declaration for an application filed under 37 CFR § 1.53(b) can be submitted after the filing date. Answers (B), (C), (D) and (E) are incorrect. 37 CFR § 1.53(b); MPEP § 601.01. 37 CFR § 1.53(b) provides that a filing date is granted on the date on which a specification as prescribed by 35 U.S.C. § 112 containing a description pursuant to § 1.71 and at least one claim pursuant to § 1.75, and any drawing required by § 1.81(a) are filed in the Office. Thus, (B), (C), (D) and (E) are needed to obtain a filing date.

The following statements relate to "multiple dependent claims." Which statement is not in accord with proper USPTO practice and procedure? A A multiple dependent claim contains all the limitations of all the alternative claims to which it refers. B A multiple dependent claim contains in any one embodiment only those limitations of the particular claim referred to for the embodiment under consideration. C A multiple dependent claim must be considered in the same manner as a plurality of single dependent claims. D Restriction may be required between the embodiments of a multiple dependent claim. E The limitations or elements of each claim incorporated by reference into a multiple dependent claim must be considered separately.

A Related MPEP Chapter(s): MPEP 600 - Parts, Form, and Content of Application Answer Reasoning:ANSWER: (A) is the most correct answer. The answer is inconsistent with 35 U.S.C. § 112 and MPEP § 608.01(n), subpart I.B.4. (B), (C), and (E) are wrong answers because they are consistent with 35 U.S.C. § 112 and MPEP § 608.01(n), subpart I.B.4. (D) is wrong because it is consistent with MPEP § 608.01(n), subpart I. C.

A registered practitioner filed a utility application on February 11, 2016. On April 4, 2016, the practitioner filed an information disclosure statement (IDS) in the application. The practitioner received a notice of allowance dated January 3, 2017 soon after it was mailed. When discussing the application with the practitioner on January 21, 2017, and before paying the issue fee, the client notices for the first time that a reference, which is one of many patents obtained by the client's competitor, was inadvertently omitted from the IDS. The client has been aware of this reference since before the application was filed. The client is anxious to have this reference appear on the face of the patent as having been considered by the USPTO. Which of the following actions, if taken by the practitioner, would not be in accord with the patent law, rules and procedures as related by the MPEP? A Before paying the issue fee, timely file an IDS citing the reference, along with the certification specified in 37 CFR 1.97(e), and any necessary fees. B Within three months of the mail date of the notice of allowance, without paying the issue fee, timely file a Request for Continued Examination (RCE) under 37 CFR 1.114, accompanied by the fee for filing an RCE, and an IDS citing the reference. C Within three months of the mail date of the notice of allowance, without paying the issue fee, timely file a continuing application under 37 CFR 1.53(b), an IDS citing the reference, and any necessary fees. D After paying the issue fee, timely file a petition to withdraw the application from issue to permit the express abandonment of the application in favor of a continuing application, a continuation application under 37 CFR 1.53(b), an IDS citing the reference, and any necessary fees. E After paying the issue fee, timely file a petition to withdraw the application from issue to permit consideration of a Request for Continued Examination (RCE) under 37 CFR 1.114, the fee for filing an RCE, and an IDS citing the reference.

A Related MPEP Chapter(s): MPEP 600 - Parts, Form, and Content of Application Answer Reasoning:ANSWER: (A), describing a procedure that is not in accordance with the USPTO rules and the procedures set forth in the MPEP , the most correct answer. MPEP § 609, under the heading "Minimum Requirements for an Information Disclosure Statement," under the subheading "B(3). Information Disclosure Statement Filed After B(2), but Prior to Payment of Issue Fee 37 CFR 1.97 (d)", and subheading "B(5) Statement Under 37 CFR 1.97(e)." (A) The statement specified in 37 CFR § 1.97(e) requires that the practitioner certify, after reasonable inquiry, that no item of information contained in the IDS was known to any individual designated in 37 CFR § 1.56(c) more than three months prior to the filing of the information disclosure statement. The practitioner cannot certify this because the reference was known to the client before February 11, 2002, the time of filing of the utility application, which was more than three months prior to the filing of the information disclosure statement. See (B), stating a procedure that conforms with the USPTO rules and the procedures set forth in the MPEP, is an incorrect answer. Under 37 CFR § 1.313(a), a petition to withdraw the application from issue is not required if a proper RCE is filed before payment of the issue fee. (C), stating a procedure that conforms with the USPTO rules and the procedures set forth in the MPEP, is an incorrect answer. A practitioner can file a continuing application on or before the date that the issue fee is due and permit the parent application to become abandoned for failure to pay the issue fee. (D), stating a procedure that conforms with the USPTO rules and the procedures set forth in the MPEP, is an incorrect answer. Under 37 CFR § 1.313(c)(3), a petition to withdraw the application from issue can be filed after payment of the issue fee to permit the express abandonment of the application in favor of a continuing application. (E), stating a procedure that conforms with the USPTO rules and the procedures set forth in the MPEP, is an incorrect answer. Under 37 CFR § 1.313(c)(2), a petition to withdraw the application from issue can be filed after payment of the issue fee to permit consideration of a Request for Continued Examination (RCE) under 37 CFR § 1.114. See also MPEP § 1308.

Which of the following is not in accordance with the recommended characteristics set forth in the provisions of the MPEP for the summary of the invention? A The summary is separate and distinct from the abstract and is directed toward the disclosure as a whole, rather than just the invention. B The summary may point out the advantages of the invention or how it solves problems previously existent in the prior art (and preferably indicated in the Background of the Invention). C In chemical cases the summary should point out in general terms the utility of the invention. D If possible, the summary should set forth the nature and gist of the invention or the inventive concept should be set forth. E Any stated objects of the invention should be treated briefly in the summary and only to the extent that they contribute to an understanding of the invention.

A Related MPEP Chapter(s): MPEP 600 - Parts, Form, and Content of Application Answer Reasoning:ANSWER: (A). Item (A) should read, "The summary is separate and distinct from the abstract and is directed toward the invention rather than the disclosure as a whole." As to (B) through (E), see 37 C.F.R. § 1.73; MPEP § 608.01(a) and (d).

Which of the following practices or procedures may be properly employed in accordance with the USPTO rules and the procedures set forth in the MPEP to overcome a rejection properly based on 35 USC 102(a)? A Perfecting a claim to priority under 35 USC 119(a)-(d) based on a foreign application having a foreign priority filing date that antedates the reference. B Filing a declaration under 37 CFR 1.131 showing that the cited prior art antedates the invention. C Filing a declaration under 37 CFR 1.132 showing that the reference invention is by "others." D Perfecting priority under 35 USC 119(e) or 120 by, in part, amending the declaration of the application to contain a specific reference to a prior application having a filing date prior to the reference. E (A), (B) (C), and (D).

A Related MPEP Chapter(s): MPEP 700 - Examination of Applications Answer Reasoning:ANSWER: (A) is the most correct answer. See MPEP § 706.02(b), under the heading "Overcoming a 35 U.S.C. § 102 Rejection Based on a Printed Publication or Patent." (B), and (C) are incorrect because they present showings that support the rejection. See MPEP § 706.02(b), supra. (D) are not correct because to perfect priority under 35 U.S.C. §§ 119(e) or 120 it is, inter alia, necessary to amend the specification of the application to contain a specific reference to a prior application having a filing date prior to the reference. See MPEP § 706.02(b), supra. Furthermore, the declaration is not to be amended. (E) is incorrect because (B), (C) and (D) are incorrect.

Which of the following is in accordance with the practice and procedures of Chapter 600 of the MPEP and/or 37 CFR 1.52(c)? A Minor handwritten alterations to the claims in a patent application, added by the inventor post oath execution, may be accepted if the signed oath still applies. B The Office will consider evidence of whether noninitialed and/or nondated alterations were made before or after the signing of the oath or declaration rather than require a new oath or declaration. C Any alteration to a patent application made by the applicant may be made after the application was signed and sworn to. D Non-initialed or non-dated handwritten alterations to the claims on an application filed in the USPTO are considered to be a minor informality. Thus, the Office personnel should not object to the same. E It is proper for an applicant to sign an oath or declaration even when the oath or declaration (i) does not identify a patent application or (ii) is not attached to or physically located together with the patent application.

A Related MPEP Chapter(s): MPEP 600 - Parts, Form, and Content of Application Answer Reasoning:ANSWER: (A). MPEP 608.01 provides that minor handwritten alterations are in accord with 37 C.F.R. § 1.52(c). (B) is incorrect. The Office will not consider whether noninitialed and/or nondated alterations were made before or after the signing of the oath or declaration; it does not matter as long as the oath still applies. (C) is incorrect because minor handwritten alterations are in accord with 37 C.F.R. § 1.52(c), not "any alteration." (D) is incorrect because the Office may seek to determine whether the oath applies. (E) is incorrect because it is improper for an applicant to sign an oath or declaration which is not attached to or does not identify the patent application.

A registered practitioner filed a utility application on February 11, 2016. On April 4, 2016, the practitioner filed an information disclosure statement (IDS) in the application. The practitioner received a notice of allowance dated January 3, 2017 soon after it was mailed. When discussing the application with the practitioner on January 21, 2017, and before paying the issue fee, the client notices for the first time that a reference, which is one of many patents obtained by the client's competitor, was inadvertently omitted from the IDS. The client has been aware of this reference since before the application was filed. The client is anxious to have this reference appear on the face of the patent as having been considered by the USPTO. Which of the following actions, if taken by the practitioner, would not be in accord with the USPTO rules and the procedures set forth in the MPEP? A Before paying the issue fee, timely file an IDS citing the reference, along with the certification specified in 37 CFR 1.97(e), and any necessary fees. B Within three months of the mail date of the notice of allowance, without paying the issue fee, timely file a Request for Continued Examination (RCE) under 37 CFR 1.114, accompanied by the fee for filing an RCE, and an IDS citing the reference. C Within three months of the mail date of the notice of allowance, without paying the issue fee, timely file a continuing application under 37 CFR 1.53(b), an IDS citing the reference, and any necessary fees. D After paying the issue fee, timely file a petition to withdraw the application from issue to permit the express abandonment of the application in favor of a continuing application, a continuation application under 37 CFR 1.53(b), an IDS citing the reference, and any necessary fees. E After paying the issue fee, timely file a petition to withdraw the application from issue to permit consideration of a Request for Continued Examination (RCE) under 37 CFR 1.114, the fee for filing an RCE, and an IDS citing the reference.

A Related MPEP Chapter(s): MPEP 600 - Parts, Form, and Content of Application Answer Reasoning:ANSWER: Answer (A), describing a procedure that is not in accordance with the USPTO rules and the procedures set forth in the MPEP, is the most correct answer. See MPEP § 609, under the heading "Minimum Requirements for an Information Disclosure Statement," and subheading "B(3). Information Disclosure Statement Filed After B(2), but Prior to Payment of Issue Fee 37 CFR 1.97 (d)", and subheading "B(5) Statement Under 37 CFR 1.97(e)." The statement specified in 37 CFR § 1.97(e) requires that the practitioner certify, after reasonable inquiry, that no item of information contained in the IDS was known to any individual designated in 37 CFR § 1.56(c) more than three months prior to the filing of the information disclosure statement. The practitioner cannot certify this because the reference was known to the client before February 11, 2016, the time of filing of the utility application, which was more than three months prior to the filing of the information disclosure statement. (B), stating a procedure that conforms with the USPTO rules and the procedures set forth in the MPEP, is an incorrect answer. Under 37 CFR § 1.313(a), a petition to withdraw the application from issue is not required if a proper RCE is filed before payment of the issue fee. (C), stating a procedure that conforms with the USPTO rules and the procedures set forth in the MPEP, is an incorrect answer. A practitioner can file a continuing application on or before the date that the issue fee is due and permit the parent application to become abandoned for failure to pay the issue fee. (D), stating a procedure that conforms with the USPTO rules and the procedures set forth in the MPEP, is an incorrect answer. Under 37 CFR § 1.313(c)(3), a petition to withdraw the application from issue can be filed after payment of the issue fee to permit the express abandonment of the application in favor of a continuing application. (E), stating a procedure that conforms with the USPTO rules and the procedures set forth in the MPEP, is an incorrect answer. Under 37 CFR § 1.313(c)(2), a petition to withdraw the application from issue can be filed after payment of the issue fee to permit consideration of a Request for Continued Examination (RCE) under 37 CFR § 1.114. See also MPEP § 1308.

Co-inventors Smith and Jones filed an application for a patent on a cell phone, on May 15, 2016. They received a first Office action from a primary examiner rejecting the claims under 35 USC 102(a) over a publication by Bell and Watson, published on April 5, 2016, describing a cell phone having all the same features as is claimed in the patent application. In reply, the co-inventors each submitted a declaration under 37 CFR 1.131 stating that they had actually reduced the invention to practice no later than March 13, 2016. However, the declarations failed to include two claimed features. Neither the particular antenna needed to enable the cell phone could receive transmissions from the local cellular transmitting tower, nor a detachable carrying strap was included in the declarations. As evidence of their prior reduction to practice, Smith and Jones submitted their co-authored journal article. The journal article contained a figure of the cell phone as described in the declarations. That is, the cell phone shown in the figure of the article lacked an antenna and a detachable strap. The article was received by the journal on March 13, 2016, and was published on April 30, 2016. The cell phones shown in the figure in the Bell and Watson publication, and in the Smith and Jones patent application have the particular antenna and a detachable strap. Which of the following actions, if taken by the examiner, would be the most proper in accordance with the patent laws, rules and the procedures as related in the MPEP? A The examiner should maintain the rejection of the claims under 35 USC 102(a) and make the rejection final inasmuch as a 1.131 affidavit has no application in an AIA case. B The examiner should withdraw the rejection and look for references which have a publication date prior to May 15, 2015. C The examiner should withdraw the rejection and notify Smith and Jones that their application is in condition for allowance. D The examiner should maintain the rejection, but indicate that the claims would be allowable if Smith and Jones provided an original copy of the figure published in their journal article as factual support for their declarations. E The examiner should maintain the rejection and inform Smith and Jones that the declarations are insufficient because they cannot "swear behind" a reference which is a statutory bar.

A Related MPEP Chapter(s): MPEP 700 - Examination of Applications Answer Reasoning: (A) is the correct answer. A 1.131 affidavit, proving a date of invention, has no application vis-a-vis an application being examined under AIA "first to file" law. Accordingly, the examiner should maintain the rejection and make it final. (B) and (C) are incorrect choices since the evidence of record shows that Smith and Jones are simply unable to overcome the prior art. (D) is wrong because an original copy of the published figure which shows that Smith and Jones were not in possession of the claimed invention prior to Bell and Watson publication cannot help their case. (E) is incorrect because prior art under 102(a) is not necessarily a statutory bar if inventors can "pre-date" with a disclosure of their own using a 1.130 affidavit.

Absent a Certificate of Mailing or Transmission, or use of Express Mail, which of the following actions requires a petition for an extension of time and the appropriate fee? A Applicant's complete first reply to a final Office action filed on the first day following the end of a shortened statutory period for reply. The Shortened Statutory Period ended on a Wednesday that was not a federal holiday, and the Office is open. B Interview with examiner conducted after the expiration of the shortened statutory period for reply to a final Office action, but within the 6 months statutory period. C Action by applicant to correct formal matters identified for the first time after a reply was made to a final Office action in an Ex parte Quayle action where the application is otherwise in condition for allowance. D Applicant's complete first reply to a final Office action filed within 2 months of the final Office action setting a 3 month shortened statutory period for reply. E Applicant's complete first reply to an Office action on the last day of a shortened statutory period for reply, where the Office action withdraws the finality of a previous Office action in view of a new ground of rejection.

A Related MPEP Chapter(s): MPEP 700 - Examination of Applications Answer Reasoning:ANSWER: (A) is correct. MPEP § 710.01.

Roberto files a U.S. patent application fourteen months after he perfects an invention in Europe in 2009. Which of the following would establish a statutory bar against the granting of a U.S. patent to Roberto? A A foreign patent issued to Roberto 11 months prior to the filing date of Roberto's U.S. patent application. The foreign patent was granted on an application that was filed 23 months prior to the effective filing date of Roberto's U.S. patent application. The foreign patent application and the U.S. patent application claim the same invention. B The invention was described in a printed publication in the United States, 11 months prior to the filing date of the U.S. patent application. C The invention was in public use in the United States, less than one year prior to the filing date of the U.S. patent application. D The invention was on sale in a foreign (NAFTA member) country, more than one year prior to the filing date of the U.S. patent application. E None of the above.

A Related MPEP Chapter(s): MPEP 700 - Examination of Applications Answer Reasoning:ANSWER: (A) is the correct answer. 35 U.S.C. § 102 (d), and MPEP § 706.02(c). (A) is correct because the foreign patent establishes a bar under 35 U.S.C. §102 (d). MPEP § 706.02 (e). (B) is incorrect because the invention is not described in a printed publication more than one year prior to the date of the U.S. application. 35 U.S.C. § 102(b). (C) is incorrect because the invention is not in public use more than one year prior to the date of the U.S. application. MPEP § 2133. (D) is incorrect because the sale is not in the United States. 35 U.S.C. § 102(b); MPEP §§ 706.02(c) and 2133.03(d). (E) is incorrect because (A) is correct.

In 2012, Rolland files a U.S. patent application fourteen months after he perfects an invention in Europe. In accordance with the USPTO rules and the procedures set forth in the MPEP, which of the following would establish a statutory bar against the granting of a U.S. patent to Rolland? A A foreign patent issued to Rolland 13 months prior to the filing date of Rolland's U.S. patent application. The foreign patent was granted on an application that was filed 23 months prior to the effective filing date of Rolland's U.S. patent application. The foreign patent application and the U.S. patent application claim the same invention. B The invention was described in a printed publication in the United States, 11 months prior to the filing date of the U.S. patent application. C The invention was in public use in the United States, less than one year prior to the filing date of the U.S. patent application. D The invention was on sale in a foreign (NAFTA member) country, less than one year prior to the filing date of the U.S. patent application. E None of the above.

A Related MPEP Chapter(s): MPEP 700 - Examination of Applications Answer Reasoning:ANSWER: (A) is the correct answer. See 35 U.S.C. § 102 and MPEP § 706. (A) is correct because the foreign patent establishes a bar under 35 U.S.C. §102. (B) is incorrect because the invention is not described in a printed publication more than one year prior to the date of the U.S. application. 35 U.S.C. § 102. (C) is incorrect because the invention is not in public use more than one year prior to the date of the U.S. application. MPEP § 2133. (D) is incorrect because the sale is less than one year before filing. 35 U.S.C. § 102; MPEP §§ 706.02(c) and 2133.03(d). (E) is incorrect because (A) is correct.

Registered practitioner Rick drafted a patent application for inventor Sam. The application was filed in the USPTO on May 15, 2016, with a power of attorney appointing Rick. On March 15, 2017, Sam filed a revocation of the power of attorney to Rick, and a new power of attorney appointing registered practitioner Dave. In a non-final Office action dated September 12, 2017, the examiner included a requirement for information, requiring Dave to submit a copy of any non-patent literature, published application, or patent that was used to draft the application. Which of the following, if timely submitted by Dave in reply to the requirement for information, will be accepted as a complete reply to the requirement for information? A A statement by Dave that the information required to be submitted is unknown and is not readily available to Dave. B A statement by Dave that the requirement for information is improper because it was included in a non-final Office action. C A statement by Dave that the requirement for information is improper because Dave is not an individual identified under 37 CFR 1.56(c). D A statement by Dave that the requirement for information is improper because information used to draft a patent application may not be required unless the examiner identifies the existence of a relevant database known by Sam that could be searched for a particular aspect of the invention. E None of the above.

A Related MPEP Chapter(s): MPEP 700 - Examination of Applications Answer Reasoning:ANSWER: (A) is the correct answer. See 37 CFR § 1.105(a)(3); MPEP § 704.12(b). MPEP § 704.12(b) states "A reply stating that the information required to be submitted is unknown and/or is not readily available to the party or parties from which it was requested will generally be sufficient unless, for example, it is clear the applicant did not understand the requirement, or the reply was ambiguous and a more specific answer is possible." The given facts do not state that the applicant did not understand the requirement, or the reply was ambiguous and a more specific answer is possible. (B) is incorrect because the requirement for information may be included in an Office action, or sent separately. 37 CFR § 1.105(b). (C) is incorrect because 37 CFR § 1.56(c) includes each attorney or agent who prepares or prosecutes the application. 37 CFR § 1.56(c)(2). (D) is incorrect because information used to draft a patent application may be required and there is no support for (D) in 37 CFR § 1.105. (E) is incorrect because (A) is correct.

The sole claim in an application filed by A and having an effective filing date of June 5, 2016, recites an electrical signal amplifier comprising a plurality of germanium transistors connected together in a particular configuration. The claim is rejected under 35 USC 103 as being obviousness over a primary nonpatent reference publication (Reference P) in view of a secondary nonpatent reference publication (Reference S). Reference P has an effective date of April 3, 2016, and names A and B as the authors. Reference S has an effective date of December 10, 2015, and names C as the sole author. Reference P discloses an electrical signal amplifier including a plurality of silicon transistors connected together in the same configuration as that set forth in the claim. Reference S discloses a signal amplifier employing germanium transistors connected in a configuration different from the claimed configuration. The applicant does not deny that the references render the claimed subject matter prima facie obvious. Which, if any, of the declarations under 37 CFR 1.130 set forth below should be sufficient under the patent laws, rules and procedures as related in the MPEP to overcome the rejection? A An uncontradicted declaration by A asserting that the subject matter relied on by the examiner in reference P constitutes A's sole invention, with the result that Reference P is not available as prior art against the claim. B A declaration by A asserting that "the claimed amplifier has satisfied a long-felt need in the art." C A declaration by A and accompanying copies of competitors' advertisements which conclusively show that those competitors have exactly copied appellant's commercial embodiment of the claimed amplifier. D A declaration by A and supporting documentation establishing that ever since the filing date of A's application, sales of the commercial embodiment of A's claimed amplifier have consistently constituted ninety percent or more of the relevant signal amplifier market in the United States. E None of the above.

A Related MPEP Chapter(s): MPEP 700 - Examination of Applications Answer Reasoning:ANSWER: (A) is the correct answer. See MPEP § 717.01 (III), which states "Under certain circumstances an affidavit or declaration may be submitted which attempts to attribute an activity, a reference or part of a reference to the applicant. If successful, the activity or the reference is no longer applicable. . . . An uncontradicted 'unequivocal statement' from the applicant regarding the subject matter disclosed in an article, patent, or published application will be accepted as establishing inventorship. In re DeBaun, 687 F.2d 459, 463, 214 USPQ 933, 936 (CCPA 1982)." (B) is insufficient in the absence of "objective evidence that an art recognized problem existed in the art for a long period of time without solution." MPEP § 716.04 under heading "THE CLAIMED INVENTION MUST SATISFY A LONG-FELT NEED WHICH WAS RECOGNIZED, PERSISTENT, AND NOT SOLVED BY OTHERS." Regarding (C), see MPEP § 716.06: "[M]ore than the mere fact of copying is necessary to make that action significant because copying may be attributable to other factors such as a lack of concern for patent property or contempt for the patentees ability to enforce the patent. Cable Electric Products, Inc. v. Genmark, Inc., 770 F.2d 1015, 226 USPQ 881 (Fed. Cir. 1985)." (D) is insufficient in the absence of evidence demonstrating that the sales are attributable to the technical merits of the invention rather than to other factors, such as "heavy promotion or advertising, shift in advertising, consumption by purchasers normally tied to applicant or assignee, or other business events extraneous to the merits of the claimed invention, etc. In re Mageli, 470 F.2d 1380, 176 USPQ 305 (CCPA 1973) (conclusory statements or opinions that increased sales were due to the merits of the invention are entitled to little weight); In re Noznick, 478 F.2d 1260, 178 USPQ 43 (CCPA 1973)." MPEP § 716.03(b) under heading "COMMERCIAL SUCCESS MUST BE DERIVED FROM THE CLAIMED INVENTION." (E) is incorrect because (A) is correct.

Which of the following would comply with the patent laws, rules and procedures as related in the MPEP and would be a fully responsive reply to a non-final Office action on the merits rejecting all the claims in the application as being unpatentable under 35 USC 102 and/or 103 over prior art references? A A timely filed and properly signed written reply which does not include an amendment to the claims, but includes a request for the examiner's rejections to be reconsidered supported by arguments replying to every ground of rejection and distinctly and specifically points out the supposed errors in every rejection. and pointing out the specific distinctions believed to render the claims patentable over any applied references. B A timely filed and properly signed written reply which includes an amendment canceling all the claims in the application and adding new claims, and a request for the examiner's rejections to be reconsidered in view of the newly presented claims. C A timely filed and properly signed written reply which does not include an amendment to the claims, but does generally alleges that the claims define a patentable invention. D A timely filed and properly signed written request for continued examination (RCE). E All of the above.

A Related MPEP Chapter(s): MPEP 700 - Examination of Applications Answer Reasoning:ANSWER: (A) is the most correct answer. 37 CFR § 1.111; MPEP § 714.02. Section 1.111 states in pertinent part: "(a)(1) If the Office action after the first examination (§ 1.104) is adverse in any respect, the applicant or patent owner, ... must reply and request reconsideration or further examination, with or without amendment. ... (b) In order to be entitled to reconsideration or further examination, the applicant or patent owner must reply to the Office action. The reply by the applicant or patent owner must be reduced to a writing which distinctly and specifically points out the supposed errors in the examiner's action and must reply to every ground of objection and rejection in the prior Office action. The reply must present arguments pointing out the specific distinctions believed to render the claims, including any newly presented claims, patentable over any applied references. ...The applicant's or patent owner's reply must appear throughout to be a bona fide attempt to advance the application or the reexamination proceeding to final action. A general allegation that the claims define a patentable invention without specifically pointing out how the language of the claims patentably distinguishes them from the references does not comply with the requirements of this section." MPEP § 714.02 states "In all cases where reply to a requirement is indicated as necessary to further consideration of the claims ... a complete reply must either comply with the formal requirements or specifically traverse each one not complied with." (B) and (C) are not the most correct answers. 37 CFR § 1.111; MPEP §§ 714.02 and 714.04. Neither reply specifically points out the supposed errors in the examiner's action and neither reply present arguments pointing out how the newly presented claims overcome the rejections. (D) is not the most correct answer. See 37 CFR § 1.114. A request for continued examination can only be made if prosecution of an application is closed. In this question the Office action is a non-final office action. (E) is not the most correct answer since (A) is correct and (B), (D) and (D) are incorrect.

A registered practitioner filed a patent application naming Sam as the sole inventor without an executed declaration under 37 CFR 1.63. The USPTO mailed a Notice to File Missing Parts dated January 3, 2016. The Notice to File Missing Parts set a two-month period for reply. Which of the following statements is in accordance with proper USPTO rules and the procedure set forth in the MPEP? I. Submit an appropriate reply to the Notice to File Missing Parts by filing, on August 3, 2016, a declaration under 37 CFR 1.63 executed by Sam, accompanied by a petition under 37 CFR 1.136(a) for an extension of five months, and the fee set forth in 37 CFR 1.17(a). II. In no situation can any extension requested by the practitioner carry the date on which a reply is due to the Notice to File Missing Parts beyond Monday, July 3, 2016. III. An appropriate reply by the practitioner to the Notice to File Missing Parts is to file, on August 3, 2016 a declaration under 37 CFR 1.63 executed by Sam, accompanied by a petition under 37 CFR 1.136(b). A I B II C III D I and III E None of the above.

A Related MPEP Chapter(s): MPEP 700 - Examination of Applications Answer Reasoning:ANSWER: (A) is the most correct answer. MPEP § 710.02(d), last paragraph, and 37 CFR § 1.136(a). (B) is incorrect because a Notice to File Missing Parts of an Application is not identified on the Notice as a statutory period subject to 35 U.S.C. § 133. (C) and (D) are incorrect because the provisions of 37 CFR § 1.136(a) are available. (E) is incorrect because (A) is correct.

A utility application filed in May 2016 has been prosecuted through a second action final rejection. In the final rejection some claims were allowed and other claims were finally rejected. Which of the following accords with the patent laws, rules and the procedures as related in the MPEP for a proper reply to a second action final rejection in the utility application? A An amendment canceling all rejected claims and complying with 37 CFR 1.116. B Only a Notice of Appeal. C The appropriate fee for a request for continued examination (RCE). D A continued prosecution application (CPA) under 37 CFR 1.53(d). E All of the above.

A Related MPEP Chapter(s): MPEP 700 - Examination of Applications Answer Reasoning:ANSWER: (A) is the most correct answer. The filing of an amendment complying with 37 CFR § 1.116 is a proper reply under 37 CFR § 1.113 to a final rejection. See MPEP § 714.13, under the heading "Entry Not A Matter of Right," which states, in pertinent part , "A reply under 37 CFR 1.113 is limited to: (A) an amendment complying with 37 CFR 1.116." (B) is not the most correct answer because the Notice of Appeal must be accompanied by the appeal fee required by 37 CFR § 1.17(b). (C) is not the most correct answer because the RCE must be accompanied by a submission (i.e., an amendment that meets the reply requirement of 37 CFR § 1.111). (D) is not the correct answer because CPA practice does not apply to utility or plant applications if the prior application has a filing date on or after May 29, 2000. See MPEP § 706.07(h), paragraphs I and IV. (E) is not the correct answer since (A) is a proper reply.

Co-inventors Smith and Jones filed an application for a patent on a cell phone, on May 15, 2016. They received a first Office action from a primary examiner rejecting the claims under 35 USC 102(a) over a publication by Bell and Watson, published on April 5, 2016, describing a cell phone having all the same features as is claimed in the patent application. In reply, the co-inventors each submitted a declaration under 37 CFR 1.130 stating that they had actually publicly reduced the invention to practice no later than March 13, 2016. However, the declarations failed to include two claimed features. Neither the particular antenna needed to enable the cell phone could receive transmissions from the local cellular transmitting tower, nor a detachable carrying strap was included in the declarations. As evidence of their prior reduction to practice, Smith and Jones submitted their co-authored journal article. The journal article contained a figure of the cell phone as described in the declarations. That is, the cell phone shown in the figure of the article lacked an antenna and a detachable strap. The article was received by the journal on March 13, 2016, and was published on April 30, 2016. The cell phones shown in the figure in the Bell and Watson publication, and in the Smith and Jones patent application have the particular antenna and a detachable strap. Which of the following actions, if taken by the examiner, would be the most proper in accordance with USPTO rules and the procedures set forth in the MPEP? A The examiner should maintain the rejection of the claims under 35 USC 102(a) and make the rejection final. B The examiner should withdraw the rejection and look for references which have a publication date prior to May 15, 2015. C The examiner should withdraw the rejection and notify Smith and Jones that their application is in condition for allowance. D The examiner should maintain the rejection, but indicate that the claims would be allowable if Smith and Jones provided an original copy of the figure published in their journal article as factual support for their declarations. E The examiner should maintain the rejection and inform Smith and Jones that the declarations are insufficient because they cannot "swear behind" a reference which is a statutory bar.

A Related MPEP Chapter(s): MPEP 700 - Examination of Applications Answer Reasoning:ANSWER: (A) is the most correct answer. The pre-dating disclosure must have what is shown in the intervening reference. In this instance, that is not done.

Which of the following practices or procedures may be properly employed to overcome a rejection properly based on pre-AIA 35 USC 102(a)? A Perfecting a claim to priority under 35 USC 119(a)-(d) based on a foreign application having a foreign priority filing date that antedates the reference. B Filing a declaration under 37 CFR 1.131 showing that the cited prior art antedates the invention. C Filing a declaration under 37 CFR 1.132 showing that the reference invention is by "others." D Perfecting priority under 35 USC 119(e) or 120 by, in part, amending the declaration of the application to contain a specific reference to a prior application having a filing date prior to the reference. E (A), (B) (C), and (D).

A Related MPEP Chapter(s): MPEP 700 - Examination of Applications Answer Reasoning:ANSWER: (A). See MPEP § 706.02(b) page 700-23 (8th ed.), under the heading "Overcoming a 35 U.S.C. § 102 Rejection Based on a Printed Publication or Patent." (B), and (C) are incorrect because they present showings that support the rejection. See MPEP § 706.02(b), supra. (D) are not correct because to perfect priority under 35 U.S.C. §§ 119(e) or 120 it is, inter alia, necessary to amend the specification of the application to contain a specific reference to a prior application having a filing date prior to the reference. See MPEP § 706.02(b), supra. Furthermore, the declaration is not to be amended. (E) is incorrect because (B), (C) and (D) are incorrect.

A registered practitioner filed in the USPTO a client's utility patent application on December 30, 2016. The application was filed with a request for nonpublication, certifying that the invention disclosed in the U.S. application has not and will not be the subject of an application in another country, or under a multilateral international agreement, that requires eighteen month publication. Subsequently, the client files an application in Japan on the invention and some recent improvements to the invention. The improvements are not disclosed or supported in the utility application. Japan is a country that requires eighteen month publication. Two months after filing the application in Japan, and before filing any other papers in the USPTO, the client remembers that a nonpublication request was filed and informs the practitioner about the application that was filed in Japan. Which of the following courses of action is in accordance with the USPTO rules and the procedures set forth in the MPEP? A The application is abandoned because the practitioner did not rescind the nonpublication request and provide notice of foreign filing within 45 days of having filed the application in Japan. The applicant must now file a petition and fee to revive under 37 CFR 1.137(b). B The application is abandoned because the applicant did not rescind the nonpublication request before filing the application in Japan. The applicant must now file a petition and fee to revive under 37 CFR 1.137(b). C The applicant should file an amendment to the specification of the U.S. application, adding the recent improvements to the disclosure in the specification. D The application is abandoned because the applicant did not rescind the nonpublication request by notifying the Office under 37 CFR 1.213(c) within the appropriate time. The applicant must now file a petition and fee to revive under 37 CFR 1.137(b). E The applicant could today notify the USPTO of the foreign filing. It is not necessary to file a petition and fee to revive for the application to continue to be examined in the USPTO.

A Related MPEP Chapter(s): MPEP 900 - Prior Art, Classification, and Search Answer Reasoning:ANSWER: (A) and (D) are accepted as the correct answers. Regarding answer (A), see 35 U.S.C. § 122(b)(2)(B)(iii); 37 CFR § 1.213; MPEP § 901.03 for information on nonpublication requests. See 37 CFR § 1.137(f); MPEP § 711.03(c), under the heading "3. Abandonment for Failure to Notify the Office of a Foreign Filing After Submission of a Non-Publication Request." (D) was also accepted because the statement characterizes the status of the application as being abandoned, though the application has not necessarily attained abandoned status. The course of action postulated in (D) is a proper reply if the application was abandoned. Accordingly, (D) was accepted as a correct answer under these circumstances. (B) is incorrect. The notice of foreign filing can be filed as late as 45 days after the foreign filing before the U.S. application becomes abandoned. (C) is incorrect. See MPEP § 608.04(a). The improvements would constitute new matter and new matter cannot be added to the disclosure of an application after the filing date of the application. (E) is not correct. The applicant was required to provide notice of foreign filing within 45 days of filing in Japan, and two months have passed. As a result, a petition to revive under 37 CFR § 1.137(b) is required for examination to continue. Also see 37 CFR § 1.137(f).

A registered practitioner filed in the USPTO a client's utility patent application on December 30, 2016. The application was filed with a request for nonpublication, certifying that the invention disclosed in the U.S. application has not and will not be the subject of an application in another country, or under a multilateral international agreement, that requires eighteen month publication. Subsequently, the client files an application in Japan on the invention and some recent improvements to the invention. The improvements are not disclosed or supported in the utility application. Japan is a country that requires eighteen month publication. Two months after filing the application in Japan, and before filing any other papers in the USPTO, the client remembers that a nonpublication request was filed and informs the practitioner about the application that was filed in Japan. Which of the following courses of action is in accordance with the patent laws, rules and procedures as related in the MPEP? A The application is abandoned because the practitioner did not rescind the nonpublication request and provide notice of foreign filing within 45 days of having filed the application in Japan. The applicant must now file a petition and fee to revive under 37 CFR 1.137(b). B The application is abandoned because the applicant did not rescind the nonpublication request before filing the application in Japan. The applicant must now file a petition and fee to revive under 37 CFR 1.137(b). C The applicant should file an amendment to the specification of the U.S. application, adding the recent improvements to the disclosure in the specification. D The application is abandoned because the applicant did not rescind the nonpublication request by notifying the Office under 37 CFR 1.213(c) within the appropriate time. The applicant must now file a petition and fee to revive under 37 CFR 1.137(b). E The applicant could today notify the USPTO of the foreign filing. It is not necessary to file a petition and fee to revive for the application to continue to be examined in the USPTO.

A Related MPEP Chapter(s): MPEP 900 - Prior Art, Classification, and Search Answer Reasoning:ANSWER: (A) is the most correct answer. See 35 U.S.C. 122(b)(2)(B)(iii); 37 CFR § 1.213; MPEP § 901.03 for information on nonpublication requests. See 37 CFR § 1.137(f); MPEP § 711.03(c), under the heading "3. Abandonment for Failure to Notify the Office of a Foreign Filing After Submission of a Non-Publication Request." (B) is incorrect. The notice of foreign filing can be filed as late as 45 days after the foreign filing before the U.S. application becomes abandoned. (C) is incorrect. See MPEP § 608.04(a). The improvements would constitute new matter and new matter cannot be added to the disclosure of an application after the filing date of the application. (D) is not correct. The applicant is required to provide notice of foreign filing, not merely rescind the nonpublication request within the appropriate time. (E) is not correct. The applicant was required to provide notice of foreign filing within 45 days of filing in Japan, and two months have passed. As a result, a petition to revive under 37 CFR § 1.137(b) is required for examination to continue. Also see 37 CFR § 1.137(f).

Inventor Joe is anxious to get a patent with the broadest claim coverage possible for the invention. Joe retained a registered practitioner, Jane, to obtain the advantage of legal counsel in obtaining broad protection. Jane filed a patent application for the invention. The inventor heard that, although patent prosecution is conducted in writing, it is possible to get interviews with examiners. Joe believes an interview might hasten the grant of a patent by providing the examiner a better understanding of the true novelty of the invention. Which of the following are consistent with the patent law, rules and procedures as related by the MPEP regarding usage of interviews? A Prior to the first Office action being mailed the inventor calls the examiner to whom the application is docketed to offer help in understanding the specification. B After receiving the first Office action Jane calls the examiner for an interview for the purpose of clarifying the structure and operation of the invention as claimed and disclosed, because the examiner's analysis regarding patentability in the rejection is novel and suggests that the examiner is interpreting the claimed invention in a manner very different from the inventor's intent. C Jane has Larry, a registered practitioner in the Washington D.C. area, who is more familiar with interview practice to call the examiner. Jane gives Larry a copy of the first Office action, which suggests that the primary examiner's analysis is incorrect, and offers to explain why. Jane instructs Larry that because Larry is unfamiliar with the inventor, Larry should not agree to possible ways in which claims could be modified, or at least indicate to the examiner that Jane would have to approve of any such agreement. D Jane calls the primary examiner after receiving the final rejection, demanding that the examiner withdraw the finality of the final action. When the examiner states that the final rejection is proper, Jane demands an interview as a matter of right to explain the arguments. E (B) and (D).

ANSWER: (B) is the most correct answer. MPEP § 713.01, under the heading "Scheduling And Conducting An Interview," states "[a]n interview should be had only when the nature of the case is such that the interview could serve to develop and clarify specific issues and lead to a mutual understanding between the examiner and the applicant, and thereby advance the prosecution of the application." (A) is incorrect. 37 CFR § 1.133(a)(2); MPEP § 713.02. Section 713.02 states that although "[a] request for an interview prior to the first Office action is ordinarily granted in continuing or substitute applications[,] [a] request for an interview in all other applications before the first action is untimely and will not be acknowledged if written, or granted if oral. 37 CFR 1.133(a)." (C) is incorrect. MPEP § 713.03. Larry is only sounding out the examiner and has no authority to commit Joe to any agreement reached with the examiner. (D) is incorrect. MPEP § 713.09. Jane has no right to an interview following the final rejection. Although such an interview may be granted if the examiner is convinced that disposal or clarification for appeal may be accomplished with only nominal further consideration, interviews merely to restate arguments of record or to discuss new limitations which would require more than nominal reconsideration or new search should be denied. (E) is incorrect because (D) is incorrect.

Assuming that a rejection has been properly made final, which of the following statements is not in accordance with the patent laws, rules and procedures as related in the MPEP? A An objection and requirement to delete new matter from the specification is subject to supervisory review by petition under 37 CFR 1.181. B A rejection of claims for lack of support by the specification (new matter) is reviewable by appeal to the Board of Patent Appeals and Interferences. C If both the claims and the specification contain the same new matter, and there has been both a rejection and objection by the primary examiner, the new matter issue should be decided by petition, and is not appealable. D If both the claims and the specification contain the same new matter, and there has been both a rejection and objection by the examiner, the new matter issue is appealable, and should not be decided by petition. E None of the above.

ANSWER: (C) is the most correct answer. MPEP § 2163.06, under the heading "Review Of New Matter Objections And Rejections," states "A rejection of claims is reviewable by the Board of Patent Appeals and Interferences, whereas an objection and requirement to delete new matter is subject to supervisory review by petition under 37 CFR 1.181. If both the claims and specification contain new matter either directly or indirectly, and there has been both a rejection and objection by the examiner, the issue becomes appealable and should not be decided by petition." Answer (C) is not accord with the USPTO rules and the procedures set forth in the MPEP. (A), (B) and (D) are incorrect. They are in accord with proper USPTO procedure. See MPEP § 2163.06, under the heading "Review Of New Matter Objections And Rejections." (E) is not correct because (C) is correct. MPEP § 2163.06.

On January 2, 2016, a registered practitioner filed a patent application with the USPTO for inventor Beck. The application includes a specification and a single claim to the invention which reads as follows: 1. Mixture Y made by the process Q1. In the specification, Mr. Beck discloses that mixture Y has a melting point of 150 degrees F. On June 2, 2016, the practitioner received an Office action from the primary examiner rejecting the claim. The claim is rejected under 35 USC 102/103 as being clearly anticipated by or obvious over Patent A. The examiner states "Patent A teaches mixture Y but made by a different process Q2." Beck believes he is entitled to a patent to mixture Y. In accordance with USPTO rules and procedures set forth in the MPEP, which of the following would be the best reply to the rejection of his claim? A An argument that the claimed product has an unexpectedly low melting point of 150 degrees F, supported by an affidavit showing that the mixture Y made by process Q2 exhibits a melting point of 300 degrees F. B An argument that the processes used by applicant and patent A are different, supported by a third-party declaration stating only that the processes are different. C An argument that the claimed product has an unexpectedly low melting point of 150 degrees F, supported by a third-party declaration stating only that the products are different. D An argument that the processes used by applicant and patent A are different, supported by an affidavit showing that the mixture Y made by process Q2 exhibits a melting point of 300 degrees F. E An argument that the claimed product has an unexpectedly low melting point of 150 degrees F because the claimed mixture Y has a melting point of 150 degrees F and the mixture Y of patent A has a melting point of 300 degrees F.

An argument that the claimed product has an unexpectedly low melting point of 150 degrees F, supported by an affidavit showing that the mixture Y made by process Q2 exhibits a melting point of 300 degrees F.

A registered practitioner receives an Office action for Application X, a patent application filed after November 29, 2016. The action contains a rejection of all the claims as being obvious under 35 USC 103 over Patent A in view of Patent B. Patent A is only available as prior art under 35 USC 102(a)(2). Patent B is available under 35 USC 102(a)(1). The practitioner seeks to disqualify Patent A as prior art under 35 USC 102(c). Which of the following would be sufficient evidence to disqualify Patent A as prior art in accordance with the patent laws, rules and procedures as related in the MPEP? A A declaration signed by an employee of Corporation K, who is not empowered to act on behalf of Corporation K, stating that at the time the invention claimed in Application X was made, the claimed invention and Patent A were commonly owned by Corporation K. B A declaration signed by the inventor of Patent A stating that at the time the invention claimed in Application X was made and later filed, the invention claimed in Application X and the invention claimed in Patent A were both subject to an obligation of assignment to the same person. C A statement by the inventor Jones, the sole inventor of Application X, saying that at the present time, Application X and Patent A are commonly owned. D A statement by the practitioner stating that Application X and Patent A were, at the time the invention claimed in Application X was made, commonly owned by the same person. E A statement by inventor Jones, the sole inventor of Application X, saying that at the time the invention claimed in Application X was made, Jones owned a majority interest in Patent A.

B D Related MPEP Chapter(s): MPEP 700 - Examination of Applications Answer Reasoning:ANSWER: (B) or (D) is accepted as the correct answer. See 102(a)(1), 102(a)(2) and 102(b)(2)(c).

A patent application has claims 1-10 pending. Claims 1 and 7 are independent claims. Claims 2-6 depend directly from claim 1 while claims 8-10 depend directly from claim 7. Claims 1-10 have been twice rejected by the primary examiner under 35 USC 103 as being unpatentable over Smith patent in view of Jones patent. The applicant has appealed the rejection to the Board of Patent Appeals and Interferences. In the brief under the "Grounds of Rejection to be Reviewed" section, appellant states that each of the claims is separately patentable. In the arguments section of the brief, appellant separately argues only claims 1, 4 and 6. In the examiner's answer, the examiner disagrees with appellant's claim grouping because all the claims present a similar issue of patentability. The examiner states that the claims all stand or fall together as a single group. In accordance with the patent laws, rules and procedures as related in the MPEP, which claim(s) must the Board consider separately on the merits? A The Board must consider each of claims 1-10 separately on the merits. B The Board must only consider claims 1, 4 and 6 separately on the merits. C The Board must only consider claim 1 separately on the merits. D The Board must consider claim 1 and claim 7 separately on the merits as representative of all the claims on appeal. E The Board must determine which claim is representative of all the claims on appeal and consider only that claim separately on the merits.

B Related MPEP Chapter(s): MPEP 1200 - Appeal Answer Reasoning:ANSWER: (B) is the most correct answer. 37 CFR § 41.37; MPEP § 1205.02, under the heading "Appeal Brief Content". The Board considers the claims as grouped and argued in Applicant's Brief. (A), (C) and (D) are incorrect answers.

In accordance with the USPTO rules and the procedures set forth in the MPEP, which of the following is true? A When the subject matter of an appeal is particularly difficult to understand, a patentability report is prepared by an examiner in order to present the technical background of the case to the Board of Patent Appeals and Interferences. B In those appeals in which an oral hearing has been confirmed and either the Board of Patent Appeals and Interferences or the primary examiner has indicated a desire for the examiner to participate in the oral argument, oral argument may be presented by the examiner whether or not the appellant appears. C If a patent applicant files a notice of appeal which is unsigned, it will be returned for signature, but the applicant will still receive the filing date of the unsigned notice of appeal. D Statements made in information disclosure statements are not binding on an applicant once the patent has issued since the sole purpose of the statement is to satisfy the duty of disclosure before the Office. E None of the above.

B Related MPEP Chapter(s): MPEP 1200 - Appeal Answer Reasoning:ANSWER: (B) is the most correct answer. See MPEP § 1209, under the heading "Participation by Examiner." As to (A), see MPEP § 705. As to (C) signature requirement does not apply. 37 C.F.R. § 1.196(b); MPEP § 1205. The notice will not be returned. As to (D), see Gentry Gallery v. Berkline Corp., 134 F.3d 1473, 45 U.S.P.Q.2d 1498 (Fed. Cir. 1998).

In accordance with the MPEP and USPTO rules and procedure, an applicant for a patent dissatisfied with the primary examiner's decision may appeal to the Board of Patent Appeals and Interferences ("the Board") in certain situations. In which of the following situations may the applicant properly appeal to the Board? A Applicant's claims have been twice objected to, but have not been rejected. B Applicant's claims have been rejected once in a non-final Office action during examination of a parent application, and once in a non-final Office action during examination of a continuing application. C Applicant's claims in an original application have been rejected only once. D Applicant's claims have been objected to only once, and have been rejected only once in a non-final Office action. E All of the above.

B Related MPEP Chapter(s): MPEP 1200 - Appeal Answer Reasoning:ANSWER: Choice (B) is the correct answer. MPEP § 1205, and 37 C.F.R. § Part 41. MPEP § 1205 states that "A notice of appeal may be filed after any of the claims has been twice rejected, regardless of whether the claim(s) has/have been finally rejected. The limitation of 'twice or finally...rejected' does not have to be related to a particular application. For example, if any claim was rejected in a parent application, and the claim is again rejected in a continuing application, then applicant will be entitled to file an appeal in the continuing application, even if the claim was rejected only once in the continuing application." Choices (A), (C), and (D) are each incorrect because Applicant's claims were not twice or finally rejected. 37 C.F.R. § Part 41. MPEP § 706.01 in pertinent part provides, "The practical difference between a rejection and an objection is that a rejection, involving the merits of the claim, is subject to review by the Board of Patent Appeals and Interferences, while an objection, if persisted, may be reviewed only by way of petition to the Commissioner." Choice (E) is incorrect because Choices (A), (C), and (D) are incorrect.

Harriet filed a nonprovisional patent application in the USPTO containing a written assertion of small entity status. Based upon the USPTO rules and the procedures set forth in the MPEP, which of the following statements is correct? A If Harriet files a related, continuing application wherein small entity status is appropriate and desired, it will not be necessary to specifically establish assertion of such status in the continuing application. B If Harriet files a related, reissue application wherein small entity status is appropriate and desired, it will be necessary to specifically establish assertion of such status in the reissue application. C If Harriet files a related, divisional application under 37 CFR 1.53, wherein small entity status is appropriate and desired, it will not be necessary to specifically establish assertion of such status in the divisional application. D If Harriet refiles her application as a continued prosecution application under 37 CFR 1.53(d), wherein small entity status is appropriate and desired, it will not be necessary to specifically establish assertion of such status in the continued prosecution application. E If Harriet subsequently assigns her rights to another party for whom small entity status is appropriate and desired, it will be necessary to specifically re-establish assertion of such status.

B Related MPEP Chapter(s): MPEP 200 - Types, Cross-Noting, and Status of Application Answer Reasoning:ANSWER: (B) is correct and (A), (C) and (D) are wrong. 37 CFR § 1.27(c)(4) ("The refiling of an application under § 1.53 as a continuation, divisional, or continuation-in-part application, including a continued prosecution application under § 1.53(d), or the filing of a reissue application, requires a new assertion as to continued entitlement to small entity status for the continuing or reissue application."). (E) is also wrong. 37 CFR § 1.27(e)(1) ("Where an assignment of rights...to other parties who are small entities occurs subsequent to an assertion of small entity status, a second assertion is not required.")

Applicant filed a provisional patent application in the USPTO under 35 USC 111(b) on Tuesday, November 30, 1999. On Tuesday, November 28, 2000, applicant filed a nonprovisional application in the USPTO under 35 USC 111(a) that properly claimed priority under 35 USC 119(e) to the filing date of the provisional application. On Wednesday, November 29, 2000, applicant filed an international application for patent in the USPTO under the Patent Cooperation Treaty that designated the United States and properly claimed priority to both the provisional and the nonprovisional applications. On Friday, July 28, 2001, applicant filed a national stage application in the USPTO under 35 USC 371, providing all of the requirements under 35 USC 371 and properly claiming benefit to the filing date of the provisional application under 35 USC 119(e) and the nonprovisional application under 35 USC 120. The national stage application was published on Tuesday, January 30, 2002 and issued as a patent on Tuesday, February 4, 2003. Assuming no patent term extension or adjustment, the patent term ends on the date that is 20 years from which of the following dates in accordance with the patent laws, rules and procedures as related in the MPEP? A Tuesday, November 30, 1999 B Tuesday, November 28, 2000 C Wednesday, November 29, 2000 D Friday, July 28, 2001 E Tuesday, February 4, 2003

B Related MPEP Chapter(s): MPEP 200 - Types, Cross-Noting, and Status of Application Answer Reasoning:ANSWER: The filing date of the nonprovisional application, (B), is the correct answer. See MPEP § 201.04(b), which states "[t]he [Uruguay Agreement Round Act] provides a mechanism to enable domestic applicants to quickly and inexpensively file provisional applications. Under the provisions of 35 U.S.C. § 119(e) applicants are entitled to claim the benefit of priority in a given application in the United States. The domestic priority period will not count in the measurement of the 20-year patent term. See 35 U.S.C. 154(a)(3). Thus, domestic applicants are placed on the same footing with foreign applicants with respect to the patent term." A provisional application is filed under 35 U.S.C. § 119(e) and according to 35 U.S.C. § 154(a)(3), such a filing date is not taken into account in determining patent term. Therefore, (A) is incorrect. The fact pattern states that benefit was properly claimed in the international application to both the provisional application and the national application and that the national stage application filed under 35 U.S.C. § 371 claimed benefit to the filing date of the nonprovisional application under 35 U.S.C. § 120. According to 35 U.S.C. § 154(a)(2), where an application contains a reference to an earlier filed application or applications under 35 U.S.C. § 120, 121, or 365(c), the patent term ends 20 years from the date on which the earliest such application was filed; in this fact pattern that date would be (B), the filing date of the nonprovisional application. The filing date of the international application, (C), is not correct in view of 35 U.S.C. § 154(a)(2) since the international application claimed the benefit under 35 .U.S.C. § 120 to the filing date of the nonprovisional application. The date of commencement of the national stage in the United States, (D), is not correct, since the date of commencement of the national stage in the U.S. is not relevant in the determination of the patent term of a patent issuing form the national stage of the international application. Furthermore, as noted in (B) above, since the international application claims the benefit under 35 U.S.C. §120 to the nonprovisional application, the patent term of the patent issuing from the national stage is measured from the filing date of the nonprovisional application. Finally, the issue date, (E), is not correct, because in 1994, 35 U.S.C. § 154 was amended to provide that for applications filed on or after June 8, 1995, the term of a patent begins on the date the patent issues and ends on the date that is twenty years from the date on which the application for the patent was filed in the United States or, of the application contains a specific reference to an earlier filed application or applications under 35 U.S.C. § 120, 121, or 365(c), twenty years from the filing date of the earliest of such application(s).

The specification in a patent application has been objected to for lack of enablement. To overcome this objection in accordance with the USPTO rules and the procedures set forth in the MPEP, a registered practitioner may do any of the following except: A traverse the objection and specifically argue how the specification is enabling. B traverse the objection and submit an additional drawing to make the specification enabling. C file a continuation-in-part application that has an enabling specification. D traverse the objection and file an amendment without adding new matter in an attempt to show enablement. E traverse the objection and refer to prior art cited in the specification that would demonstrate that the specification is enabling to one of ordinary skill.

B Related MPEP Chapter(s): MPEP 200 - Types, Cross-Noting, and Status of Application MPEP 600 - Parts, Form, and Content of Application Answer Reasoning:ANSWER: (B) is the most correct answer. 35 U.S.C. § 113 reads "Drawings submitted after the filing date of the application may not be used (i) to overcome any insufficiency of the specification due to lack of an enabling disclosure." Since choice (A) may be done, 37 CFR § 1.111, it is an incorrect answer to the above question. Since choice (C) may be done, 35 U.S.C. § 120, it is an incorrect answer to the above question. Since choice (D) may be done, 37 CFR § 1.121, it is an incorrect answer to the above question. Since choice (E) may be done, 37 CFR § 1.111, it also is an incorrect answer to the above question.

In accordance with the MPEP and USPTO rules and procedure, certain individuals owe a duty to the USPTO to disclose all information known to be material to patentability of the claim(s) pending in an application. Which of the following parties does not have the duty? A An inventor named in the application who relies on a patent attorney to prepare and prosecute the application. B A corporation to which an assignment of the entire interest in the application is on record at the USPTO. C An agent who prepares the application. D An attorney who prosecutes the application. E A person, who is not an inventor named in the application, who is substantively involved in the preparation and prosecution of the application, and who is associated with an inventor named in the application.

B Related MPEP Chapter(s): MPEP 2000 - Duty of Disclosure Answer Reasoning:ANSWER: Choice (B) is the correct answer. MPEP § 2001.01, and 37 C.F.R. § 1.56(c). MPEP § 2001.01 states that "the duty applies only to individuals, not to organizations...the duty of disclosure would not apply to a corporation or institution as such." (A) is incorrect because the duty of disclosure applies to each inventor named in the application. 37 C.F.R. § 1.56(c)(1). (C) and (D) are each incorrect because the duty of disclosure applies to an each attorney or agent who prepares or prosecutes the application. 37 C.F.R. § 1.56(c)(2). (E) is incorrect because, in the case of a person who is not a named inventor or an attorney or agent but who prepares or prosecutes the application, the duty of disclosure applies to each such person who is substantively involved in the preparation or prosecution of the application and who is associated with the inventor, with the assignee or with anyone to whom there is an obligation to assign the application. 37 C.F.R. § 1.56(c)(3).

Paul, a registered patent practitioner and counsel for Superior Aircraft, Inc. ("Superior"), filed a patent application naming chief engineer Davis as sole inventor, and claiming a titanium and aluminum alloy designed for use in advanced gas turbine engines in aircraft. The application described the alloy as having unexpectedly excellent and improved room temperature ductility. The application was filed with an assignment document transferring all right, title and interest in the application to Superior. During prosecution of the application, the examiner had an interview with Paul and Davis of Superior. The examiner noted the existence of a prior art publication that disclosed test data demonstrating that the claimed alloys exhibited poor room temperature ductility, and stated that he had personal knowledge that the alloy was old and well known. Davis agreed with the examiner, and stated that such information was "old hat," but that they overcame the ductility problem by simply resorting to a 3-step process of microstructure refinement. Paul concurred and pointed to the fact that not only had they disclosed the process in the application, but that microstructure refinement of alloys to improve ductility was so well-known that the technique was even taught in metallurgy courses in college. Which of the following statements is false? A The examiner may reject the alloy claims on the basis of the prior art publication. B The examiner may not reject the alloy claims on the basis of the prior art publication, because the alloys of the application are characterized by unexpected, improved ductility properties. C The examiner may rely upon the chief engineer's statement as an admission against patentability. D The examiner may rely upon the patent counsel's statement as an admission against patentability. E The examiner, having facts within his or her personal knowledge, may rely on the facts in rejecting the alloy claims.

B Related MPEP Chapter(s): MPEP 2100 - Patentability Answer Reasoning:ANSWER: (B) is best choice because it is a false statement. MPEP § 2112.01 cites Titanium Metals Corp. v. Banner, 778 F.2d 660. 227 USPQ 773 (Fed. Cir. 1985), as stating, "it was immaterial what properties the alloys had...because the composition is the same and thus must necessarily exhibit the properties." (A) is not correct because it is a true statement. (C), (D) and (E) are incorrect because the stated reliance is permitted. 37 C.F.R. § 1.104(c)(3); MPEP § 706.

A U.S. patent application discloses a first embodiment of an invention, a composition made of known materials in equal amounts by weight of A, B, and C. The application discloses a second embodiment of the invention comprising equal amounts by weight of A, B, and C, and an effective amount of D, a known material, to reduce excess moisture from the composition. The application also discloses a third embodiment of the invention comprising equal amounts by weight of A, B, and C, and an effective amount of D to reduce the acidity of the composition. The application fully discloses guidelines for determining an effective amount of D to reduce excess moisture from the composition, and determining an effective amount of D to reduce the acidity of the composition. The application discloses that the amount of D needed to reduce excess moisture from the composition differs from the amount of D needed to reduce the acidity of the composition. Which of the following claims, if included in the application, provides a proper basis for a rejection under 35 USC 112, second paragraph in accordance with the USPTO rules and the procedures set forth in the MPEP? A Claim 1. A composition comprising: equal amounts by weight of A, B, and C, and an effective amount of D to reduce the acidity of the composition. B Claim 1. A composition comprising: equal amounts by weight of A, B, and C, and an effective amount of D. C Claim 1. A composition comprising: equal amounts by weight of A, B, and C, and an effective amount of D to reduce excess moisture from the composition. D Claim 1. A composition comprising: equal amounts by weight of A, B, and C. E None of the above.

B Related MPEP Chapter(s): MPEP 2100 - Patentability Answer Reasoning:ANSWER: (B) is the most correct answer. 35 U.S.C. § 112, second paragraph, and MPEP § 2173.05(c), under the heading "III. Effective Amount." The claim presented in (B) is improper as "an effective amount" has been held to be indefinite when the claim fails to state the function that is to be achieved and more than one effect can be implied from the specification. In re Fredericksen 213 F.2d 547, 102 USPQ 35 (CCPA 1954). It is unclear whether "an effective amount" in (B) is an effective amount to reduce acidity or an effective amount to reduce moisture. The claims presented in (A) and (C) find support in the disclosure, which provides guidelines for determining "an effective amount" for each of the claims in (A) and (C). See MPEP § 2173.05(c) under the heading "III. Effective Amount." The claim presented in (D) is not indefinite, given that A, B, and C are known materials as set forth in the question and the composition can be determined by the claim language. (E) is incorrect because (B) is correct.

A patent application includes the following Claim 1: Claim 1. A method of making an electrical device comprising the steps of: (i) heating a base made of carbon to a first temperature in the range of 1875 degrees C to 1925 degrees C; (ii) passing a first gas over said heated base, said first gas comprising a mixture of hydrogen, SiCl4, phosphorus, and methane, whereby said first gas decomposes over said heated base and thereby forms a first deposited layer of silicon, phosphorus and carbon on said heated base; (iii) heating said base having said deposited layer to a second temperature of approximately 1620 degrees C; and (iv) passing a second gas over said base heated to said second temperature, said second gas consisting of a mixture of hydrogen, SiCl4, AlCl3, and methane, whereby said second gas decomposes over said heated base to form a second deposit layer adjacent said first layer, said second layer comprising silicon, aluminum and carbon. Assuming proper support in the specification, which of the following claims, if presented in the same application, is a proper claim in accordance with the USPTO rules and the procedures set forth in the MPEP? A Claim 2. The method of claim 1, wherein said first temperature is in the range of 1800 degrees C to 2000 degrees C. B Claim 3. The method of claim 1, wherein said first gas further comprises an inert gas. C Claim 4. The method of claim 1, wherein said second gas further comprises Argon. D Claim 5. The method of claim 1, wherein said first gas is an inert gas such as Argon. E Claim 6. The method of claim 1, wherein said second gas consists of a mixture of hydrogen, SiCl4 and AlCl3 only.

B Related MPEP Chapter(s): MPEP 2100 - Patentability Answer Reasoning:ANSWER: (B) is the most correct answer. 37 CFR § 1.75(c). Answers (A) and (E) are incorrect because they improperly seek to broaden the parent claim. 37 CFR § 1.75(c). Answer (A) broadens the range by going below the stated limit. Answer (E) broadens by trying to remove a recited component of the second gas, and covering subject matter that is not covered by the parent claim. Answer (C) is incorrect because claim 1 uses the close ended claim term "consists" in connection with the second gas, which precludes the addition of further components to the second gas in claim 4. Answer (D) is incorrect because the use of the exemplary language "such as" is improper is improper under 35 U.S.C. § 112, second paragraph, and because it is inconsistent with claim 1. See MPEP § 2173.05(d).

In accordance with the USPTO rules and the procedures in the MPEP, in which of the following instances is the reference properly available as prior art under 35 USC 102? A A U.S. patent application is filed within the one year anniversary date of the filing date of the foreign application. The reference is the foreign application. B The applicant files a foreign application, later timely files a U.S. application claiming priority based on the foreign application, and then files a continuation-in-part (CIP) application, and the claims in the CIP are not entitled to the filing date of the U.S. parent application. The foreign application issues as a patent before the filing date of the CIP application and is used to reject the claims directed to the added subject matter under 35 USC 102/103. The reference is the foreign application. C The applicant files a foreign application, and later timely files a U.S. application claiming priority based on the foreign application. The examined foreign application has been allowed by the examiner and has not been published before the U.S. application was filed. D The cited date is the filing date and the reference is a defensive publication. E All of the above.

B Related MPEP Chapter(s): MPEP 2100 - Patentability Answer Reasoning:ANSWER: (B) is the most correct answer. See 35 U.S.C. § 102; MPEP § 2135.01, under the heading "A Continuation - In - Part Breaks The Chain Of Priority As To Foreign As Well As U.S. Parents." If an applicant files a foreign application, later files a U.S. application claiming priority based on the foreign application, and then files a continuation - in - part (CIP) application whose claims are not entitled to the filing date of the U.S. parent, the effective filing date of the CIP application is the filing date of the CIP. The applicant cannot obtain the benefit of either the U.S. parent or foreign application filing dates. In re van Langenhoven, 173 USPQ 426, 429 (CCPA 1972); Ex parte Appeal No. 242 - 47, 196 USPQ 828 (Bd. App. 1976). (A) is incorrect. 35 U.S.C. § 102. (C) is not correct. 35 U.S.C. § 102. (D) is not correct. 35 U.S.C. § 102(d); MPEP § 2136, under the heading "Defensive Publications Are Not Prior Art As Of Their Filing Date," citing Ex parte Osmond, 191 USPQ 334 (Bd. App. 1973). (E) is not correct inasmuch as (A), (C) and (D) are not correct.

A claim in a pending patent application is rejected under 35 USC 103 as being obvious over Barry in view of Foreman. The Barry reference is a U.S. Patent that was issued on an application filed before the date of the application in question. In accordance with USPTO rules and procedures set forth in the MPEP, which of the following arguments, if true, would overcome the rejection? A The Foreman reference is nonanalogous art, but the reference may be reasonably pertinent to Barry's endeavor to solving the particular problem with which Barry was concerned. B The rejection does not address a claimed limitation, and neither of the references teaches the claimed limitation. C The Barry patent issued after the filing date of the pending patent application. D The original specification states that the results achieved by the claimed invention are unexpected. (The statement is unsubstantiated by evidence). E The Foreman patent issued 105 years before the filing date of the pending patent application.

B Related MPEP Chapter(s): MPEP 2100 - Patentability Answer Reasoning:ANSWER: (B) is the most correct answer. See MPEP § 2143.03. To establish prima facie obviousness of a claimed invention, all of the claimed limitations must be taught or suggested by the prior art. (A) is incorrect. See MPEP § 2141.01(a). Although an argument that the reference is nonanalogous art may be appropriate, it is overcome by the acknowledgment that the reference may be reasonably pertinent to the applicant's endeavor to solving the particular problem with which the inventor was concerned. As discussed in MPEP § 2141.01(a), under the heading "To Rely On A Reference Under 35 U.S.C. 103, It Must Be Analogous Prior Art," which quotes In re Oetiker, 977 F.2d 1443, 24 USPQ2d 1443, 1445 (Fed. Cir. 1992), to rely on a reference as a basis for rejection of the applicant's invention, "the reference must either be in the field of the applicant's endeavor or, if not, be reasonably pertinent to the particular problem with which the inventor was concerned." Here, the argument acknowledges that reference may be reasonably pertinent to the applicant's problem solving endeavor. (C) is incorrect. U.S. patents may b used as of their filing dates to show that the claimed subject matter is anticipated or obvious. See MPEP § 2136.02 under the heading "The Supreme Court Has Authorized 35 U.S.C. 103 Rejections Based On 35 USC 102(e)." (D) is incorrect. MPEP § 716.01(c), under the heading, "To Be Of Probative Value, Any Objective Evidence Should Be Supported By Actual Proof," states "Objective evidence which must be factually supported by an appropriate affidavit or declaration to be of probative value includes evidence of unexpected results...." It also quotes from In re De Blauwe, 736 F.2d 699, 222 USPQ 191, 196 (Fed. Cir. 1984), "It is well settled that unexpected results must be established by factual evidence. Mere argument or conclusory statements in the specification does not suffice." De Blauwe, 736 F.2d at 705, 222 USPQ at 196. Here, the conclusory statement in the specification does not suffice. (E) is incorrect. MPEP § 2145, subsection VIII (under the heading "Arguing About The Age Of References"), quoting from In re Wright, 569 F.2d 1124, 193 USPQ 332, 335 (CCPA 1977), states "The mere age of the references is not persuasive of the unobviousness of the combination of their teachings, absent evidence that, notwithstanding knowledge of the references, the art tried and failed to solve the problem." Here, the mere fact that the Foreman patent issued 105 years before the filing date of the pending patent application is unpersuasive of the non-obviousness of the applicant's claim

In accordance with the USPTO rules and the procedures set forth in the MPEP, which of the following presents a Markush group in accordance with proper PTO practice and procedure? A R is selected from the group consisting of A, B, C, or D. B R is selected from the group consisting of A, B, C, and D. C R is selected from the group comprising A, B, C, and D. D R is selected from the group comprising A, B, C or D. E R is A, B, C, and D.

B Related MPEP Chapter(s): MPEP 2100 - Patentability Answer Reasoning:ANSWER: (B) is the most correct answer. See MPEP § 2173.05(h). A Markush group is an acceptable form of alternative expression provided the introductory phrase "consisting of," and the conjunctive "and" are used. (A) and (D) are incorrect because the conjunctive "or" is used. (C) and (D) are incorrect because the introductory phrase "comprising" is used. (E) is incorrect because R must simultaneous be A, B, C, and D, as opposed to being a single member of the group, i.e., no language provides for the selection of one of the members of the group of A, B, C, and D. MPEP § 2173.05(h).

With respect to the examiner's burden in making an enablement rejection under 35 USC 112(a), which of the following statements is or are in accordance with the patent laws, rules and procedures as related in the MPEP? (1) The examiner may properly make an enablement rejection before construing the claims. (2) The examiner has the initial burden to establish a reasonable basis to question the enablement provided for the claimed invention. (3) The examiner need not give reasons for the uncertainty of the enablement when there is no evidence of operability beyond the disclosed embodiments. A Statement (1) only B Statement (2) only C Statement (3) only D Statements (1) and (2) E Statements (1) and (3)

B Related MPEP Chapter(s): MPEP 2100 - Patentability Answer Reasoning:ANSWER: (B) is the most correct, as only statement (2) is true. The examiner has the initial burden to establish a reasonable basis to question the enablement provided. MPEP § 2164.04 states "[i]n order to make a rejection, the examiner has the initial burden to establish a reasonable basis to question the enablement provided for the claimed invention. In re Wright, 999 F.2d 1557, 1562, 27 USPQ2d 1510, 1513 (Fed. Cir. 1993) (examiner must provide a reasonable explanation as to why the scope of protection provided by a claim is not adequately enabled by the disclosure)." Answer (A) is incorrect, because statement (1) is not true. The examiner may not analyze enablement before construing the claims. MPEP § 2164.04. Answer (C) is incorrect, because statement (3) is not true. The examiner must give reasons for the uncertainty of the enablement, even when there is no evidence of operability without undue experimentation other than the disclosed embodiments. In re Brana, 51 F.3d 1560, 1566, 34 USPQ2d 1436, 1441 (Fed. Cir. 1995). MPEP § 2164.04 states "[a]ccording to In re Bowen, 492 F.2d 859, 862-63, 181 USPQ 48, 51 (CCPA 1974), the minimal requirement is for the examiner to give reasons for the uncertainty of the enablement." Answer (D) is incorrect because it includes false statement (1). Answer (E) is incorrect because it includes false statements (1) and (3).

In accordance with the patent laws, rules and procedures as related in the MPEP, which of the following statements regarding claim interpretation during patent prosecution is incorrect? A A claim is to be given its broadest reasonable interpretation in light of the supporting disclosure in the specification. B Because a claim is read in light of the specification, the claim may properly be narrowed by interpreting it as including elements or steps disclosed in the specification but not recited in the claim. C If an applicant does not define a claim term in the specification, that term is given its ordinary meaning in the art. D When an explicit definition of a claim term is provided in an applicant's specification, that definition controls the interpretation of the term as it is used in the claims. E Means plus function language in claims which defines the characteristics of a machine or manufacture includes only the corresponding structures or materials disclosed in the specification and equivalents thereof.

B Related MPEP Chapter(s): MPEP 2100 - Patentability Answer Reasoning:ANSWER: (B) is the most proper answer. MPEP § 2111, under the heading "Claims Must Be Given Their Broadest Reasonable Interpretation," states, in reference to In re Prater, 415 F.2d 1393, 1404-05, 162 USPQ 541, 550-51 (CCPA 1969): "The court explained that 'reading a claim in light of the specification,['] to thereby interpret limitations explicitly recited in the claim, is a quite different thing from 'reading limitations of the specification into a claim,' to thereby narrow the scope of the claim by implicitly adding disclosed [sic, disclosed] limitations which have no express basis in the claim." Answer (A) is an improper response to the question because it is a correct statement of claim interpretation during patent prosecution. As pointed out in MPEP § 2111.01, the court in In re Marosi, 710 F.2d 799, 802, 218 USPQ 289, 292 (Fed. Cir. 1983) (quoting In re Okuzawa, 537 F.2d 545, 548, 190 USPQ 464, 466 (CCPA 1976)), states: "It is well settled that 'claims are not to be read in a vacuum and limitations therein are to be interpreted in light of the specification in giving them their "broadest reasonable interpretation."'" Answer (C) is an improper response to the question because it is a correct statement of claim interpretation during patent prosecution. MPEP § 2111.01, under the heading "Plain Meaning Refers To The Meaning Given to The Term By Those Of Ordinary Skill In The Art," states that "[w]hen not defined by applicant in the specification, the words of a claim must be given their plain meaning." Answer (D) is an improper response to the question because it is a correct statement of claim interpretation during patent prosecution. MPEP § 2111.01 states that it is only when the specification provides a definition for terms appearing in the claims can the specification be used to interpret the claim language. Answer (E) is an improper response to the question because it is a correct statement of claim interpretation during patent prosecution. See MPEP § 2111.01, under the heading "Plain Meaning Refers To The Meaning Given to The Term By Those Of Ordinary Skill In The Art," states, in reliance upon In re Donaldson, 16 F.3d 1189, 1193, 29 USPQ2d 1845, 1848 (Fed. Cir. 1994), that there is "one exception, and that is when an element is claimed using language falling under the scope of 35 U.S.C. 112, 6th paragraph (often broadly referred to as means or step plus function language). In that case, the specification must be consulted to determine the structure, material, or acts corresponding to the function recited in the claim."

In accordance with USPTO rules and procedures set forth in the MPEP, which of the following statements regarding claim interpretation during patent prosecution is incorrect? A A claim is to be given its broadest reasonable interpretation in light of the supporting disclosure in the specification. B Because a claim is read in light of the specification, the claim may properly be narrowed by interpreting it as including elements or steps disclosed in the specification but not recited in the claim. C If an applicant does not define a claim term in the specification, that term is given its ordinary meaning in the art. D When an explicit definition of a claim term is provided in an applicant's specification, that definition controls the interpretation of the term as it is used in the claims. E Means plus function language in claims which defines the characteristics of a machine or manufacture includes only the corresponding structures or materials disclosed in the specification and equivalents thereof.

B Related MPEP Chapter(s): MPEP 2100 - Patentability Answer Reasoning:ANSWER: (B) is the most proper answer. MPEP § 2111, under the heading "Claims Must Be Given Their Broadest Reasonable Interpretation," states, in reference to In re Prater, 415 F.2d 1393, 1404-05, 162 USPQ 541, 550-51 (CCPA 1969): "The court explained that 'reading a claim in light of the specification,['] to thereby interpret limitations explicitly recited in the claim, is a quite different thing from 'reading limitations of the specification into a claim,' to thereby narrow the scope of the claim by implicitly adding disclosed [sic, disclosed] limitations which have no express basis in the claim." Answer (A) is incorrect because, as pointed out in MPEP § 2111.01, the court in In re Marosi, 710 F.2d 799, 802, 218 USPQ 289, 292 (Fed. Cir. 1983) (quoting In re Okuzawa, 537 F.2d 545, 548, 190 USPQ 464, 466 (CCPA 1976)), states: "It is well settled that 'claims are not to be read in a vacuum and limitations therein are to be interpreted in light of the specification in giving them their "broadest reasonable interpretation."'" Answer (C) is incorrect because MPEP § 2111.01, under the heading "Plain Meaning Refers To The Meaning Given to The Term By Those Of Ordinary Skill In The Art," states that "[w]hen not defined by applicant in the specification, the words of a claim must be given their plain meaning." Answer (D) is incorrect because MPEP § 2111.01 states that it is only when the specification provides a definition for terms appearing in the claims can the specification be used to interpret the claim language. Answer (E) is incorrect. See MPEP § 2111.01, under the heading "Plain Meaning Refers To The Meaning Given to The Term By Those Of Ordinary Skill In The Art," states, in reliance upon In re Donaldson, 16 F.3d 1189, 1193, 29 USPQ2d 1845, 1848 (Fed. Cir. 1994), that there is "one exception, and that is when an element is claimed using language falling under the scope of 35 U.S.C. 112, 6th paragraph (often broadly referred to as means or step plus function language). In that case, the specification must be consulted to determine the structure, material, or acts corresponding to the function recited in the claim."

In accordance with the patent laws, rules and procedures as related in the MPEP, which of the following statements regarding publications as references is incorrect? A A doctoral thesis indexed and shelved in a library can be sufficiently accessible to the public to constitute prior art as a printed publication. B Evidence showing routine business practices is never sufficient to establish the date on which a publication became accessible to the public. C A paper which is orally presented in a forum open to all interested persons can constitute a "printed publication" if written copies are disseminated without restriction. D Documents distributed only internally within an organization, which has an existing policy of confidentiality or agreement to remain confidential are not "printed publications" even if many copies are distributed. E A publication disseminated by mail is not available as prior art until it is received by at least one member of the public.

B Related MPEP Chapter(s): MPEP 2100 - Patentability Answer Reasoning:ANSWER: (B) is the most proper answer. MPEP § 2128.02, under the heading "Date of Accessibility Can Be Shown Through Evidence of Routine Business Practices," states, in reliance upon Constant v. Advanced Micro-Devices, Inc., 848 F.2d 1560, 7 USPQ2d 1057 (Fed. Cir.), cert. denied, 988 U.S. 892 (1988), and In re Hall, 781 F.2d 897, 228 USPQ 453 (Fed. Cir. 1986), "Evidence showing routine business practices can be used to establish the date on which publication became accessible to the public. Specific evidence showing when the specific document actually became available is not always necessary." Answer (A) is incorrect. MPEP § 2128.01, under the heading "A Thesis Placed In A University Library May Be Prior Art If Sufficiently Accessible To The Public," states "[a] doctoral thesis indexed and shelved in a library is sufficiently accessible to the public to constitute prior art as a 'printed publication.' In re Hall, 781 F.2d 897, 228 USPQ 453 (Fed. Cir. 1986). Even if access to the library is restricted, a reference will constitute a 'printed publication' as long as a presumption is raised that the portion of the public concerned with the art would know of the invention. In re Bayer, 568 F.2d 1357, 196 USPQ 670 (CCPA 1978)." Answer (C) is incorrect. MPEP § 2128.01, under the heading "Orally Presented Paper Can Constitute A 'Printed Publication' If Written Copies Are Available Without Restriction," states, in reliance upon Massachusetts Institute of Technology v. AB Fortia, 774 F.2d 1104, 1109, 227 USPQ 428, 432 (Fed. Cir. 1985): "[a] paper which is orally presented in a forum open to all interested persons constitutes a 'printed publication' if written copies are disseminated without restriction." Answer (D) is incorrect. MPEP § 2128.01, under the heading "Internal Documents Intended To Be Confidential Are Not Printed Publications," states, in reliance upon In re George, 2 USPQ2d 1880 (Bd. Pat. App. & Int. 1987), Garret Corp. v. United States, 422 F.2d 874, 878, 164 USPQ 521, 524 (Ct. Cl. 1970), and Northern Telecom Inc. v. Datapoint Corp., 908 F.2d 931, 15 USPQ2d 1321 (Fed. Cir. 1990). "[d]ocuments and items only distributed internally within an organization which are intended to remain confidential are not 'printed publications' no matter how many copies are distributed." Answer (E) is incorrect. MPEP § 2128.02, under the heading "A Journal Article or Other Publication Becomes Available As Prior Art on Date of It Is Received by a Member of the Public," states, in reliance upon In re Schlittler, 234 F.2d 882, 110 USPQ 304 (CCPA 1956): "[a] publication disseminated by mail is not prior art until it is received by at least one member of the public."

Office policy has consistently been to follow Graham v. John Deere Co., 383 U.S. 1, 148 USPQ 459 (1966), in the consideration and determination of obviousness under 35 USC 103. Each of the following are the four factual inquires enunciated therein as a background for determining obviousness except: A Determining the scope and contents of the prior art. B Resolving any issue of indefiniteness in favor of clarity. C Ascertaining the differences between the prior art and the claims in issue. D Resolving the level of ordinary skill in the pertinent art. E Evaluating evidence of secondary considerations.

B Related MPEP Chapter(s): MPEP 2100 - Patentability Answer Reasoning:ANSWER: The most correct answer is (B). 35 U.S.C. § 103; Graham v. John Deere Co., 383 U.S. 1, 148 USPQ 459 (1966); MPEP § 2141. Resolving any issue of indefiniteness in favor of clarity is not among the factual inquiries enunciated in Graham. The four factual inquiries are set forth in answers (A), (C), (D), and (E).

An application includes independent claims 1 and 2. Which of the following, in a reply to a non-final Office action, provides the proper basis for a rejection under 35 USC 112, first paragraph? A Applicant amends claim 2 of the originally filed application by adding a limitation which was previously written only in claim 1 of the originally filed application, and one of ordinary skill in the art is enabled by the original disclosure to make and use the invention as claimed in amended claim 2. B Applicant amends claim 1 of the originally filed application by adding a limitation that was written in the original disclosure of the application, but the original disclosure does not enable one of ordinary skill in the art to make or use the invention as claimed in amended claim 1. C Applicant amends and broadens claim 2 by removing a limitation which was written in the original disclosure of the application, and one of ordinary skill in the art is enabled by the original disclosure to make and use the invention as claimed in amended claim 2. D Applicant adds new matter to the disclosure, but does not amend the claims of the originally filed application, and one of ordinary skill in the art is enabled by the original disclosure to make and use the invention as described in each of the claims. E None of the above.

B Related MPEP Chapter(s): MPEP 2100 - Patentability Answer Reasoning:ANSWER: The most correct answer is (B). MPEP § 2163.01. (A) is incorrect because the claims as filed in the original application are part of the disclosure, MPEP §§ 2163.03 and 2163.06(III), and claim 2 is enabled by the original disclosure. (C) is incorrect. The original disclosure enables claim 2. (D) is incorrect because although the specification should be objected to, the original disclosure enables each of the claims. MPEP § 2163.06(I). (E) is incorrect because (B) is correct.

In an Exparte reexamination proceeding a non-final Office action dated November 8, 2016 set a shortened statutory period of 2 months to reply. The patent owner, represented by a registered practitioner, filed a response on March 7, 2017, which included an amendment of the claims. No request for an extension of time was received. As of May 8, 2017, which of the following actions would be in accord with the patent laws, rules and procedures as related in the MPEP? A The registered practitioner should file a request and fee for an extension of time of two months. B The registered practitioner should file a petition for revival of a terminated reexamination proceeding showing the delay was unintentional, and the appropriate petition fee for entry of late papers. C The primary examiner responsible for the reexamination should mail a Notice of Allowance and grant a new patent. The patent owner's failure to timely respond to the outstanding Office action does not affect the allowability of the claims in the patent. D The examiner should provide an Office action based upon the claims in existence prior to the patent owner's late amendment, and mail a Final Office action. E The registered practitioner should request an extension of time of four months, and file a Notice of Appeal.

B Related MPEP Chapter(s): MPEP 2200 - Citation of Prior Art and Ex Parte Reexamination of Patents Answer Reasoning:ANSWER: (B) is the most correct answer. 37 CFR § 1.137; and MPEP § 2268. The patent owner will need to file a petition for entry of late papers in order to have their response entered, considered and acted upon. According to MPEP 2268, "[p]ursuant to 37 CFR 1.550(d), an ex parte reexamination proceeding is terminated if the patent owner fails to file a timely and appropriate response to any Office . . . An ex parte reexamination proceeding terminated under 37 CFR 1.550(d) can be revived if the delay in response by the patent . . . was unavoidable in accordance with 37 CFR 1.137(a), or unintentional in accordance with 37 CFR 1.137(b)." (A) is not the most correct answer. In a reexamination proceeding, requests for extensions of time must be filed on or before the day on which action by the patent owner is due pursuant to 37 CFR § 1.550(c). See MPEP § 2265. (C) is incorrect. (C) is inconsistent with MPEP § 2266, which states that if the patent owner fails to file a timely response to any Office action, the reexamination proceeding will be terminated, and after the proceeding is terminated, the Commissioner will proceed to issue a reexamination certificate. There is no provision for issuing a notice of allowance in a reexamination proceeding. Further, (C) is incorrect inasmuch as the examiner should not mail a Notice of Allowance and grant a new patent. (D) is not the most correct answer. In a reexamination proceeding where patent owner fails to file a timely and appropriate response to any Office action, the reexamination proceeding will be terminated via issuance of the Notice of Intent to Issue Reexamination Certificate. See MPEP § 2266. (E) is not the most correct answer. In a reexamination proceeding, requests for extensions of time must be filed on or before the day on which action by the patent owner is due pursuant to 37 C.F.R. § 1.550(c).

In accordance with the USPTO rules and the procedures set forth in the MPEP, which of the following, if any, is true? A The loser in an interference in the PTO is estopped from later claiming he or she was the first to invent in a Federal District Court since the loser must win in the USPTO or he/she will lose the right to contest priority. B A person being sued for infringement may file a request for Ex-parte reexamination without first obtaining the permission of the Court in which the litigation is taking place. C A practitioner may not represent spouses, family members or relatives before the USPTO since such representation inherently creates a conflict of interest and a practitioner is likely to engage in favoritism over his/her other clients. D Employees of the USPTO may not apply for a patent during the period of their employment and for two years thereafter. E None of the above.

B Related MPEP Chapter(s): MPEP 2200 - Citation of Prior Art and Ex Parte Reexamination of Patents Answer Reasoning:ANSWER: (B) is the most correct answer. Any person at any time may file a request for reexamination. 35 U.S.C. § 302. As to (A) loser may appeal to District Court under 35 U.S.C. § 146. As to (C), there is no prohibition regarding spouses, family members, and other relatives. As to (D) according to 35 U.S.C. § 4, employees are prohibited during the period of their employment and one year thereafter. As to (E), (B) is true.

Which of the following is not prohibited conduct for a practitioner under the USPTO Code of Professional Responsibility? A Entering into an agreement with the client to limit the amount of any damages which the client may collect for any mistakes the practitioner may make during prosecution of the client's patent application in exchange for prosecuting the application at a reduced fee. B Encouraging the client to meet with an opposing party for settlement discussions. C Failing to disclose controlling legal authority which is adverse to the practitioner's client's interest when arguing the patentability of claims in a patent application. D In reply to an Office action, stating honestly and truthfully in the remarks accompanying an amendment that the practitioner has personally used the device and found it to be very efficient and better than the prior art. E Investing the funds the client advanced for the practitioner legal fees (not costs and expenses) in long term United States Treasury Bills in order to obtain guaranteed protection of the principal.

B Related MPEP Chapter(s): MPEP 400 - Representative of Inventor or Owner Answer Reasoning:ANSWER: (B) is the most correct answer. CFR Part 11 generally. As to (A), practitioner may not limit damages. As to (C), see Rule 56. As to (D), personal use is entitled to no weight. As to (E), client funds advanced for legal services are required to be deposited in a bank account.

In accordance with the MPEP and USPTO rules and procedure, a registered attorney or agent may be appointed to prosecute a patent application for an applicant. In which of the following situations will the USPTO not recognize the appointment of an attorney or agent to prosecute a patent application for an applicant? A The principal agent of record appoints a registered associate attorney to prosecute the application. B The applicant executes a power of attorney naming only a law firm to prosecute the application. C The applicant executes a power of attorney appointing more than one registered individual to prosecute the application. D The principal attorney of record appoints an associate registered agent to prosecute the application. E The assignee of the entire interest of record, who has established the right to prosecute the patent application, executes a power of attorney appointing a registered attorney to prosecute the patent application.

B Related MPEP Chapter(s): MPEP 400 - Representative of Inventor or Owner Answer Reasoning:ANSWER: Choice (B) is the correct answer. MPEP § 402; 37 C.F.R. § 1.34. MPEP § 402 states that "Powers of attorney or authorizations of agent naming firms of attorneys or agents filed in patent applications will not be recognized." (A) and (D) are each incorrect because according to 37 C.F.R. § 1.34(b), "A principal registered attorney or agent, so appointed, may appoint an associate registered attorney or agent who shall also then be of record." (C) is incorrect. As stated in MPEP § 402, "Powers of attorney and authorizations of agent under 37 C.F.R. § 1.34(b) naming one or more registered individuals, or all registered practitioners associated with a Customer Number, may be made. See MPEP § 403 for Customer Number practice." (E) is incorrect because, as set forth in MPEP § 402.07, "A power of attorney by the assignee of the entire interest revokes all powers given by the applicant and prior assignees if the assignee establishes their right to take action as provided in 37 C.F.R. § 3.73(b)."

Which of the following statement(s) is in accordance with patent laws, rules and procedures as related in the MPEP regarding filing of an Application Data Sheet (ADS) in the USPTO? A All non-provisional applications must include an ADS when the application is originally filed. B If an ADS is filed at the same time as an oath or declaration under 37 CFR 1.63 or 1.67 and the information supplied in the two documents is inconsistent, the information provided in the ADS will always govern. C If an ADS is filed at the same time as an oath or declaration under 37 CFR 1.63 or 1.67 and the information supplied in the two documents is inconsistent, the oath or declaration will govern any inconsistency related the claiming of benefit under 35 USC 119(e), 120, 121 or 365(c). D If an ADS is filed after an oath or declaration under 37 CFR 1.63 or 1.67 is filed, and the information supplied in the two documents is inconsistent, the information provided in the ADS will always govern. E The oath or declaration under 37 CFR 1.63 or 1.67 governs inconsistencies with the ADS when the inconsistency concerns setting forth the name(s) of the inventor(s) under 35 USC 115.

B Related MPEP Chapter(s): MPEP 600 - Parts, Form, and Content of Application Answer Reasoning:ANSWER: (B) is correct. See 37 CFR § 1.76(d), post-AIA.

Which of the following statements relevant to a third-party submission in a published patent application accords with proper USPTO practice and procedure? A A submission of patents by a member of the public must be made within 2 months of the date of publication of the application. B A submission of patents by a member of the public must be made prior to the mailing of a Notice of Allowance. C A submission of patents by a member of the public must be made within 2 months of the date of publication of the application or prior to the mailing of a Notice of Allowance, whichever is later. D A submission of patents by a member of the public must be made within 2 months of the date of publication of the application or prior to the mailing of a Notice of Allowance, whichever is earlier. E Any submission not filed within the period set forth in the patent rules will be accepted provided it is accompanied by the processing fee set forth in 37 CFR 1.17(i).

B Related MPEP Chapter(s): MPEP 600 - Parts, Form, and Content of Application Answer Reasoning:ANSWER: (B) is correct. The Notice of Allowance cuts off all filings of this type, regardless of any other date. 37 C.F.R. §1.290. (A), (C) and (D) are incorrect because 37 C.F.R. § 1.290 provides a six-month filing window post-publication. (E) is wrong because no fee is necesswary if an applicant submits three or fewer references in an initial submission, but timeliness is essential! See generaly MPEP 1134.01.

9. Which of the following is not in accordance with the provisions of the MPEP? A The title of the invention should be placed at the top of the first page of the specification unless it is provided in the application data sheet. B The title need not be technically accurate, but should be descriptive and should contain fewer than 10 words. C Inasmuch as the words "improved," "improvement of," and "improvement in" are not considered as part of the title of an invention, these words should not be included at the beginning of the title of the invention and will be deleted when the Office enters the title into the Office's computer records, and when any patent issues. D If a satisfactory title is not supplied by the applicant the examiner may, at the time of allowance, change the title by examiner's amendment. If the change to the title is the only change being made by the examiner at the time of allowance, a separate examiner's amendment need not be prepared and the examiner is to indicate the change in title in the file. E A title in a U.S. application need not be identical to the corresponding foreign filed application.

B Related MPEP Chapter(s): MPEP 600 - Parts, Form, and Content of Application Answer Reasoning:ANSWER: (B) is the most correct answer inasmuch as the title needs to be technically accurate and the limitation is 500 characters, not 10 words. See MPEP § 606. As to (A), (C) and (D), see MPEP §§ 606 and 606.01 on p. 600-50. As to (E), the title can be amended by the examiner.

The specification in your client's patent application has been objected to for lack of enablement. To overcome this objection, your client may do any of the following except: A traverse the objection and specifically argue how the specification is enabling. B traverse the objection and submit an additional drawing to make the specification enabling. C file a continuation-in-part application that has an enabling specification. D traverse the objection and file an amendment without adding new matter in an attempt to show enablement. E traverse the objection and refer to prior art cited in the specification that would demonstrate that the specification is enabling to one of ordinary skill.

B Related MPEP Chapter(s): MPEP 600 - Parts, Form, and Content of Application Answer Reasoning:ANSWER: (B) is the most correct answer. 35 U.S.C. § 113 reads "Drawings submitted after the filing date of the application may not be used (i) to overcome any insufficiency of the specification due to lack of an enabling disclosure." Since choice (A) may be done, 37 C.F.R. § 1.111, it is an incorrect answer to the above question. Since choice (C) may be done, 35 U.S.C. § 120, it is an incorrect answer to the above question. Since choice (D) may be done, 37 C.F.R. § 1.121, it is an incorrect answer to the above question. Since choice (E) may be done, 37 C.F.R. § 1.111, it also is an incorrect answer to the above question.

A complete continuation application by the same inventors as those named in the prior application may be filed under 35 USC 111(a) using the procedures of 37 CFR 1.53(b) by providing, in accordance with the USPTO rules and the procedures set forth in the MPEP: A A copy of the prior application, including a copy of the signed declaration in the prior application, as amended. B A new and proper specification (including one or more claims), any necessary drawings, a copy of the signed declaration as filed in the prior application (the new specification, claim(s), and drawings do not contain any subject matter that would have been new matter in the prior application), and all required fees. C A new specification and drawings and a newly executed declaration. The new specification and drawings may contain any subject matter that would have been new matter in the prior application. D A new specification and drawings, and all required fees. E (A), (B), (C) and (D).

B Related MPEP Chapter(s): MPEP 600 - Parts, Form, and Content of Application Answer Reasoning:ANSWER: (B) is the most correct answer. See 37 CFR §§ 1.51(b), 1.53(b), and 1.63(d)(1)(iv); MPEP § 201.06(c), under the heading "Specification and Drawings," and MPEP § 602.05(a). (A) is incorrect. As indicated by MPEP § 201.06(c), a continuation application may be filed under 35 U.S.C. § 111(a) by providing a copy of the prior application, including a copy of the signed declaration in the prior application, as filed. (C) is incorrect. As indicated by MPEP § 201.06(c), a continuation application may be filed under 35 U.S.C. § 111(a) by providing a new specification and drawings and a newly executed declaration provided the new specification and drawings do not contain any subject matter that would have been new matter in the prior application. (D) is incorrect. The oath or declaration is needed to name the same inventor in the continuation application. 37 CFR § 1.53(b); MPEP § 201.06(c). (E) is incorrect because (A), (C) and (D) are incorrect.

A patent application filed in the USPTO contains the following three original claims, including product by process Claim 3: Claim 1. A method for making an Ethernet cable comprising the steps of A, B and C. Claim 2. The method of claim 1, further characterized by the step of D. Claim 3. The Ethernet cable as in any one of the preceding claims. In the first Office action, the examiner objects to Claim 3 as being an improper dependent claim and requires cancellation of the claim. Following proper USPTO practices and procedures, which of the following replies best overcomes the examiner's objection and provides the client with the broadest patent protection? A Amend Claim 3 to read: "The Ethernet cable as made by the process set forth in claims 1-2." B Cancel Claim 3. Add Claim 4, which reads: "An Ethernet cable made by a process comprising the steps of A, B and C." Add Claim 5, which reads: "An Ethernet cable made by a process comprising the steps of A, B, C and D." C Cancel Claim 3 and add Claim 4, which reads: "An Ethernet cable made by a process comprising the steps of A, B and C." D Cancel Claim 3 and add Claim 4, which reads: "An Ethernet cable made by a process comprising the steps of A, B, C and D." E Cancel Claim 3.

B Related MPEP Chapter(s): MPEP 600 - Parts, Form, and Content of Application Answer Reasoning:ANSWER: (B) is the most correct answer. The cancellation of Claim 3 overcomes the examiner's objection. The addition of Claims 4 and 5 provide the client with patent protection in product by process format for the cable by both methods of manufacture. Thus, if Claim 4 is invalid, Claim 5 may remain valid. Answer (A) is incorrect because it is an improper multiple dependent claim. 35 U.S.C. § 112 ¶ 5; 37 C.F.R. § 1.75(c); MPEP § 608.01(n), part (I)(B)(1). Answer (C) alone is not the most correct answer because even though canceling Claim 3 will overcome the rejection and provides protection for the Ethernet cable made by the process comprising the steps A, B and C, it will also leave the application without a claim to the Ethernet cable made using the processes comprising the steps of A, B, C, and D. Answer (D) alone is not the most correct answer because even though canceling Claim 3 will overcome the rejection and provides protection for the Ethernet cable made by the process comprising the steps A, B, C, and D, it will also leave the application without a claim to the Ethernet cable made using the processes comprising the steps of A, B, and C. Answer (E) alone is incorrect because, even though canceling the claim will overcome the rejection, it will also leave the application without a claim to the Ethernet cable made using the processes set forth in either Claim 1 or Claim 2.

Regarding the specification of a nonprovisional patent application, which of the following practices is in accordance with proper USPTO practice and procedure? A The specification may include graphical illustrations or flowcharts. B The specification may include tables and chemical formulas. C The specification may include hyperlinks or other forms of browser-executable code embedded in the text. D The specification must begin with one or more claims. E The specification may include a reservation for a future application of subject matter disclosed but not claimed in the application.

B Related MPEP Chapter(s): MPEP 600 - Parts, Form, and Content of Application Answer Reasoning:ANSWER: (B). MPEP § 608.01, subsection "Illustrations In the Specification." 37 C.F.R. § 1.58(a) permits tables and chemical formulas in the specification in lieu of formal drawings. (A) is incorrect. MPEP § 608.01, subsection "Illustrations In the Specification." Graphical illustrations, diagrammatic views, flowcharts, and diagrams in the descriptive portion of the specification do not come within the purview of 37 C.F.R. § 1.58(a). The examiner should object to such descriptive illustrations in the specification and request formal drawings in accordance with 37 C.F.R. § 1.81 when an application contains graphs in the specification. (C) is incorrect. MPEP § 608.01, subsection "Hyperlinks And Other Forms Of Browser-Executable Code In The Specification." USPTO policy does not permit the USPTO to link to any commercial sites since the USPTO exercises no control over the organization, views, or accuracy of the information contained on these outside sites. (D) is incorrect. 37 C.F.R. § 1.75(a). The specification must conclude with one or more claims. (E) is incorrect. 37 C.F.R. § 1.79. A reservation for a future application of subject matter disclosed but not claimed in a pending application will not be permitted in the pending application.

A patent application is filed disclosing and claiming a system for detecting expired parking meters. The specification fully supports the original, sole claim. The application discloses that the "electronics control unit" contains a comparator and an alarm. The application includes several drawings. One of the drawings shows a block diagram of the system, illustrating the electronics control unit as a box, labeled "electronics control unit." The sole claim of the application is as follows: The claim. A system for detecting expired parking meters, comprising: a timer mechanism; an infrared sensor for detecting the presence of a parked vehicle; and an electronics control unit, including a comparator and an alarm, coupled to the infrared sensor and the timer mechanism. A final Office action, dated February 3, 2016, indicates that the sole claim contains allowable subject matter, but includes an objection to the specification, on the grounds that the subject matter of the electronics control unit, though described in a sufficiently specific and detailed manner in the original specification, was required to be shown in the drawings under 37 CFR 1.83. The Office action did not set a period for reply. Determine which of the following actions, if any, comports with the patent laws, rules and procedures as related in the MPEP for overcoming the objection. A On April 1, 2016, a Notice of Appeal is filed together with appropriate fees, and a brief pointing out that a patent should issue since the subject matter of the electronics control unit was adequately described in the original specification. B On April 1, 2016, a drawing is filed in the USPTO illustrating only the comparator and alarm of the electronics control unit that was described in the original specification. C On April 1, 2016, a Notice of Appeal of appeal is filed together with appropriate fees, and a brief pointing out that the addition of a drawing showing the electronics control unit would not constitute addition of new matter since the electronics control unit was adequately described in the original specification. D On September 1, 2016, a petition is filed urging that no further drawing should be required because the subject matter of the electronics control unit, for purposes of the application, was adequately disclosed in the block diagram drawing. E None of the above.

B Related MPEP Chapter(s): MPEP 600 - Parts, Form, and Content of Application MPEP 800 - Restriction in Applications Filed Under 35 U.S.C. 111; Double Patenting Answer Reasoning:ANSWER: (B). 37 CFR § 1.83(a); MPEP §§ 608.01(l) and 706.03(o). MPEP § 608.01(l) states "[w]here subject matter not shown in a drawing...is claimed in the specification as filed, and such original claim itself constitutes a clear disclosure of this subject matter, then the claim should be treated on its merits, and requirement made to amend the drawing and description to show this subject matter...It is the drawing...that [is] defective, not the claim. It is, of course, to be understood that this disclosure in the claim must be sufficiently specific and detailed to support the necessary amendment of the drawing..." MPEP § 608.02(d) and 706.03(o). MPEP § 706.03(o), state "[i]f subject matter capable of illustration is originally claimed and it is not shown in the drawing, the claim is not rejected but applicant is required to add it to the drawing. See MPEP § 608.01(l)." (A), (C), and (D) are incorrect. See the foregoing quotation from MPEP § 706.03(o). (A) and (C) also are incorrect because objections to drawings are petitionable, not appealable. MPEP §§ 608.02, under the heading "Receipt of Drawing After Filing Date," and 1002. (D) is incorrect because the petition, filed more than six months after the final rejection, is not timely. A timely petition would be filed within two months from the mailing date of the action or notice from which relief is requested. See 35 U.S.C. § 133; 37 CFR §§ 1.181(f), 1.134 and 1.135(a); and MPEP § 710.01. (E) is incorrect because (B) is correct.

Assume that conception of applicant's complex invention occurred prior to the date of the reference, but reduction to practice occurred after the date of the reference in a pre-AIA filing. Which of the following is sufficient to overcome the reference in accordance with the USPTO rules and the procedures set forth in the MPEP? A In a 37 CFR 1.131 affidavit or declaration, it is sufficient to allege that applicant or patent owner has been diligent. B In a 37 CFR 1.131 affidavit or declaration, it is sufficient to clearly establish conception of the invention prior to the effective date of the reference, and diligence from just prior to the effective date of the reference to actual reduction to practice. The presence of a lapse of time between the reduction to practice of an invention and the filing of an application thereon is not relevant. C In a 37 CFR 1.131 affidavit or declaration, it is sufficient to clearly establish conception of the invention prior to the effective date of the reference. Diligence need not be considered. D In a 37 CFR 1.131 affidavit or declaration, it is sufficient to show conception and reduction to practice in any country. E In a 37 CFR 1.131 affidavit or declaration, it is always sufficient to prove actual reduction to practice for all mechanical inventions by showing plans for the construction of the claimed apparatus.

B Related MPEP Chapter(s): MPEP 700 - Examination of Applications Answer Reasoning:ANSWER (B) is the most correct answer. See Ex parte Merz, 75 USPQ 296 (Bd. App. 1947) (holding that the "lapse of time between the completion or reduction to practice of an invention and the filing of an application thereon" is not relevant to an affidavit or declaration under 37 CFR § 1.131(b)); MPEP § 715.07(a). (A) is incorrect. Ex parte Hunter, 1889 C.D. 218, 49 O.G. 733 (Comm'r Pat. 1889); MPEP § 715.07(a). Applicant must show evidence of facts establishing diligence. (C) is incorrect. Ex parte Kantor, 177 USPQ 455 (Bd. App. 1958) (after conception has been clearly established, diligence must be considered prior to the effective date is clearly established, since diligence then comes into question); MPEP § 715.07(a). (D) is incorrect. MPEP § 715.07(c). 37 CFR § 1.131(a) provides for the establishment of a date of completion of the invention in a NAFTA or WTO member country, as well as in the United States, an applicant can establish a date of completion in a NAFTA member country on or after December 8, 1993, the effective date of section 331 of Public Law 103 - 182, the North American Free Trade Agreement Act, and can establish a date of completion in a WTO member country other than a NAFTA member country on or after January 1, 1996, the effective date of section 531 of Public Law 103 - 465, the Uruguay Round Agreements Act. Not all countries are members of NAFTA or WTO, and prior invention in a foreign country cannot be shown without regard for when the reduction to practice occurred. (E) is incorrect. MPEP § 715.07. Actual reduction to practice generally, but not always, requires a showing that the apparatus actually existed and worked, "There are some devices so simple that a mere construction of them is all that is necessary to constitute reduction to practice." In re Asahi/America Inc., 68 F.3d 442, 37 USPQ2d 1204 (Fed. Cir. 1995) (citing Newkirk v. Lulegian, 825 F.2d 1581, 3USPQ2d 1793 (Fed. Cir. 1987) and Sachs v. Wadsworth, 48 F.2d 928, 929, 9 USPQ 252, 253 (CCPA 1931). The claimed restraint coupling held to be so simple a device that mere construction of it was sufficient to constitute reduction to practice. Photographs, coupled with articles and a technical report describing the coupling in detail were sufficient to show reduction to practice.).

Which of the following requests by the registered practitioner of record for an interview with an examiner concerning an application will be granted in accordance with proper USPTO rules and procedure? A A request for an interview in a substitute application prior to the first Office action, for the examiner and attorney of record to meet in the practitioner's office without the authority of the Commissioner. B A request for an interview in a continued prosecution application prior to the first Office action, to be held in the examiner's office. C A request for an interview in a non-continuing and non-substitute application, prior to the first Office action to be held in the examiner's office. D None of the above. E All of the above.

B Related MPEP Chapter(s): MPEP 700 - Examination of Applications Answer Reasoning:ANSWER: (B) is the correct answer. 37 C.F.R. § 1.133 (effective November 7, 2000); "Changes To Implement the Patent Business Goals; Final Rule," 65 FR 54604, 54640-54641 (September 8, 2000). As stated in 65 FR at 54641, left column, "Comment 65: One comment urged that interviews be allowed in a CPA prior to a first action. Response: The comment has been adopted in a broader manner to apply to all continuations and substitute applications that conform to practice set forth in the MPEP." Thus, (B) is correct. (A) is incorrect because interview will not be permitted off Office premises without the authority of the Commissioner. 37 C.F.R. § 1.133(a)(1). (C) is incorrect because an interview for the discussion of the patentability of a pending application will not occur before the first Office action, unless the application is a continuing or substitute application. 37 C.F.R. § 1.133(a)(2). (D) is incorrect because (B) is correct. (E) is incorrect because (A) and (C) are incorrect.

John, unaware of the existence of Jane's U.S. patent, which issued on Tuesday, July 11, 2012, files a patent application on Friday, January 11, 2013. John's application and Jane's patent are not commonly owned. On Thursday, July 11, 2013, in reply to an Office action rejecting all of his claims, John files an amendment canceling all of his claims and adding claims setting forth, for the first time, "substantially the same subject matter" as is claimed in Jane's patent. The examiner rejects John's claims on the basis of 35 USC 135(b). Which of the following statements accords with the USPTO rules and the procedures set forth in the MPEP? A The rejection is improper because 35 USC 135(b) relates to interferences. B The rejection is proper because 35 USC 135(b) is not limited to inter partes proceedings, but may be used as a basis for ex parte rejections. C Since John's claims would interfere with Jane's unexpired patent, the proper procedure is for the examiner to declare an interference rather than to reject John's claims. D The rejection is proper merely by reason of the fact that John's claims are broad enough to cover the patent claims. E The rejection is improper inasmuch as John is claiming "substantially the same subject matter" as is claimed in the patent.

B Related MPEP Chapter(s): MPEP 700 - Examination of Applications Answer Reasoning:ANSWER: (B) is the most correct answer, and (A) and (C) are wrong. MPEP § 715.05 states "[i]f the patent is claiming the same invention as the application and its issue date is one year or more prior to the presentation of claims to that invention in the application, a rejection of the claims of the application under 35 U.S.C. § 135(b) should be made. See In re McGrew, 120 F.3d 1236, 1238, 43 USPQ2d 1632, 1635 (Fed.Cir. 1997) (holding that application of 35 U.S.C. § 135(b) is not limited to inter partes interference proceedings, but may be used as a basis for ex parte rejections.)". (D) is wrong. See MPEP § 2307 ("The fact that the application claim may be broad enough to cover the patent claim is not sufficient. In re Frey, 182 F.2d 184, 86 USPQ 99 (CCPA 1950)"). (E) is also wrong. See MPEP § 2307 ("If the claim presented or identified as corresponding to the proposed count was added to the application by an amendment filed more than one year after issuance of the patent...then under the provisions of 35 U.S.C. § 135(b), an interference will not be declared unless at least one of the claims which were in the application...prior to expiration of the one-year period was for 'substantially the same subject matter' as at least one of the claims of the patent.").

Applicant files a patent application in Japan on January 5, 2000. Applicant files a PCT international application designating the United States on January 5, 2001, based on the Japanese application. The international application is published in English on July 5, 2001. The international application enters the national stage in the United States on September 5, 2001. The USPTO publishes the application on June 6, 2002. The application issues as a United States patent on December 3, 2002. What is its earliest possible 35 USC 102(e) prior art date for the application published by the United States, in view of the amendment to Title 35 by the American Inventors Protection Act of 1999 and the Intellectual Property and High Technology Technical Amendments Act of 2002? A January 5, 2000. B January 5, 2001. C July 5, 2001. D June 6, 2002. E December 3, 2002.

B Related MPEP Chapter(s): MPEP 700 - Examination of Applications Answer Reasoning:ANSWER: (B) is the most correct answer. 35 U.S.C. § 102(e)(1) provides that a US published application of a national stage of an international application filed on or after November 29, 2000 has a prior art effect as of its international filing date, if the international application designated the United States, and was published in English. Because in the above fact pattern, the international application designated the United States and was published in English, and was filed on or after November 29, 2000, the USPTO published application is entitled to its international filing date of January 5, 2001 for prior art purposes under 35 U.S.C. § 102(e)(1). See Example 4 of MPEP § 706.02(f)(1). (A) is wrong because the Japanese filing date is relevant under 35 U.S.C. § 119(a) only for priority and not prior art purposes. (C) and (E) are wrong because they recite prior art dates that are later than January 5, 2001. (D) is wrong because the prior art date under 35 U.S.C. § 102(e)(1) is earlier than the application publication date, June 6, 2002.

Which of the following requests by the registered practitioner of record for an interview with an examiner concerning an application will be granted in accordance with proper USPTO rules and procedure? A A request for an interview in a substitute application prior to the first Office action, for the examiner and attorney of record to meet in the practitioner's office without the authority of the Commissioner. B A request for an interview in a continued prosecution application prior to the first Office action, to be held in the examiner's office. C A request for an interview in a non-continuing and non-substitute application, prior to the first Office action to be held in the examiner's office. D All of the above. E None of the above.

B Related MPEP Chapter(s): MPEP 700 - Examination of Applications Answer Reasoning:ANSWER: (B) is the most correct answer. 37 CFR § 1.133; MPEP § 713.02. As stated in MPEP § 713.02, "[a] request for an interview prior to the first Office action is ordinarily granted in continuing or substitute applications. A request for an interview in all other applications before the first action is untimely and will not be acknowledged if written, or granted if oral. 37 CFR 1.133(a)." (A) is incorrect because interview will not be permitted off Office premises without the authority of the Commissioner. 37 CFR § 1.133(a)(1). (C) is incorrect because an interview for the discussion of the patentability of a pending application will not occur before the first Office action, unless the application is a continuing or substitute application. 37 CFR § 1.133(a)(2). (D) is incorrect because (A) and (C) are incorrect. (E) is incorrect because (B) is correct.

In accordance with the USPTO rules and the procedures set forth in the MPEP, which of the following is true? A There is no practical difference between an objection and rejection of a claim. B If the form of the claim (as distinguished from its substance) is improper, an objection is made. C An objection, if maintained by an examiner, is subject to review by the Board of Patent Appeals and Interferences. D An example of a proper objection is where the claims are refused because they fail to comply with the second paragraph of 35 USC 112. E An example of a proper rejection is a rejection of a dependent claim for being dependent on a claim that has been rejected only over prior art, where the dependent claim is otherwise allowable.

B Related MPEP Chapter(s): MPEP 700 - Examination of Applications Answer Reasoning:ANSWER: (B) is the most correct answer. MPEP § 706.01. (A) and (C) are incorrect. As stated by MPEP § 706.01, "The practical difference between a rejection and an objection is that a rejection, involving the merits of the claim, is subject to review by the Board of Patent Appeals and Interferences, while an objection, if persisted, may be reviewed only by way of petition to the Commissioner." (D) is incorrect. MPEP § 706.03(d). (E) is incorrect. As stated in MPEP § 706.01, "If the form of the claim (as distinguished from its substance) is improper, an "objection" is made. An example of a matter of form as to which objection is made is dependency of a claim on a rejected claim, if the dependent claim is otherwise allowable. See MPEP § 608.01(n)."

Which of the following best describes a situation for which a reply to the examiner's Office action in a pre-AIA filed application including both an affidavit filed under 37 CFR 1.131 and an affidavit filed under 37 CFR 1.132 may be in accord with the patent laws, rules and procedures as related in the MPEP? A In a timely reply to a non-final Office action, where the examiner's sole rejection of appellant's claims is based on an alleged violation of the enablement requirement of 35 USC 112. B In a timely reply to non-final Office action, where the examiner's sole rejection of appellant's claims is a rejection under 35 USC 103(a) employing a non-patent document that was published less than one year prior to the filing date of appellant's patent application. C In a timely reply to a non-final Office action, where the examiner's sole rejection of appellant's claims is a rejection under 35 USC 103(a) employing a non-commonly owned U.S. patent as prior art under 35 USC 102 (e) that claims the same invention as applicant. D In a timely reply to an examiner's answer presenting the affidavits for the first time, where in the examiner's first Office action and final rejection, the examiner maintains the same rejection under 35 USC 103(a) of all of appellant's claims based in part on a non-patent document that was published less than one year prior to the filing date of appellant's patent application. E In a timely reply to a final Office action presenting the affidavits for the first time, where in the examiner's first Office action, the examiner's sole rejection of appellant's claims is a rejection under 35 USC 103(a) employing a non-patent document that was published less than one year prior to the filing date of appellant's patent application.

B Related MPEP Chapter(s): MPEP 700 - Examination of Applications Answer Reasoning:ANSWER: (B) is the most correct answer. MPEP § 715, under the heading "Situations Where 37 CFR 1.131 Affidavits or Declarations Can Be Used," provides that an affidavit under 37 CFR § 1.131 may be used to antedate a reference that qualifies as prior art under 35 U.S.C. § 102(a) but not 35 U.S.C. § 102(b). Also, MPEP § 716 provides that objective evidence traversing a rejection may be presented in a timely submitted affidavit under 37 CFR § 1.132. As for (A), an affidavit under 37 CFR § 1.131 would not serve any useful purpose in those situations. See MPEP § 715. Regarding (C), an affidavit under 37 CFR § 1.131 would be inappropriate. See MPEP § 715, under the heading "Situations Where 37 CXFR 1.131 Affidavits or Declarations Are Inappropriate." As for (D) and (E), a reply including affidavits under 37 CFR § 1.131 and 37 CFR § 1.132 normally would be considered untimely under the circumstances set forth in those answers. See 37 CFR §§ 1.116, 1.192(a) and 1.195. Also, see MPEP §§ 715.09 and 716.01. Consequently, (B) is the most correct answer.

The Potter patent application was filed on June 6, 2015, claiming subject matter invented by Potter. The Potter application properly claims priority to a German application filed on June 6, 2014. A first Office action contains a rejection of all the claims of the application under 35 USC 103 based on a U.S. patent application publication to Smith in view of a U.S. patent to Jones. A registered practitioner prosecuting the Potter application ascertains that the relevant subject matter in Smith's published application and Potter's claimed invention were, at the time Potter's invention was made, owned by ABC Company or subject to an obligation of assignment to ABC Company. The practitioner also observes that the Smith patent application was filed on April 10, 2014 and that the patent application was published on December 5, 2015. Smith and Potter do not claim the same patentable invention. To overcome the rejection without amending the claims, which of the following timely replies would comply with the USPTO rules and the procedures set forth in the MPEP to be an effective reply for overcoming the rejection? A A reply that only contains arguments that Smith fails to teach all the elements in the only independent claim, and which specifically points out the claimed element that Smith lacks. B A reply that properly states that the invention of the Potter application and the Smith application were commonly owned by ABC Company at the time of the invention and later filing of the Potter application. C A reply that consists of an affidavit or declaration under 37 CFR 1.132 stating that the affiant has never seen the invention in the Potter application before. D A reply that consists of an affidavit or declaration under 37 CFR 1.131 properly proving invention of the claimed subject matter of Potter application only prior to June 6, 2014. E A reply that consists of a proper terminal disclaimer and affidavit or declaration under 37 CFR 1.130.

B Related MPEP Chapter(s): MPEP 700 - Examination of Applications Answer Reasoning:ANSWER: (B) is the most correct answer. See 35 U.S.C. § 102(b)(2)(c). The prior art exception in 35 U.S.C. § 102 is applicable because the Smith reference is only prior art under 35 U.S.C. § 102(a)(2), was applied in a rejection under 35 U.S.C. § 103, and was commonly owned at the time Potter made the invention claimed by Potter. (A) is not a correct answer in that one cannot show nonobviousness by attacking the references individually where the rejections are based on a combination of references. See MPEP § 2145. (C) is not a correct answer. An affirmation that the affiant has never seen the invention before is not relevant to the issue of nonobviousness of the claimed subject matter. See MPEP 716. (D) is not a correct answer. Invention must be proved prior to the effective filing date of Smith, which is April 10, 2014. See MPEP § 715. (E) is not a correct answer. A terminal disclaimer and affidavit or declaration under 37 CFR § 1.130 are not proper because the Potter application and the Smith reference are not claiming the same patentable invention.

Assume that conception of applicant's complex pre-AIA invention occurred prior to the date of the reference, but reduction to practice occurred after the date of the reference. Which of the following is sufficient to overcome the reference in accordance with proper USPTO practice and procedure? A In a 37 CFR 1.131 affidavit or declaration, it is sufficient to allege that applicant or patent owner has been diligent. B In a 37 CFR 1.131 affidavit or declaration, it is sufficient to clearly establish conception of the invention prior to the effective date of the reference, and diligence from just prior to the effective date of the reference to actual reduction to practice of an invention and the filing of an application thereon is not relevant. C In a 37 CFR 1.131 affidavit or declaration, it is sufficient to clearly establish conception of the invention prior to the effective date of the reference. Diligence need not be considered. D In a 37 CFR 1.131 affidavit or declaration, it is sufficient to show conception and reduction to practice in any country. E In a 37 CFR 1.131 affidavit or declaration, it is always sufficient to prove actual reduction to practice for all mechanical inventions by showing plans for the construction of the claimed apparatus.

B Related MPEP Chapter(s): MPEP 700 - Examination of Applications Answer Reasoning:ANSWER: (B) is the most correct answer. See Ex parte Merz, 75 USPQ 296 (Bd. App. 1947) (holding that the "lapse of time between the completion or reduction to practice of an invention and the filing of an application thereon" is not relevant to an affidavit or declaration under 37 C.F.R. § 1.131(b)); MPEP § 715.07(a). (A) is incorrect. Ex parte Hunter, 1889 C.D. 218, 49 O.G. 733 (Comm'r Pat. 1889); MPEP § 715.07(a). Applicant must show evidence of facts establishing diligence. (C) is incorrect. Ex parte Kantor, 177 USPQ 455 (Bd. App. 1958) (after conception has been clearly established, diligence must be considered prior to the effective date is clearly established, since diligence then comes into question); MPEP § 715.07(a). (D) is incorrect. MPEP § 715.07(c). 37 C.F.R. § 1.131(a) provides for the establishment of a date of completion of the invention in a NAFTA or WTO member country, as well as in the United States, an applicant can establish a date of completion in a NAFTA member country on or after December 8, 1993, the effective date of section 331 of Public Law 103 - 182, the North American Free Trade Agreement Act, and can establish a date of completion in a WTO member country other than a NAFTA member country on or after January 1, 1996, the effective date of section 531 of Public Law 103 - 465, the Uruguay Round Agreements Act. Not all countries are members of NAFTA or WTO, and prior invention in a foreign country cannot be shown without regard for when the reduction to practice occurred. (E) is incorrect. MPEP § 715.07. Actual reduction to practice generally, but not always, requires a showing that the apparatus actually existed and worked, "There are some devices so simple that a mere construction of them is all that is necessary to constitute reduction to practice." In re Asahi/America Inc., 68 F.3d 442, 37 USPQ2d 1204 (Fed. Cir. 1995) (citing Newkirk v. Lulegian, 825 F.2d 1581, 3USPQ2d 1793 (Fed. Cir. 1987) and Sachs v. Wadsworth, 48 F.2d 928, 929, 9 USPQ 252, 253 (CCPA 1931). The claimed restraint coupling held to be so simple a device that mere construction of it was sufficient to constitute reduction to practice. Photographs, coupled with articles and a technical report describing the coupling in detail were sufficient to show reduction to practice.).

n accordance with the USPTO rules and the procedures set forth in the MPEP, a petition to make a patent application special may be filed without fee in which of the following cases? A The petition is supported by applicant's birth certificate showing applicant's age is 62. B The petition is supported by applicant's unverified statement that applicant's age is 65. C The petition is supported by applicant's statement that there is an infringing device actually on the market, that a rigid comparison of the alleged infringing device with the claims of the application has been made, and that applicant has made a careful and thorough search of the prior art. D The petition is accompanied by a statement under 37 CFR 1.102 by applicant explaining the relationship of the invention to safety of research in the field of recombinant DNA research. E The petition is accompanied by applicant's statement explaining how the invention contributes to the diagnosis, treatment or prevention of HIV/AIDS or cancer.

B Related MPEP Chapter(s): MPEP 700 - Examination of Applications Answer Reasoning:ANSWER: (B) is the most correct answer. See MPEP § 708.02, under the heading "IV. Applicant's Age." (A) is wrong because MPEP § 708.02, under the heading "IV. Applicant's Age, states, "[a]n application may be made special upon filing a petition including any evidence showing that the applicant is 65 years of age, or more, such as a birth certificate or applicant's statement. No fee is required with such a petition." (C), (D), and (E) are wrong because a fee is required with respect to each petition. MPEP § 708.02, under the headings "II. Infringement," "VII, Inventions Relating To Recombinant DNA," and "X. Inventions Relating To HIV/AIDS and Cancer," respectively.

Inventor Joe is anxious to get a patent with the broadest claim coverage possible for the invention. Joe retained a registered practitioner, Jane, to obtain the advantage of legal counsel in obtaining broad protection. Jane filed a patent application for the invention. The inventor heard that, although patent prosecution is conducted in writing, it is possible to get interviews with examiners. Joe believes an interview might hasten the grant of a patent by providing the examiner a better understanding of the true novelty of the invention. Which of the following are consistent with the USPTO rules and the procedures set forth in the MPEP regarding usage of interviews? A Prior to the first Office action being mailed the inventor calls the examiner to whom the application is docketed to offer help in understanding the specification. B After receiving the first Office action Jane calls the examiner for an interview for the purpose of clarifying the structure and operation of the invention as claimed and disclosed, because the examiner's analysis regarding patentability in the rejection is novel and suggests that the examiner is interpreting the claimed invention in a manner very different from the inventor's intent. C Jane has Larry, a registered practitioner in the Washington D.C. area, who is more familiar with interview practice to call the examiner. Jane gives Larry a copy of the first Office action, which suggests that the primary examiner's analysis is incorrect, and offers to explain why. Jane instructs Larry that because Larry is unfamiliar with the inventor, Larry should not agree to possible ways in which the claims could be modified, or at least indicate to the examiner that Jane would have to approve of any such agreement. D Jane calls the primary examiner after receiving the final rejection, demanding that the examiner withdraw the finality of the final action. When the examiner states that the final rejection is proper, Jane demands an interview as a matter of right to explain the arguments. E (B) and (D).

B Related MPEP Chapter(s): MPEP 700 - Examination of Applications Answer Reasoning:ANSWER: (B) is the most correct answer. See MPEP § 713.01. An interview should be had only when the nature of the case is such that the interview could serve to develop and clarify specific issues and lead to a mutual understanding between the examiner and the applicant, and thereby advance the prosecution of the application. (A) is incorrect. 37 CFR § 1.133(a)(2); MPEP § 713.02. Section 713.02 states that although "[a] request for an interview prior to the first Office action is ordinarily granted in continuing or substitute applications[,] [a] request for an interview in all other applications before the first action is untimely and will not be acknowledged if written, or granted if oral. 37 CFR 1.133(a)." (C) is incorrect. MPEP § 713.03. Larry is only sounding out the examiner and has no authority to commit Joe to any agreement reached with the examiner. (D) is incorrect. MPEP § 713.09. Jane has no right to an interview following the final rejection. Although such an interview may be granted if the examiner is convinced that disposal or clarification for appeal may be accomplished with only nominal further consideration, interviews merely to restate arguments of record or to discuss new limitations which would require more than nominal reconsideration or new search should be denied. (E) is incorrect because D is incorrect.

Which of the following is true? A There is no practical difference between an objection and rejection of a claim. B If the form of the claim (as distinguished from its substance) is improper, an objection is made. C An objection, if maintained by an examiner, is subject to review by the Board of Patent Appeals and Interferences. D An example of a proper objection is where the claims are refused because they fail to comply with the second paragraph of 35 USC 112. E An example of a proper rejection is a rejection of a dependent claim for being dependent on a claim that has been rejected only over prior art, where the dependent claim is otherwise allowable.

B Related MPEP Chapter(s): MPEP 700 - Examination of Applications Answer Reasoning:ANSWER: (B). MPEP § 706.01. (A) and (C) are incorrect. As stated by MPEP § 706.01, "The practical difference between a rejection and an objection is that a rejection, involving the merits of the claim, is subject to review by the Board of Patent Appeals and Interferences, while an objection, if persisted, may be reviewed only by way of petition to the Commissioner." (D) is incorrect. MPEP § 706.03(d). (E) is incorrect. As stated in MPEP § 706.01, "If the form of the claim (as distinguished from its substance) is improper, an "objection" is made. An example of a matter of form as to which objection is made is dependency of a claim on a rejected claim, if the dependent claim is otherwise allowable. See MPEP § § 608.01(n)."

If a prima facie case of obviousness is properly established by a primary examiner, how can an applicant effectively rebut the rejection in accordance with proper USPTO practice and procedure? A Rebuttal may be by way of arguments of counsel used in place of factually supported objective evidence to rebut the prima facie case. B Rebuttal may be by way of an affidavit or declaration under 37 CFR 1.132 containing objective evidence arising out of a secondary consideration related to the claimed invention. C No substantive showing is required by applicant. The burden remains on the examiner to maintain a prima facie case. D Rebuttal evidence must be found elsewhere than in the specification. E Rebuttal may be by way of arguing that the prior art did not recognize latent properties.

B Related MPEP Chapter(s): MPEP 700 - Examination of Applications Answer Reasoning:ANSWER: (B). MPEP § 716.01(a). Affidavits or declarations containing objective evidence of criticality, unexpected results, commercial success, long-felt but unsolved needs, failure of others, skepticism of experts, is considered by an examiner. (A) is incorrect. In re Schulze, 346 F.2d 600, 602, 145 USPQ 716, 718 (CCPA 1965); MPEP § 716.01(c), subsection styled "Attorney Arguments Cannot Take The Place of Evidence"; 2145, part I. Arguments of counsel cannot take the place of factually supported objective evidence. (C) is incorrect. MPEP § 2145. The burden shifts to the applicant to come forward with arguments and/or evidence to rebut the prima facie case. In re Hoeksema, 399 F.2d 269, 158 USPQ 596 (CCPA 1968). (D) is incorrect. In re Soni, 54 F.3d 746, 750, 34 USPQ2d 1684, 1687 (Fed. Cir. 1995); MPEP § 2144.05, subsections II and III. (E) is incorrect. In re Wiseman, 596 F.2d 1019, 201 USPQ 658 (CCPA 1979) (finding that mere recognition of latent properties in the prior art does not render a known invention unobvious); MPEP § 2145, subsection II. Also see KSR guidelines.

The MPEP and USPTO rules and procedure provide for ways that a nonstatutory double patenting rejection can be overcome. Which of the following is an effective way to overcome a nonstatutory double patenting rejection? A Filing a 37 CFR 1.131 affidavit to swear behind the patent on which the rejection is based. B Filing a terminal disclaimer under 37 CFR 1.321(c). C Filing a 37 CFR 1.131 affidavit arguing that the claims are for different inventions that are not patentably distinct. D Filing a reply arguing that there is only one common inventor regarding the claims of the application and the claims of the patent. E All of the above.

B Related MPEP Chapter(s): MPEP 800 - Restriction in Applications Filed Under 35 U.S.C. 111; Double Patenting Answer Reasoning:ANSWER: Choice (B) is the correct answer. MPEP § 804.02, subpart (II) reads, "A rejection based on a nonstatutory type of double patenting can be avoided by filing a terminal disclaimer in the application or proceeding in which the rejection is made." Choices (A) and (C) are each incorrect. MPEP § 804.02, reads, "The use of a 37 C.F.R. § 1.131 affidavit in overcoming a double patenting rejection is inappropriate...37 C.F.R. § 1.131 is inapplicable if the claims of the application and the patent are 'directed to substantially the same invention.' It is also inapplicable if there is a lack of 'patentable distinctness' between the claimed subject matter." Choice (C) is further incorrect since a nonstatutory double patenting rejection can be based on the claims not being patentably distinct. MPEP § 804, subpart (II)(B)(1). Choice (D) is incorrect because MPEP § 804, subpart (I)(A) reads, "Double patenting may exist between an issued patent and an application filed by the same inventive entity, or by an inventive entity having a common inventor with the patent." Choice (E) is incorrect because choices (A), (C), and (D) are each incorrect.

Inventors B and C are employed by Corporation D, which authorized registered practitioner E to prepare and file a patent application claiming subject matter invented by B and C. Inventor B signed the oath, an assignment to Corporation D, and a power of attorney authorizing practitioner E to prosecute the application. Inventor C refused to sign the oath and any assignment documents for the application. The employment contract between inventor C and Corporation D contains no language obligating C to assign any invention to Corporation D. A patent application was properly filed in the USPTO under 37 CFR 1.47 naming B and C as inventors, but without inventor C signing the oath. C has now started his own company competing with Corporation D producing a product with the invention in the application. Inventor B is a friend of inventor C and wants C to have continued access to the application. Which of the following statements is in accordance with the patent laws, rules and procedures as related in the MPEP? A Inventor C, who has not signed the oath or declaration, may revoke the power of attorney to practitioner E and appoint practitioner F to prosecute the application. B Inventor C cannot be excluded from access t the application because inventor B has not agreed to exclude inventor C. In order to exclude a co-inventor from access to an application, all the remaining inventors must agree to exclude that coinventor. C Inasmuch as one of the named joint inventors has not assigned his or her rights to Corporation D, the corporation is not an assignee of the entire right and interest, and therefore cannot exclude inventor C from access to the application. D An inventor who did not sign the oath or declaration filed in an application can always be excluded from access to an application. E An assignee filing an application can control access to an application and exclude inventors who have not assigned their rights and other assignees from inspecting the application.

C Related MPEP Chapter(s): MPEP 100 - Secrecy, Access, National Security, and Foreign Filing Answer Reasoning:ANSWER: (C) is the most correct answer. MPEP § 106 states "[t]he assignee of record of the entire interest in an application may intervene in the prosecution of the application, appointing an attorney or agent of his or her own choice. See 37 CFR § 3.71. Such intervention, however, does not exclude the applicant from access to the application to see that it is being prosecuted properly, unless the assignee makes specific request to that effect." (A), (B), (D), and (E) are incorrect. MPEP § 409.03(i) is directly contrary to answer (A), and provides that a non-signing inventor cannot revoke or give a power of attorney without agreement of all named inventors or the 37 CFR § 1.47(b) applicant. (B) is incorrect. MPEP § 106 does not empower an inventor who has assigned his or her rights to exclude a non-signing joint inventor from accessing an application in which the latter party is named as a joint inventor. (E) is incorrect. MPEP § 106. Corporation D, as an assignee of a part interest, cannot exclude the non-signing joint inventor from access to the application. See also, MPEP § 106.01, which states "While it is only the assignee of record of the entire interest who can intervene in the prosecution of an application or interference to the exclusion of the applicant, an assignee of a part interest or a licensee of exclusive right is entitled to inspect the application." (D) is incorrect because MPEP § 409.03(i) states that a nonsigning inventor is entitled to inspect any papers in the application, and order copies at the price set forth in 37 C FR § 1.19.

Inventors B and C are employed by Corporation D, which authorized registered practitioner E to prepare and file a patent application claiming subject matter invented by B and C. Inventor B signed the oath, an assignment to Corporation D, and a power of attorney authorizing practitioner E to prosecute the application. Inventor C refused to sign the oath and any assignment documents for the application. The employment contract between inventor C and Corporation D contains no language obligating C to assign any invention to Corporation D. A patent application was properly filed in the USPTO under 37 CFR 1.47 naming B and C as inventors, but without inventor C signing the oath. C has now started his own company competing with Corporation D producing a product with the invention in the application. Inventor B is a friend of inventor C and wants C to have continued access to the application. Which of the following statements is in accordance with the USPTO rules and the procedures set forth in the MPEP? A Inventor C, who has not signed the oath or declaration, may revoke the power of attorney to practitioner E and appoint practitioner F to prosecute the application. B Inventor C cannot be excluded from access to the application because inventor B has not agreed to exclude inventor C. In order to exclude a co-inventor from access to an application, all the remaining inventors must agree to exclude that co-inventor. C Inasmuch as one of the named joint inventors has not assigned his or her rights to Corporation D, the corporation is not an assignee of the entire right and interest, and therefore cannot exclude inventor C from access to the application. D An inventor who did not sign the oath or declaration filed in an application can always be excluded from access to an application. E An assignee filing an application can control access to an application and exclude inventors who have not assigned their rights and other assignees from inspecting the application.

C Related MPEP Chapter(s): MPEP 100 - Secrecy, Access, National Security, and Foreign Filing Answer Reasoning:ANSWER: (C) is the most correct answer. MPEP § 106 states: "[t]he assignee of record of the entire interest in an application may intervene in the prosecution of the application, appointing an attorney or agent of his or her own choice. See 37 CFR § 3.71. Such intervention, however, does not exclude the applicant from access to the application to see that it is being prosecuted properly, unless the assignee makes specific request to that effect." (A), (B), (D), and (E) are incorrect. MPEP § 409.03(i) is directly contrary to answer (A), and provides that a non-signing inventor cannot revoke or give a power of attorney without agreement of all named inventors or the 37 CFR § 1.47(b) applicant. (B) is incorrect. MPEP § 106 does not empower an inventor who has assigned his or her rights to exclude a non-signing joint inventor from accessing an application in which the latter party is named as a joint inventor. (E) is incorrect. MPEP § 106. Corporation D, as an assignee of a part interest, cannot exclude the non-signing joint inventor from access to the application. See also, MPEP § 106.01, which states "While it is only the assignee of record of the entire interest who can intervene in the prosecution of an application or interference to the exclusion of the applicant, an assignee of a part interest or a licensee of exclusive right is entitled to inspect the application." (D) is incorrect because MPEP § 409.03(i) states that a nonsigning inventor is entitled to inspect any papers in the application, and order copies at the price set forth in 37 CFR § 1.19.

The claims in a patent application having been twice or finally rejected, the applicant files a timely Notice of Appeal on January 2, 2016. In accordance with USPTO rules and procedures set forth in the MPEP, which of the following situations should the USPTO not notify the applicant that the Appeal Brief is defective and allow him an opportunity to correct the deficiency? A The Appeal Brief is filed on July 10, 2016, without a request for extension of time under 37 CFR 1.136. B The Appeal Brief is submitted unsigned. C The Appeal Brief states that the claims do not stand or fall together, and presents argument as to why the claims are separately patentable, but the primary examiner does not agree with the applicant's argument. D The Appeal Brief does not state whether the claims stand or fall together, but presents arguments why the claims subject to the same rejection are separately patentable. E The Appeal Brief does not address any of the grounds of rejection stated by the primary examiner, has no argument section, and is non-compliant as to 37 CFR 41.37.

C Related MPEP Chapter(s): MPEP 1200 - Appeal Answer Reasoning: ANSWER: (C) is the most correct answer. See MPEP § 1207. If the examiner disagrees with the reasons given, the reason for disagreement should be addressed in the Examiner's Answer. If the brief includes a statement that a grouping of claims does not stand or fall together but does not provide reasons, [the examiner is to] notify appellant of the non-compliance. If the examiner disagrees with appellant's statement in the brief that certain claims do not stand or fall together, the examiner explains in the examiner's answer why the claim grouping listed in the brief is not agreed with and why, if appropriate, e.g., the claims as listed by the appellant are not separately patentable. Answer (A) is incorrect. The Appeal Brief was filed less than seven months after the Notice of Appeal was filed. The applicant should be notified of the deficiency and provided an opportunity to request a five-month extension of time. Answer (B) is incorrect. See MPEP § 1205.03.. Answer (D) is incorrect. Where the applicant omits the statement required by 37 CFR § Part 41, yet presents arguments in the argument section of the brief, the applicant should be notified of the noncompliance and given time to correct the deficiency. See 37 CFR § Part 41; and MPEP § 1205.02. Answer (E) is incorrect. MPEP § 1205.03. Where the brief is non-compliant in some material way under 41.37, the applicant will be notified.

The primary examiner has rejected claims 1-10 under 35 USC 103 as being unpatentable over the Smith patent in view of the Jones reference. Appellant properly argues that there is no motivation or KSR rationale to combine the teachings of Smith and Jones. The examiner repeats the rejection of claims 1-10 as being "unpatentable over Smith in view of Jones." The examiner additionally cites a patent to Brown that was necessary to provide motivation for combining the teachings of Smith and Jones. The examiner does not list Brown in the statement of the rejection. Appellant timely appeals to the Board of Patent Appeals and Interferences, and files a proper appeal brief. The examiner files an examiner's answer addressing the rejection of claims 1-10 under 35 USC 103 as being unpatentable over Smith in view of Jones, and cites Brown in the argument as providing motivation to combine Smith and Jones. In accordance with the patent laws, rules and procedures as related in the MPEP, what will be the most proper decision of the Board? A The Board will affirm the rejection based on Smith and Jones only. B The Board will affirm the rejection based on Smith, Jones and Brown. C The Board will reverse the rejection based on Smith and Jones only. D The Board will reverse the rejection based on Smith, Jones and Brown. E None of the above.

C Related MPEP Chapter(s): MPEP 1200 - Appeal Answer Reasoning:ANSWER: (C) is the most correct answer. MPEP § 1207.03. If the claimed invention is rendered obvious by Smith in view of Jones and Brown, the statement of rejection must include all three references. Reliance on Brown to support the rejection is a different rejection from a rejection relying only on Smith in view of Jones. In accordance with MPEP § 1208.01, the Board will not consider the teachings of Brown because Brown was used to support the rejection, but was not listed in the statement of the rejection. As stated in MPEP § 1208.01, "Even if the prior art reference is cited to support the rejection in a minor capacity, it should be positively included in the statement of rejection. In re Hoch, 428 F.2d 1341, 1342 n.3, 166 USPQ 406, 407 n.3 (CCPA 1970)." Therefore, (B) and (D) are clearly wrong. (A) is incorrect. The decision cannot affirm the rejection since there is no motivation for combining the teachings of Smith and Jones absent the teachings of Brown. Therefore, the rejection must be reversed, not affirmed.

In accordance with USPTO rules and the procedures set forth in the MPEP, an amendment filed with or after a notice of appeal under, but before jurisdiction has passed to the Board of Patent Appeals and Interferences, should be entered by the primary examiner where the amendment: A requests unofficial consideration by the examiner. B is less than six pages long. C removes issues from appeal. D presents more specific claims, because it is believed that they may have a better chance of being allowable even though the claims do not adopt the examiner's suggestions. E introduces new issues, allowing the examiner to rethink his position.

C Related MPEP Chapter(s): MPEP 1200 - Appeal Answer Reasoning:ANSWER: (C) is the most correct answer. See 37 CFR § 1.116; MPEP § 1207, first paragraph. Answers (A), (B), and (D) are purely fictional. With respect to answer (E), see MPEP § 1207, first paragraph. Question should reference 37 CFR Part 41.

14. Mark Twine obtains a patent directed to a machine for manufacturing string. The patent contains a single claim (Claim 1) which recites six claim elements. The entire interest in Twine's patent is assigned to the S. Clemens String Co., and Twine is available and willing to cooperate with S. Clemens String Co. to file a reissue application. A subsequent reissue application includes Claim 2, which is similar to original Claim 1. However, one of the elements recited in Claim 2 is broader than its counterpart element in the original claim. The remaining five elements are narrower than their respective counterpart elements in the original patent claim. Which of the following scenarios accords with the USPTO rules and the procedures set forth in the MPEP? A The S. Clemens String Co. files the reissue application more than 2 years after the issue date of the original patent application. B The S. Clemens String Co. files the reissue application less than 2 years after the issue date of the original patent but more than 2 years after original application filing date. C Mark Twine files the reissue application less than 2 years after the issue date of the original patent but more than 2 years after original application filing date. D Mark Twine files the reissue application more than 2 years after the issue date of the original patent. E Mark Twine and the S. Clemens String Co. jointly file the reissue application more than 2 years after the issue date of the original patent.

C Related MPEP Chapter(s): MPEP 1400 - Correction of Patents Answer Reasoning:ANSWER: (C) is the most correct answer. Answers (A), (D) and (E) are incorrect because a broadening reissue application must be filed within two years of issuance of the original patent. 35 U.S.C. § 251; MPEP § 1412.03. Answer (B) is incorrect because the assignee may not file a broadening reissue application. MPEP § 706.03(x).

In accordance with the USPTO rules and the procedures of the MPEP, which of the following is true? A If after the filing of a reissue application no errors in the original patent are found, a reissue patent will be granted on the reissue application noting no change, and the original patent will be returned to the applicant. B In order to add matter not previously found in the patent, a continuation-in-part reissue application must be filed. C In a reissue application, additions and deletions to the original patent should be made by underlining and bracketing, respectively, except for changes made in prior Certificates of Correction and disclaimer(s) of claims under 37 CFR 1.321(a). D A dependent claim may be broadened in a reissue application only in the first two years of the enforceable life of the patent. E (A), (B), and (C).

C Related MPEP Chapter(s): MPEP 1400 - Correction of Patents Answer Reasoning:ANSWER: (C) is the most correct answer. See MPEP § 1411.01. As to (A) see MPEP § 1402. A reissue patent is not granted. As to (B), new matter may not be entered in a reissue. As to (D) see MPEP § 1412.03. Since (A), and (B) are incorrect, (E) is incorrect.

In accordance with the USPTO rules and the procedures set forth in the MPEP, impermissible recapture in an application exists ________________________ A if the limitation now being added in the present reissue was originally presented/argued/stated in the original application to make the claims allowable over a rejection or objection made in the original application. B if the limitation now being omitted or broadened in the present continuation was originally presented/argued/stated in a parent application to make the claims allowable over a rejection or objection made in the parent application. C if the limitation now being omitted or broadened in the present reissue was originally presented/argued/stated in the original application to make the claims allowable over a rejection or objection made in the original application. D if the limitation now being omitted or broadened in the present reissue was being broadened for the first time more than two years after the issuance of the original patent. E None of the above.

C Related MPEP Chapter(s): MPEP 1400 - Correction of Patents Answer Reasoning:ANSWER: (C) is the most correct. See MPEP § 1412.02, Recapture. As to (A), recapture occurs when the claim is broadened. Adding a limitation would narrow the claim. As to (B), recapture does not apply to continuations. As to (D), the two-year date relates to broadening reissue applications, not to the issue of recapture. 35 U.S.C. 251 prescribes a 2-year limit for filing applications for broadening reissues: "No reissue patent shall be granted enlarging the scope of the original patent unless applied for within two years from the grant of the original patent." (E) is incorrect because a (C) is correct.

Reexamination has been ordered following receipt of a request for reexamination of U.S. Patent X, filed by the patentee. Patent X contains independent claims 1 through 4, each directed to a hydrocyclone separator apparatus. They are the only claims that were ever presented during prosecution of the application that matured into Patent X. In the first Office action during reexamination, claims 1 through 4 are rejected as being obvious under 35 USC 103 over U.S. Patent Z. The apparatus is used for separating material, including fibers suspended in a liquid suspension, into a light fraction containing the fibers, and a heavy fraction containing rejects. Assume there are no issues under 35 USC 102, 103, or 112, and that any dependent claim is properly dependent. Recommend which of the following claims, if any, would be subject to rejection under 35 USC 305 for improperly enlarging the scope of the original claim in accordance with the patent laws, rules and procedures as related in the MPEP. A Claim 5. A hydrocyclone separator apparatus according to claim 4, wherein said blades are configured in the form of generally plane surfaces curved in one plane only. B Claim 5. A hydrocyclone separator apparatus according to claim 4, wherein the outlet duct is in the form of two frustro-conical portions joined at their narrow ends. C Claim 5. A method of separating material including fibers suspended in a liquid suspension comprising the steps of separating the material into a light fraction containing the fibers and a heavy fraction containing rejects, and converting the light fraction into a pulp and paper stock suspension. D Claim 5. A hydrocyclone separator apparatus according to claim 4, wherein the separator chamber is conical in shape having at the narrow end an outlet for the heavy fraction and at its wide end an outlet for the light fraction. E None of the above.

C Related MPEP Chapter(s): MPEP 1400 - Correction of Patents MPEP 2200 - Citation of Prior Art and Ex Parte Reexamination of Patents Answer Reasoning:ANSWER: (C). 35 U.S.C. § 305; MPEP §§ 2258 and 1412.03. MPEP § 2258, under the heading "Claims In Proceeding Must Not Enlarge Scope Of The Claims Of The Patent," states "[w]here new or amended claims are presented . . . the claims of the reexamination proceeding should be examined under 35 U.S.C. 305, to determine whether they enlarge the scope of the original claims. 35 U.S.C. 305 states that 'no proposed amended or new claim enlarging the scope of the claims of the patent will be permitted in a reexamination proceeding...'." Under the further subheading "Criteria for Enlargement of the Scope of the Claims," MPEP § 2258 states "A claim presented in a reexamination proceeding 'enlarges the scope' of the claims of the patent being reexamined where the claim is broader than each and every claim of the patent. See MPEP § 1412.03 for guidance as to when the presented claim is considered to be a broadening claim as compared with the claims of the patent, i.e., what is broadening and what is not. If a claim is considered to be a broadening claim for purposes of reissue, it is likewise considered to be a broadening claim in reexamination." MPEP § 1412.03, under the heading "New Category of Invention Added In Reissue - Broadening," states "[t]he addition of process claims as a new category of invention to be claimed in the patent (i.e., where there were no method claims present in the original patent) is generally considered as being a broadening of the invention. See Ex parte Wikdahl, 10 USPQ2d 1546, 1549 (Bd. Pat. App. & Inter. 1989)." MPEP 2258, under the further subheading "Rejection of Claims Where There Is Enlargement," states "[a]ny claim in a reexamination proceeding which enlarges the scope of the claims of the patent should be rejected under 35 U.S.C. 305." Since no claims drawn to a method were ever presented during prosecution of Patent X (claims 1 through 4 "are the only claims that were ever presented during prosecution of the application that matured into Patent X"), the claim recited in (C) is not directed to "the invention as claimed." (A), (B), and (D) are all incorrect because each of their claims are directed to a hydrocyclone separator apparatus, i.e., "the invention as claimed," and they do not enlarge the scope of the claims in Patent X. (E) is an incorrect answer because (C) is the correct answer.

On Thursday, February 6, 2015, applicant files an application for a design patent in Country X, which issues the patent on the filing date. In accordance with the patent laws, rules and the procedures as related in the MPEP, what is the last date applicant can file a U.S. design application to avoid any loss of patent rights? A Friday, February 6, 2016 (assume not a Federal holiday). B Thursday, February 5, 2016 (assume not a Federal holiday). C Wednesday, August 6, 2015. D Wednesday, May 6, 2015. E None of the above are correct.

C Related MPEP Chapter(s): MPEP 1500 - Design Patents Answer Reasoning: ANSWER: The correct answer is (C). See 35 U.S.C. § 172; MPEP § 1504.02. Registration of a design abroad is considered to be equivalent to patenting under 35 U.S.C. 119(a)-(d). In order for the filing to be timely for priority purposes, the U.S. design patent application must be made within 6 months of the foreign filing. (A) and (B) are incorrect because they are after the six month period. (D) is not correct because it is not the latest date for filing as required by the question. (E) is not correct because answer (C) is correct.

Which of the following statements relating to design patents does not accord with proper USPTO practice and procedure? A Both design and utility patents may be obtained on an article if the invention resides both in its utility and ornamental appearance. B The design for an article consists of the visual characteristics embodied in or applied to an article. C Design patent applications are included in the Patent Cooperation Treaty (PCT), and the procedures followed for PCT international applications are to be followed for design patent applications. D A claim directed to a computer-generated icon shown on a computer screen complies with the "article of manufacture" requirement of 35 USC 171. E A claimed design may encompass multiple articles or multiple parts within an article.

C Related MPEP Chapter(s): MPEP 1500 - Design Patents Answer Reasoning:ANSWER: (C) is the best choice because there is no provision for design patents under PCT. MPEP § 1502.01. (A) is a true statement. MPEP § 1502.01. (B) is a true statement. MPEP § 1502. (D) is a true statement. MPEP § 1504.01(a), I.A. (E) is a true statement. MPEP § 1504.01(b).

Applicant filed an international patent application under the Patent Cooperation Treaty (PCT) designating the United States. A copy of the international application has not been submitted to the USPTO by the International Bureau. The deadline for entering the national stage under 35 USC 371(c) was August 15, 2016. Applicant submitted all of the national stage items required by 35 USC 371(c) by facsimile transmission on August 15, 2016. The facsimile transmission was successfully received by the USPTO on August 15, 2016. The submission included an authorization to charge any required fees to the valid deposit account of the registered practitioner representing applicant. The account contained sufficient funds. Assuming that applicant has made no other national stage submissions under 35 USC 371(c), which of the following statements is most correctly describes why the national stage submission in accordance with the patent laws, rules and the procedures as related in the MPEP is proper or improper? A The national stage submission was proper because facsimile transmission is a valid method of correspondence in the USPTO. B The national stage submission was proper because a copy of an originally executed oath or declaration is acceptable, but the original oath or declaration should be retained as evidence of authenticity. C The national stage submission was improper because a copy of the international application and the basic national fee necessary to enter the national stage as required by 35 USC 371(c) may not be submitted by facsimile transmission. D The national stage submission was improper because the USPTO does not accept fee payments via facsimile transmission. E The national stage submission was improper because facsimile transmission may never be used for PCT applications.

C Related MPEP Chapter(s): MPEP 1800 - Patent Cooperation Treaty Answer Reasoning:ANSWER: (C) is the most correct answer. 37 §§ CFR 1.6(d)(3); 1.8(a)(2)(i)(F); 1.495(b); MPEP § 1893.01(a)(1), 2nd paragraph. The filing of the copy of the international application and the basic national fee in order to avoid abandonment under 37 CFR § 1.495(b), as appropriate, may not be transmitted by facsimile. See 37 CFR § 1.6(d)(3) and 37 CFR § 1.8(a)(2)(i)(F). (A) is not the most correct answer because facsimile transmission is not permitted in the situations set forth in 37 CFR § 1.6(d). (B) is not the most correct answer because even though an oath or declaration may be submitted by facsimile transmission as set forth in MPEP § 602, the national stage submission was improper for the reasons discussed in (C). (D) is not the most correct answer because (C) is the most complete answer. Facsimile transmissions may not be used to file a copy of the international application necessary to enter the national stage. (E) is not the most correct answer because facsimile transmission may be used to file certain correspondence in PCT applications. See MPEP § 1805.

Which of the following is patentable subject matter under 35 USC 101 in accordance with the patent laws, rules, and procedures as set forth in the MPEP? A A claim to a new mineral discovered in the earth or a new plant found in the wild. B A claim to a method of using a computer to select a set of arbitrary measurement point values. (The selected values are not to be transformed outside of the computer into computer data). C A claim to a method of controlling a mechanical robot which relies upon storing data in a computer that represents various types of mechanical movements of the robot. D A claim to a method of updating alarm limits by changing the number value of a variable to represent the result of the calculation. E A claim to a data structure per se. (The claim does not specify any location where the data structure is stored).

C Related MPEP Chapter(s): MPEP 2100 - Patentability Answer Reasoning:(C) is the most correct answer. This is merely a method of controlling a machine. (A) is not patentable inasmuch as it is naturally occurring and is therefore an incorrect answer. As set forth in MPEP § 2105 a " new mineral discovered in the earth or a new plant found in the wild is not patentable subject matter." (B) is an incorrect answer. Merely selecting a set of arbitrary measurement point values ([In re] Sarkar, 588 F.2d [1330] at 1331, 200 USPQ [132] at 135) does not create a basis for patent eligibility. (D) is an incorrect answer. The step of 'updating alarm limits' found to constitute changing the number value of a variable to represent the result of the calculation (Parker v. Flook, 437 U.S.584, 585, 198 USPQ 193, 195 (1978)." (E) A claim to a data structure has been per se held nonstatutory. This is consistent with Bilski guidelines. See generally MPEP 2105-2106.

An applicant submits a product-by-process claim to a shoe made by a series of specific process steps. The claim is rejected over a publication under 35 USC 102 and 103. Assume for this question that the publication reasonably appears to show the identical shoe, but describes a different method of making the shoe. What is the proper procedure to try to overcome the rejection in accordance with the patent laws, rules and the procedures as related in the MPEP? A Argue that all limitations in the claim must be given weight and that rejection must be withdrawn because the reference does not disclose the claimed method of making steps. B Argue that the examiner has not carried the burden of proving that the shoes are identical. C Present evidence why the steps of the claimed process produce a patentably different structure. D Submit a declaration under 37 CFR 1.132 by the author of the publication describing in more detail how the shoe in the publication was made by a different method. E Argue that the inventor was not aware of the publication when the invention was made.

C Related MPEP Chapter(s): MPEP 2100 - Patentability Answer Reasoning:ANSWER: (C) is correct. MPEP § 2113, under the heading "Once A Product Appearing To Be Substantially Identical Is Found And A 35 U.S.C. 102/103 Rejection Made, The Burden Shifts To The Applicant To Show An Unobvious Difference," states "[o]nce the examiner provides a rationale tending to show that the claimed product appears to be the same or similar to that of the prior art, although produced by a different process, the burden shifts to applicant to come forward with evidence establishing an unobvious difference between the claimed product and the prior art product. In re Marosi, 710 F.2d 798, 802, 218 USPQ 289, 292 (Fed. Cir. 1983)." (A) is incorrect because the patentability of product-by-process claims is based on the product itself. See In re Thorpe, 227 USPQ 964, 966 (Fed. Cir. 1985). Where the end products are the same, the process of making limitations do not have to be given weight in ex parte examination. See Atlantic Thermoplastics Co. v. Faytex Corp., 23 USPQ2d 1481, 1490-91 (Fed. Cir. 1992) (product-by-process claims are treated differently for patentability purposes during ex parte examination in the USPTO than for infringement and validity purposes during litigation). (B) is incorrect because "[o]nce the Examiner provides a rationale tending to show that the claimed product appears to be the same or similar to that of the prior art, although produced by a different process, the burden shifts to applicant to come forward with evidence establishing an unobvious difference between the claimed product and the prior art product." MPEP § 2113. "To the extent that the process limitations distinguish the products over the prior art, they must be given the same consideration as traditional product characteristics." In re Hallman, 210 USPQ 609, 611 (CCPA 1981). Thus, (C) is correct because applicant can show by factual arguments and/or declarations or affidavits under 37 CFR § 1.132 that the .method of making produces a different product and that the differences are unobvious. (D) is incorrect because it does not tend to show that the products are different. (E) is incorrect because the inventor's awareness of prior art is of no consequence to patentability.

A primary examiner is examining a patent application. The application includes a specification and a single claim to the invention that reads as follows: 1. A building material to be used as an alternative to brick in the construction of a house, said building material comprising compressed refuse, the majority of which is wood. In the specification, the inventor explains that the wood to be used in the inventive building material should be balsa wood. According to the specification, balsa-containing building material has the advantage of being lighter than brick. In a first Office action mailed to the registered practitioner representing the inventor the single claim was rejected as anticipated under 35 U.S.C. 102 over Patent A. Patent A issued more than one year before the effective filing date of the application, and teaches a building material to be used as an alternative to brick in the construction of a house comprising compressed refuse, the majority of which is pine. The practitioner replies to the first Office action by arguing that the invention is different from that of Patent A. According to the practitioner, the inventor uses balsa wood, not pine. The claim has not been amended. Which of the following describes how the examiner should proceed in accordance with the patent laws, rules and procedures as related in the MPEP? A The examiner should allow the claim. B The examiner should allow the claim only after including a Reasons for Allowance pointing out that the inventor argues that her invention is directed to using balsa wood, not pine. C The examiner should issue a Final Rejection again rejecting the claim as anticipated under 35 USC102 over Patent A. D The examiner should reopen prosecution and begin anew, this time searching for a reference that shows a building material containing balsa wood. E The examiner should withdraw the rejection but issue a new Office action this time rejecting the claim under 35 USC 112, second paragraph, because the claim is broad enough to encompass using pine.

C Related MPEP Chapter(s): MPEP 2100 - Patentability Answer Reasoning:ANSWER: (C) is the best answer. 35 U.S.C. § 102; MPEP §§ 2111 and 2131. MPEP § 2131, under the heading, "To Anticipate A Claim, The Reference Must Teach Every Element Of The Claim." "A claim is anticipated only if each and every element as set forth in the claim is found, either expressly or inherently described, in a single prior art reference." See Verdegaal Bros. v. Union Oil Co. of California, 814 F.2d 628, 631, 2 USPQ2d 1051, 1053 (Fed. Cir. 1987). Here, every element of the claim is found in Patent A. See MPEP 2111, under the heading "Claims Must Be Given Their Broadest Reasonable Interpretation," where it explained that "[d]uring patent examination, the pending claims must be 'given the broadest reasonable interpretation consistent with the specification," and cites In re Prater, 415 F.2d 1393, 1404-05, 162 USPQ 541, 550-51 (CCPA 1969) to explain that "reading a claim in light of the specification, to thereby interpret limitations explicitly recited in the claim, is a quite different thing from 'reading limitations of the specification into a claim,' to thereby narrow the scope of the claim by implicitly adding disclosed limitations which have no express basis in the claim." (A) and (B) are incorrect. MPEP § 2111. The claim, as written, is not allowable over Patent A since Patent A disclosed every element recited in the claim. (D) is incorrect. There is no need to search again for a building material, this time looking for balsa wood. The claim has not been amended to be directed to balsa wood. Since it still broadly recites "wood," Patent A that discloses pine remains germane and anticipates the claim. (E) is incorrect for one or more reasons. It is incorrect because it wrongly agrees with the practitioner's argument that Patent A is poor reference. It also is incorrect because it seeks to reject the claim over 35 U.S.C. § 112, second paragraph, for indefiniteness. The claim is clear on its face and there is nothing indefinite about what the claim says. This answer makes the mistake of confusing breadth with indefiniteness. The claim is broad but it is definite.

Which of the following is not in accordance with Office practice under 35 USC 42? A The Director may refund any fee paid by mistake or any amount paid in excess of that required. B A change of purpose after the payment of a fee, such as when a party desires to withdraw a patent for which the fee was paid, including an application, an appeal or a request for an oral hearing, will not entitle a party to a refund of such fee. C Even if an applicant specifically requests a refund, the Office will not refund amounts of twenty-five dollars or less. D Any refund of fee paid by credit card will be by a credit to the credit card account to which the fee was charged. E If an applicant mistakenly files an application, the filing fee is not considered a fee paid by mistake.

C Related MPEP Chapter(s): MPEP 600 - Parts, Form, and Content of Application Answer Reasoning:ANSWER: (C) is incorrect as the Office will refund amounts of twenty-five dollars or less if requested to do so by the applicant. See MPEP § 607.02 at p. 600-51. As to (A), (B), (D) and (E), see MPEP § 607.02.

On January 2, 2017, a registered practitioner filed a patent application with the USPTO for inventor Bloc. The application includes a specification and a single claim to the invention which reads as follows: 1. Compound Y. In the specification, Bloc explains that compound Y is an intermediate in the chemical manufacture of synthetic Z. With respect to synthetic Z, the specification discloses its structural formula and further states that synthetic Z is modeled on the natural form of Z to give it the same therapeutic ability to alleviate pain. The specification goes on to state that synthetic Z is also a cure for cancer. On June 2, 2017, the practitioner received an Office action from the primary examiner rejecting the claim. The claim is rejected under 35 U.S.C. 101 as being inoperative; that is, the synthetic Z does not operate to produce a cure for cancer (i.e., incredible utility). Bloc believes he is entitled to a patent to his compound Y. In accordance with the patent laws, rules and procedures as related in the MPEP, how best should the practitioner reply to the rejection of the claim? A Advise Bloc that he should give up because a cure for cancer is indeed incredible and is unproven. B File a reply arguing that a cure for cancer is not incredible and he can prove it if given the chance. C File a reply arguing that whether or not a cure for cancer is incredible is superfluous since Bloc has disclosed another utility - alleviating pain, which is not incredible. D File a reply arguing that the claim is directed to compound Y, not synthetic Z. E File a reply arguing that synthetic Z is modeled on the natural form of Z.

C Related MPEP Chapter(s): MPEP 2100 - Patentability Answer Reasoning:ANSWER: (C) is the best answer. MPEP §§ 2107.01 and 2107.02. MPEP § 2107.01, under the heading "Therapeutic or Pharmacological Utility," cites In re Chilowsky, 229 F.2d 457, 461-2, 108 USPQ 321, 325 (CCPA 1956); In re Gazave, 379 F.2d 973, 978, 154 USPQ 92, 96 (CCPA 1967); and Nelson v. Bowler, 626 F.2d 853, 856, 206 USPQ 881, 883 (CCPA 1980) as taking the position that "[i]nventions asserted to have utility in the treatment of human or animal disorders are subject to the same legal requirements for utility as inventions in any other field of technology." MPEP § 2107.02, under the heading "The Claimed Invention Is The Focus Of The Utility Requirement," states ". . . regardless of the category of invention that is claimed (e.g., product or process), an applicant need only make one credible assertion of specific utility for the claimed invention to satisfy 35 U.S.C. 101 and 35 U.S.C. 112; additional statements of utility, even if not "credible," do not render the claimed invention lacking in utility. See, e.g., . . . In re Gottlieb, 328 F.2d 1016, 1019, 140 USPQ 665, 668 (CCPA 1964) ('Having found that the antibiotic is useful for some purpose, it becomes unnecessary to decide whether it is in fact useful for the other purposes 'indicated' in the specification as possibly useful.')." The issue is whether Mr. Bloc has disclosed a specific utility for the claimed compound Y sufficient to satisfy the practical utility requirement of 35 U.S.C § 101. According to the set of facts, we know that compound Y is an intermediate in the chemical manufacture of synthetic Z. We are given two utilities for synthetic Z: 1) alleviating pain, a utility it shares with the natural form of Z; and, 2) curing cancer. The examiner focuses on the disclosure that synthetic Z is a cure for cancer. Even if one were to agree that synthetic Z's ability to cure cancer amounts to an incredible utility, a claim to the intermediate compound Y would not run afoul of the utility requirement of 35 U.S.C. § 101 where another substantial, credible and specific utility is alternatively demonstrated. Here, the specification discloses that synthetic Z, like the natural form of Z, alleviates pain. The alleviation of pain is another substantial, credible and specific utility and serves to give compound Y an alternative utility to that of being used to make a cancer-curing substance. An applicant need not show that all disclosed utilities are credible. An applicant need only show that one of the disclosed utilities is in fact credible. In re Gottlieb, supra. The establishment of a credible, substantial and specific utility renders the disclosure of an additional incredible utility superfluous, and therefore ultimately irrelevant. Accordingly, Mr. Bloc's best course of action is to make the argument that he has disclosed another substantial, credible, and specific utility, notwithstanding the disclosure of curing cancer. (A) is not the most correct answer. The advice could prevent him from getting a patent to which he may be entitled. (B) is not the most correct answer. A cure for cancer is ostensibly incredible. It is hardly a response to the examiner's rejection to ask for the chance to prove one can cure cancer. (D) is not the most correct answer. While it is true that the utility requirement is addressed to the claimed invention, which here is compound Y not synthetic Z, it is not enough to respond by repeating what the invention is but, rather, to show that the invention has indeed a substantial, credible, and specific utility. Whatever is claimed as the invention, it must comply with the utility requirement of 35 U.S.C. § 101. Here the examiner states that the claim does not comply, as evidenced by the incredible utility of the final product. It is Mr. Bloc's responsibility to then show that compound Y does comply with 35 U.S.C. § 101 by showing that its end product has a substantial, credible, and specific utility. (E) is not the most correct answer. Noting that synthetic Z is modeled on natural Z does not go far enough in establishing a substantial, credible and specific utility for compound Y. It is synthetic Z's therapeutic ability to alleviate pain which establishes the necessary alternative utility.

On January 2, 2016, a registered practitioner filed a patent application with the USPTO for inventor Bloc. The application includes a specification and a single claim to the invention which reads as follows: 1. Compound Y. In the specification, Bloc explains that compound Y is an intermediate in the chemical manufacture of synthetic Z. With respect to synthetic Z, the specification discloses its structural formula and further states that synthetic Z is modeled on the natural form of Z to give it the same therapeutic ability to alleviate pain. The specification goes on to state that synthetic Z is also a cure for cancer. On June 2, 2016, the practitioner received an Office action from the primary examiner rejecting the claim. The claim is rejected under 35 USC 101 as being inoperative; that is, the synthetic Z does not operate to produce a cure for cancer (i.e., incredible utility). Bloc believes he is entitled to a patent to his compound Y. In accordance with USPTO rules and procedures set forth in the MPEP, how best should the practitioner reply to the rejection of the claim? A Advise Bloc that he should give up because a cure for cancer is indeed incredible and is unproven. B File a reply arguing that a cure for cancer is not incredible and he can prove it if given the chance. C File a reply arguing that whether or not a cure for cancer is incredible is superfluous since Bloc has disclosed another utility - alleviating pain, which is not incredible. D File a reply arguing that the claim is directed to compound Y, not synthetic Z. E File a reply arguing that synthetic Z is modeled on the natural form of Z.

C Related MPEP Chapter(s): MPEP 2100 - Patentability Answer Reasoning:ANSWER: (C) is the best answer. MPEP §§ 2107.01 and 2107.02. MPEP § 2107.01, under the heading "Therapeutic or Pharmacological Utility," cites In re Chilowsky, 229 F.2d 457, 461-2, 108 USPQ 321, 325 (CCPA 1956); In re Gazave, 379 F.2d 973, 978, 154 USPQ 92, 96 (CCPA 1967); and Nelson v. Bowler, 626 F.2d 853, 856, 206 USPQ 881, 883 (CCPA 1980) as taking the position that "[i]nventions asserted to have utility in the treatment of human or animal disorders are subject to the same legal requirements for utility as inventions in any other field of technology." MPEP § 2107.02, under the heading "The Claimed Invention Is The Focus Of The Utility Requirement," states "...regardless of the category of invention that is claimed (e.g., product or process), an applicant need only make one credible assertion of specific utility for the claimed invention to satisfy 35 U.S.C. 101 and 35 U.S.C. 112; additional statements of utility, even if not "credible," do not render the claimed invention lacking in utility. See, e.g.,...In re Gottlieb, 328 F.2d 1016, 1019, 140 USPQ 665, 668 (CCPA 1964) ('Having found that the antibiotic is useful for some purpose, it becomes unnecessary to decide whether it is in fact useful for the other purposes 'indicated' in the specification as possibly useful.')." The issue is whether Mr. Bloc has disclosed a specific utility for the claimed compound Y sufficient to satisfy the practical utility requirement of 35 U.S.C § 101. According to the set of facts, we know that compound Y is an intermediate in the chemical manufacture of synthetic Z. We are given two utilities for synthetic Z: 1) alleviating pain, a utility it shares with the natural form of Z; and, 2) curing cancer. The examiner focuses on the disclosure that synthetic Z is a cure for cancer. Even if one were to agree that synthetic Z's ability to cure cancer amounts to an incredible utility, a claim to the intermediate compound Y would not run afoul of the utility requirement of 35 U.S.C. § 101 where another substantial, credible and specific utility is alternatively demonstrated. Here, the specification discloses that synthetic Z, like the natural form of Z, alleviates pain. The alleviation of pain is another substantial, credible and specific utility and serves to give compound Y an alternative utility to that of being used to make a cancer-curing substance. An applicant need not show that all disclosed utilities are credible. An applicant need only show that one of the disclosed utilities is in fact credible. In re Gottlieb, supra. The establishment of a credible, substantial and specific utility renders the disclosure of an additional incredible utility superfluous, and therefore ultimately irrelevant. Accordingly, Mr. Bloc's best course of action is to make the argument that he has disclosed another substantial, credible, and specific utility, notwithstanding the disclosure of curing cancer. (A) is not the most correct answer. The advice could prevent him from getting a patent to which he may be entitled. (B) is not the most correct answer. A cure for cancer is ostensibly incredible. It is hardly a response to the examiner's rejection to ask for the chance to prove one can cure cancer. (D) is not the most correct answer. While it is true that the utility requirement is addressed to the claimed invention, which here is compound Y not synthetic Z, it is not enough to respond by repeating what the invention is but, rather, to show that the invention has indeed a substantial, credible, and specific utility. Whatever is claimed as the invention, it must comply with the utility requirement of 35 U.S.C. § 101. Here the examiner states that the claim does not comply, as evidenced by the incredible utility of the final product. It is Mr. Bloc's responsibility to then show that compound Y does comply with 35 U.S.C. § 101. (E) is not the most correct answer. Noting that synthetic Z is modeled on natural Z does not go far enough in establishing a substantial, credible and specific utility for compound Y. It is synthetic Z's therapeutic ability to alleviate pain which establishes the necessary alternative utility.

The following facts apply: Claims 1 and 2, fully disclosed and supported in the specification of a patent application having an effective filing date of March 15, 2000, for sole inventor Ted, state the following: Claim 1. An apparatus intended to be used for aerating water in a fish tank, comprising: (i) an oxygen source connected to a tube, and (ii) a valve connected to the tube. Claim 2. An apparatus as in claim 1, further comprising an oxygen sensor connected to the valve. Which of the following, if relied on by an examiner in a rejection of claim 2, can be a statutory bar under 35 USC 102 of claim 2? A A U.S. patent to John, issued February 2, 1999, that discloses and claims an apparatus intended to be used for aerating ice cream, having an oxygen source connected to a tube, a valve connected to the tube, and a battery coupled to the oxygen source. B A U.S. patent to John, issued April 6, 1999, that discloses and claims an apparatus intended to be used for aerating water in a fish tank, having an oxygen source connected to a tube, a valve connected to the tube, and an oxygen sensor connected to the tube. C A U.S. patent to Ned, issued February 9, 1999, that discloses, but does not claim, an apparatus intended to be used for aerating ice cream, having an oxygen source connected to a tube, a valve connected to the tube, an oxygen sensor connected to the valve, and a battery coupled to the oxygen source. D A foreign patent to Ted issued April 12, 2000, on an application filed on March 12, 1997. The foreign patent discloses and claims an apparatus intended to be used for aerating water in a fish tank, having an oxygen source connected to a tube, a valve connected to the tube, and an oxygen sensor connected to the tube. E None of the above.

C Related MPEP Chapter(s): MPEP 2100 - Patentability Answer Reasoning:ANSWER: (C) is the correct answer. 35 U.S.C. § 102(b). MPEP § 2111.02 provides that the preamble generally is not accorded patentable weight where it merely recites the intended use of a structure. (A) is incorrect because it does not disclose an oxygen sensor. (B) is incorrect because the patent is not more than one year prior to the date of the Ted's application. (D) is incorrect because the Japanese patent application issued after the date of Ted's application. 35 U.S.C. § 102(d). (E) is incorrect because (C) is correct.

In accordance with the patent laws, rules and procedures as related in the MPEP, which of the following is patentable subject matter under 35 USC 101? A A novel and unobvious abstract idea. B A previously undiscovered law of nature. C A billing process containing mathematical algorithms producing a written invoice. D A novel and unobvious discovery of a physical phenomenon. E All of the above.

C Related MPEP Chapter(s): MPEP 2100 - Patentability Answer Reasoning:ANSWER: (C) is the most correct answer. 35 U.S.C. § 101; MPEP 2106, under the heading "A. Identify and Understand Any Practical Application Asserted for the Invention." With regard to computer-related inventions, MPEP § 2106 states that "[a]lthough the courts have yet to define the terms useful, concrete, and tangible in the context of the practical application requirement for such inventions, the following example illustrates claimed inventions that have a practical application because they produce useful, concrete, and tangible results: 'Claims drawn to a long-distance telephone billing process containing mathematical algorithms were held to be directed to patentable subject matter because the claimed process applies the Boolean principle to produce a useful, concrete, tangible result without pre-empting other uses of the mathematical principle.' AT&T Corp. v. Excel Communications, Inc., 172 F.3d 1352, 1358, 50 USPQ2d 1447, 1452 (Fed. Cir. 1999)." See also, State Street Bank & Trust Co. v. Signature Financial Group Inc., 149 F. 3d 1368, 1374, 47 USPQ2d 1596, 1601-02 (Fed. Cir. 1998). Answers (A), (B) and (D) are incorrect. MPEP § 2105 states that abstract ideas, laws of nature and physical phenomena have been held by the Supreme Court to be unpatentable subject matter under 35 U.S.C. § 101 in accord with Bilski guidelines. Answer (E) is incorrect because answers (A), (B) and (D) are incorrect.

To rely in a rejection under pre-AIA 35 USC 102(a) on an invention that is known or publicly used in accordance with patent laws, rules and procedures as related in the MPEP, the invention: A must be known or used in NAFTA or WTO member countries. B must be known or used in a NAFTA member country, but only if the filing date of the application is after the effective date of the North American Free Trade Agreement Implementation Act. C must be known or used in this country. D can be known or used in any country. E must be known or used in a WTO member country, but only if the filing date of the application is after the effective date of the implementation of the Uruguay Round (WTO) Agreements Act.

C Related MPEP Chapter(s): MPEP 2100 - Patentability Answer Reasoning:ANSWER: (C) is the most correct answer. 35 U.S.C. § 102(a); MPEP § 2132. As set forth in MPEP § 2132, under the heading "II. 'In This Country,'" subheading "Only Knowledge or Use In The U.S. Can Be Used in a 35 U.S.C. 102(a) Rejection," states "[t]he knowledge or use relied on in a 35 U.S.C. 102(a) rejection must be knowledge or use 'in this country.' Prior knowledge or use which is not present in the United States, even if widespread in a foreign country, cannot be the basis of a rejection under 35 U.S.C. 102(a). In re Ekenstam, 256 F.2d 321, 118 USPQ 349 (CCPA 1958). Note that the changes made to 35 U.S.C.104 by NAFTA (Public Law 103-182) and Uruguay Round Agreements Act (Public Law 103-465) do not modify the meaning of 'in this country' as used in 35 U.S.C. 102(a) and thus 'in this country" still means in the United States for purposes of 35 U.S.C. 102(a) rejections.'" See also MPEP § 706.02(c), "[t]he language 'in this country' means in the United States only and does not include other WTO or NAFTA member countries." Since "in this country" means in the United States for purposes of 35 U.S.C. § 102(a) rejections, (A), (B), (D) and (E) are incorrect.

Buddy is a recent father and a machinist at a local machine shop. One day while driving to work, Buddy conceived an idea for an improved baby stroller. He quickly worked out many of the details of how to build such an improved stroller, but he still had questions. Buddy later explained his idea to his employer and showed the employer detailed preliminary drawings of the stroller without any agreement as to confidentiality. Buddy wanted use of his employer's machine shop to build a model. Buddy's employer was also excited about the stroller idea and its commercial potential, and the two quickly reached an oral agreement. Buddy would have free use of the machine shop equipment and supplies after regular business hours to work on his model. In exchange, Buddy agreed to assign any patent rights in his invention to the employer for $1,000.00. Only Buddy and, occasionally, his employer were ever present in the shop when Buddy was working on the stroller. Buddy finalized the design just over a year later, and a nonprovisional patent application was on file within a month of finalization along with a recently executed written assignment of the rights in the invention to Buddy's employer. During prosecution of the patent application, the examiner learned of the oral agreement between Buddy and his employer, and rejected the claims on the basis that the invention was on sale more than one year before the application filing date. Determine which of the following would provide the most reasonable basis for traversing the rejection in accordance with the patent laws, rules and procedures as related in the MPEP. A The examiner cannot properly make the rejection because it is not based on prior art patents or printed publications. B The oral agreement was a private transaction between Buddy and his employer and no private transaction can provide a basis for an on-sale bar. C An assignment or sale of the rights in an invention and potential patent rights is not a sale of "the invention" that would operate as a bar to patentability under 35 USC 102. D There can be no on-sale bar even though there was no express requirement of confidentiality because no one other than Buddy's employer was present in the shop when Buddy was working on the stroller and the oral agreement was not public. E Although the oral agreement to assign the patent to Buddy's employer was made more than a year before the filing date, the written assignment was less than a year before the filing date, and under the Statute of Frauds, sales for more than $500.00 require a written agreement. A rejection based on the on-sale bar can never be made unless there is an actual sale.

C Related MPEP Chapter(s): MPEP 2100 - Patentability Answer Reasoning:ANSWER: (C) is the most correct answer. 35 U.S.C. § 102(b); MPEP § 2133.03(b). MPEP § 2133.03(b), under the heading "I. The Meaning Of "Sale," and subheading "D. A Sale of Rights Is Not a Sale of the Invention and Will Not in Itself Bar a Patent," states "'[a]n assignment or sale of the rights, such as patent rights, in the invention is not a sale of 'the invention' within the meaning of section 102(b).'" The sale must involve the delivery of the physical invention itself. Moleculon Research Corp. v. CBS, Inc., 793 F.2d 1261, 1265, 229 USPQ 805, 809 (Fed. Cir. 1986)." (A) is incorrect. Although reexaminations are limited to prior art patents and printed publications, that limitation is not present in original prosecution. MPEP § 2133.03(b) states "An impermissible sale has occurred if there was a definite sale, or offer to sell, more than 1 year before the effective filing date of the U.S. application and the subject matter of the sale, or offer to sell, fully anticipated the claimed invention or would have rendered the claimed invention obvious by its addition to the prior art. Ferag AG v. Quipp, Inc., 45 F.3d 1562, 1565, 33 USPQ2d 1512, 1514 (Fed. Cir. 1995)." (B) and (D) are incorrect. There is no requirement that on-sale activity be public. See MPEP § 2133.03(b), under the heading "III. Sale By Inventor, Assignee Or Others Associated With The Inventor In The Course Of Business," and subheading "A. Sale Activity Need Not Be Public." (E) is wrong at least because an on-sale bar does not require an actual sale. A bar can also be based on an offer to sell. MPEP § 2133.03(b), under the heading "II. Offers For Sale."

A claim in an application recites "[a] composition containing: (a) 35-55% polypropylene; and (b) 45-65% polyethylene." The sole prior art reference describes, as the only relevant disclosure, a composition containing 34.9% polypropylene and 65.1% polyethylene. In accordance with USPTO rules and procedures set forth in the MPEP, the primary examiner should properly: A Indicate the claim allowable over the prior art because there is no teaching, motivation or suggestion to increase the amount of polypropylene from 34.9% to 35% and decrease the amount of polyethylene from 65.1% to 65%. B Reject the claim under 35 USC 102 as anticipated by the prior art reference. C Reject the claim under 35 USC 103 as obvious over the prior art reference. D Reject the claim alternatively under 35 USC 102 as anticipated by or under 35 USC 103 as obvious over the prior art reference. E None of the above.

C Related MPEP Chapter(s): MPEP 2100 - Patentability Answer Reasoning:ANSWER: (C) is the most correct answer. A prima facie case of obviousness exists where the claimed ranges and the prior art are close enough that one of ordinary skill in the art would have expected them to have the same properties. See MPEP § 2144.05. In Titanium Metals Corp. v. Banner, 778 F.2d 775, 783, 227 USPQ 773, 779 (Fed. Cir. 1985), a claim recited a titanium base alloy consisting essentially of 0.8% nickel, 0.3% molybdenum, up to 0.1% maximum iron, and the balance titanium. A prior art reference described two similar alloys: (i) one with 0.25% molybdenum, 0.75% nickel, and balance titanium; and (ii) another with 0.31% molybdenum, 0.94% nickel, and balance titanium. The court held: As admitted by appellee's affidavit evidence from James A. Hall, the Russian article discloses two alloys having compositions very close to that of claim 3, which is 0.3% Mo and 0.8% Ni, balance titanium. The two alloys in the prior art have 0.25% Mo-0.75% Ni and 0.31% Mo-0.94% Ni, respectively. The proportions are so close that prima facie one skilled in the art would have expected them to have the same properties. Appellee produced no evidence to rebut that prima facie case. The specific alloy of claim 3 must therefore be considered to have been obvious from known alloys. Id. Thus, (A) is incorrect. (B) and (D) are incorrect because a claim is anticipated by a prior art reference only when the prior art discloses, either expressly or inherently, every limitation of the claimed invention. (E) is incorrect because (C) is correct.

In accordance with the USPTO rules and the procedures set forth in the MPEP, which of the following is true? A A claim to a process omitting a step in a disclosed process, where the step is disclosed in the specification to be essential to the invention, may not be properly rejected under 35 USC 112, first paragraph, for lack of enablement where the specification provides an enabling disclosure only for the process which includes the essential step. B Failure to disclose the best mode must rise to the level of active concealment or grossly inequitable conduct in order to support a rejection under 35 USC 112, first paragraph. C A claim failing to interrelate essential elements of the invention, as defined by the applicant in the specification, where the interrelation is critical to the invention may be properly rejected under 35 USC 112, second paragraph, for failure to properly point out and distinctly claim the invention. D Where the best mode contemplated by the inventor at the time of filing the application is not disclosed, a proposed amendment adding a specific mode of practicing the invention would not be new matter. E The best mode requirement is the same as the enablement requirement of the first paragraph of 35 USC 112.

C Related MPEP Chapter(s): MPEP 2100 - Patentability Answer Reasoning:ANSWER: (C) is the most correct answer. As stated in MPEP § 2172.01, "a claim which fails to interrelate essential elements of the invention as defined by applicant(s) in the specification may be rejected under 35 U.S.C. 112, second paragraph, for failure to point out and distinctly claim the invention. See In re Venezia, 530 F.2d 956, 189 USPQ 149 (CCPA 1976); In re Collier, 397 F.2d 1003, 158 USPQ 266 (CCPA 1968)." (A) is incorrect. As stated in MPEP § 2172.01, "A claim which omits matter disclosed to be essential to the invention as described in the specification or in other statements of record may be rejected under 35 U.S.C. § 112, first paragraph, as not enabling. In re Mayhew, 527 F.2d 1229, 188 USPQ 356 (CCPA 1976)"; MPEP § 2164.08(c). (B) is incorrect. As stated in MPEP § 2165, "Failure to disclose the best mode need not rise to the level of active concealment or grossly inequitable conduct in order to support a rejection or invalidate a patent. Where an inventor knows of a specific material that will make possible the successful reproduction of the effects claimed by the patent, but does not disclose it, speaking instead in terms of broad categories, the best mode requirement has not been satisfied. Union Carbide Corp. v. Borg - Warner, 550 F.2d 555, 193 USPQ 1 (6th Cir. 1977)." (D) is incorrect. MPEP § 2165.01, under the heading "Defect In Best Mode Cannot Be Cured By New Matter," indicates that if there is no disclosure of the best mode contemplated by the inventor at the time the application is filed, such a defect cannot be cured by submitting an amendment seeking to put into the specification something required to be there when the patent application was originally filed. In re Hay, 534 F.2d 917, 189 USPQ 790 (CCPA 1976). Any proposed amendment of this type should be treated as new matter. MPEP § 2165.01. (E) is incorrect. As stated in MPEP § 2165.02, "The best mode requirement is a separate and distinct requirement from the enablement requirement of the first paragraph of 35 U.S.C. § 112. In re Newton, 414 F.2d 1400, 163 USPQ 34 (CCPA 1969).

Which of the following is true? A A claim to a process omitting a step in a disclosed process, where the step is disclosed in the specification to be essential to the invention, may not be properly rejected under 35 USC 112, first paragraph, for lack of enablement where the specification provides an enabling disclosure only for the process which includes the essential step. B Failure to disclose the best mode must rise to the level of active concealment or grossly inequitable conduct in order to support a rejection under 35 USC 112, first paragraph. C A claim failing to interrelate essential elements of the invention, as defined by the applicant in the specification, where the interrelation is critical to the invention may be properly rejected under 35 USC 112, second paragraph, for failure to properly point out and distinctly claim the invention. D Where the best mode contemplated by the inventor at the time of filing the application is not disclosed, a proposed amendment adding a specific mode of practicing the invention would not be new matter. E The best mode requirement is the same as the enablement requirement of the first paragraph of 35 USC 112.

C Related MPEP Chapter(s): MPEP 2100 - Patentability Answer Reasoning:ANSWER: (C) is the most correct answer. As stated in MPEP § 2172.01, "a claim which fails to interrelate essential elements of the invention as defined by applicant(s) in the specification may be rejected under 35 U.S.C. § 112, second paragraph, for failure to point out and distinctly claim the invention. See In re Venezia, 530 F.2d 956, 189 USPQ 149 (CCPA 1976); In re Collier, 397 F.2d 1003, 158 USPQ 266 (CCPA 1968)." (A) is incorrect. As stated in MPEP § 2172.01, "A claim which omits matter disclosed to be essential to the invention as described in the specification or in other statements of record may be rejected under 35 U.S.C. § 112, first paragraph, as not enabling. In re Mayhew, 527 F.2d 1229, 188 USPQ 356 (CCPA 1976)"; MPEP § 2164.08(c). (B) is incorrect. As stated in MPEP § 2165, "Failure to disclose the best mode need not rise to the level of active concealment or grossly inequitable conduct in order to support a rejection or invalidate a patent. Where an inventor knows of a specific material that will make possible the successful reproduction of the effects claimed by the patent, but does not disclose it, speaking instead in terms of broad categories, the best mode requirement has not been satisfied. Union Carbide Corp. v. Borg - Warner, 550 F.2d 555, 193 USPQ 1 (6th Cir. 1977)." (D) is incorrect. MPEP § 2165.01, part V indicates that if there is no disclosure of the best mode contemplated by the inventor at the time the application is filed, such a defect cannot be cured by submitting an amendment seeking to put into the specification something required to be there when the patent application was originally filed. In re Hay, 534 F.2d 917, 189 USPQ 790 (CCPA 1976). Any proposed amendment of this type should be treated as new matter. MPEP § 2165.01. (E) is incorrect. As stated in MPEP § 2165.02, "The best mode requirement is a separate and distinct requirement from the enablement requirement of the first paragraph of 35 U.S.C. § 112. In re Newton, 414 F.2d 1400, 163 USPQ 34 (CCPA 1969).

In accordance with the patent laws, rules and procedures as related in the MPEP, satisfaction of the written description requirement may not be demonstrated by: A including in the specification a description of an actual reduction to practice. B describing the claimed invention with all of its limitations using such descriptive means as words, structures, figures, diagrams, and formulas that fully set forth the claimed invention. C describing during prosecution of a new or amended claim an element or limitation (omitted from the original disclosure in the specification) as an essential or critical feature of the invention. D including in the specification a description of distinguishing identifying characteristics sufficient to show that the applicant was in possession of the claimed invention at the time of filing. E including in the patent application disclosure of drawings or structural chemical formulas showing that the invention is complete.

C Related MPEP Chapter(s): MPEP 2100 - Patentability Answer Reasoning:ANSWER: (C) is the most correct answer. MPEP § 2163, under the heading "GENERAL PRINCIPLES GOVERNING COMPLIANCE WITH THE "WRITTEN DESCRIPTION" REQUIREMENT FOR APPLICATIONS," and subheading "New or Amended Claims," states "A claim that omits an element which applicant describes as an essential or critical feature of the invention originally disclosed does not comply with the written description requirement. See Gentry Gallery, 134 F.3d at 1480, 45 USPQ2d at 1503; In re Sus, 306 F.2d 494, 504, 134 USPQ 301, 309 (CCPA 1962) ('[O]ne skilled in this art would not be taught by the written description of the invention in the specification that any 'aryl or substituted aryl radical' would be suitable for the purposes of the invention but rather that only certain aryl radicals and certain specifically substituted aryl radicals [i.e., aryl azides] would be suitable for such purposes.')." (A), (B), (D) and (E) are incorrect. Each lists a proper way to demonstrate satisfaction of the written description requirement. MPEP § 2163.02, under the heading "STANDARD FOR DETERMINING COMPLIANCE WITH THE WRITTEN DESCRIPTION REQUIREMENT," provides that the written description requirement is met "when the specification conveys with reasonable clarity to those skilled in the art that, as of the filing date sought, applicant was in possession of the invention as now claimed. See, e.g., Vas-Cath, Inc. v. Mahurkar, 935 F.2d 1555, 1563-64, 19 USPQ2d 1111, 1117 (Fed. Cir. 1991)." Possession may be shown in a variety of ways "including description of an actual reduction to practice, or by showing that the invention was 'ready for patenting' such as by the disclosure of drawings or structural chemical formulas that show that the invention was complete, or by describing distinguishing identifying characteristics sufficient to show that the applicant was in possession of the claimed invention. See, e.g., Pfaff v. Wells Electronics, Inc., 525 U.S. 55, 68, 119 S.Ct. 304, 312, 48 USPQ2d 1641, 1647 (1998); Regents of the University of California v. Eli Lilly, 119 .F.3d 1559, 1568, 43 USPQ2d 1398, 1405 (Fed. Cir. 1997); Amgen, Inc. v. Chugai Pharmaceutical, 927 F.2d 1200, 1206, 18 USPQ2d 1016, 1021 (Fed. Cir. 1991)."

Assuming that any rejection has been properly made final, which of the following statements is not in accordance with the USPTO rules and the procedures set forth in the MPEP? A An objection and requirement to delete new matter from the specification is subject to supervisory review by petition under 37 CFR 1.181. B A rejection of claims for lack of support by the specification (new matter) is reviewable by appeal to the Board of Patent Appeals and Interferences. C If both the claims and the specification contain the same new matter, and there has been both a rejection and objection by the primary examiner, the new matter issue should be decided by petition, and is not appealable. D If both the claims and the specification contain the same new matter, and there has been both a rejection and objection by the examiner, the new matter issue is appealable, and should not be decided by petition. E None of the above.

C Related MPEP Chapter(s): MPEP 2100 - Patentability Answer Reasoning:ANSWER: (C) is the most correct answer. MPEP § 2163.06, under the heading "Review Of New Matter Objections And Rejections," states "[a] rejection of claims is reviewable by the Board of Patent Appeals and Interferences, whereas an objection and requirement to delete new matter is subject to supervisory review by petition under 37 CFR 1.181 . If both the claims and specification contain new matter either directly or indirectly, and there has been both a rejection and objection by the examiner, the issue becomes appealable and should not be decided by petition." Answer (C) is not in accordance with the USPTO rules and the procedures set forth in the MPEP. (A), (B) and (D) are incorrect. They are in accord with proper USPTO procedure. See MPEP § 2163.06, under the heading "Review Of New Matter Objections And Rejections." (E) is not correct because (C) is correct. MPEP § 2163.06.

Which of the following is the best way to recite a claim limitation so that it will be interpreted by the examiner in accordance with 35 USC 112, paragraph 6? A dot matrix printer for printing indicia on a first surface of a label. B dot matrix printer means coupled to a computer. C means for printing indicia on a first surface of a label. D printer station for printing indicia on a first surface of a label. E All of the above.

C Related MPEP Chapter(s): MPEP 2100 - Patentability Answer Reasoning:ANSWER: (C) is the most correct answer. MPEP § 2181 expressly requires that for a claim limitation to be interpreted in accordance with 35 U.S.C. § 112, paragraph 6, that limitation must (1) use the phrase "means for", (2) the "means for" must be modified by functional language, and (3) the "means for" must not be modified by sufficient structure for achieving the specified function. In the above fact pattern, only answer choice (C) satisfies the above requirements. (A) is wrong because it does not use the phrase "means for" and recites structure for achieving the specified function ("printer"). (B) is wrong because it modifies the "means" with structure, and also fails to modify the "means" with functional language. (D) is wrong because it does not use the phrase "means for" and also recites structure modifying "mechanism."

23. Applicant files a claim which includes the following limitation: "a step for crossing the road." The specification recites the following acts: "(1) go to the curb, (2) look both ways, (3) if the road appears safe, walk across the road, (4) step up onto the far curb, (5) continue walking." The primary examiner properly construes the step limitation to cover the foregoing acts. A prior art reference, published two years before the application was filed, expressly describes acts (1)-(4), but not (5). This same reference also discloses the remaining limitations recited in applicant's claim, i.e., those other than the step plus function limitation. The examiner rejects the claim under 35 USC 102 as being anticipated by the prior art reference. In accordance with the patent laws, rules and procedures as related in the MPEP, which of the following is the most complete reply to overcome the rejection under these circumstances? A An argument explaining that since act (5) is not disclosed in the reference, it does not anticipate the claim. B An amendment to the specification deleting act (5) - continue walking. C An argument showing that neither the equivalent of act (5) nor act (5) is disclosed in the reference, which therefore does not anticipate the claim. D An amendment to the claim by adding a negative limitation to expressly exclude act (5) from crossing the road. E (B) and (D).

C Related MPEP Chapter(s): MPEP 2100 - Patentability Answer Reasoning:ANSWER: (C) is the most correct answer. Pursuant to 35 U.S.C. § 112, paragraph 6, In re Donaldson Co., 16 F.3d 1189, 1193, 29 USPQ2d 1845, 1849 (Fed. Cir. 1994) (in banc), and MPEP § 2181, under the heading "Written Description Necessary To Support A Claim Limitation Which Invokes 35 U.S.C. 112, Sixth Paragraph," "step" plus function limitations shall be construed to cover the corresponding acts disclosed in the specification and their equivalents. Accordingly, the step plus function imitation correspondingly includes acts (1)-(5) and their equivalents. Thus, in order to anticipate, a prior art reference must disclose each and every act, or its equivalent, for the step plus function. If the reference is shown to not disclose one of the acts, or its equivalents, then the reference fails to anticipate, which is the answer set forth in (C). Thus, (C) is the most complete answer. (A) is not the most complete answer because acts (1)-(4) are disclosed in the reference and the equivalent of act (5) has to be dealt with, i.e., the equivalent of continuing to walk may still be met by the reference unless the applicant shows through argument that the reference also fails to contain any equivalent for act (5). Thus, the most complete answer is (C) as compared to (A). (B) is not the most correct answer because once act (5) is removed from the specification, the prior art reference clearly anticipates (since it otherwise expressly has acts (1)-(4) and the other claim limitations) under the above recited facts absent act (5) in the specification. See Donaldson, 16 F.3d at 1193, 29 USPQ2d at 1849; MPEP § 2181. (D) is not the most complete answer the prior art still anticipates the claim. (E) is not the most correct answer because it includes two incorrect answer choices, (B) and (D).

Which of the following does not represent prior art? A The preamble of a Jepson claim. B A technical journal as of its date of publication which is accessible to the public as of the date of its publication. C A disclosure publicly posted on the INTERNET, but containing no publication or retrieval date. D A doctoral thesis indexed, cataloged and shelved in a university library. E Applicant's labeling of one of the figures in the drawings submitted with his application as prior art.

C Related MPEP Chapter(s): MPEP 2100 - Patentability Answer Reasoning:ANSWER: (C) is the most correct answer. See MPEP § 2128 under the subheading "Date of Availability," of the heading "Electronic Publications As Prior Art." (A) is wrong. See MPEP § 2129 under the heading "A Jepson Claim Results In An Implied Admission That Preamble Is Prior Art." (B) is wrong. See MPEP § 2128.02. A journal article or other publication becomes available as prior art on date it is received by at least one member of the public. (D) is wrong. See MPEP § 2128.01 under the heading "A Thesis Placed In A University Library May Be Prior Art If Sufficiently Accessible To The Public." (E) is wrong. See In re Nomiya, 184 USPQ 607, 610 (CCPA 1975); MPEP § 2129 under the heading "Admissions By Applicant Constitute Prior Art."

In accordance with USPTO rules and procedures set forth in the MPEP, which of the following is not a "printed publication" under 35 USC 102, with respect to a patent application filed June 1, 2016? A A paper that was orally presented at a meeting held May 1, 2015, where the meeting was open to all interested persons and the paper was distributed in written form to six people without restriction. B A doctoral thesis that was indexed, cataloged, and shelved May 1, 2015, in a single, university library. C A research report distributed May 1, 2015, in numerous copies but only internally within an organization and intended to remain confidential. D A reference available only in electronic form on the Internet, which states that it was publicly posted May 1, 2015. E A technical manual that was shelved and cataloged in a public library as of May 1, 2015, where there is no evidence that anyone ever actually looked at the manual.

C Related MPEP Chapter(s): MPEP 2100 - Patentability Answer Reasoning:ANSWER: (C) is the most correct answer. The internal report was intended to be confidential and therefore is not a "printed publication" under 35 U.S.C. § 102(b). See MPEP § 2128.01. Answer (A) is incorrect. An orally presented paper can be a "printed publication" if copies are available without restriction. The paper is a "printed publication" under 35 U.S.C. § 102(b). See MPEP § 2128.01. Answer (B) is incorrect. The thesis is a "printed publication" under 35 U.S.C. § 102(b). See MPEP § 2128.01. Answer (D) is incorrect. An electronic publication disclosed on the Internet is considered to be publicly available as of the date the item was posted. The reference is a "printed publication" under 35 U.S.C. § 102(b). See MPEP § 2128. Answer (E) is incorrect. There is no need to prove that anyone actually looked at a document. The manual is a "printed publication" under 35 U.S.C. § 102(b). See MPEP § 2128.

Which of the following statements is or are in accord with the patent laws, rules and procedures as related in the MPEP? (1) In a 35 USC 103 obviousness analysis, the proper question is whether the differences between the prior art and the claims would have been obvious to one of ordinary skill in the art. (2) In a 35 USC 103 obviousness analysis, an inventor's assertion the he has discovered the source or cause of an identified problem should never be considered. (3) A 35 USC 103 obviousness analysis requires consideration not just of what is literally recited in the claims, but also of any properties inherent in the claimed subject matter that are disclosed in the specification. A Statement 1 B Statement 2 C Statement 3 D Statements 1 and 2 E Statements 1 and 3

C Related MPEP Chapter(s): MPEP 2100 - Patentability Answer Reasoning:ANSWER: (C) is the most correct answer. The principle in Statement 3, that consideration of inherent properties is part of proper consideration of the invention as a whole, is recited in MPEP § 2141.02, under the heading "Disclosed Inherent Properties Are Part Of 'As A Whole' Inquiry," and in In re Antonie, 559 F.2d 618, 620, 195 USPQ 6, 8 (CCPA 1977). (A) is incorrect, because the proper question is whether the invention as a whole, not just the differences, would have been obvious. See MPEP § 2141.02, under the heading "The Claimed Invention As A Whole Must Be Considered," (citing Stratoflex, Inc. v. Aeroquip Corp., 713 F.2d 1530, 218 USPQ 871 (Fed. Cir. 1983). (B) is incorrect because an examiner should consider such assertions by an inventor as part of the "subject matter as a whole." See MPEP § 2141.02 (citing In re Sponnoble, 405 F.2d 578, 585, 160 USPQ 237, 243 (CCPA 1969)). (D) and (E) are incorrect because they include incorrect Statements 1 and/or 2.

Claim 1 of an application recites "[a]n article comprising: (a) a copper substrate; and (b) a electrically insulating layer on said substrate." The specification defines the term "copper" as being elemental copper or copper alloys. In accordance with USPTO rules and procedures set forth in the MPEP, for purposes of searching and examining the claim, the examiner should interpret the term "copper" in the claim as reading on: A Elemental copper only, based on the plain meaning of "copper." B Copper alloys only, based on the special definition in the specification. C Elemental copper and copper alloys, based on the special definition in the specification. D Any material that contains copper, including copper compounds. E None of the above.

C Related MPEP Chapter(s): MPEP 2100 - Patentability Answer Reasoning:ANSWER: (C) is the most correct answer. When the specification expressly provides a special definition for a term used in the claims, the term must be given that special meaning. See MPEP § 2111.01. (A) is incorrect because a term is given its plain meaning only when the specification does not provide a definition for the term. See MPEP § 2111.01 (B) is incorrect because the specification defines the term as being inclusive of elemental copper. See MPEP § 2111.01. (D) is incorrect because it does not take into account the definition of copper found in the specification. See MPEP § 2111.01.

In accordance with the patent laws, rules and procedures as related in the MPEP, which of the following facts are required for pre-AIA 35 USC 102(g) to form the basis for an ex parte rejection: (1) The subject matter at issue has been actually reduced to practice by another before the applicant's invention. (2) There has been no abandonment, suppression or concealment. (3) A U.S. patent application for the subject matter at issue has been filed by another prior to the filing of the applicant's application. (4) A U.S. patent has been granted for the subject matter at issue prior to the filing of the applicant's application. A Fact (1) only B Fact (2) only C Facts (1) and (2) D Facts (1), (2) and (3) E Facts (1), (2), (3) and (4)

C Related MPEP Chapter(s): MPEP 2100 - Patentability Answer Reasoning:ANSWER: (C) is the most correct, as a 35 U.S.C. § 102(g) rejection requires actual reduction to practice by another, and lack of abandonment, suppression, or concealment. MPEP § 2138 states "35 U.S.C. 102(g) may form the basis for an ex parte rejection if: (1) the subject matter at issue has been actually reduced to practice by another before the applicant's invention; and (2) there has been no abandonment, suppression or concealment. See, e.g., Amgen, Inc. v. Chugai Pharmaceutical Co., 927 F.2d 1200, 1205, 18 USPQ2d 1016, 1020 (Fed. Cir. 1991); New Idea Farm Equipment Corp. v. Sperry Corp., 916 F.2d 1561, 1566, 16 USPQ2d 1424, 1428 (Fed. Cir. 1990); E.I. DuPont de Nemours & Co. v. Phillips Petroleum Co., 849 F.2d 1430, 1434, 7 USPQ2d 1129, 1132 (Fed. Cir. 1988); Kimberly-Clark v. Johnson & Johnson, 745 F.2d 1437, 1444-46, 223 USPQ 603, 606-08 (Fed. Cir. 1984)." (A) is incorrect, as actual reduction to practice is not sufficient to establish a 35 U.S.C. § 102(g) rejection where the subject matter has been abandoned, suppressed, or concealed. MPEP § 2138. (B) is incorrect, as abandonment, suppression, or concealment is not sufficient to establish a 35 U.S.C. § 102(g) rejection where the subject matter has been reduced to practice in that conception alone is not sufficient. See Kimberly-Clark v. Johnson & Johnson, 745 F.2d 1437, 1445, 223 USPQ 603, 607 (Fed. Cir. 1984). MPEP § 2138. (D) is incorrect because no prior patent application is required for a § 102(g) rejection. MPEP § 2138. Similarly, (E) is incorrect, because no prior patent application nor issued patent is required for a rejection under 35 U.S.C. § 102(g).

In accordance with the USPTO rules and the procedures set forth in the MPEP, which of the following statements is most correct? A The same evidence sufficient to establish a constructive reduction to practice is necessarily also sufficient to establish actual reduction to practice. B Proof of constructive reduction to practice does not require sufficient disclosure to satisfy the "how to use" and "how to make" requirements of 35 USC 112, first paragraph. C A process is reduced to actual practice when it is successfully performed. D The diligence of 35 USC 102(g) requires an inventor to drop all other work and concentrate on the particular invention. E The diligence of 35 USC 102(g) does not impose on a registered practitioner any need for diligence in preparing and filing a patent application inasmuch as such the practitioner's acts do not inure to the benefit of the inventor.

C Related MPEP Chapter(s): MPEP 2100 - Patentability Answer Reasoning:ANSWER: (C) is the most correct. Corona v. Dovan 273 U.S. 692, 1928 CD 252 (1928); MPEP § 2138.05 under the heading "Requirements To Establish Actual Reduction To Practice." (A) is incorrect. MPEP § 2138.05, under the heading "Requirements To Establish Actual Reduction To Practice." The same evidence sufficient to establish a constructive reduction to practice is not necessarily sufficient to establish actual reduction to practice, which requires a showing of the invention in a physical or tangible form containing every element of the count. Wetmore v. Quick, 536 F.2d 937, 942 190 USPQ 223 227 (CCPA 1976). (B) is incorrect. MPEP § 2138.05, under the heading "Constructive Reduction To Practice Requires Compliance With 35 U.S.C. 112, First Paragraph." Kawai v. Metlesics, 489 F.2d 880, 886, 178 USPQ 158, 163 (CCPA 1973). (D) is incorrect. Keizer v. Bradley, 270 F.2d 396, 397, 123 USPQ 215, 216 (CCPA 1959) (the diligence of 35 U.S.C. § 102(g) does not require "an inventor or his attorney to drop all other work and concentrate on the particular invention involved"); MPEP § 2138.06. (E) is incorrect. The diligence of a practitioner in preparing and filing an application inures to the benefit of the inventor. See MPEP § 2138.06, under the heading "Diligence Required In Preparing And Filing Patent Application." Haskell v. Coleburne, 671 F.2d 1362, 213 USPQ 192,195 (CCPA 1982) (six days to execute and file application was acceptable).

In which of the following situations, considered independently of each other, is the original, new, or amended claim supported in the application as filed? A An amendment to the specification changing the definition of "holder" from "is a hook" to "is a hook, clasp, crimp, or tong" and no amendment is made of the claim, which uses the term "holder." The amendment is filed one month after the application was filed. There was no previous supporting disclosure in the specification of the holder being a clasp, crimp, or tong. B An amendment to the specification and claims changing the definition of "holder" from "is a hook" to "is a hook, clasp, crimp, or tong." The amendment is filed one month after the application was filed. There was no previous supporting disclosure in the specification of the holder being a clasp, crimp, or tong. C Original claim 1 in the application refers to "a holder," and original claim 2 depends from and refers to claim 1 stating, "said holder is a hook, clasp, crimp, or tong." There is no disclosure in the specification preceding the claims in the application as filed for the holder to be a clasp, crimp, or tong. D An amendment is filed presenting a claim to an electrical insulating device, copied from a patent for the purpose of provoking an interference. The claim refers to "nonconductive plastic holder." The application as filed contains a broad generic disclosure describing electrical insulating devices. The holder is described in the specification of the application as "conducting electricity." There is no disclosure in the specification of the holder being "nonconductive." E All of the above.

C Related MPEP Chapter(s): MPEP 2100 - Patentability Answer Reasoning:ANSWER: (C). MPEP 2163.03, item I. Original claims constitute their own description. In re Koller, 613 F.2d 819, 204 USPQ 702 (CCPA 1980). (A) and (B) are incorrect. As stated in MPEP 2163.03, item I, "An amendment to the specification (e.g., a change in the definition of a term used both in the specification and claim) may indirectly affect a claim even though no actual amendment is made to the claim." There is no supporting disclosure in the original description of the invention for the holder to a clasp, crimp, or tong. (D) is incorrect. MPEP 2163.03, item IV. A broad generic disclosure is not necessarily a sufficient written description of a specific embodiment, especially where the broad generic disclosure conflicts with the remainder of the disclosure. Fields v. Conover, 443 F.2d 1386, 170 USPQ 276 (CCPA 1970). (E) is not correct because (C) is correct.

In accordance with the patent law, rules and procedures as related by the MPEP, which of the following is not a "printed publication" under 35 USC 102(a), with respect to a patent application filed June 1, 2016? A A paper that was orally presented at a meeting held May 1, 2015, where the meeting was open to all interested persons and the paper was distributed in written form to six people without restriction. B A doctoral thesis that was indexed, cataloged, and shelved May 1, 2015, in a single, university library. C A research report distributed May 1, 2015, in numerous copies but only internally within an organization to persons who understood the organization's unwritten policy of confidentiality regarding such reports. D A reference available only in electronic form on the Internet, which states that it was publicly posted May 1, 2015. E A technical manual that was shelved and cataloged in a public library as of May 1, 2015, where there is no evidence that anyone ever actually looked at the manual.

C Related MPEP Chapter(s): MPEP 2100 - Patentability Answer Reasoning:ANSWER: The correct answer is (C). The internal report was intended to be confidential and therefore is not a "printed publication" under 35 U.S.C. § 102(b). See MPEP § 2128.01, under the heading "Internal Documents Intended To Be Confidential Are Not 'Printed Publications," citing In re George, , 2 USPQ2d 1880 (Bd. Pat. App. & Int. 1987) states "Research reports disseminated in-house to only those persons who understood the policy of confidentiality regarding such reports are not printed publications even though the policy was not specifically stated in writing." Answer (A) is incorrect. An orally presented paper can be a "printed publication" if copies are available without restriction. The paper is a "printed publication" under 35 U.S.C. § 102(b). See MPEP § 2128.01. Answer (B) is incorrect. The thesis is a "printed publication" under 35 U.S.C. § 102(b). See MPEP § 2128.01. Answer (D) is incorrect. An electronic publication disclosed on the Internet is considered to be publicly available as of the date the item was posted. The reference is a "printed publication" under 35 U.S.C. § 102(b). See MPEP § 2128. Answer (E) is incorrect. There is no need to prove that anyone actually looked at a document. The manual is a "printed publication" under 35 U.S.C. § 102(b). See MPEP § 2128.

Recommend which of the following rejections under 35 USC 102 in a Exparte reexamination proceeding is in accordance with the patent laws, rules and procedures as related in the MPEP for pre-03/16/13 filed patents. A A rejection under 35 USC 102(a) based on an affidavit that the invention was known or used by others before the invention thereof by the applicant for patent. B A rejection under 35 USC 102(b) based on an affidavit that the invention was in the public use in this country more than one year prior to the date of the application for a patent in the United States. C A rejection under 35 USC 102(e) that the invention was described in a patent by another filed in the United States before the invention thereof by the patent applicant. D A rejection under 35 USC 102(f) based on an affidavit that the applicant did not himself invent the subject matter sought to be patented. E A rejection under 35 USC 102(b) that the invention was on sale in this country, more than one year prior to the date of the application for patent in the United States.

C Related MPEP Chapter(s): MPEP 2200 - Citation of Prior Art and Ex Parte Reexamination of Patents Answer Reasoning:ANSWER: (C) is the most correct answer. 35 U.S.C. § 302; 37 CFR § 1.552; and MPEP § 2258. MPEP § 2258, under the heading "Prior Patents Or Printed Publications," states "[r]ejections on prior art in reexamination proceedings may only be made on the basis of prior patents or printed publications. Prior art rejections may be based upon the following portions of 35 U.S.C. 102: . . .(e)." (A), (B), (D) and (E) are incorrect. MPEP § 2258, under the heading "Matters Other Than Patents or Printed Publications," states "[r]ejections will not be based on matters other than patents or printed publications, such as public use or sale, inventorship, 35 U.S.C. 101, fraud, etc. In this regard, see In re Lanham, 1 USPQ2d 1877 (Comm'r Pat. 1986), and Stewart Systems v. Comm'r of Patents and Trademarks, 1 USPQ2d 1879 (E.D. Va. 1986). A rejection on prior public use or sale, insufficiency of disclosure, etc., cannot be made even if it relies on a prior patent or printed publication. Prior patents or printed publications must be applied under an appropriate portion of 35 U.S.C. 102 and/or 103 when making a rejection." Reexamination is limited to substantially new questions of patentability based on patents and publications.

John filed a nonprovisional patent application in the USPTO claiming two distinct inventions, a combination and a subcombination. At the time of filing the nonprovisional application, he recorded an assignment of all right, title, and interest in the inventions claimed in the application to ABC Corporation. In the first Office action, the examiner required restriction, and John elected the combination. A year later, during the pendency of the nonprovisional application, John filed a divisional patent application claiming the subcombination. At the time of filing the divisional application, John assigned all right, title, and interest in the inventions claimed in the divisional application to XYZ Corporation, and the latter party recorded the assignment within three months of the assignment. Following recordation of the assignment to XYZ Corporation, which of the following statements is false? A The Office should treat John as having no ownership rights in the combination. B The Office should treat John as having no ownership rights in the subcombination. C ABC Corporation has no ownership rights in the subcombination. D XYZ Corporation has no ownership rights in the combination. E XYZ Corporation has no ownership rights in the subcombination.

C Related MPEP Chapter(s): MPEP 300 - Ownership and Assignment Answer Reasoning:ANSWER: (C) is a false statement and therefore the correct answer. Under 35 U.S.C. § 261, "An assignment, grant, or conveyance shall be void as against any subsequent purchaser or mortgagee for a valuable consideration, without notice, unless it is recorded in the Patent and Trademark Office within three months from its date or prior to the date of such subsequent purchase or mortgage." (C) is correct because ABC Corporation acquired all of John's ownership rights in the original patent application, including the subcombination claimed in the original nonprovisional and divisional patent applications. The assignment of the rights to ABC Corporation was recorded in the USPTO prior to the subsequent acquisition of the subcombination by XYZ Corporation. U.S.C. § 261. ABC Corporation's recordation of its assignment gave constructive notice to XYZ Corporation. MPEP § 306 recites that in the case of a division...application, a prior assignment recorded against the original application is applied to the division...application because the assignment recorded against the original application gives the assignee rights to the subject matter common to both applications. (A) and (B) are true statements and therefore wrong answers. John gave up his ownership rights when he executed the assignment to ABC Corporation. The assignment to ABC Corporation carries with it the transfer of the bundle of rights associated with subject matter common to the original patent application, e.g., the divisional patent application. (D) and (E) are true statements and therefore wrong answers because XYZ Corporation acquired no rights in the original or divisional patent applications. MPEP § 306.

The USPTO notifies John, a registered patent agent who is representing applicant A, that after a reasonable search, the USPTO has been unable to locate applicant A's patent application. By which of the following procedures may John avoid abandonment of applicant A's application within the time period set by the USPTO? A Provide the USPTO with a copy of his record of all the correspondence between his office and the USPTO, assuming the existence of such record. B Provide the USPTO with a list of all the correspondence between his office and the USPTO, assuming the existence of such list, and a statement that the list is complete and accurate. C Provide the USPTO with a statement that he does not possess any record of the correspondence between his office and the USPTO because his files were destroyed. D Provide the USPTO with a record of all the correspondence between his office and the USPTO, and a statement that the papers produced are his complete record of all the correspondence between his office and the USPTO, assuming the existence of such record. E Provide the USPTO with a copy of his record of all the correspondence between his office and the USPTO, assuming the existence of such record, a list of all such correspondence, and a statement that he is not aware of any correspondence between his office and the USPTO that is not among his records.

C Related MPEP Chapter(s): MPEP 500 - Receipt and Handling of Mail and Papers Answer Reasoning:ANSWER: (C) is correct. 37 C.F.R. § 1.251(a)(3). (C) is correct because there is compliance with 37 C.F.R. § 1.251(a)(3). (A) is wrong because along with a copy of the record, he is required to provide a list of all correspondence, and a statement that the copy is complete and accurate and that he is not aware of any correspondence between his office and the USPTO that is not among his records. 37 C.F.R. § 1.251(a)(1)(ii). (B) is wrong because along with a list of all correspondence and a statement that the copy of his record of all the correspondence is complete and accurate, he is required to provide a copy of his record of all the correspondence, and the statement must recite that he is not aware of any correspondence between his office and the USPTO that is not among his records. 37 C.F.R. § 1.251(a)(1)(i). (D) is wrong because the statement omits the recitation that he is not aware of any correspondence between his office and the USPTO that is not among his records. 37 C.F.R. § 1.251(a)(2)(ii). (E) is wrong because the statement omits the recitation that the copy of his record of all the correspondence is complete and accurate. 37 C.F.R. § 1.251(a)(1)(iii).

A registered practitioner files an international application submission that includes a description, claims and drawings in the United States Receiving Office (RO/US) on Wednesday, January 8, 2003. The submission did not include the required request, international and search fees, or the designation of a PCT contracting State. The RO/US mails an "Invitation to Correct the Purported International Application," dated January 10, 2003, to the practitioner indicating that the designation of at least one Contracting State, as required by PCT Article 11(1)(iii)(b), was not included. A one-month period for response is set in the Invitation. On Monday, February 10, 2003, the practitioner submits by facsimile a designation sheet of the Request Form designating every available Contracting State, and authorization to charge all required fees. In accordance with the patent laws, rules and procedures as related in the MPEP, will the application be accorded an international filing date? A Yes. The application will be accorded a filing date of January 8, 2003. B Yes. The application will be accorded an international filing date of February 10, 2003. C No. The application will not be accorded an international filing date because the failure to designate at least one contracting State cannot be cured by a facsimile transmission. D No. The application was given a one-month period for response. The practitioner would have had to have filed the response on Friday, February 7, 2003 in order to have been timely. E None of the above.

C Related MPEP Chapter(s): MPEP 500 - Receipt and Handling of Mail and Papers Answer Reasoning:ANSWER: (C) is the correct answer. 37 CFR §§ 1.6(d)(3) and 1.8(a)(2)(i)(d); MPEP § 502 (reproducing Rule 1.6(d)(3)); MPEP § 512 (reproducing Rule 1.8(a)(2)(i)(d)); and MPEP §1817.01. As stated in MPEP § 1817.01, "[a]ll designations must be made in the international application on filing; none may be added later." The application will not be accorded an international filing date since the practitioner has tried to cure the failure to designate at least one contracting State by filing a paper using facsimile which is not permitted according to 37 CFR §§ 1.6(d)(3) and 1.8(a)(2)(i)(d). (But, bear in mind since 1/1/04 - a designation is automatic as a part of a PCT filing.) (A) is wrong because applicant has failed to comply with Article 11(1)(iii)(b) on such date. See MPEP § 1810 (reproducing PCT Article 11(1)(iii)(b). (B) is wrong because according to 37 CFR 1.6(d)(3) and 37 CFR 1.8 (a)(2)(i)(d), applicant cannot file an international application by facsimile. See MPEP § 502 (reproducing 37 CFR § 1.6(d)(3)); MPEP § 512 (reproducing 37 CFR § 1.8(a)(2)(i)(d)). Since no designations were included on filing, the application papers cannot be accorded an international filing date. See PCT Article 11(1)(iii)(b). Applicant cannot correct this by filing the designation sheet by facsimile. See MPEP § 502 (reproducing Rule 1.6(d)(3)); MPEP § 512 (reproducing Rule 1.8(a)(2)(i)(d)). (D) is wrong because according to PCT Rule 80.5, when a response is due on a day where the receiving Office is not open for business, applicant has until the next business day. See Appendix T of the MPEP. (E) is incorrect because (C) is correct.

Sam is a sole proprietor of Sam's Labs, which has no other employees. Sam invented a new drug while doing research under a Government contract. Sam desires to file a patent application for his invention and assign it to Sam's Labs. Sam has licensed Rick, also a sole proprietor with no employees, to make and use his invention. Sam wants to claim small entity status when filing a patent application for his invention. Sam also wants to grant the Government a license, but will not do so if he will be denied small entity status. Sam has limited resources and wants to know whether, how, and to what extent he may claim small entity status. Which of the following is not in accord with the USPTO rules and the procedures set forth in the MPEP in relation to applications filed on or after January 1, 2001? A Sam's Labs is a small business concern for the purposes of claiming small entity status for fee reduction purposes. B If Sam grants a license to the Government resulting from a rights determination under Executive Order 10096, it will not constitute a license so as to prohibit claiming small entity status. C The establishment of small entity status permits the recipient to pay reduced fees for all patent application processing fees charged by the USPTO. D Sam may establish small entity status by a written assertion of entitlement to small entity status. A written assertion must: (i) be clearly identifiable; (ii) be signed; and (iii) convey the concept of entitlement to small entity status, such as by stating that applicant is a small entity, or that small entity status is entitled to be asserted for the application or patent. E While no specific words or wording are required to assert small entity status, the intent to assert small entity status must be clearly indicated in order to comply with the assertion requirement.

C Related MPEP Chapter(s): MPEP 500 - Receipt and Handling of Mail and Papers Answer Reasoning:ANSWER: (C) is the most correct answer. Not all fees are subject to the small entity reduction. See, for example, 37 CFR § 1.17(q). As to (A), a small business concern for the purposes of claiming small entity status for fee reduction purposes is any business concern that: (i) has not assigned, granted, conveyed, or licensed, and is under no obligation under contract or law to assign, grant, convey, or license, any rights in the invention to any person, concern, or organization which would not qualify for small entity status as a person, small business concern, or nonprofit organization. and (ii) meets the standards set forth in the appropriate section of the code of federal regulations to be eligible for reduced patent fees. Sam's Labs meets all of the elements required by 37 CFR § 1.27 (a)(2). Statement (B) contains all of the elements required by 37 CFR § 1.27 (a)(4). Statement (D) contains all of the elements required by 37 CFR § 1.27 (c)(1). Statement (E) contains all of the elements required by 37 CFR § 1.27 (c)(1)(iii).

Which of the following is not a USPTO recommendation or requirement? A Product and process claims should be separately grouped. B Claims should be arranged in order of scope so that the first claim presented is the least restrictive. C Every application should contain no more than three dependent claims. D A claim which depends from a dependent claim should not be separated from that dependent claim by any claim which does not also depend from the dependent claim. E Each claim should start with a capital letter and end with a period.

C Related MPEP Chapter(s): MPEP 500 - Receipt and Handling of Mail and Papers Answer Reasoning:ANSWER: (C) is the most correct answer. The USPTO does not require or recommend a minimum or maximum number of dependent claims. 37 C.F.R. § 1.75(c). (A) is a USPTO recommendation. See MPEP § 608.01(m) ("Similarly, product and process claims should be separately grouped."). (B) is a USPTO recommendation. See MPEP § 608.01(m) ("Claims should preferably be arranged in order of scope so that the first claim presented is the least restrictive."). (D) is a PTO recommendation. See MPEP § 608.01(n), part IV. (E) is a USPTO requirement. See MPEP § 608.01(m) ("Each claim begins with a capital letter and ends with a period.").

The following facts: Claims 1 and 2, fully disclosed and supported in the specification of a patent application having an effective filing date of March 15, 2000, for sole inventor Ted, state the following: Claim 1. An apparatus intended to be used for aerating water in a fish tank, comprising: (i) an oxygen source connected to a tube, and (ii) a valve connected to the tube. Claim 2. An apparatus as in claim 1, further comprising an oxygen sensor connected to the valve. Which of the following claims, if fully disclosed and supported in the specification, and included in the application, provides a proper basis for an objection under 37 CFR 1.75(c)? A Claim 3. An apparatus as in any one of the preceding claims, in which the tube is plastic. B Claim 3. An apparatus according to claims 1 or 2, further comprising a temperature sensor connected to the valve. C Claim 3. An apparatus as in the preceding claims, in which the tube is plastic. D Claim 3. An apparatus as in any preceding claim, in which the tube is plastic. E Claim 3. An apparatus as in either claim 1 or claim 2, further comprising a temperature sensor connected to the valve.

C Related MPEP Chapter(s): MPEP 600 - Parts, Form, and Content of Application Answer Reasoning:ANSWER: (C) is the correct answer. Claim 3 in answer (C) employs improper multiple dependent claim wording. MPEP § 608.01(n)(I)(B). (A), (B), (D), and (E) are incorrect as each uses acceptable multiple dependent claim wording. MPEP § 608.01(n)(I)(A).

A registered practitioner filed a design patent application on December 30, 2016. The application was filed with an inventor-executed declaration naming Jon Jones as the sole inventor, who has not assigned the invention and is not under an obligation to assign his invention. In reviewing the filing receipt, the practitioner realizes that the typed name of the inventor contained a typographical error (an "h" was missing) and that the correct spelling was John Jones. Which of the following would be the course of action at the least expense to correct the error in accordance with the patent laws, rules and procedures as related in the MPEP? A The practitioner should file a request under 37 CFR 1.48 to correct the inventorship of the application with a new declaration under 37 CFR 1.63 signed by John Jones (with the correct spelling of this name), a statement by Mr. Jones as to how the error occurred and that the error was without deceptive intention, and the processing fee set forth in 37 CFR 1.17(q). B The practitioner should file a petition under 37 CFR 1.182 and the petition fee set forth in 37 CFR 1.17(h), requesting correction of the spelling of the inventor's name. C The practitioner should file a request including (1) a corrected application data sheet that identifies each inventor by his or her properly spelled legal name, and (2) the processing fee. D The practitioner should expressly abandon the application, and file a continuation with a new declaration with the correct spelling. E The practitioner should call the examiner and tell the examiner that the inventor's name is wrong, and ask for the examiner to change the name on the declaration.

C Related MPEP Chapter(s): MPEP 600 - Parts, Form, and Content of Application Answer Reasoning:ANSWER: (C) is the most correct answer. "Except for correction of a typographical or transliteration error in the spelling of an inventor's name, a request to have the name changed to the signed version or any other corrections in the name of the inventor(s) will not be entertained...When a typographical or transliteration error in the spelling of an inventor's name is discovered during pendency of an application, a petition is not required, nor is a new oath or declaration under 37 CFR 1.63 needed. The U.S.[PTO] should simply be notified of the error and reference to the notification paper will be made on the previously filed oath or declaration by the Office." (A), (B) and (D) could result in the spelling of Jon's name being corrected in USPTO records, but would do so at a higher cost to applicant, and therefore neither one is the most correct answer. Furthermore, (A) is also not correct in that if a request to add John Jones as an inventor was to be filed, another request (and fee) to delete Jon Jones would be required. (B) is wrong because a petition under 37 CFR § 1.182 is not required if the error in the name is a typographical error, and the facts specify that the error in the spelling of "John" as "Jon" is a typographical error. (D) is not correct because not only would filing a continuation create an additional expensive, but filing a new application could also delay examination. (E) is not correct because pursuant to 37 CFR § 1.2, business with the Office is to be conducted in writing, and, even more importantly, because "it is improper for anyone, including counsel, to alter, rewrite, or partly fill in any part of the application, including the oath or declaration, after execution of the oath or declaration by the applicant." MPEP § 605.04(a).

In accordance with USPTO rules and the procedures set forth in the MPEP, claims in a patent application may not contain: A chemical formulas. B mathematical equations. C drawings or flow diagrams. D only one sentence. E tables not necessary to conform with 35 USC 112.

C Related MPEP Chapter(s): MPEP 600 - Parts, Form, and Content of Application Answer Reasoning:ANSWER: (C) is the most correct answer. MPEP § 608.01, under the heading "Illustrations In The Specification," states "[t]he specification, including any claims...may not contain drawings or flow diagrams." With respect to answers (A) and (B), see MPEP § 608.01, under the heading "Illustrations In The Specification," states, "[t]he specification, including any claims, may contain chemical formulas and mathematical equations..." As to answer (D), see MPEP § 608.01(m), which states that periods may not be used elsewhere in the claim except for abbreviations and that each claim begins with a capital letter and ends with a period. As to (E), see MPEP § 608.01, which states, "The...claims may contain tables only if necessary to conform to 35 U.S.C. 112."

Lucy, new associate of a registered practitioner, wants to know whether she must file an application data sheet with a provisional patent application of an applicant and what information she should include on the application data sheet. Lucy has previously submitted an application data sheet with a previously filed application for another applicant, but has discovered a discrepancy with the information contained in the declaration and application data sheet. Lucy wonders if she needs to correct the error if the correct information is contained in the declaration. She also asks how errors may be corrected. With respect to the filing of an application data sheet, which of the following is not in accord with the USPTO rules and the procedures set forth in the MPEP for applications filed on or after January 1, 2017? A An application data sheet is a sheet or sheets that may be voluntarily submitted in either provisional or nonprovisional applications, which contains bibliographic data, arranged in a format specified by the Office. If an application data sheet is provided, the application data sheet is part of the provisional or nonprovisional application for which it has been submitted. B Bibliographic data on an application data sheet includes: (1) applicant information, (2) correspondence information, (3) application information, (4) representative information, (5) domestic priority information, (6) foreign priority information, and (7) assignee information. C Once captured by the Office, bibliographic information derived from an application data sheet containing errors may not be corrected and recaptured by a request therefore accompanied by the submission of a corrected application data sheet, an oath or declaration under 37 CFR 1.63 or 1.67; nor will a letter pursuant to 37 CFR 1.33(b) be acceptable. D In general, corrected application data sheets may be subsequently supplied prior to payment of the issue fee either to correct or update information in a previously submitted application data sheet. E The Office will initially capture bibliographic information from the application data sheet notwithstanding whether an oath or declaration governs the information. Thus, the Office shall generally not look to an oath or declaration under 37 CFR 1.63 to see if the bibliographic information contained therein is consistent with the bibliographic information captured from an application data sheet (whether the oath or declaration

C Related MPEP Chapter(s): MPEP 600 - Parts, Form, and Content of Application Answer Reasoning:ANSWER: (C) is the most correct answer. This is not true since 37 CFR § 1.76(d)(4) provides, in part, "(4)...Captured bibliographic information derived from an application data sheet containing errors may be recaptured by a request therefor and the submission of a supplemental application data sheet, an oath or declaration under 37 CFR § 1.63 or § 1.67, or a letter pursuant to 37 CFR 1.33(b)." (A) is in accordance with 37 CFR § 1.76(a). (B) is in accordance with 37 CFR § 1.76(b). (D) is in accordance with 37 CFR § 1.76(c). (E) is in accordance with 37 CFR § 1.76 (d)(4).

Where subject matter for which there is an enabling disclosure, but is not shown in the drawing or described in the detailed description preceding the claim(s), which of the following is not in accordance with the provisions of the MPEP? A In establishing a disclosure, applicant may rely not only on the description and drawing as filed but also on the original claims to show compliance with the first paragraph of 35 USC 112. B Where subject matter not shown in the drawing or described in the description is claimed in the application as filed, and such original claim itself constitutes a clear disclosure of this subject matter, then the claim should be treated on its merits, and the applicant should be required to amend the drawing and description to show this subject matter. C If subject matter appearing in the original claim is not found in the drawing or detailed description, the claim should be rejected for noncompliance with the first paragraph of 35 USC 112. D If the subject matter found in the claim is lacking in the drawing or detailed description, it is the drawing and description that are defective, not the claim. E The subject matter found in the original claims, but lacking in the drawing or detailed description, must be sufficiently specific and detailed to support an amendment of the drawing and detailed description.

C Related MPEP Chapter(s): MPEP 600 - Parts, Form, and Content of Application Answer Reasoning:ANSWER: (C). According to MPEP § 608.01(l), "[t]he claim should not be attacked either by objection or rejection because this subject matter is lacking in the drawing and description." As to (A), (B), (D), and (E), see MPEP § 608.01(l).

Which of the following is not in accordance with the provisions of the MPEP regarding an abstract of the disclosure? A The abstract of the disclosure has been interpreted to be a part of the specification for the purpose of compliance with paragraph 1 of 35 USC 112. B Any submission of a new abstract of the disclosure or amendment to an existing abstract should be carefully reviewed for introduction of new matter. C If an application is otherwise in condition for allowance except that the abstract of the disclosure does not comply with the guidelines, the examiner generally cannot make any necessary revisions by examiner's amendment, but should instead issue an Ex parte Quayle action requiring applicant to make the necessary revisions D Under current practice, in all instances where the application contains an abstract of the disclosure when sent to issue, the abstract will be printed on the patent. E The content of a patent abstract should be such as to enable the reader thereof, regardless of his or her degree of familiarity with patent documents, to ascertain quickly the character of the subject matter covered by the technical disclosure and should include that which is new in the art to which the invention pertains.

C Related MPEP Chapter(s): MPEP 600 - Parts, Form, and Content of Application Answer Reasoning:ANSWER: (C). As indicated in MPEP § 608.01(b), if an application is otherwise in condition for allowance except that the abstract does not comply with the guidelines, the examiner generally should make any necessary revisions by examiner's amendment rather than issuing an Ex parte Quayle action requiring applicant to make the necessary revisions. As to (A), (B), (D), and (E), see MPEP § 608.01(b).

Which of the following is not in accordance with the provisions of the MPEP? A If there is a discrepancy between the information submitted in an application data sheet and the information submitted elsewhere in the application, the application data sheet will control. B A patent examiner should object to text of a patent application if it contains an embedded hyperlink and /or other form of browser-executable code. C All patent applicants should use the English units of measurement followed by the equivalent metric units when describing their inventions in the specifications of patent applications. D The paper used for patent applications must have a surface such that amendments may be written thereon in ink; so-called "Easily Erasable" paper having a special coating so that erasures can be made more easily may not provide a permanent copy as is required. E The following documents may be submitted to the Office on a compact disc: a computer program listing, a sequence listing, and a table that has more than 50 pages of text.

C Related MPEP Chapter(s): MPEP 600 - Parts, Form, and Content of Application Answer Reasoning:ANSWER: (C). As to (C), according to MPEP § 608.01 at p. 600-58, "In order to minimize the necessity in the future for converting dimensions given in the English system of measurements to the metric system of measurements when using printed patents as research and prior art search documents, all patent applicants should use the metric units (SI) followed by the equivalent English units when describing their inventions in the specifications of patent applications." As to (A), see MPEP § 601.05 at p. 600-21 and 37 C.F.R. § 1.76 (d)(3), regarding the oath or declaration controlling over the application data sheet. See also MPEP § 608.01(a) at p. 600-61. As to (B), see MPEP § 608.01(a) at p. 600-60 regarding hyperlinks. As to (D), see 37 C.F.R. § 1.52(a); and MPEP § 608.01. As to (E) see MPEP § 608.01 at p. 600-56 and 37 C.F.R. § 1.52.

In accordance with the patent laws, rules and procedures as related in the MPEP, where the independent claim in an application is to an article of manufacture, then a dependent claim to the article of manufacture does not comply with 35 USC 112(d) if: A the further limitation changes the scope of the dependent claim from that of the claim from which it depends. B the further limitation of the dependent claim is not significant. C it does not refer back to and further limit the claim from which it depends. D it relates to a separate invention. E it is separately classified from the claim from which it depends.

C Related MPEP Chapter(s): MPEP 600 - Parts, Form, and Content of Application Answer Reasoning:ANSWER: The answer is (C). See 37 CFR § 1.75(c); MPEP 608.01(n). Rule 1.75(c) provides that "[o]ne or more claims may be presented in dependent form, referring back to and further limiting another claim or claims in the same application." See also MPEP § 608.01(n), under the heading "III Infringement Test," second paragraph, wherein it states, "[t]he test for a proper dependent claim under the fourth paragraph of 35 U.S.C. 112 is whether the dependent claim includes every limitation of the claim from which it depends." For answer (A), see MPEP § 608.01(n), under the heading "III Infringement Test," second paragraph, wherein it states, "[t]he test is not one of whether the claims differ in scope." For answer (B), see MPEP § 608.01(n), under the heading "III Infringement Test," second paragraph, wherein it states, "[a] dependent claim does not lack compliance with 35 U.S.C. 112, fourth paragraph, simply because there is a question as to (1) the significance of the further limitation added by the dependent claim." For answers (D) and (E), see MPEP § 608.01 (n), under the heading "III Infringement Test," fifth paragraph, wherein it states, "[t]he fact that a dependent claim which is otherwise proper might relate to a separate invention which would require a separate search or be separately classified from the claim on which it depends would not render it an improper dependent claim, although it might result in a requirement for restriction."

Application A was filed after November 29, 2016. Reference X and application A were commonly owned at the time the invention of application A was made, and later filed. In accordance with the patent laws, rules and procedures as related in the MPEP the prior art exclusion of 35 USC 102(c) can be properly invoked to obviate which of the following rejections? A A rejection under 35 USC 102 based on reference X, if reference X is prior art under 35 USC 102(a)(1). B A double patenting rejection based on reference X, if reference X is available as prior art under 35 USC 102. C A rejection under 35 USC 103(a) based on reference X, if reference X is available as prior art only under 35 USC 102(a)(2). D (B) and (C). E All of the above.

C Related MPEP Chapter(s): MPEP 700 - Examination of Applications Answer Reasoning:(A) is wrong because the 102(c) exception does not apply to 102(a)(1) prior art. (B) is wrong because the 102(c) exception does not apply to either obviousness-type or same-invention double patenting. (D) and (E) are wrong because of the above.

Xavier files a complete first reply exactly 2 months after the mailing date of a final Office action which sets a 3 month shortened statutory period for reply. An Examiner's Amendment is necessary for the purpose of placing the application in condition for allowance. Which of the following statements is true? A If the Examiner's Amendment is mailed exactly 5 months after Xavier's reply, the application will be allowed. B The Examiner's Amendment must be made within the 3 month shortened statutory period of the final Office action to avoid abandonment of the application. C If the Examiner's Amendment is made exactly 4 months after Xavier's reply, the application will be allowed. D The Examiner's Amendment may be made at any time within 6 months of Xavier's reply to avoid abandonment. E Abandonment of the application will be avoided if Xavier accompanies his reply with a request for extension of time accompanied by the proper fee and the Examiner's Amendment is made within 6 months of Xavier's reply.

C Related MPEP Chapter(s): MPEP 700 - Examination of Applications Answer Reasoning:ANSWER: (C) is correct and (B) is wrong because MPEP § 706.07(f), part (H), states, "Where a complete first reply to a final Office action has been filed within 2 months of the final Office action, an examiner's amendment to place the application in condition for allowance may be made without the payment of extension fees even if the examiner's amendment is made more than 3 months from the date of the final Office action." (A), (D) and (E) are wrong because MPEP § 706.07(f), part (H), states, "Note that an examiner's amendment may not be made more than 6 months from the date of the final Office action, as the application would be abandoned at that point by operation of law." In (A), when an examiner's amendment is mailed exactly 5 months after Xavier's reply, the examiner's amendment would be made more than 6 months after the Office action.

A registered practitioner timely files a petition under 37 CFR 1.181 while the application is pending before the primary examiner to challenge the prematureness of the final rejection that set a shortened statutory period for reply. Assume the petition is filed within two months of the date on the final rejection. What is the next response that should be docketed by the practitioner in accordance with the patent laws, rules and the procedures as related in the MPEP to avoid a penalty or payment of fees? A A reply to the final rejection within 6 months. B A status inquiry 6 months after filing the petition. C A reply to the final rejection within the shortened statutory time period set for reply in the final rejection. D No reply is necessary until a decision is received on the petition. E All of the above.

C Related MPEP Chapter(s): MPEP 700 - Examination of Applications Answer Reasoning:ANSWER: (C) is correct. 37 CFR § 1.181(f); MPEP §§ 714.13 and 1002: 37 CFR § 1.181(f) provides "The mere filing of a petition will not stay the period for reply to an Examiner's action which may be running against an application, nor act as a stay of other proceedings." MPEP §§ 714.13 states "[i]t should be noted that under 37 CFR 1.181(f), the filing of a 37 CFR 1.181 petition will not stay the period for reply to an examiner's action which may be running against an application." See also MPEP § 1002. Thus, if a petition to vacate a final rejection as premature is filed within 2 months from the date of the final rejection, the period for reply to the final rejection is not extended even if the petition is not reached for decision within that period. However, if the petition is granted and the applicant has filed an otherwise full reply to the rejection within the period for reply, the case is not abandoned. (C) is correct because the petition does not stay the time for responding to the final and a reply should be filed within the shortened statutory period to avoid fees. (A) is incorrect because any reply after the shorted statutory period set for reply in the final rejection will require at least payment of fees for an extension of time. MPEP § 1002 states "The mere filing of a petition will not stay the period for replying to an examiner's action which may be running against an application, nor act as a stay of other proceedings (37 CFR 1.181(f)). For example, if a petition to vacate a final rejection as premature is filed within 2 months from the date of the final rejection, the period for reply to the final rejection is not extended even if the petition is not reached for decision within that period." (B) is incorrect because the application will be abandoned 6 months after the date of the final rejection for lack of reply to the final rejection, 35 U.S.C. 133, and a status inquiry filed 6 months after filing the petition would be in an abandoned application. Moreover, a petition status inquiry filed at any time is not a proper reply to the final rejection. See 37 CFR §§ 1.113(c) and 1.116. (D) is incorrect because the filing of a petition does not stay the period for reply. 37 CFR § 1.181(f); MPEP §§ 714.13 and 1002. (E) is incorrect because (C) is correct, and (A), (B) and (D) are incorrect.

Which of the following statements is true in a pre-AIA filing? A In the context of 35 USC 102(b), a magazine need only be placed in the mail to be effective as a printed publication. B The earliest date declassified printed material may be taken as prima facie evidence of prior knowledge under 35 USC 102(a) is as of the date the material is cataloged and placed on the shelf of a public library. C Declassified printed material is effective as a printed publication under 35 USC 102(b) as of the date of its release following declassification. D The American Inventors Protection Act (AIPA) amended 35 USC 102(e) to provide that U.S. patents, U.S. application publications, and certain international application publications can be used as prior art under 35 USC 102(e) based on their earliest effective filing date only against applications filed on or after November 29, 2000. E The American Inventors Protection Act (AIPA) amended 35 USC 102(e) to provide that U.S. patents, U.S. application publications, and certain international application publications can be used as prior art under 35 USC 102(e) based on their earliest effective filing date only against applications filed prior to November 29, 2000 which have been voluntarily published.

C Related MPEP Chapter(s): MPEP 700 - Examination of Applications Answer Reasoning:ANSWER: (C) is correct. MPEP § 707.05(f) states, "In the use of [declassified material] ... as an anticipatory publication, the date of release following declassification is the effective date of publication within the meaning of the statute." (A) is wrong. MPEP § 706.02(a) states, "A magazine is effective as a printed publication under 35 U.S.C. § 102(b) as of the date it reached the addressee and not the date it was placed in the mail." (B) is wrong. MPEP § 707.05(f) states, "For the purpose of anticipation predicated upon prior knowledge under 35 U.S.C. §102(a) the above noted declassified material may be taken as prima facie evidence of such prior knowledge as of its printing date even though such material was classified at that time." (D) and (E) are wrong. The AIPA amended 35 U.S.C. § 102(e) to provide that U.S. patents, U.S. application publications, and certain international application publications can be used as prior art under 35 U.S.C. § 102(e) based on their earliest effective filing date against applications filed on or after November 29, 2000, and applications filed prior to November 29, 2000 which have been voluntarily published. MPEP § 706.02(a).

Where a final rejection of claims has been made, any question of prematureness of the final rejection should be raised, if at all: A as a ground of appeal. B as the basis of a complaint before the Board of Patent Appeals and Interferences. C by petition under 37 CFR 1.181 while the application is pending before the examiner. D after 2 months from the examiner's answer plus mail room time, if no reply brief has been timely filed during an appeal to the Board of Patent Appeals and Interferences. E after a supplemental examiner's answer, pursuant to a remand from the Board of Patent Appeals and Interferences has been mailed.

C Related MPEP Chapter(s): MPEP 700 - Examination of Applications Answer Reasoning:ANSWER: (C) is most correct. 37 C.F.R. § 1.181; MPEP § 706.07(c). (A) and (B) are wrong. 37 C.F.R. § 1.181(a)(1); MPEP § 706.07(c). Prematureness of a final rejection is not appealable. 37 C.F.R. § 1.191(a). (D) and (E) are wrong because MPEP § 706.07(c) states, "Any question as to prematureness of a final rejection should be raised, if at all, while the application is still pending before the primary examiner." MPEP § 1210 indicates that jurisdiction is before the Board at the times set forth in (D) and (E).

The Office mailed an Office action containing a proper final rejection dated July 8, 2016. The Office action did not set a period for reply. On January 7, 2017, in reply to the final rejection, a registered practitioner filed a request for continued examination under 37 CFR 1.114, a request for a suspension of action under 37 CFR 1.103(c) to suspend action for three months, and proper payment all required fees. No submission in reply to the outstanding Office action accompanied the request for continued examination. No other paper was submitted and no communication with the Office was held until after Midnight, January 8, 2017. Which of the following statements accords with the USPTO rules and the procedures set forth in the MPEP? A If an appropriate reply is submitted within the three month period of suspension permitted under 37 CFR 1.103(c), the application will not be held abandoned. B The application will not be held abandoned if an appropriate reply is submitted within the three month period of suspension and it is accompanied by a showing that the reply could not have been submitted within the period set in the final rejection. For example, the reply includes a showing based on an experiment that required 8 months to conduct. C No reply will prevent the application from being held abandoned. D If, on January 10, 2017, the primary examiner and applicant agree to an examiner's amendment that places the application in condition for allowance and a notice of allowance is mailed within the three month period of suspension, application X will not be held abandoned. E No other submission by applicant is necessary because application X is still pending. The examiner is required to act on the request for continued examination after expiration of the three month period of suspension.

C Related MPEP Chapter(s): MPEP 700 - Examination of Applications Answer Reasoning:ANSWER: (C) is the most correct answer. As stated in MPEP § 709, under the heading "Request By The Applicant," subheading "Request for Suspension Under 37 CFR 1.103(b) or (c)," "The Office will not grant the requested suspension of action unless the following requirements are met: (A) the request must be filed with the filing of a CPA or an RCE...(1) if the request is filed with an RCE, the RCE must be in compliance with 37 CFR 1.114, i.e., the RCE must be accompanied by a submission and the fee set forth in 37 CFR 1.17(e). Note that the payment of the RCE filing fee may not be deferred and the request for suspension cannot substitute for the submission." The RCE was improper because no submission in reply to the outstanding Office action accompanied the RCE. Since the RCE was improper, the Office will not recognize the request for suspension. The time period set in the final rejection continues to run from the mail date of the Office action. Since the Office action did not set a period for reply, applicant has a maximum period of six months for reply. A reply was due on January 8, 2017. Since the RCE was improper and the Office did not recognize the request for suspension, the application became abandoned at Midnight of January 8, 2017.

n which of the following final Office action rejections is the finality of the Office action rejection in accordance with the USPTO rules and the procedures set forth in the MPEP? A The final Office action rejection is in a second Office action and uses newly cited art under pre-AIA 35 USC 102(b) to reject unamended claims that were objected to but not rejected in a first Office action. B The final Office action rejection is in a first Office action in a continuation-in-part application where at least one claim includes subject matter not present in the parent application. C The final Office action rejection is in a first Office action in a continuing application, all claims are drawn to the same invention claimed in the parent application, and the claims would have been properly finally rejected on the grounds and art of record in the next Office action if they had been entered in the parent application. D The final Office action rejection is in a first Office action in a substitute application that contains material that was presented after final rejection in an earlier application but was denied entry because the issue of new matter was raised. E None of the above.

C Related MPEP Chapter(s): MPEP 700 - Examination of Applications Answer Reasoning:ANSWER: (C) is the most correct answer. See MPEP § 706.07(b). (A) is incorrect because a final rejection is not proper on a second action if it includes a rejection on newly cited art other than information submitted in an information disclosure statement under 37 CFR 1.97(c). MPEP § 706.07(a). (B) is incorrect because it is improper to make final a first Office action in a continuation-in-part application where any claim includes subject matter not present in the parent application. MPEP § 706.07(b). (D) is incorrect because it is improper to make final a first Office action in a substitute application where that application contains material, which was presented in the earlier application after final rejection, or closing of prosecution but was denied entry because the issue of new matter was raised. MPEP § 706.07(b). (E) is incorrect because (C) is correct.

To satisfy the written description requirement of the first paragraph of 35 USC 112, an applicant must show possession of the invention. An applicant's lack of possession of the invention may be evidenced by: A Describing an actual reduction to practice of the claimed invention. B Describing the claimed invention with all of its limitations using such descriptive means as words, structures, figures, diagrams, and formulas that fully set forth the claimed invention. C Requiring an essential feature in the original claims, where the feature is not described in the specification or the claims, and is not conventional in the art or known to one of ordinary skill in the art. D Amending a claim to add a limitation that is supported in the specification through implicit or inherent disclosure. E Amending a claim to correct an obvious error by the appropriate correction.

C Related MPEP Chapter(s): MPEP 700 - Examination of Applications Answer Reasoning:ANSWER: (C) is the most correct answer. See, "Guidelines for Examination of Patent Applications under 35 U.S.C. § 112, ¶ 1, 'Written Description' Requirement," MPEP 2163. "The claimed invention as a whole may not be adequately described if the claims require an essential or critical feature that is not described in the specification and is not conventional in the art or known to one of ordinary skill in the art." (A) is not the most correct answer. See, "Guidelines for Examination of Patent Applications under 35 U.S.C. § 112, ¶ 1, 'Written Description' Requirement," MPEP 2163. Describing an actual reduction to practice of the claimed invention is a means of showing possession of the invention. (B) is not the most correct answer. See, "Guidelines for Examination of Patent Applications under 35 U.S.C. § 112, ¶ 1, 'Written Description' Requirement," MPEP 2163. (D) is not the most correct answer. See, "Guidelines for Examination of Patent Applications under 35 U.S.C. § 112, ¶ 1, 'Written Description' Requirement," MPEP 2163, which states, "While there is no in haec verba requirement, newly added claim limitations must be supported by in the specification through express, implicit, or inherent disclosure." (E) is not the most correct answer. See, "Guidelines for Examination of Patent Applications under 35 U.S.C. § 112, ¶ 1, 'Written Description' Requirement," MPEP 2163, left column, second paragraph, which states, "An amendment to correct an obvious error does not constitute new matter where one skilled in the art would not only recognize the existence of the error in the specification, but also recognize the appropriate correction."

Applicant received a Final Rejection with a mail date of Tuesday, February 29, 2016. The Final Rejection set a 3-month shortened statutory period for reply. Applicant files an Amendment and a Notice of Appeal on Monday, March 27, 2016. The examiner indicates in an Advisory Action that the Amendment will be entered for appeal purposes, and how the individual rejection(s) set forth in the final Office action will be used to reject any added or amended claim(s). The mail date of the examiner's Advisory Action is Wednesday, May 31, 2016. In accordance with the USPTO rules and the procedures set forth in the MPEP, which of the following dates is the last date for filing a Brief on Appeal without an extension of time? A Saturday, May 27, 2016. B Monday, May 29, 2016 (a Federal holiday, Memorial Day). C Tuesday, May 30, 2016. D Wednesday, May 31, 2016. E Tuesday, August 29, 2016.

C Related MPEP Chapter(s): MPEP 700 - Examination of Applications Answer Reasoning:ANSWER: (C). Although the "two month rule" was invoked by a rapid applicant response after a final rejection, the Appeal Brief is still due two months from the filing date of the Notice of Appeal unless an extension has been requested and paid for. Hence, Tuesday, May 30.

A patent application was filed on November 1, 2016 for the invention of J.J. Smithy. The application has no priority or benefit claims to any other application. Claims in the application are separately rejected under 35 USC 102 as being anticipated by each of the following references. Which reference can be properly applied under 35 U.S.C. 102(a)(2) in accordance with the patent laws, rules and procedures as related in the MPEP? A A WIPO publication of an international application under PCT Article 21(2), which has an international filing date of October 3, 2017, was published in English and designated the United States. B A U.S. patent by J.J. Smithy that has a filing date of September 5, 2016. C A U.S. application publication under 35 U.S.C. 122(b) by inventor Jones that was filed on August 8, 2016. D A journal article by Marks published on October 11, 2016. E All of the above.

C Related MPEP Chapter(s): MPEP 700 - Examination of Applications Answer Reasoning:ANSWER: The correct answer is (C). 35 U.S.C. § 102(a)(2). The application publication is a proper reference under 35 U.S.C. 102(a)(2) because it was filed by another prior to the filing date of the invention. (A) is incorrect. The reference in answer (A) is not a proper reference because its international filing date was too late. The reference in (B) is not a proper reference under 35.U.S.C. § 102(a)(2) because the reference is not by another. The reference in (D) is not a proper reference under 35 U.S.C. § 102(a)(2) because 35 U.S.C. § 102(a)(2) refers to patents and patent applications, not journal articles. (E) is not correct because (C) is correct and (A), (B) and (D) are incorrect.

A first Office action on the merits rejecting Claim 1 under 35 USC 103 as being obvious in view of reference A set a three month shortened statutory period for reply. A registered practitioner files a timely response (without an extension of time) to the first Office action amending Claim 1 to include a limitation not found in reference A or any other prior art of record. However, the limitation also lacks support in applicant's original disclosure, i.e., it is new matter. Which of the following courses of action, if taken by the primary examiner, would be in accord with the USPTO rules and the procedures set forth in the MPEP? A Hold the application abandoned after expiration of the three month shortened statutory period for reply because an amendment adding new matter to the claims is not a bona fide response. B Consider the new matter and reject Claim 1 under 35 USC 101 because a claim that recites new matter lacks utility. C Consider the new matter and treat Claim 1, determining whether the invention as claimed with the new matter, would have been obvious in view of reference A, and reject Claim 1 under 35 USC 112, first paragraph, for lack of support in the original disclosure for new matter. D Ignore the new matter and reject Claim 1 again under § 103 in view of reference A. E All of the above.

C Related MPEP Chapter(s): MPEP 700 - Examination of Applications MPEP 2100 - Patentability Answer Reasoning:ANSWER: (C) is the most correct answer. See MPEP §§ 706.03(o) and 2143.03. MPEP § 2143.03, under the heading "Limitations Which Do Not Find Support In The Original Specification Must Be Considered," states: "When evaluating claims for obviousness under 35 U.S.C. § 103, all the limitations of the claims must be considered and given weight, including limitations which do not find support in the specification as originally filed (i.e., new matter)." In (C), the examiner considered the new matter as required. MPEP § 706.03(o) states, "In amended cases, subject matter not disclosed in the original application is sometimes added and a claim directed thereto. Such a claim is rejected on the ground that it recites elements without support in the original disclosure under 35 U.S.C. 112, first paragraph, Waldemar Link, GmbH & Co. v. Osteonics Corp. 32 F.3d 556, 559, 31 USPQ2d 1855, 1857 (Fed. Cir. 1994); In re Rasmussen, 650 F.2d 1212, 211 USPQ 323 (CCPA 1981)." (A) is incorrect. An amendment adding new matter is not necessarily a non-bona fide response as (A) implies. Moreover, abandonment is not proper after the expiration of the period for response since even if the response is considered non-responsive, applicant would be notified and given the remaining time period plus available extensions of time to reply. See MPEP § 714.03. (B) is incorrect. The mere fact that a claim recites new matter does not mean that the claim lacks utility. See MPEP § 2107.01, under the heading "Relationship Between 35 U.S.C. 112, First Paragraph, And 35 U.S.C. 101," discussing the difference between new matter under 35 U.S.C. § 112(1) and lack of utility under 34 U.S.C. § 101. (D) is incorrect because it contradicts MPEP § 2143.03, which requires the examiner to consider new matter. (E) is incorrect inasmuch as (A), (B), and (D) are incorrect and (C) is correct.

Adams filed Application X on March 1, 2001. Beth filed application Y on May 1, 2001. Neither application has been published. Applications X and Y are copending and commonly assigned. Earlier filed application X claims the same invention as claimed in application Y using identical language. In accordance with the MPEP, which of the following actions should the examiner or assignee follow? A The claims to the same invention in application Y should be rejected under 35 USC 102(a) as being anticipated by application X. B The claims to the same invention in application Y should be rejected under 35 USC 102(b) as being anticipated by application X. C The claims to the same invention in application Y should be rejected under 35 USC 102(e) as being provisionally anticipated by application X. D The common assignee should file a terminal disclaimer in application Y to avoid any question of double patenting. E The claims to the same invention in application Y should be rejected under 35 USC 102(e) as being anticipated by application X.

C Related MPEP Chapter(s): MPEP 800 - Restriction in Applications Filed Under 35 U.S.C. 111; Double Patenting Answer Reasoning:ANSWER: (C) is correct 35 U.S.C. § 102(e); MPEP § 804, Chart III-A. (C) is correct because section I. of MPEP § 706.02(f) states, "If (1)...the applications are commonly assigned and (2) the effective filing dates are different, then a provisional rejection of the later filed application should be made." (A) is wrong because the facts do not indicate prior knowledge or use by others, or that the application Y is patented or published. (B) is wrong because the facts do not indicate that the application Y is patented or published, or the existence of "public use" or "on sale" bars. (D) is wrong. This is a statutory double patenting situation that cannot be avoided by filing a terminal disclaimer. See In re Bartfeld, 17 USPQ2d 1885 (Fed. Cir. 1991). MPEP § 706.02(f), section I; MPEP § 804.02, part I. (E) is wrong. It is improper to make a nonprovisional rejection under § 102(e) in the circumstances described in the question. Choice (E) provides for an improper nonprovisional rejection under § 102(e). MPEP § 804, Chart I-A.

A U.S. patent application for inventor William Tull discloses a target-shooting gun for improved accuracy, and a bullet impregnated with a new chemical composition. The new chemical composition minimizes damage to a target struck by the bullet. In a non-final Office action, an examiner includes a restriction requirement between a group of claims drawn to the target-shooting gun (Group 1), and a group of claims drawn to the bullet (Group 2). Which of the following, included in a timely reply to the non-final Office action, preserves Tull's right to petition for review of the restriction requirement, if the requirement is made final? A A reply that distinctly points out supposed errors in the restriction requirement, and also states, "The restriction requirement is traversed, and no election is made, thereby preserving Applicant's right to petition for review of the restriction requirement." B A reply that states, "Applicant elects Group 2 and traverses the restriction requirement because the requirement for restriction between Group 1 and Group 2 is in error." C A reply that distinctly and specifically points out supposed errors in the restriction requirement, and states, "Applicant traverses the restriction requirement and elects Group 2." D A reply that states, "The restriction requirement between Group 1 and Group 2 is traversed because it is in error, and no election is made, thereby preserving Applicant's right to petition for review of the restriction requirement." E None of the above.

C Related MPEP Chapter(s): MPEP 800 - Restriction in Applications Filed Under 35 U.S.C. 111; Double Patenting Answer Reasoning:ANSWER: (C) is the correct answer. 37 C.F.R. 1.144; MPEP §§ 818.03(a)-(c). (A), (B), and (D) are each incorrect because no supposed errors in the restriction requirement are distinctly and specifically pointed out. (A) and (D) are further incorrect because no election is made. (E) is incorrect because (C) is correct.

A claim in a pending patent application stands rejected under 35 USC 103 as being obvious over Kim in view of Lance. The Kim and Lance references are both U.S. Patents issued on respective applications filed before the date of the application in question. In the rejection, the primary examiner asserted that no determination of the level of ordinary skill in the art was necessary because the subject matter of the application and of Kim and Lance were so easily understandable; and that the Kim reference relates to the applicant's endeavor. The examiner properly found motive in Kim and Lance for combining the references, but the motive would produce a benefit different from that offered by applicant's invention. Neither reference teaches or suggests the ambiguous limitation. In the rejection under 35 USC 103, the examiner did not address an ambiguous limitation in the claim. However, the examiner separately rejected under 35 USC 112(b) as indefinite due to the ambiguity. According to the patent laws, rules and procedures as related in the MPEP, which of the following arguments, if true, would overcome the rejection? A The examiner improperly asserted that, because the subject matter of the application and of Kim and Lance were so easily understandable, a factual determination of the level of skill in the art was unnecessary. B The Kim reference is nonanalogous art because, although it relates to the field of the applicant's endeavor, it is not pertinent to the particular problem with which the applicant was concerned. C The reason given by the examiner to combine Kim and Lance is to obtain a benefit different from that offered by the applicant's invention. D Neither the Kim nor Lance references teaches or suggests the ambiguous claimed limitation that the examiner separately rejected as indefinite. E All of the above.

CREDIT GIVEN FOR ALL ANSWERS. (A) would be the most likely answer in view of KSR.

In accordance with the patent laws, rules and procedures as related in the MPEP, which of the following statements regarding claim interpretation is the most correct? A A claim having the transition term "comprising" is limited to only the limitations, elements or steps recited in the claim, and is not inclusive or open-ended of other unrecited elements or steps. B The transition term "consisting essentially of" limits the claim to the limitations recited in the claim and additional elements or steps which do not materially affect the basic and novel characteristics of the claimed invention. C A claim having the transition term "consisting of" is not limited to the elements or steps recited in the claim, but can include elements or steps other than those recited in addition to any impurities ordinarily associated therewith. D A claim which depends from a claim which claims an invention "consisting of" the recited elements or steps can add an element or step to further limit the claimed invention. E All of the above.

Correct Answer(s): B Related MPEP Chapter(s): MPEP 2100 - Patentability Answer Reasoning:ANSWER: (B) is the most correct answer. MPEP § 2111.03 (fourth paragraph) states, in reliance upon In re Herz, 537 F.2d 549, 551-52, 190 USPQ 461, 463 (CCPA 1976), that "[t]he transitional phrase 'consisting essentially of' limits the scope of a claim to the specified materials or steps 'and those that do not materially affect the basic and novel characteristic(s)' of the claimed invention." (A) is incorrect. The statement is contradicted by MPEP § 2111.03 (second paragraph), which states, in reliance upon Genentech, Inc. v. Chiron Corp., 112, F.3d 495, 501, 42 USPQ2d 1608, 1613 (Fed. Cir. 1997) that "[t]he transition term 'comprising', which is synonymous with 'including,' 'containing,' or 'characterized by,' is inclusive or open-ended and does not exclude additional, unrecited elements or method steps. (C) is incorrect. The statement is contradicted by MPEP § 2111.03 (third paragraph), which states, in reliance upon In re Gray, 53 F.2d 520, 11 USPQ 255 (CCPA 1931) that "[t]he transitional phrase 'consisting of' excludes any element, step , or ingredient not specified in the claim." (D) is incorrect. The statement is directly contradicted by MPEP § 2111.03 (third paragraph), which states "[a] claim which depends from a claim which 'consists of' the recited elements or steps cannot add an element or step." (E) is incorrect because (A), (B) and (C) are incorrect.

In accordance with the patent laws, rules and procedures as related in the MPEP, which of the following fees may not be reduced by 50 percent for "small entities"? A The basic filing fee for a design patent application. B The fee for a utility examination. C The fee for a petition for an extension of time. D The fee for recording a document affecting title. E The maintenance fee due at 3 years and six months after grant.

Correct Answer(s): D Related MPEP Chapter(s): MPEP 300 - Ownership and Assignment Answer Reasoning:ANSWER: (D) is the most correct answer. 35 U.S.C. § 41(h); MPEP §§ 302.06; 509.02. 35 U.S.C. § 41(h) specifies that the fees "charged under subsection (a) or (b) shall be reduced by 50 percent with respect to their application to any small business concern as defined under section 3 of the Small Business Act, and to any independent inventor or nonprofit organization as defined in regulations issued by the Director." Since the fee for a document affecting title is charged pursuant to 35 U.S.C. § 41(d)(1), it not subsection (a) or (b), and it is not entitled to a small entity discount. See also MPEP 509.02, which states, "[o]ther fees, established under section 41 (c) or (d) of Title 35, United States Code, are not reduced for small entities since such a reduction is not permitted or authorized by Public Law 97-247. Fees which are not reduced include . . . miscellaneous fees and charges, 37 CFR 1.21." Fees for recording documents affecting title are set under 37 CFR § 1.21(h). See MPEP § 302.06. (A) is entitled to a small entity discount because it is a fee charged pursuant to 35 U.S.C. 41(a)(3)(A). (B) is entitled to a small entity discount because it is charged pursuant to 37 CFR 1.16(o). (C) is entitled to a small entity discount because it is charged pursuant to 35 U.S.C. 41(a)(8). (E) is entitled to a small entity discount because it is charged pursuant to 35 U.S.C. 41(b)(1).

A registered practitioner filed a utility patent application on May 15, 2000 pursuant to 35 USC 111(a) claiming a detergent composition. On May 15, 2002 the Office mailed a non-final Office action setting a 3-month period for reply. A proper reply was mailed on August 15, 2002 by first-class mail with sufficient postage to the USPTO. The reply was received by the USPTO on September 15, 2002. On September 30, 2002, the Office mailed a final Office action. On October 15, 2002, the Office received a Request for Continued Examination (RCE) meeting all of the requirements of 37 CFR 1.114. On October 30, 2002, the USPTO mailed a Notice of Allowance in view of the RCE and amendment. The utility application issued on February 11, 2003. Which of the following statements is in accord with the patent laws, rules and procedures as related in the MPEP concerning the amount of additional term applicant X would receive because of Patent Term Adjustment (PTA)? A The patentee would earn PTA because the Office failed to mail at least one notification under 35 USC 132 or notice of allowance under 35 USC 151 no later than fourteen months after the date the application was filed under 35 USC 111(a) but would lose some earned PTA because applicant did not file a response to the non-final rejection within three months. B The patentee would earn PTA because the Office failed to mail at least one notification under 35 USC 132 or notice of allowance under 35 USC 151 no later than fourteen months after the date the application was filed under 35 USC 111(a) and would not lose any earned PTA because applicant filed a response to the non-final rejection within three months. C The patentee would not earn any additional time under the provisions of PTA because the application is utility application, not a design application. D The patentee would not earn any additional time because the application was filed prior to May 29, 2000 and the filing of the RCE would not make the application eligible for PTA. E The patentee would earn PTA because the filing of the RCE on October 15, 2002 makes the application eligible for PTA and the Office did not mail at least one notification under 35 USC or notice of allowance under 35 USC 151 within 14 months of the date the application was filed under 35 USC 111(a).

Correct Answer(s): D Related MPEP Chapter(s): MPEP 700 - Examination of Applications Answer Reasoning:ANSWER: The most correct answer is (D). See 35 U.S.C. § 154(b); 37 CFR § 1.702(f); MPEP § 2730 (quoting section 1.702(f)). The application was filed prior to May 29, 2000 and is ineligible for the provisions of Patent Term Adjustment (PTA). Moreover, the filing of a Request for Continued Examination (RCE) under 35 U.S.C. § 132(b) and 37 CFR § 1.114 does not cause an application filed before May 29, 2000 to be entitled to the benefits PTA under the provisions of 35 U.S.C. § 154(b) and 37 CFR §§ 1.702-1.705. See MPEP § 2730. (A) and (B) are not correct answers because the application was filed prior to May 29, 2000, the eligibility date for applications to receive the benefit of PTA provisions of 35 U.S.C. § 154(b) and 37 CFR §§ 1.702 through 705. Answer choice (C) is not correct because utility applications, not design applications are subject to the PTA provisions and the answer suggests that design applications are eligible for PTA. Answer choice (E) is not a correct answer because the application is not eligible for PTA and filing an RCE does not make an ineligible application eligible for PTA. Design patents are granted for fourteen year terms from the grant of the patent. 35 U.S.C. § 171. Utility patents are subject to patent term adjustment. 35 U.S.C. § 154(b)

Which of the following documents is not open to public inspection? A The abandoned parent application of a divisional application. A patent was granted on the divisional application, which refers to the abandoned parent application. B Assignment document relating to both an issued patent and a patent application not published under 35 USC 122(b). C Assignment document relating to a pending reissue application. D Copy of assignment record relating to both a pending patent application and an abandoned patent application not published under 35 USC 122(b). E Assignment document relating to both an abandoned patent application not published under 35 USC 122(b) and a pending reissue application.

D Related MPEP Chapter(s): MPEP 100 - Secrecy, Access, National Security, and Foreign Filing Answer Reasoning:ANSWER: (D) is correct. (A) is wrong. 37 C.F.R. § 1.14(e)(2); MPEP § 103, application files are available upon request because the divisional application refers to the abandoned parent application, and the division issued as a patent, causing the application to be open to inspection. (B), (C) and (E) are wrong and (D) is correct. MPEP § 301.01.

In accordance with Chapter 100 of the Manual of Patent Examining Procedure and 35 USC 122, which of the following statements is not true? A All requests for reexamination and related patent files are available to the public subject to the availability of the reexamination file. B The Board of Patent Appeals and Interferences handles all petitions for access to applications involved in an interference. C An abandoned application referenced in a U.S. patent application publication, U.S. patent or a U.S. application that is open to public inspection may be ordered for inspection by any member of the public. D The assignee of record of a part interest in an application may always intervene in the prosecution of the application, appointing a registered attorney or agent of his or her own choice, without participation by any or all other assignees. E All provisional patent applications are screened upon receipt in the USPTO for subject matter that, if disclosed, might impact the national security, and such applications are referred to appropriate agencies for consideration of restrictions on disclosure of the subject matter.

D Related MPEP Chapter(s): MPEP 100 - Secrecy, Access, National Security, and Foreign Filing Answer Reasoning:ANSWER: The best choice is (D). Choices (A), (B), (C) and (E) are each TRUE statements. See MPEP at pp. 100-13, 100-12, 100-8 and 100-18, respectively. Choice (D) is NOT TRUE because only the assignee of record of the entire interest in an application may intervene in the prosecution of the application, appointing an attorney or agent of his or her own choice. MPEP at p. 100-16. An assignee of record of a part interest is, however, entitled to inspect the application. Id.

Prosecution before the primary examiner results in the rejection of claim 1. Claim 2 was objected to as being allowable except for its dependency from claim 1. Independent claim 3 has been allowed. The rejection of claim 1 is properly appealed to the Board of Patent Appeals and Interferences. The Board properly affirms the rejection of claim 1. Appellant has filed no response to the decision of the Board, the appellant has taken no action, and the time for filing an appeal to the court or a civil action has expired. In accordance with the patent laws, rules and procedures as related in the MPEP, which of the following actions is the most appropriate response by the examiner? A The examiner should hold the application abandoned. B The examiner should cancel claim 1, convert dependent claim 2 into independent form by examiner's amendment, and allow the application. C The examiner should set a 1-month time limit in which appellant may rewrite the dependent claim in independent form. D The examiner should cancel claims 1 and 2 and allow the application with claim 3 only. E None of the above.

D Related MPEP Chapter(s): MPEP 1200 - Appeal Answer Reasoning:ANSWER: (D) is the most correct answer. 37 CFR § Part 41; MPEP § 1214.06. This case is specifically set forth in MPEP § 1214.06 under the heading "Claims Stand Allowed." Answers (A), (B) and (C) apply only if no claims stand allowed in the application. They are incorrect because the facts state that claim 3 was allowed. See MPEP § 1214.06, under the heading "No Claims Stand Allowed." (B) is incorrect. See MPEP § 1214.06 under the heading "Claims Stand Allowed." Where one or more other claims stand allowed, the examiner is not authorized to convert to independent form a dependent claim that has been objected to (but not allowed or rejected) based on its dependency to a rejected claim. (C) is incorrect. See MPEP § 1214.06 under the heading "Claims Stand Allowed." Where one or more other claims stand allowed, the examiner is not authorized to provide appellant with time to rewrite a dependent claim into independent form where the dependent claim was objected to (but not allowed or rejected) based on its dependency to a rejected claim.

In accordance with USPTO rules and procedures set forth in the MPEP, which of the following is not a proper basis on which the PTAB may remand a case to the examiner? A Remand for a fuller description of the stated rejection. B Remand for a clearer explanation of the pertinence of the references. C Remand for a selection by the primary examiner of a preferred or best ground of rejection when multiple rejections of a cumulative nature have been made by the examiner. D Remand to the primary examiner with instructions to consider an affidavit not entered by the examiner which was filed after the final rejection but before the appeal. E Remand to the primary examiner to prepare a supplemental examiner's answer in response to a reply brief.

D Related MPEP Chapter(s): MPEP 1200 - Appeal Answer Reasoning:ANSWER: (D) is the most correct answer. See MPEP § 1211.02. (D) is not a proper basis for remand because the Board has no authority to require the examiner to consider an affidavit which has not been entered after final rejection and which was filed while the application was pending before the examiner. Pursuant to 37 CFR § Part 41, "[a]ffidavits...submitted after the case has been appealed will not be admitted without a showing of good and sufficient reasons why they were not earlier presented." The facts are silent regarding whether such a showing was made. However, as discussed in MPEP § 715.09, review of an examiner's refusal to enter and consider an affidavit as untimely if by petition and not by appeal to the Board. In re Deters, 515 F.2d 1152, 185 USPQ 644 (CCPA 1975); Ex parte Hale, 49 USPQ 209 (Bd. App. 1941)." Thus, remand by the Board cannot be expected. Support for each of answers (A), (B), (C) and (E) is specifically provided for in MPEP § 1211.

A U.S. patent was granted on May 8, 2015. The sole independent claim in the patent is directed to a combination of elements ABCD. A registered practitioner filed a reissue application on April 11, 2017 to narrow sole independent claim. In the reissue application, the independent claim is amended to a combination to elements ABCDE. The reissue application is accompanied by a transmittal letter stating that the application was filed to narrow a claim, that all inventors could not be located to sign the reissue oath or declaration at that time, and that a declaration would be submitted in due course. No other amendments to the claims were filed on April 11, 2017. On May 8, 2017, a declaration signed by all inventors is filed declaring that they had claimed less than they had a right to claim, and that the error arose without deceptive intent. The inventors also filed on May 8, 2017 a preliminary amendment deleting element A from the sole independent claim leaving elements BCDE. The amendment and declaration are filed using the provisions of 37 CFR 1.10. The practitioner included an authorization to charge the practitioner's deposit account for any necessary fees. Which of the following actions by the primary the examiner in the first Office action is in accordance with the patent laws, rules and procedures as related in the MPEP? A Reject all the claims based upon a broadening reissue outside the two year statutory period authorized by 35 USC 251 since applicant did not file a broadened reissue claim at the time of filing. B Reject all the claims based upon a broadening reissue outside the two year statutory period authorized by 35 USC 251 since applicant did not file a claim to a broadened reissue claim within the two year period set by 35 USC 251. C Reject all the claims based upon a broadening reissue outside the two year statutory period authorized by 35 USC 251 since applicant's indication in the transmittal letter indicated that the filing of the reissue application was a narrowing reissue and that the broadening amendment was not permissible even if filed within the two-years from the grant of the original patent. D Determine that the application is a proper broadening reissue and perform an examination and issue an Office action in due course. E Determine that the application is a proper broadening reissue and reject the claims under the recapture doctrine since the claims are broader than the issued claims.

D Related MPEP Chapter(s): MPEP 1400 - Correction of Patents Answer Reasoning:ANSWER: (D) is the most correct answer. MPEP §§ 1403 and 1412.03, under the heading "When A Broadened Claim Can Be Presented." A broadening reissue claim must be filed within the two years from the grant of the original patent. (D) is the most correct and the examiner should examine the case as any other application and address appropriate issues concerning reissue examination. See Switzer v. Sockman, 333 F.2d 935, 142 USPQ 226 (CCPA 1964) (a similar rule in interferences). Since applicant filed the amendment by Express Mail, the amendment is treated as being filed with the USPTO on the date of deposit with the US Postal Service. Therefore, (A), (B) and (C) are incorrect answers. A reissue application can be granted a filing date without an oath or declaration, or without the filing fee being present. See 37 CFR § 1.53(f). Applicant will be given a period of time to provide the missing parts and to pay the surcharge under 37 CFR § 1.16(e). See MPEP § 1410.01. Choice (E) is not correct since the mere deletion of an element of a claim does not automatically raise a ground of rejection based on the recapture doctrine. See MPEP § 1412.02

An international application is filed in the United States Receiving Office on September 18, 2016. In accordance with the PCT and USPTO rules and the procedures set forth in the MPEP, which of the following will result in the application not being accorded an international filing date of September 18, 2016? A The description and claims are in German. B The Request is signed by a registered attorney rather than the applicant. C The sole applicant is a Canadian resident and national. D The application does not contain a claim. E The application is not accompanied by any fees.

D Related MPEP Chapter(s): MPEP 1800 - Patent Cooperation Treaty Answer Reasoning:ANSWER: The correct answer is (D). PCT Article 11(1)(iii)(e); 35 U.S.C. § 363; 37 CFR § 1.431(a); MPEP § 1810. Under PCT Article 11(1)(iii)(e), to be accorded an international filing date, an application must have "a part which on the face of it appears to be a claim or claims." (A) and (C) are incorrect. Under PCT Rule 11, if an application is not filed in the prescribed language or is filed by an applicant for which the Office to which the application is submitted is not competent, such application will be forwarded to the International Bureau which will act as receiving Office and accord a filing date as of the date of receipt in the USPTO. (B) is not correct. The Request may be signed by an attorney or agent who is registered to practice before the USPTO. In such a situation the application will be accorded an international filing date of September 18, 2016, and under PCT Article 14 an invitation to correct the defect will be mailed. See MPEP § 1805, paragraph 7; MPEP 1810, under the heading "The 'International Filing Date," second paragraph. (E) is also incorrect. Under PCT Rules 14.1(c), 15.4(a), 16.1(f), and 16bis.1 the fees may be paid at a date later than the original receipt date.

Which of the following statements does not accord with proper USPTO practice and procedure? A A protest may be filed by an attorney or other representative on behalf of an unnamed principal. B Information which may be relied on in a protest includes information indicating violation of the duty of disclosure under 37 CFR 1.56. C While a protest must be complete and contain a copy of every document relied on by the protestor, a protest without copies of prior art documents will not necessarily be ignored. D A pre-AIA protest must be submitted prior to the date the application was published or the mailing of a notice of allowance, whichever occurs later, provided the application is pending. E Since a protestor is not authorized to participate in the prosecution of a pending application, the examiner must not communicate in any manner with the protestor.

D Related MPEP Chapter(s): MPEP 1900 - Protest Answer Reasoning:ANSWER: (D) is the best answer. MPEP § 1901.04. (A) is a true statement. MPEP § 1901.01. (B) is a true statement. MPEP § 1901.02, paragraph (G). (C) is a true statement. MPEP § 1901.03. (E) is a true statement. MPEP § 1907.

A claim, as required by the second through fifth paragraphs of section 112, shall not be required in a _____________ patent application. A reissue B design C continuation D provisional E plant

D Related MPEP Chapter(s): MPEP 200 - Types, Cross-Noting, and Status of Application Answer Reasoning:ANSWER: (D) is correct. (D) is correct because 35 U.S.C. § 111(b)(2) states, "A claim, as required by the second through fifth paragraphs of section 112, shall not be required in a provisional application." MPEP § 201. (A) is wrong because an application for reissue must contain the entire specification, including the claims, and the drawings of the patent. 37 C.F.R. § 1.173(a). (B) is wrong because a design patent application contains a single claim. 37 C.F.R. § 1.53(b); MPEP § 1503.03. (C) is wrong because a continuation patent application is a second application for the same invention claimed in a prior nonprovisional application and filed before the original becomes abandoned or patented. MPEP § 201.07. (E) is wrong because 35 U.S.C. § 162 states, "The claim in the specification shall be in formal terms to the plant shown and described." MPEP § 1605.

Which of the following is not in accordance with the provisions of the MPEP? A Where joint inventors are named, the examiner should not inquire of the patent applicant concerning the inventors and the invention dates for the subject matter of the various claims until it becomes necessary to do so in order to properly examine the application. B Under 35 USC 119(a), the foreign priority benefit may be claimed to any foreign application that names a U.S. inventor as long as the U.S. named inventor was the inventor of the foreign application invention and 35 USC 119(a)-(d) requirements are met. C Where two or more foreign applications are combined in a single U.S. application, to take advantage of the changes to 35 USC 103 or 35 USC 116, the U.S. application may claim benefit under 35 USC 119(a) to each of the foreign applications provided all the requirements of 35 USC 119(a)-(d) are met. D One of the conditions for benefit under 35 USC 119(a) is that the foreign application must be for the same or a nonobvious improvement of the invention described in the United States application. E If a foreign application for which priority is being claimed under 35 USC 119 is filed in a country which does not afford similar privileges in the case of applications filed in the United States or to citizens of the United States and the foreign country is not a WTO member country, any claim for the foreign priority thereto by a U.S. application will not be effective.

D Related MPEP Chapter(s): MPEP 200 - Types, Cross-Noting, and Status of Application Answer Reasoning:ANSWER: (D) is the best answer since the inventions do not have to be identical in the respective foreign and U.S. applications. Rather, the US claim must only find support in the foreign filed specification. As to (A) through (C), see MPEP § 605.07 at p. 600-49, right column. As to (E), see 35 U.S.C. § 119 which provides that the previously filed application must have been filed in a country that affords similar privileges in the case of applications filed in the United States or to citizens of the United States or in a WTO member country.

The procedures in the MPEP do not require an applicant claiming foreign priority in a nonprovisional utility application to: A submit the processing fee set forth in 37 CFR 1.17(i) if the claim for priority or submission of the certified copy of the priority document is made after payment of the issue fee and before the patent is granted. B identify the foreign application for which priority is being claimed as well as any foreign application for the same subject matter having a filing date before that of the application for which priority is being claimed. C file the claim in the application. D have the same inventive entity listed in the foreign application as in the U.S. application in which the priority claim has been filed. E identify the intellectual property authority or country in or for which the foreign application was filed.

D Related MPEP Chapter(s): MPEP 200 - Types, Cross-Noting, and Status of Application Answer Reasoning:ANSWER: (D). There is no requirement as to the inventive entity being the same. As to (A), see 37 C.F.R. § 1.55(a)(2). As to (E), see MPEP 201.14 at p. 200-82 (right column). As to (C), see MPEP 201.14(a). As to (B), see 37 C.F.R. § 1.55(a)(1)(i) and MPEP 201.14(a).

In accordance with proper USPTO practice and procedure, a submission for a request for continued examination does not include: A An amendment of the drawings. B New arguments in support of patentability. C New evidence in support of patentability. D An appeal brief or reply brief (or related papers). E An amendment of the claims.

D Related MPEP Chapter(s): MPEP 700 - Examination of Applications Answer Reasoning:ANSWER: (D) is the most correct answer. 37 C.F.R. § 1.114(d), last sentence. (A), (B), (C), and (E) are not the most correct answers. Each is recognized as being a "submission" within the scope of 37 C.F.R. § 1.114(c).

Inventor Tip, a scientist in a pencil research laboratory, theorized that, based on the abrasive properties of moon dust, a highly efficient erasure can be made by adding a trace amount of moon dust to a normal pencil erasure formulation. Point, in the Sales department, determined that this would be perfect for a high end product. A U.S. patent application has been filed claiming a pencil erasure formulation with a trace amount of moon dust. An example of how to make the formulation with specified percentages of moon dust is presented therein. Thereafter, Tip learns about the duty to disclose information and he recalls signing a declaration under 37 CFR 1.63 stating that he had reviewed and understood the contents of the specification including the claims. Tip becomes concerned that the use of moon dust was only a theory and that to obtain patent would mislead the public to conclude that moon dust was actually used and found to be effective. The application has been allowed, but the issue fee has not yet been paid. Which of the following is most in accord with patent laws, rules and procedures as related in the MPEP? A Point is under a duty to disclose material information to the USPTO. B Tip is under a duty to disclose his concern regarding the moon rock information to the USPTO. C Both Point and Tip are under a duty to disclose material information to the UPSTO. D There is no duty to disclose information regarding how the moon rock formulation was developed to the USPTO. E Inasmuch as the application is allowed, an appropriate Request for Continued Prosecution pursuant to 37 CFR 1.114 needs to be filed accompanied by a information disclosure regarding the possibility of rejections under 35 USC 101, and 112, first paragraph.

D Related MPEP Chapter(s): MPEP 2000 - Duty of Disclosure Answer Reasoning:ANSWER: (D) is the most correct answer. 37 CFR § 1.56; MPEP § 2001.05. 37 CFR § 1.56(a) sets forth a duty to disclose information that is material to patentability. MPEP § 2001.05 states that "information is not material unless is comes within the definition of 37 CFR 1.56(b)(1) or (b)(2). If information is not material, there is no duty to disclose the information to the Office." The information that moon dust was never actually used is not material as defined under 37 CFR § 1.56(b)(1) or (2) which state that information is material if "(b)(1) It establishes, by itself or in combination with other information, a prima facie case of unpatentability of a claim; or (2) It refutes, or is inconsistent with, a position the applicant takes in: (i) Opposing an argument of unpatentability relied on b y the Office or, (ii) Asserting an argument of patentability." That the use of the moon dust as part of an erasure formulation was only theorized and not actually used is acceptable as is an example for making it. MPEP § 608.01(p), II, under the heading "Simulated or Predicted Test Results Or Prophetic Examples," states that "[s]imulated or predicted test results and prophetical examples (paper examples) are permitted in patent applications. ... Paper examples describe the manner and process of making an embodiment of the invention which has not actually been conducted." Care, however, must be taken not to state that an experiment was actually run or conducted when it was not and that "[n]o results should be presented as actual results unless they have actually been achieved." MPEP § 2004, item 8. (A) is incorrect. 37 CFR § 1.56(a) requires that individuals associated with the filing and prosecution of a patent application have a duty to disclose information to the Office. 37 CFR § 1.56(c) defines which individuals are associated with the filing and prosecution of a patent application and that "(c) Individuals associated with the filing or prosecution of a patent application within the meaning of this section are: (1) Each inventor named in the application; (2) Each attorney or agent who prepares or prosecutes the application; and (3) Every other person who is substantively involved in the preparation or prosecution of the application and who is associated with the inventor, with the assignee or with anyone to whom there is an obligation to assign the application." Point is part of the Sales department and no facts were presented that substantively involved him in the preparation or prosecution of the application. Additionally, as noted in the explanation relating answer to (D), the information given to Point by Tip was not material information. (B) is incorrect. While Tip would be an individual identified under 37 CFR § 1.56(c), there is no material information to be disclosed as noted in the explanation to (D). (C) is incorrect. As noted in the explanation relating to (D), the information is not material. Additionally, as noted in the explanation to answer (A), Point is not an individual defined by 37 CFR 1.56(c) as owing a duty. (E) is incorrect. As there is no requirement that it be explicitly stated that an invention has or has not been actually conducted, as noted in the explanation of (D), the prosecution need not be continued for the purpose of supplying an information disclosure statement regarding the development of the moon rock erasure formulation.

Which of the following phrases taken from an independent claim has an antecedent basis problem according to the patent laws, rules and the procedures as related in the MPEP? A "the center of the circle having ...," where the claim does not previously recite that the circle has a "center." B "the major diameter of the ellipse being ...," where the claim does not previously recite that the ellipse has a "major diameter." C "the outer surface of the sphere being ...," where the claim does not previously recite that the sphere has an "outer surface." D "the lever of the machine being located ...," where the claim does not previously recite a "lever." E "the area of the rectangle being ...," where the claim does not previously define an "area."

D Related MPEP Chapter(s): MPEP 2100 - Patentability Answer Reasoning:ANSWER: (D) is correct. "Inherent components of elements recited have antecedent basis in the recitation of the components themselves." MPEP § 2173.05(e). The MPEP provides an analogous example: "the limitation 'the outer surface of said sphere' would not require an antecedent recitation that the sphere have an outer surface." Id. (A), (B), (C), and (E) are all examples of things which inherently have the claimed characteristic and do not have an antecedent basis problem; that is, all circles have a center, all ellipses have a major diameter, all spheres have an outer surface, and all rectangles have an area, and these characteristics need not be provided with express antecedent basis. The ellipse example is from Bose Corp. v. JBL Inc., 61 USPQ2d 1216, 1219 (Fed. Cir. 2001) ("There can be no dispute that mathematically an inherent characteristic of an ellipse is a major diameter."). The lever recited in (D) is not an inherent component of a machine and therefore requires express antecedent basis.

An applicant's claim stands rejected under 35 USC 103 as being obvious over Larry in view of Morris. Larry and Morris are references published more than one year before applicant's effective filing. Although the examiner cites no suggestion or motivation for combining the references, they are, in fact, combinable. Which of the following arguments could properly show that the claim is not obvious? A The inventions disclosed by Larry and Morris cannot be physically combined. B Neither Larry nor Morris provides an express suggestion to combine the references. C As recognized by businessmen, the high cost of Larry's device teaches away from combining it with the simpler device of Morris. D Absent a suggestion or motivation or other permissible obviousness rationale, the examiner has not shown that combining Larry's with Morris's device would have been within the level of ordinary skill of the art. E None of the above.

D Related MPEP Chapter(s): MPEP 2100 - Patentability Answer Reasoning:ANSWER: (D) is correct. "The mere fact that references can be combined or modified does not render the resultant combination obvious unless the prior art also suggests the desirability of the combination." MPEP § 2143.01 (citing In re Mills, 916 F.2d 680, 16 USPQ2d 1430 (Fed. Cir. 1990)). Here, the examiner fails to show that substituting Larry's device for another type of device in Morris would have been desirable. (A) is incorrect. The test of obviousness is not whether the features or elements of the references are physically combinable. In re Keller, 642 F.2d 413, 425, 208 USPQ 871, 881 (CCPA 1981); In re Sneed, 710 F.2d 1544,1550, 218 USPQ 385,389 (Fed. Cir. 1983). (B) is incorrect. "The rationale to modify or combine the prior art does not have to be expressly stated in the prior art; the rationale may be expressly or impliedly contained in the prior art or it may be reasoned from knowledge generally available to one of ordinary skill in the art, established scientific principles, or legal precedent established by prior case law." MPEP § 2144 (citing In re Fine, 837 F.2d 1071, 5 USPQ2d 1596 (Fed. Cir. 1988); In re Jones, 958 F.2d 347, 21 USPQ2d 1941 (Fed. Cir. 1992)). Here, the argument overlooks the fact that a suggestion to combine Larry and Morris may be reasoned from knowledge generally available to one of ordinary skill in the art, established scientific principles, or legal precedent established by prior case law. (C) is incorrect. "The fact that a combination would not be made by businessmen for economic reasons does not mean that a person of ordinary skill in the art would not make the combination because of some technological incompatibility." MPEP § 2145 (citing In re Farrenkopf, 713 F.2d 714, 718, 219 USPQ 1, 4 (Fed. Cir. 1983)). Here, the high cost of Barry's device does not teach away from a person of ordinary skill in the art combining it with Lance's device. Also see KSR guidelines - TSM rationale.

The specification of an application does not disclose the utility of the claimed composition. In fact, the claimed invention is useful for shrinking a specific class of tumors. In a first Office action, the primary examiner has properly determined that the claims lack utility, and has rejected all of the composition claims under the first paragraph of 35 USC 112 as lacking utility. Which of the following responses is in accord with the USPTO rules and the procedures of the MPEP for persuading the examiner that the rejection is improper? A Explain that the rejection is statutorily improper because the first paragraph of section 112 is concerned with enablement and written description issues and therefore does not support a rejection for lack of utility. B Point out that the rejection is based on an erroneous finding by the examiner because the specification, in fact, clearly discloses that the composition in question possesses "useful biological" properties. C Show that the rejection is improper by filing probative evidence that the claimed composition has unambiguously proven to be useful for shrinking a specific class of tumors. D File declarations by persons with ordinary skill in the art stating that they would immediately appreciate that the claimed composition is useful for shrinking a specific class of tumors due to the fact that similar compositions having the same characteristics as applicant's claimed composition were known to be effective for this purpose. E Argue that the rejection is improper because the examiner has failed to present evidence in support of his position that the claimed composition has no utility.

D Related MPEP Chapter(s): MPEP 2100 - Patentability Answer Reasoning:ANSWER: (D) is most correct answer. As explained at MPEP § 2107.02, II, B, under the heading "No Statement of Utility for the Claimed Invention in the Specification Does Not Per Se Negate Utility," the fact that a specification does not contain a statement of utility for the claimed invention does not per se negate utility. This is because a claimed invention may have a well-established utility, and an invention has a well-established utility if (i) a person of ordinary skill in the art would immediately appreciate why the invention is useful based on the characteristics of the invention and (ii) the utility is specific, substantial, and credible. In this case, the declarations specify a specific substantial and credible utility and explain why the declarants (i.e., persons of ordinary skill in the art) would immediately appreciate that the applicant's claimed composition would possess this utility. (A) is incorrect. A lack of utility deficiency under 35 U.S.C. § 101 also creates a lack of utility deficiency under the first paragraph of 35 U.S.C. § 112 as fully explained at MPEP § 2107.01, under the heading "IV. Relationship Between 35 U.S.C. 112, First Paragraph, And 35 U.S.C. 101." (B) is not the most correct answer. 35 U.S.C. § 101 (and the first paragraph of 35 U.S.C. § 112) requires that the utility be specific. Therefore, the disclosure of a general utility such as "useful biological" properties does not satisfy this requirement as fully explained at MPEP § 2107.01, under the heading "I. Specific And Substantial Requirements." Response (C) also would not be persuasive since the rejection is based on the fact that the applicant's specification fails to identify any .specific and substantial utility for the claimed composition or fails to disclose enough information about the invention to make its usefulness immediately apparent to those familiar with the technological field of the invention. This is explained at MPEP § 2107.01. The fact that the claimed composition has unambiguously proven to be useful for curing a form of cancer previously thought to be incurable does not negate these specification deficiencies. That is, notwithstanding this unambiguous proof, the fact remains that the applicant's specification fails to identify any specific and substantial utility for the composition. Moreover, it is clear that the specification would not make this specific usefulness immediately apparent to those familiar with the technological field of the composition since the cancer was previously thought to be incurable. Finally, response (E) also would not be persuasive. Under current USPTO policy and procedure, the examiner is not required to present evidence in support of a rejection based on lack of utility where, as here, the specification does not identify a specific, substantial and credible utility and does not appear to provide sufficient information such that a well-established utility would be apparent to a person with ordinary skill in the art. See MPEP § 2107, under the heading "II. Examination Guidelines For The Utility Requirement."

Inventor files an application for a non-theoretical metal alloy. The application as originally filed contains the following Claim 1: 1. A metal alloy comprising at least 20% by volume of iron; at least 10% by volume of gallium, and at least 10% by volume of copper. In accordance with the patent law, rules and procedures as related by the MPEP, which of the following claims would be properly held indefinite under 35 USC 112(b)? A Claim 2: The alloy of claim 1 containing 66% by volume of gallium and 14% by volume of copper. B Claim 2: The alloy of claim 1 containing at least 21% by volume of iron, 11% by volume of gallium, and 10.01% by volume of copper. C Claim 2: The alloy of claim 1 containing 20% by volume of iron, 10% by volume of gallium, and 10% by volume of copper. D Claim 2: The alloy of claim 1 containing 54% by volume of copper and 27% by volume of gallium. E Claim 2: The alloy of claim 1 containing at least 1% by volume of silver.

D Related MPEP Chapter(s): MPEP 2100 - Patentability Answer Reasoning:ANSWER: (D) is the correct answer. See MPEP § 2173.05(c), under the heading "Open-Ended Numerical Ranges." Paraphrasing the explanation therein, when an independent claim recites a composition comprising "at least 20% iron" and a dependent claim sets forth specific amounts of non-iron ingredients which add up to100%, apparently to the exclusion of iron, an ambiguity is created with regard to the "at least" limitation unless the percentages of the non-iron ingredients are based on the weight of the non-iron ingredients. On the other hand, a composition claimed to have a theoretical content greater than 100% (i.e., 20-80% of iron, 20-80% of gallium, and 1-25% of copper) is not necessarily indefinite simply because the claims may be read in theory to include compositions that are impossible in fact to formulate. Here, because the invention is a non-theoretical alloy, the sum of the claimed constituents cannot exceed 100% unless the percentage is based on weight. In (D), the sum of elements (B) and (C).is 81% by volume, leaving only 19% for iron. Claim 1, however, requires "at least 20% iron," rendering Claim 2 ambiguous as to the percentage of element A. (A) is incorrect. The sum of gallium and copper components is 80%, leaving a possible 20% of the composition for element iron. Claim 1 requires "at least 20% iron," which includes 20% iron. Therefore, the sum of iron, gallium and copper components in Claim 2 is 100%. (B) is incorrect. "At least 20% iron" includes 21% iron, "at least 10% gallium includes 11% gallium, and "at least 10% copper includes 10.01% copper. (C) is incorrect. "At least 20% iron" includes 20% iron, "at least 10% gallium includes 10% gallium, and "at least 10% copper" includes 10% copper. (E) is incorrect because Claim 1 uses the open transition phrase "comprising," which permits additional elements to be added to the composition. Nothing in the problem indicates that an additional component, silver, cannot be added to the composition.

Applicant files an application claiming a nutritional supplement comprising ingredients (1) through (9) on September 6, 2001. The examiner's search on November 12, 2001 retrieved several documents, each of which provides an enabling disclosure of a nutritional supplement comprising ingredients (1) through (9). Which of the following documents retrieved by the examiner may be properly used by the examiner to reject applicant's claims under 35 USC 102(b)? A An advertisement in the September 2000 issue of Dieticians and Nutritionists Health Weekly where the examiner is not able to determine the actual date of publication. B A printout on November 12, 2001 by the examiner of a MEDLINE database abstract 123456 of an article by Food et al., "Nutritional supplements for infants," published in Azerbijan Pediatrics, Vol. 33, No. 8, pp. 33-37 (September 2000). The printout does not include the date on which the MEDLINE abstract was publicly posted. C A printout, on November 12, 2001 by the examiner, of a product brochure from the Internet website of PRO-BIOTICS VITAMIN CORP. The examiner determines that the brochure was posted on September 7, 2000 on the website. D A Japanese patent application published on September 1, 2000. E All of the above.

D Related MPEP Chapter(s): MPEP 2100 - Patentability Answer Reasoning:ANSWER: (D) is the most correct answer. A reference is a "printed publication" if one of ordinary skill in the art can locate it with reasonable diligence. Its availability as prior art under § 102(b) depends upon proof of when the reference was "published" or became publicly accessible. Here, (D) is the correct answer because the Japanese patent application was published, i.e., "laid open," more than 1 year before applicant's filing date. (C) is incorrect because it was posted or published less than one year after applicant's filing date. (B) is incorrect because (1) the database retrieval date is after applicant's filing date, (2) the printout does not include the date on which the MEDLINE abstract was publicly posted and (3) reliance is on the printout per se not the actual article (reliance on the actual article would require getting the article and an English translation as well as determining the date when the journal was publicly available). (A) is incorrect because there is no evidence when the journal was publicly available. The examiner was unable to determine the actual date of publication.(E) is incorrect because (A), (B) and (C) are incorrect

35 USC 102(d) pre-AIA establishes four conditions which, if all are present, establish a bar against the granting of a patent in this country. In accordance with the patent laws, rules and procedures as related in the MPEP, which of the following is not one of the four conditions established by 35 USC 102(d)? A The foreign application must be filed more than 12 months before the effective U.S. filing date. B The foreign application must have been filed by the same applicant as in the United States or by his or her legal representatives or assigns. C The foreign patent or inventor's certificate must be actually granted before the U.S. filing date. D The foreign patent or inventor's certificate must be actually granted and published before the U.S. filing date. E The same invention must be involved.

D Related MPEP Chapter(s): MPEP 2100 - Patentability Answer Reasoning:ANSWER: (D) is the most correct answer. As set forth in MPEP § 2135, under the heading "General Requirements of 35 U.S.C. 102(d)," states "(C) The foreign patent or inventor's certificate must be actually granted (e.g., by sealing of the papers in Great Britain) before the U.S. filing date. It need not be published." (A) is incorrect because it is one of the four conditions established by 35 U.S.C. § 102(d). MPEP § 2135, under the heading "General Requirements of 35 U.S.C. 102(d)," states "(A) The foreign application must be filed more than 12 months before the effective U.S. filing date...." (B) is incorrect because it is one of the four conditions established by 35 U.S.C. § 102(d). MPEP § 2135, under the heading "General Requirements of 35 U.S.C. 102(d)," states "(B) The foreign application must have been filed by the same applicant as in the United States or by his or her legal representatives or assigns." (C) is incorrect because it is one of the four conditions established by 35 U.S.C. § 102(d). MPEP § 2135, under the heading "General Requirements of 35 U.S.C. 102(d)," states "(C) The foreign patent or inventor's certificate must be actually granted (e.g., by sealing of the papers in Great Britain) before the U.S. filing date. It need not be published." (E) is incorrect because it is one of the four conditions established by 35 U.S.C. § 102(d). MPEP § 2135, under the heading "General Requirement of 35 U.S.C. 102(d)" states "(D) The same invention must be involved." See also MPEP § 2135.01(IV).

36. Which of the following is not a policy underlying the public use bar of 35 USC 102? A Discouraging the removal, from the public domain, of inventions that the public reasonably has come to believe are freely available. B Favoring the prompt and widespread disclosure of inventions. C Allowing the inventor(s) a reasonable amount of time following sales activity to determine the potential economic value of a patent. D Increasing the economic value of a patent by extending the effective term of the patent up to one year. E Prohibiting the inventor(s) from commercially exploiting the invention for a period greater than the statutorily prescribed time.

D Related MPEP Chapter(s): MPEP 2100 - Patentability Answer Reasoning:ANSWER: (D) is the most correct answer. Extending patent term is not a policy underlying any section of 35 U.S.C. § 102. Answers (A), (B), (C) and (E) do state policies underlying the public use bar. Lough v. Brunswick Corp., 86 F.3d 1113, 39 USPQ2d 1100 (Fed. Cir. 1996).

A registered practitioner filed an application for an applicant claiming a "a means for pulling the door open." The specification describes a handle and a knob as being used together as a corresponding structure for pulling the door open. A prior art patent discloses a door opened by pulling on an attached bar. The primary examiner issued an Office action rejecting the claim under 35 USC 102 as being anticipated. In the action, the examiner properly identified the corresponding structure described in applicant's specification as the means for pulling the door open, and properly explained why the prior art attached bar is the equivalent of the structure described in applicant's specification. In accordance with the patent laws, rules and procedures as related in the MPEP, which of the following is the most correct reply to overcome the rejection under these circumstances? A An amendment to the claim changing the pulling means to expressly include an attached bar. B Only argue that the claimed pulling means is not found in the prior art relied-upon reference and therefore the claim is patentable. C An amendment to the specification that adds an attached bar to correspond to the prior art. D An amendment to the claim substituting for the term "means for pulling the door open" the structure of a handle and a knob. E An amendment to the specification that excludes an attached bar as a pulling means.

D Related MPEP Chapter(s): MPEP 2100 - Patentability Answer Reasoning:ANSWER: (D) is the most correct answer. MPEP § 2181 under the heading "Procedures For Determining Whether The Written Description Adequately Describes The Corresponding Structure, Material, Or Acts Necessary To Support A Claim Limitation Which Invokes 35 U.S.C. 112, Sixth Paragraph." 35 U.S.C. 112, sixth paragraph states that a claim limitation expressed in means plus function language "shall be construed to cover the corresponding structure, materials, or acts described in the specification and 'equivalents thereof.'" See also B. Braun Medical, Inc. v. Abbott Lab., 124 F.3d 1419, 1424, 43 USPQ2d 1896, 1899 (Fed. Cir. 1997)." The examiner has made a prima facie case of equivalent in the Office action to support the rejection based on 35 U.S.C. § 102. By amending the claim to no longer include the means limitation in question, the claim becomes narrower inasmuch as it no longer includes equivalents under 35 U.S.C. § 112, paragraph 6 for examination purposes. Thus, (D) overcomes the lack of novelty rejection under these circumstances. (A) is not the most correct answer because such an amended claim would continue to lack novelty, since both it and the prior art would have the attached bar expressly. Furthermore, such an amendment would introduce new matter lacking support in the application as originally filed. 35 U.S.C. § 112, first paragraph. (B) is not the most correct answer because the "not found in the prior art" argument does not rebut the prima facie case of equivalents raised by the examiner. (C) is not the most correct answer because it does not address the rejection. (E) is not the most correct answer because the amendment would raise a new matter issue.

Paprika is a known product. A patent application discloses a composition which is made by subjecting paprika to processing steps X, Y and Z. The composition is disclosed to be useful in treating cancer. The application was filed June 1, 2016. A reference published May 1, 2015 discloses a food product made by subjecting paprika to processing steps X, Y and Z. The reference does not disclose that the resulting composition has any properties that would make it useful for treating cancer. In accordance with USPTO rules and procedures set forth in the MPEP, which of the following claims is not anticipated by the reference? A A composition made by the process of subjecting paprika to processing steps X, Y and Z, wherein the composition is effective for treating cancer. B A composition for treating cancer, made by the process of subjecting paprika to processing steps X, Y and Z. C A method of making a cancer-treating composition, comprising subjecting paprika to processing steps X, Y and Z. D A method of treating cancer, comprising administering an effective amount of a composition made by subjecting paprika to processing steps X, Y and Z. E All of the above.

D Related MPEP Chapter(s): MPEP 2100 - Patentability Answer Reasoning:ANSWER: (D) is the most correct answer. See 35 U.S.C. § 102(b); MPEP § 2131. Citing Verdegaal Bros. v. Union Oil Co. of California, 814 F.2d 628, 631, 2 USPQ2d 1051, 1053 (Fed. Cir. 1987), MPEP § 2131, under the heading, "To Anticipate A Claim, The Reference Must Teach Every Element Of The Claim" states, "A claim is anticipated only if each and every element as set forth in the claim is found, either expressly or inherently described, in a single prior art reference.". The claim is directed to a method of use that is not disclosed by the reference. Answer (A) is incorrect. 35 U.S.C. § 102(b); MPEP §§ 2112, 2112.01. The claimed composition is the same as that disclosed in the prior art, because it is made from the same starting material subjected to the same processing steps. The recitation of "the composition is effective for treating cancer," is only a statement of the inherent properties of the composition. Where the claimed and prior art products are identical in structure or composition, or are produced by identical processes, a prima facie case of anticipation has been established. In re Best, 195 USPQ 430, 433 (CCPA 1977). The burden is shifted to applicant to show that the prior art product does not necessarily possess the characteristics of the claimed product. The reference is prior art under 35 U.S.C. § 102(b), and therefore the claim is anticipated. Answer (B) is incorrect. 35 U.S.C. § 102(b); MPEP §§ 2112, 2112.01, and 2112.02. The claimed composition is the same as that disclosed in the prior art, because it is made from the same starting material subjected to the same processing steps. The recitation of a composition "for treating cancer" reflects only a preamble statement of an intended use of the claimed composition, which does not limit the scope of the claim. Answer (C) is incorrect. See 35 U.S.C. § 102(b); MPEP §§ 2112, 2112.01. The claimed method is the same as that disclosed in the prior art, because it subjects the same starting material to the same manipulative steps. The recitation of making "a cancer-treating composition" reflects only a preamble's statement of an intended use of the claimed composition, which does not further limit the claimed method. Answer (E) is incorrect, because (A), (B), and (C) are incorrect.

Inventor files an application for a non-theoretical metal alloy. The application as originally filed contains the following Claim 1: Claim 1. A metal alloy comprising at least 20% by volume of iron; at least 10% by volume of gallium, and at least 10% by volume of copper. In accordance with the USPTO rules and the procedures set forth in the MPEP, which of the following claims would be properly held indefinite under 35 USC 112(b)? A Claim 2: The alloy of claim 1 containing 66% by volume of gallium and 14% by volume of copper. B Claim 2: The alloy of claim 1 containing at least 21% by volume of iron, 11% by volume of gallium, and 10.01% by volume of copper. C Claim 2: The alloy of claim 1 containing 20% by volume of iron, 10% by volume of gallium, and 10% by volume of copper. D Claim 2: The alloy of claim 1 containing 54% by volume of copper and 27% by volume of gallium. E Claim 2: The alloy of claim 1 containing at least 1% by volume of silver.

D Related MPEP Chapter(s): MPEP 2100 - Patentability Answer Reasoning:ANSWER: (D) is the most correct answer. See MPEP § 2173.05(c), under the heading "Open-Ended Numerical Ranges." Paraphrasing the explanation therein, when an independent claim recites a composition comprising "at least 20% iron" and a dependent claim sets forth specific amounts of non-iron ingredients which add up to100%, apparently to the exclusion of iron, an ambiguity is created with regard to the "at least" limitation unless the percentages of the non-iron ingredients are based on the weight of the non-iron ingredients. On the other hand, a composition claimed to have a theoretical content greater than 100% (i.e., 20-80% of iron, 20-80% of gallium, and 1-25% of copper) is not necessarily indefinite simply because the claims may be read in theory to include compositions that are impossible in fact to formulate. Here, because the invention is a non-theoretical alloy, the sum of the claimed constituents cannot exceed 100% unless the percentage is based on weight. In (D), the sum of elements (B) and (C) is 81% by volume, leaving only 19% for iron. Claim 1, however, requires "at least 20% iron," rendering Claim 2 ambiguous as to the percentage of element A. (A) is incorrect. The sum of gallium and copper components is 80%, leaving a possible 20% of the composition for element iron. Claim 1 requires "at least 20% iron," which includes 20% iron. Therefore, the sum of iron, gallium and copper components in Claim 2 is 100%. (B) is incorrect. "At least 20% iron" includes 21% iron, "at least 10% gallium includes 11% gallium, and "at least 10% copper includes 10.01% copper. (C) is incorrect. "At least 20% iron" includes 20% iron, "at least 10% gallium includes 10% gallium, and "at least 10% copper" includes 10% copper. (E) is incorrect because Claim 1 uses the open transition phrase "comprising," which permits additional elements to be added to the composition. Nothing in the problem indicates that an additional component, silver, cannot be added to the composition.

During his summer vacation to the mountains, Eric discovered and isolated a microorganism which secretes a novel compound. Eric purified and tested the compound in tumor-containing control mice and found that the tumors disappeared after one week; whereas tumor-containing mice which did not receive the compound died. Eric was very excited about his results and so he did a few additional experiments to characterize the microorganism and the compound which it was secreting. Eric determined that the microorganism was an S. spectaculus, and that the secreted compound was so unlike any other compounds that Eric named it spectaculysem. Eric told his friend Sam about his discovery, who urged him to apply for a U.S. patent on the microorganism and the secreted product. Eric did so, but to his amazement, a primary examiner rejected all the claims to his inventions. Which of the following, if made by the examiner, would be a proper rejection in accordance with USPTO rules and procedures set forth in the MPEP? A The examiner's rejection of the claims to the microorganism under 35 USC 101 as being unpatentable because microorganisms are living matter and living matter is non-statutory subject matter. B The examiner's rejection of the claims to the compound under 35 USC 101 as having no credible utility because Eric has only tested the compound in mice and curing mice of cancer has no "real world" value. The examiner also states that Eric must demonstrate that the compound works in humans in order to show that it has a patentable utility. C The examiner's rejection of the claims to the compound under 35 USC 103, stating that it would have been obvious to one of ordinary skill in the art to test the by product of a newly-discovered microorganism for therapeutic uses. D The examiner's rejection of the claims to the microorganism under 35 USC 102/103 over a reference which teaches an S. spectaculus microorganism stating that Eric's claimed microorganism is the same as, or substantially the same as, the microorganism described in the reference. E None of the above.

D Related MPEP Chapter(s): MPEP 2100 - Patentability Answer Reasoning:ANSWER: (D) is the most correct answer. See MPEP §§ 2112.01 and 2131. MPEP § 2112.01, under the heading "Product and Apparatus Claims - When the Structure Recited in the Reference is Substantially Identical to that of the Claims, Claimed Properties or Functions are Presumed to be Inherent" states that "[w]here the claimed and prior art products are identical or substantially identical in structure or composition, or are produced by identical or substantially identical processes, a prima facie case of either anticipation or obviousness has been established. In re Best, 562 F.2d 1252, 1255, 195 USPQ 430, 433 (CCPA 1977)." Here, the claimed microorganism and the prior art microorganism appear to be the identical. (A) is an incorrect choice. MPEP § 2105, under the heading "Patentable Subject Matter - Living Subject Matter," states "...the question of whether or not an invention embraces living matter is irrelevant to the issue of patentability." The Supreme Court has held that biological materials such as microorganisms, and non-human animals, is patentable subject matter, provided it is made by man. Here, Eric's isolation and purification the microorganism from its natural state (environment) makes it a product of human ingenuity, as opposed to a product of nature. Diamond v. Chakrabarty, 447 U.S. 303, 206 USPQ 193 (1980). (B) is an incorrect choice. MPEP § 2107.01, under the heading "III. Therapeutic or Pharmacological Utility," states that the "courts have found utility for therapeutic invention despite the fact that an applicant is at a very early stage in the development of a pharmaceutical product or therapeutic regimen based on a claimed pharmacological or bioactive compound or composition." See, Cross v. Iizuka, 753 F.2d 1040, 1051, 224 USPQ 739, 747-48 (Fed. Cir. 1985). See also, In re Brana, 51 F.3d 1560, 34 USPQ2d 1436 (Fed. Cir. 1995). Citing Brana, MPEP § 2107.01 states, "Accordingly, Office personnel should not construe 35 USC 101, under the logic of 'practical' utility or otherwise, to require that an applicant demonstrate that a therapeutic agent based on a claimed invention is a safe or fully effective drug for humans." See also, MPEP § 2107.03, under the heading "Special Considerations for Asserted Therapeutic or Pharmacological Utilities, I. A Reasonable Correlation Between the Evidence and the Asserted Utility is Sufficient," which states "As a general matter, evidence of pharmacological or other biological activity of a compound will be relevant to an asserted therapeutic use if there is a reasonable correlation between the activity in question and the asserted utility. Cross v. Iizuka, 753 F.2d 1040, 224 USPQ 739 (Fed. Cir. 1985); In re Jolles, 628 F.2d 1322, 206 USPQ 885 (CCPA 1980); Nelson v. Bowler, 626 F.2d 853, 206 F.2d 881 (CCPA 1981). An applicant can establish this reasonable correlation by relying on statistically relevant data documenting the activity of a compound or composition, arguments or reasoning, documentary evidence (e.g., articles in scientific journals), or any combination thereof. The applicant does not have to prove that a correlation exists between a particular activity and an asserted therapeutic use of a compound as a matter of statistical certainty, nor does he or she have to provide actual evidence of success in treating humans where such utility is asserted. Instead, as the courts have repeatedly held, all that is required is a reasonable correlation between the activity and the asserted use." Since mice are routinely used to test anti-cancer drugs for their tumoricidal activity, it is reasonable to assume that the compound, spectaculysem, may be useful as a therapeutic agent. (C) is an incorrect choice. MPEP § 2145, under the subheading "X. Arguing Improper Rationales for Combining References. A. Impermissible Hindsight." If, as acknowledged by the examiner, a novel microorganism has been discovered, then any product which it makes could not have been anticipated by, or obvious over, the prior art. The examiner's rejection is based purely on hindsight derived from his or her reading of the applicant's specification.

A patent application filed in the USPTO claims a nylon rope coated with element E for the purpose of preventing breakage of the rope. In the first Office action, the examiner rejects the claim as obvious over P in view of a trade journal publication, T. P teaches a nylon rope coated with resin for the purpose of making the rope waterproof. T teaches a nylon tent fabric coated with element E for the purpose of making the tent waterproof, and suggests the use of element E for making other nylon products waterproof. Following proper USPTO practices and procedures, the combination of P and T: A cannot support a prima facie case of obviousness because T lacks a suggestion to combine with P for the purpose of preventing breakage in nylon rope. B cannot support a prima facie case of obviousness because P lacks a suggestion to combine with T for the purpose of preventing breakage in nylon rope. C cannot support a prima facie case of obviousness because T only contains a suggestion to combine with P for the purpose of waterproofing nylon rope. D can support a prima facie case of obviousness, even though T only contains a suggestion to combine with P for the purpose of waterproofing nylon rope. E can support a prima facie case of obviousness because the applicant is always under an obligation to submit evidence of non-obviousness regardless of whether the examiner fully establishes a prima facie case of obviousness.

D Related MPEP Chapter(s): MPEP 2100 - Patentability Answer Reasoning:ANSWER: (D). "It is not necessary in order to establish a prima facie case of obviousness...that there be a suggestion or expectation from the prior art that the claimed [invention] will have the same or a similar utility as one newly discovered by the applicant." In re Dillon, 919 F.2d 688, 692, 16 USPQ2d 1897, 1900 (Fed. Cir. 1990) (emphasis in original). Thus, "[i]t is not necessary that the prior art suggest the combination to achieve the same advantage or result discovered by applicant." MPEP § 2144 ("Rationale Different from Applicant's is Permissible"). Here, T suggests the combination with P to achieve a different advantage or result, i.e., waterproofing, from that discovered by applicant, i.e., reducing breakage. Answers (A) - (C) are incorrect because the suggestion to combine does not need to be for the same purpose as applicant discloses in the application. Dillon, 919 F.2d at 692, 16 USPQ2d at 1900; MPEP § 2144 ("Rationale Different from Applicant's is Permissible"). Answer (E) is incorrect because an applicant is under no obligation to submit evidence of non-obviousness unless the examiner meets his or her initial burden to fully establish a prima facie case of obviousness. MPEP § 2142.

In accordance with the USPTO rules and the procedures set forth in the MPEP, which of the following is true? A Interferences will generally be declared even when the applications involved are owned by the same assignee since only one patent may issue for any given invention. B A senior party in an interference is necessarily the party who obtains the earliest actual filing date in the USPTO. C Reexamination proceedings may not be merged with reissue applications since third parties are not permitted in reissue applications. D After a reexamination proceeding is terminated and the certificate has issued, any member of the public may obtain a copy of the certificate by ordering a copy of the patent. E None of the above.

D Related MPEP Chapter(s): MPEP 2200 - Citation of Prior Art and Ex Parte Reexamination of Patents Answer Reasoning:ANSWER: (D) is the most correct answer. See MPEP § 2292. As to (A) see 37 CFR § Part 41. As to (B) see 37 CFR § Part 41, which provides that the senior party has earliest effective filing date. As to (C), see MPEP § 2285 regarding merger of reissues and reexamination proceedings. As to (E), (D) is true.

On Monday, May 13, 2002, John's secretary deposited in an "Express Mail" drop box prior to the last scheduled pick-up for that day, an envelope properly addressed to the USPTO for delivery to the USPTO by the "Express Mail Post Office to Addressee" service. The envelope was received by the USPTO on Wednesday, May 15, 2002, containing a reply to an Office action which set a shortened statutory period ("SSP") for reply ending on Tuesday, May 14, 2002. The reply was marked by the Office as being received on May 15, 2002. The number of the "Express Mail" mailing label had not been placed on the response papers, and upon receipt of the "Express Mail" mailing label John learned that the "date in" was not clearly marked. John promptly filed a petition requesting the filing date to be the date of deposit. The petition included a showing that the date of deposit accompanied by evidence of USPS corroboration of the deposit. Accordingly, A The reply will be regarded as timely filed in the USPTO on May 15, 2002. B The reply will be regarded as timely filed in the USPTO on May 14, 2002. C The reply will be regarded as timely filed in the USPTO on May 13, 2002. D The reply will be regarded as timely filed in the USPTO if a petition with proper fee for a one month extension of time is filed in the USPTO on or before June 14, 2002. E The reply will be regarded as timely filed in the USPTO if the number of the "Express Mail" mailing label is placed on each page of a copy of the original response and hand carried to the USPTO on May 15, 2002, rather than being sent by "Express Mail."

D Related MPEP Chapter(s): MPEP 500 - Receipt and Handling of Mail and Papers Answer Reasoning:ANSWER: (D) is correct. 37 C.F.R. § 1.136(a). 37 C.F.R. § 1.136(a) states, "[A]pplicant may extend the time period for reply up to the earlier of the expiration of any maximum period set by statute or five months after the time period set for reply, if a petition for an extension of time and the fee set in § 1.17(a) are filed..." (A) is wrong because the response was not timely filed since it was received by the USPTO after the SSP expired. (B) and (C) are wrong. The reply was not filed on May 14, 2002, because the conditions of 37 C.F.R. § 1.10(b) were not satisfied. For example, the number of the "Express Mail" mailing label must have been placed on each page of the response prior to the original mailing by "Express Mail." The petition should not be expected to be granted inasmuch as the papers did not include the number of the "Express Mail" mailing label on them. See § 1.10(c)(2), (d)(2), and (e)(2). (E) is wrong because 37 C.F.R. § 1.10(b) requires that "the number of the 'Express Mail' mailing label must have been placed on each page of the response prior to the original mailing by 'Express Mail.'" Emphasis added.

Which of the following is not a proper incorporation by reference in an application prior to allowance according to the USPTO rules and the procedures set forth in the MPEP? A Incorporating material necessary to describe the best mode of the claimed invention by reference to a commonly owned, abandoned U.S. application that is less than 20 years old. B Incorporating non-essential material by reference to a prior filed, commonly owned pending U.S. application. C Incorporating material that is necessary to provide an enabling disclosure of the claimed invention by reference to a U.S. patent. D Incorporating non-essential material by reference to a hyperlink. E Incorporating material indicating the background of the invention by reference to a U.S. patent which incorporates essential material.

D Related MPEP Chapter(s): MPEP 500 - Receipt and Handling of Mail and Papers Answer Reasoning:ANSWER: (D) is the correct answer. See MPEP § 608.01(p). (A) is incorrect because abandoned applications less than 20 years old can be incorporated by reference to the same extent as copending applications. (B) is incorrect because non-essential material may be incorporated by reference to patents or applications published by the United States. (C) is incorrect because material necessary to provide an enabling disclosure is essential material, which may be incorporated by reference to a U.S. patent. (E) is incorrect because non-essential material may be incorporated by reference to a U.S. patent which incorporates essential material.

accordance with the USPTO rules and the procedures set forth in the MPEP, which of the following may not be filed by facsimile transmission? A A request for continued examination under 37 CFR 1.114 along with a submission. B A continued prosecution application under 37 CFR 1.53(d). C An amendment in reply to a non-final Office action. D The filing of a provisional patent application specification and drawing for the purpose of obtaining an application filing date. E (B) and (D).

D Related MPEP Chapter(s): MPEP 500 - Receipt and Handling of Mail and Papers Answer Reasoning:ANSWER: (D) is the most correct answer. 37 CFR § 1.6(d)(3); MPEP § 502.01. MPEP § 501.01, under the heading "Correspondence Relative To Patents And Patent Applications Where Filing By Facsimile Transmission Is Not Permitted," identifies among the correspondence not permitted to be filed by facsimile transmission "(B) A national patent application specification and drawing (provisional or nonprovisional) or other correspondence for the purpose of obtaining an application filing date, other than a continued prosecution application filed under 37 CFR 1.53(d) " (A), (B) and (C) are incorrect. See 37 CFR § 1.6(d)(3); MPEP §§ 201.06(d), 502.01, 706.07(h) and 714. A request for continued examination (RCE) under 37 CFR § 1.114, which is not a new application, a continued prosecution application (CPA) under 37 CFR § 1.53(d) and an amendment in reply to a non-final Office action may be filed by facsimile transmission.

In accordance with the USPTO rules and the procedures set forth in the MPEP, in which of the following cases is the date of actual receipt by the USPTO not accorded as the application filing date? A Provisional application filed without claims. B Non-provisional application filed containing an error in inventorship. C Non-provisional application filed which fails to identify the inventor(s). D Non-provisional application filed "by reference" where a complete copy of the reference filing is not submitted within 4 months of filing, or sixteen months from the priority date. E Non-provisional application filed using a certificate of mailing in accordance with 37 CFR 1.8.

D Related MPEP Chapter(s): MPEP 500 - Receipt and Handling of Mail and Papers Answer Reasoning:ANSWER: (D) is the most correct answer. A non-provisional application filed by reference and not followed by the timely submission of a complete copy of the priority document, will be treated as though it had never been filed. See PLTIA. (A) is wrong because the component parts of a provisional application necessary to obtain a filing date do not include claims. 35 U.S.C. § 111(b); 37 CFR § 1.53(c); MPEP § 506 under heading "Incomplete Provisional Applications." (B) and (C) are wrong because "[a]n error in or failure to identify inventorship does not raise a filing date issue." MPEP § 506.02. (E) is wrong. Under 37 CFR § 1.8(a)(2)(i)(A) no benefit is accorded to a certificate of mailing date. The effective date is the actual date of receipt, and not the certificate of mailing date.

Prior to filing a patent application for a client, a registered practitioner determined that the client was entitled to claim small entity status under 37 CFR 1.27. The practitioner filed a patent application for the client on November 1, 2016 together with a claim for small entity status under 37 CFR 1.27. On December 2, 2016, a Notice to File Missing Parts was mailed setting a two month period for reply and requiring the basic filing fee and the surcharge under 37 CFR 1.16(e). The practitioner timely submitted the small entity fees for the basic filing fee and the surcharge as required in the Notice. Shortly thereafter, the practitioner discovered that on October 31, 2016, the day before the application was filed, the client, without advising the practitioner, had assigned all rights in the invention that is the subject of the application to an entity that would not qualify for small entity status under 37 CFR 1.27. In accordance with the USPTO rules and the procedures set forth in the MPEP, which of the following actions would be the best action for the practitioner to take? A File a continuing application under 37 CFR 1.53(b) with the large entity filing fee and then file a letter of express abandonment under 37 CFR 1.138 in the original application after the continuing application has been accorded a filing date. B Promptly file a notification of loss of small entity status under 37 CFR 1.27(g) and, thereafter, pay large entity fees whenever any subsequent fees are required. C Wait until a Notice of Allowance is received and then timely submit the large entity issue fee along with a notification of loss of small entity status under 37 CFR 1.27(g). D File a paper under 37 CFR 1.28(c) requesting that the good faith error in claiming small entity status be excused and complying with the separate submission and itemization requirements of 37 CFR 1.28(c) and including payment of the deficiency owed. E Pay the difference between the large entity filing fee and small entity filing fee and the difference between the large entity surcharge and small entity surcharge within two months from the mail date of the Notice to File Missing Parts.

D Related MPEP Chapter(s): MPEP 500 - Receipt and Handling of Mail and Papers Answer Reasoning:ANSWER: (D) is the most correct answer. MPEP § 509.03, under the heading "Correcting Errors In Small Entity Status," states "37 CFR 1.28(c) provides that if small entity status is established in good faith and the small entity fees are paid in good faith, and it is later discovered that such status as a small entity was established in error or through error the Office was not notified of a change of status, the error will be excused upon compliance with the separate submission and itemization requirements of 37 CFR 1.28(c)(1) and (c)(2), and the deficiency payment requirement of 37 CFR 1.28(c)(2)." (A), (B), (C), and (E) are not correct. Small entity status was not appropriate when the assertion of small entity status was filed with the application on November 1, 2002 and none of the actions recited in (A), (B), (C), and (E) would correct the error in claiming small entity status. The only mechanism for correcting a good faith error in claiming small entity status is by filing a request in compliance with 37 CFR § 1.28(c).

In accordance with the USPTO rules and the procedures set forth in the MPEP, a grant of small entity status entitles an applicant to which of the following? A Applicant will receive an accelerated examination by having the application advanced out of order. B Applicant can use a certificate of mailing under 37 CFR 1.8 to obtain a U.S. filing date that is earlier than the actual USPTO receipt date of the application. C Applicant will obtain a refund of all fees paid to the USPTO where applicant demonstrates: (i) a changed purpose for which the fees were paid, (ii) the fees were not paid by mistake, and (iii) the fees were not paid in excess of the amount required. D Applicant can pay a fee to file a request for continued examination pursuant to 37 CFR 1.114 that is less than the fee paid by other than a small entity. E None of the above.

D Related MPEP Chapter(s): MPEP 500 - Receipt and Handling of Mail and Papers Answer Reasoning:ANSWER: (D) is the most correct answer. See 35 U.S.C. § 41(h)(1); 37 CFR §§ 1.17(e) and 1.114; and MPEP § 509.02. (A) is incorrect because there is no support for (A) in 37 CFR § 1.102. (B) is incorrect because there is no support for (B) in 37 CFR § 1.8. (C) is incorrect because it is inconsistent with 35 U.S.C. § 42(d); 37 CFR § 1.26. Miessner v. United States, 228 F.2d 643, 644 (D.C. Cir. 1955). (E) is incorrect because (D) is correct.

Which of the following timely actions should you take to accord maximum patent protection at minimum government fees for your client whose invention is described in a provisional patent application that was filed 6 months ago with no claim? A File a request to convert the provisional application to a nonprovisional application, accompanied by a proper executed declaration, an amendment including at least one claim as prescribed by paragraph 2 of 35 USC 112 and the proper fee set forth in 37 CFR 1.17(i). B File a request to convert the provisional application to a nonprovisional application, accompanied by a proper executed declaration, an amendment including at least one claim as prescribed by paragraph 2 of 35 USC 112, the proper fee set forth in 37 1.17(i), and the basic filing fee for the nonprovisional application. C File a request to convert the provisional application to a nonprovisional application, accompanied by a proper executed declaration, an amendment including at least one claim as prescribed by paragraph 2 of 35 USC 112, the proper fee set forth in 37 CFR 1.17(i), the basic filing fee for the nonprovisional application, and the surcharge required by 37 CFR 1.16(e). D File a nonprovisional application including at least one claim accompanied by a proper executed declaration, and the basic filing fee. The application contains a specific reference to the provisional application in compliance with 37 CFR 1.78(a)(5). E File a nonprovisional application including at least one claim accompanied by a proper executed declaration but without the basic filing fee. The application contains a specific reference to the provisional application in compliance with 37 CFR 1.78(a)(5).

D Related MPEP Chapter(s): MPEP 600 - Parts, Form, and Content of Application Answer Reasoning: ANSWER: (D) is correct. (A), (B) and (C) are wrong because MPEP § 601.01(c) states, "Claiming priority is less expensive [than conversion] and will result in a longer patent term." Conversion requires payment of the conversion fee. (D) is correct because MPEP § 601.01(c) states, "In addition, if the provisional application was not filed with an executed oath or declaration and the filing fee for a non-provisional application, the surcharge set forth in 37 C.F.R. § 1.16(e) is required. (E) is wrong because the action taken results in an additional late filing fee surcharge being required.

Which of the following is not in accordance with the provisions of the USPTO rules and the procedures set forth in the MPEP? A Where joint inventors are named, the examiner should not inquire of the patent applicant concerning the inventors and the invention dates for the subject matter of the various claims until it becomes necessary to do so in order to properly examine the application. B Under 35 USC 119(a), the foreign priority benefit may be claimed to any foreign application that names a U.S. inventor as long as the U.S. named inventor was the inventor of the foreign application invention and 35 USC 119(a)-(d) requirements are met. C Where two or more foreign applications are combined in a single U.S. application, to take advantage of the changes to 35 USC 103 or 35 USC 116, the U.S. application may claim benefit under 35 USC 119(a) to each of the foreign applications provided all the requirements of 35 USC 119(a)-(d) are met. D One of the conditions for benefit under 35 USC 119(a) is that the foreign application must be for the same or a nonobvious improvement of the invention described in the United States application. E If a foreign application for which priority is being claimed under 35 USC 119 is filed in a country which does not afford similar privileges in the case of applications filed in the United States or to citizens of the United States and the foreign country is not a WTO member country, any claim for the foreign priority thereto by a U.S. application will not be effective.

D Related MPEP Chapter(s): MPEP 600 - Parts, Form, and Content of Application Answer Reasoning:ANSWER: (D) is the best answer as the inventions must be the same in the foreign and U.S. applications. 35 U.S.C. § 119(a). As to (A) through (C), see MPEP § 213. As to (E), see 35 U.S.C. § 119, which provides that the previously filed application must have been filed in a country that affords similar privileges in the case of applications filed in the United States or to citizens of the United States or in a WTO member country.

Ben hires a registered practitioner to prosecute his patent application. The practitioner drafted an application having fifteen claims. Claim 1 is independent, and each of claims 2-15 are singularly dependent upon claim 1. A proper non-final Office action is mailed to the practitioner. Following consultation with Ben, the practitioner timely prepared, signed, and filed a reply to the Office action containing an amendment that does not add new matter, but does add claims 16-27. Each of claims 16-27 is directed to the same invention sought to be patented through claims 1-15. The dependency of each of claims 16-27 reads "any of claims 5-15." For purposes of fee calculation in accordance with the patent laws, rules and procedures as related in the MPEP, how many total claims are contained in the application after the amendment is entered? A One hundred thirty-six. B One hundred thirty-five. C Twenty-seven. D One hundred forty-seven. E Fifteen.

D Related MPEP Chapter(s): MPEP 600 - Parts, Form, and Content of Application Answer Reasoning:ANSWER: (D) is the most correct answer. 37 CFR § 1.75; MPEP § 608.01(n). As explained in MPEP § 608.01(n), under the heading "Multiple Dependent Claims," subheading "Acceptable Multiple Dependent Claim Wording" the multiple dependent claim wording of new claims 16-27 is proper. See, for example, "any one of the preceding claims," and "in any of claims 1-3 or 7-9." 37 CFR § 1.75(c) states "For fee calculation purposes under § 1.16, a multiple dependent claim will be considered to be that number of claims to which direct reference is made therein." Therefore, claims 16-27 would each have a claim value of eleven and the total number of claims for fee calculation is one hundred forty-seven (12 x 11 = 132 + 15 = 147). Answers (A) and (B) are incorrect because they are not the correct total. Answer (C) is incorrect because the multiple dependent claims have not been calculated in accordance with 37 CFR § 1.75. Answer (E) is incorrect because the question asks for the total after the amendment adding claims 16-27 has been entered.

en hires a registered practitioner to prosecute his patent application. The practitioner drafted an application having fifteen claims. Claim 1 is independent, and each of claims 2-15 are singularly dependent upon claim 1. A proper non-final Office action is mailed to the practitioner. Following consultation with Ben, the practitioner timely prepared, signed, and filed a reply to the Office action containing an amendment that does not add new matter, but does add claims 16-27. Each of claims 16-27 is directed to the same invention sought to be patented through claims 1-15. The dependency of each of claims 16-27 reads "any of claims 5-15." For purposes of fee calculation in accordance with the USPTO rules and the procedures set forth in the MPEP, how many total claims are contained in the application after the amendment is entered? A One hundred thirty-six. B One hundred thirty-five. C Twenty-seven. D One hundred forty-seven. E Fifteen.

D Related MPEP Chapter(s): MPEP 600 - Parts, Form, and Content of Application Answer Reasoning:ANSWER: (D) is the most correct answer. 37 CFR § 1.75; MPEP § 608.01(n). As explained in MPEP § 608.01(n), under the heading "Multiple Dependent Claims," subheading "Acceptable Multiple Dependent Claim Wording" the multiple dependent claim wording of new claims 16-27 is proper. See, for example, "any one of the preceding claims," and "in any of claims 1-3 or 7-9." 37 CFR § 1.75(c) states "For fee calculation purposes under § 1.16, a multiple dependent claim will be considered to be that number of claims to which direct reference is made therein." Therefore, claims 16-27 would each have a claim value of eleven and the total number of claims for fee calculation is one hundred forty-seven (12 x 11 = 132 + 15 = 147). Answers (A) and (B) are incorrect because they are not the correct total. Answer (C) is incorrect because the multiple dependent claims have not been calculated in accordance with 37 CFR § 1.75. Answer (E) is incorrect because the question asks for the total after the amendment adding claims 16-27 has been entered.

Claims 1 and 2 in a patent application state the following: Claim 1. An apparatus for sitting comprising: (i) a square shaped base member; (ii) four elongated members mounted to the bottom of the base member; and (iii) a circular back member mounted to the base member. Claim 2. An apparatus as in claim 1, further comprising a spring connected to the back member and to the base member. Which, if any, of the following claims fully supported by the specification and presented in the application, is in accordance with USPTO rules and procedure? A 3. An apparatus as in claim 1, wherein the base member is rectangularly shaped. B 3. An apparatus as in claim 2, wherein the wheels connected to each of the elongated members are plastic. C 3. An apparatus as in the preceding claims, further comprising a pressure-sensing device connected to the base member. D 3. An apparatus as in any of the preceding claims, in which the circular back member is wooden. E None of the above.

D Related MPEP Chapter(s): MPEP 600 - Parts, Form, and Content of Application Answer Reasoning:ANSWER: (D) is the most correct answer. MPEP § 608.01(n). (A) is incorrect because a dependent claim must further limit the subject matter of a previous claim. 37 C.F.R. § 1.75(c). The claim in (B) is actually inconsistent with claim 1. (B) is incorrect because there is no antecedent basis for the wheels. MPEP § 2173.05(e). (C) is incorrect because it does not refer back in the alternative only. MPEP § 608.01(n). (E) is incorrect because (D) is correct.

Which of the following is not in accordance with the recommended characteristics set forth in the MPEP for the detailed description of the invention? A The detailed description of the invention must include a description of the preferred embodiment(s) of the invention as required in 37 CFR 1.71. B The detailed description should be as short and specific as is necessary to describe the invention adequately and accurately. C Where elements or groups of elements, compounds, and processes, which are conventional and generally widely known in the field of the invention described, and their exact nature or type is not necessary for an understanding and use of the invention by a person skilled in the art, they should not be described in detail. D The detailed description of the invention may include reference characters to the parts of the invention that do not appear in the drawings. E Where particularly complicated nonessential subject matter is involved or where the elements, compounds, or processes may not be commonly or widely known in the field, absent disclosure elsewhere in the application, the specification should refer to another patent or readily available publication that adequately describes the subject matter.

D Related MPEP Chapter(s): MPEP 600 - Parts, Form, and Content of Application Answer Reasoning:ANSWER: (D) is the most correct answer. Reference characteristics mentioned in the detailed description must appear in the drawings. 37 C.F.R. §§ 1.83(a) and 1.84(p)(5); MPEP § 608.02. As to (A), (B), (C), and (E), see MPEP § 608.01(a).

n accordance with USPTO rules and the procedure set forth in the MPEP, which one of the following is not required for a provisional application filed in the USPTO? A A specification. B A drawing as prescribed by 35 USC 113. C An application fee. D A claim. E A cover sheet complying with the rule.

D Related MPEP Chapter(s): MPEP 600 - Parts, Form, and Content of Application Answer Reasoning:ANSWER: (D) is the most correct answer. See 35 U.S.C. § 111(b)(2); 37 CFR § 1.51(c); MPEP §§ 601 and 601.01(b). 35 U.S.C. § 111(b)(2) states that "[a] claim, as required by the second through fifth paragraphs of section 112, shall not be required in a provisional application." The documents and other components recited in answers (A) through (C) and (E) are required in a provisional application. See 35 U.S.C. § 111(b); 37 CFR § 1.51(c).

Al files an application for a patent. After the Notice of Allowance is mailed and the issue fee has been paid Al discovers a prior art reference which is material to patentability. What should Al do in accordance with the USPTO rules and the procedures set forth in the MPEP? A Al should file a prior art statement under 37 CFR 1.501 that will be placed in the patent file upon issuance of the application as a patent. B Since the issue fee has been paid, Al no longer has a duty to disclose to the Office material prior art. He is under no obligation to submit the prior art reference to the Office. C Since the issue fee has been paid, it is too late to have the examiner consider the reference in this application. Al should file a continuation application to have the reference considered and allow the original patent application to issue as a patent. D Al should file a petition to have the application withdrawn from issuance, citing the finding of additional material prior art as the reason for withdrawal. A continuation application should also be filed with an information disclosure statement containing the reference in order to have the reference considered. E Al should file an amendment under 37 CFR. 1.312 deleting all of the claims which are unpatentable over the reference since an amendment deleting claims is entitled to entry as a matter of right.

D Related MPEP Chapter(s): MPEP 600 - Parts, Form, and Content of Application Answer Reasoning:ANSWER: (D) is the most correct answer. See 37 CFR § 1.313(b); MPEP §§ 609, subpart (B)(4) and 1308. After payment of the issue fee it is impractical for the Office to consider any information disclosures. As to (A), a prior art statement is applicable only to patent, not application, files. 37 CFR § 1.501. As to (B), duty of disclosure continues until the patent is issued. As to (C), the patent should not be allowed to issue since it may contain invalid claims. As to (E) no amendment is entitled to entry after payment of the issue fee. 37 CFR. § 1.312(b).

Practitioner Smith filed a utility patent application on January 5, 2016, with informal drawings. Upon review of the drawings, the USPTO concluded that the drawings were not in compliance with the 37 CFR 1.84(a)(1) and (k), and were not suitable for reproduction. In an Office communication, Smith was notified of the objection and given two months to correct the drawings so that the application can be forwarded to a Technology Center for examination. Which of the following complies with the USPTO rules and the procedures set forth in the MPEP for a complete bona fide attempt to advance the application to final action? A Smith timely files a response requesting that the objections to the drawings be held in abeyance until allowable subject matter is indicated. B Smith timely files a response requesting that the objections to the drawings be held in abeyance since the requirement increases up-front costs for the patent applicant, and the costs can be avoided if patentable subject matter is not found. C Smith timely files a response requesting that the objections to the drawings be held in abeyance until fourteen months from the earliest claimed priority date. D Smith timely files a response correcting the drawings to comply with 37 CFR 1.84(a)(1) and (k), and making them suitable for reproduction. E All of the above.

D Related MPEP Chapter(s): MPEP 600 - Parts, Form, and Content of Application Answer Reasoning:ANSWER: (D) is the most correct answer. Under 37 CFR § 1.85(a), correcting the drawings to comply with 37 CFR § 1.84(a)(1) and (k), and making them suitable for reproduction is a bona fide response. (A), (B), and (C) are not the most correct answer. In each, Smith seeks to hold the requirement in abeyance. As stated in 37 CFR § 1.85(a), "Unless applicant is otherwise notified in an Office action, objections to the drawings in a utility or plant application will not be held in abeyance, and a request to hold objections to the drawings in abeyance will not be considered a bona fide attempt to advance the application to final action." See also, MPEP § 608.02(b), under the heading "Informal Drawings," which states "[u]nless applicant is otherwise notified in an Office action, objections to the drawings in a utility or plant application will not be held in abeyance. A request to hold objections to the drawings in abeyance will not be considered a bona fide attempt to advance the application to final action (37 CFR 1.135(c))." (E) is not the most correct answer inasmuch as (A), (B), and (C) are not the most correct answers.

Xavier files a complete first reply exactly 10 weeks after the mailing date of a final Office action that sets a 3 month shortened statutory period for reply. An Examiner's Amendment is necessary for the purpose of placing the application in condition for allowance. Which of the following statements is true? A If Xavier gives authorization for the Examiner's Amendment exactly 2 months after his reply, the application will be allowed. B Authorization for the Examiner's Amendment may be made at any time within 6 months of Xavier's reply to avoid abandonment of the application. C Unless Xavier gives authorization for the Examiner's Amendment within the 3 months shortened statutory period for reply, the application will be abandoned. D If Xavier gives authorization for the Examiner's Amendment exactly 2 months after his reply, the application will be abandoned unless accompanied by a proper petition and fee for an extension of time. E Abandonment of the application will be avoided if Xavier gives authorization for the Examiner's Amendment any time within 6 months of the mail date of a final Office action. No extension of time need be filed if Xavier gives the authorization between 3 months and 6 months after the Office action.

D Related MPEP Chapter(s): MPEP 700 - Examination of Applications Answer Reasoning:ANSWER: (D) is correct. (A), (C), and (E) are wrong because MPEP § 706.07(f), paragraph (I), states, "Where a complete first reply to a final Office action has not been filed within 2 months of the final Office action, applicant's authorization to make an amendment to place the application in condition for allowance must be made either within the 3 month shortened statutory period or within an extended period for reply that has been petitioned and paid for by applicant pursuant to 37 C.F.R. § 1.136(a)." (B) is wrong because MPEP § 706.07(f), paragraph (H), states, "Note that an examiner's amendment may not be made more than 6 months from the date of the final Office action, as the application would be abandoned at that point by operation of law."

Patent application A was filed on January 12, 2015, containing claims 1-10. A primary examiner rejects the claims under 35 USC 102 as being anticipated by a U.S. patent issued on June 2, 2012. The rejection also relies on a technical paper published March 12, 2013 to show that a characteristic is inherent in the patent, although not expressed in its disclosure. According to the patent laws, rules and procedures as related in the MPEP, which of the following actions is most likely to overcome the rejection? A Filing a declaration and exhibits under 37 CFR 1.131 to antedate the reference U.S. patent. B Filing evidence under 37 CFR 1.132 tending to show commercial success of the invention. C Filing evidence under 37 CFR 1.132 tending to show unexpected results of the invention. D Amending the ADS of application A to claim priority under 35 USC 120 to a prior copending application B that was filed before June 2, 2012 by the same inventor and discloses, in its entirety, the invention claimed in claims 1-10 of application A in the manner provided by 35 USC 112(a).. E Submitting arguments pointing out that the rejection under 35 USC 102 relies on more than one reference.

D Related MPEP Chapter(s): MPEP 700 - Examination of Applications Answer Reasoning:ANSWER: (D) is most correct. MPEP § 706.02(b) states that "[a] rejection based on 35 U.S.C. § 102 may be overcome by...(C) perfecting priority under...35 .U.S.C. § 120 by amending the specification of the application to contain a specific reference to a prior application..." (A) is incorrect because a declaration and evidence filed under 37 CFR § 1.131 cannot antedate a reference that qualifies as prior art under AIA 35 U.S.C. § 102. (B) and (C) are incorrect because, as noted in MPEP § 2131.04, evidence of secondary considerations, such as unexpected results or commercial success, is irrelevant to 35 U.S.C. § 102 rejections and thus cannot overcome a rejection so based. (E) is incorrect because to serve as an anticipation when the reference is silent about an asserted inherent characteristic, such gap in the reference may be filled with recourse to extrinsic evidence. Continental Can Co. USA v. Monsanto Co., 948 F.2d 1264, 1268, 20 USPQ2d 1746, 1749 (Fed. Cir. 1991). See also MPEP § 2131.01, "Multiple Reference 35 U.S.C. 102 Rejections."

According to the USPTO rules and the procedures set forth in the MPEP, in which of the following situations would the finality of an Office action rejection be improper? I. The final Office action rejection is in a first Office action in a substitute application that contains material which was presented in the earlier application after final rejection but was denied entry because the issue of new matter was raised. II. The final Office action rejection is in a first Office action in a continuing application, all claims are drawn to the same invention claimed in the earlier application, and the claims would have been properly finally rejected on the grounds and art of record in the next Office action if they had been entered in the earlier application. III. The final Office action rejection is in a first Office action in a continuation-in-part application where at least one claim includes subject matter not present in the earlier application. A I B II C III D I and III E II and III

D Related MPEP Chapter(s): MPEP 700 - Examination of Applications Answer Reasoning:ANSWER: (D) is the correct answer. MPEP § 706.07(b). In both I and III the finality is improper. MPEP § 706.07(b). Therefore (A) and (C) are incorrect. In II the finality is proper. MPEP § 706.07(b). Therefore (B) and (E) are incorrect.

A registered practitioner filed a patent application in the Office in 2016. Following examination and a final rejection, the practitioner timely filed a proper notice of appeal and a proper appeal brief in the application wherein claims 1-3 stand rejected, claims 4 and 5, which depend from claim 1, stand objected to as depending from a rejected claim but are otherwise allowable, and claims 6-10 stand allowed. The appeal involves claims 1-3. After the brief was filed but prior to a decision by the Board of Patent Appeals and Interferences, the practitioner filed a request for continued examination (RCE) with a submission in accordance with 37 CFR 1.114 without paying the fee set forth in 37 CFR 1.17(e). In accordance with the patent laws, rules and procedures as related in the MPEP, what effect does the filing of the RCE without the fee set forth in Rule 1.17(e) have on the application under appeal? A The application is abandoned. B The application is still pending and under appeal awaiting a decision by the Board of Patent Appeals and Interferences, because the RCE was improper. C The application is still pending; the appeal is considered withdrawn and the application will be passed to issue with claims 1-3 canceled and claims 4-10 allowed. D The application is still pending; the appeal is considered withdrawn and the application will be passed to issue with claims 1-5 canceled and claims 6-10 allowed. E The appeal is withdrawn; the application is returned to the primary examiner and prosecution is reopened as to claims 1-10.

D Related MPEP Chapter(s): MPEP 700 - Examination of Applications Answer Reasoning:ANSWER: (D) is the correct answer. See MPEP §§ 706.07(h), under the heading "X. After Appeal But Before Decision By Board," and 1215.01. As explained in MPEP § 1215.01, "The filing of an RCE will be treated as a withdrawal of the appeal by the applicant, regardless of whether the RCE includes the appropriate fee or a submission." Thus, the filing of the RCE without the fee results in the withdrawal of the appeal in this application and passage of the application to issue with the allowed claims 6-10 after the cancellation of both rejected claims 13 and claims 4 and 5 which are allowable except for their dependency from rejected claim 1 (A) is incorrect. As also explained in MPEP § 1215.01, although an application under appeal having no allowed claims will be considered abandoned by the filing of an improper RCE, an application having allowed claims will be passed to issue with the allowed claims. Upon withdrawal of appeal, claims which are allowable except for their dependency from rejected claims will be treated as if they were rejected. See MPEP § 1215.01. All rejected claims, such as claims 1-3, and claims which are allowable except for their dependency from rejected claims, such as claims 4 and 5, will be canceled. (B) is incorrect. As explained in MPEP § 706.07(h), under the heading "After Appeal But Before Decision By The Board," proceedings as to the rejected claims are terminated and the application is passed to issue with the allowed claims. MPEP § 1215.01 explains that the filing of an RCE will be treated as a withdrawal of the appeal by the applicant, regardless of whether the RCE includes the appropriate fee or a submission. (C) is incorrect for the reasons explained for (A), and because claims 4 and 5 will be canceled. (E) is incorrect. The RCE, which was filed without the fee, is improper. Thus, as explained in MPEP § 706.07(h),. under the heading, "After Appeal But Before Decision By The Board," proceedings as to the rejected claims are terminated and the application is passed to issue with the allowed claims. MPEP § 1215.01 explains that the filing of an RCE will be treated as a withdrawal of the appeal by the applicant, regardless of whether the RCE includes the appropriate fee or a submission.

Evidence of unexpected results is relied upon to overcome a prima facie case of obviousness. Which of the following is incorrect? A The evidence must compare the claimed invention to the closest prior art. B The evidence must be commensurate in scope with the claims. C Data relied upon to show unexpected results need not cover the full range of the claims if one of ordinary skill in the art could ascertain a trend in the data that would allow that person to reasonably extend the probative value of the data to the full scope of the claims. D Unexpected results can be shown by factual evidence or, if no factual evidence is available to the applicant, by sound argument by the applicant's agent or attorney. E The evidence need not be in an affidavit or declaration under 37 CFR 1.132 if the evidence is presented in the specification of an application to which the applicant has attested.

D Related MPEP Chapter(s): MPEP 700 - Examination of Applications Answer Reasoning:ANSWER: (D) is the most correct answer because mere attorney argument, unsupported by factual evidence, is insufficient to establish unexpected results. See In re Geisler, 116 F.3d 1465, 1470-71, 43 USPQ2d 1362, 1365-66 (Fed. Cir. 1997). (A) is not the proper choice because such a comparison is required. See In re Baxter Travenol Labs., 952 F.2d 388, 392, 21 USPQ2d 1281, 1285 (Fed. Cir. 1991); In re De Blauwe, 736 F.2d 699, 705, 222 USPQ 191, 196 (Fed. Cir. 1984), and MPEP § 716.02(e). (B) is not the proper choice because evidence relied upon for overcoming a prima facie case of obviousness must be commensurate in scope with the claims. See In re Grasselli, 713 F.2d 731, 743, 218 USPQ 769, 778 (Fed. Cir. 1983); In re Clemens, 622 F.2d 1029, 1035, 206 USPQ 289, 296 (CCPA 1980), and MPEP § 716.03(a). (C) is not the proper choice because the unobviousness of a broader range recited in a claim can be proven by a narrower range of data provided that one of ordinary skill in the art could ascertain a trend in the data which would allow that person to reasonably extend the probative value of the data to the broader range. See In re Kollman, 595 F.2d 48, 56, 201 USPQ 193, 199 (CCPA 1979), and MPEP § 717.02(d). (E) is not the proper choice because the relied-upon evidence can be in the specification. See In re Soni, 54 F.3d 746, 750, 34 USPQ2d 1684, 1687 (Fed. Cir. 1995).

When, in accordance with the patent laws, rules and procedures as related in the MPEP, is a supplemental oath or declaration treated as an amendment under 37 CFR 1.312? A When filed in a nonprovisional application after the Notice of Allowance has been mailed. B When filed in a reissue application at any point during the prosecution. C When filed in a nonprovisional application after the payment of the Issue Fee. D When filed in a reissue application after the Notice of Allowance has been mailed. E (A) and (D).

D Related MPEP Chapter(s): MPEP 700 - Examination of Applications Answer Reasoning:ANSWER: (D) is the most correct answer. MPEP § 714.16, third paragraph, states "a supplemental reissue oath or declaration is treated as an amendment under 37 CFR 1.312 because the correction of the patent which it provides is an amendment of the patent, even though no amendment is physically entered into the specification or claim(s)." (A) is incorrect because a supplemental oath or declaration is not treated as an amendment under 37 CFR 1.312 except when submitted in a reissue. See MPEP § 603.01. (B) is incorrect because a supplemental oath or declaration in a reissue will be treated as an amendment under 37 CFR § 1.312 only if filed after allowance. (C) is incorrect because amendments filed after the date the issue fee has been paid are no longer permitted under 37 CFR § 1.312. (E) is wrong because (A) is incorrect.

The Potter patent application was filed on June 6, 2016, claiming subject matter invented by Potter. The Potter application properly claims priority to a German application filed on June 6, 2015. In a first Office action, all the claims of the Potter application are rejected under 35 USC 102 based on a U.S. patent application publication to Smith et al ("Smith"). A registered practitioner prosecuting the Potter application ascertains that the relevant subject matter in Smith's published application and Potter's claimed invention were, at the time Potter's invention was made, owned by ABC Company or subject to an obligation of assignment to ABC Company. The practitioner ascertains that the Smith application was filed on April 10, 2015 and that the Smith application was published on December 5, 2016. Smith and Potter do not claim the same patentable invention. To overcome the rejection without amending the claims, which of the following replies would comply with the USPTO rules and the procedures set forth in the MPEP to be an effective reply for overcoming the rejection? A A reply that only contains arguments that Smith fails to teach all the elements in the only independent claim, and which specifically points out the claimed element that Smith lacks. B A reply that consists of an affidavit or declaration under 37 CFR 1.131 properly proving invention of the claimed subject matter of the Potter application prior to April 10, 2001. C A reply that consists of an affidavit or declaration under 37 CFR 1.132 properly showing that Smith's invention is not by "another." D A reply that properly states that the invention of the Potter application and the Smith application were commonly owned by ABC Company at the time of the filng of the Potter application. E All of the above.

D Related MPEP Chapter(s): MPEP 700 - Examination of Applications Answer Reasoning:ANSWER: (D) is the most correct answer. See 35 USC §§ 102(a)(2) and 102(b)(2).

Jack, a registered patent agent, received a final rejection of all of the claims in an application directed to an article of manufacture. Jack is about to file a timely Notice of Appeal to the Board of Patent Appeals and Interferences. Before filing his notice of appeal, Jack would like to tie up some loose ends by amendment. Which of the following reply (replies) may he file without triggering the requirements of 37 CFR 1.116(b)? A A reply that presents his argument in a more defensible light and adds additional claims. B A reply amending the claims into process claims. C A reply amending all of the independent claims, accompanied by a declaration from the inventor. D A reply complying with a requirement of form expressly set forth in the previous Office action. E (A) and (D).

D Related MPEP Chapter(s): MPEP 700 - Examination of Applications Answer Reasoning:ANSWER: (D). 37 C.F.R. § 1.116; MPEP § 714.13, Entry Not Matter of Right [p. 700-118]. The reply in (D) is directed to a reply permitted to be made under 37 C.F.R. § 1.116(a). (A), (B), and (C) are directed to the merits of the application, and are not in accord with 37 C.F.R. § 1.116(a).

Applicant Jones filed a request for a first continued prosecution application (CPA) on December 29, 2000 in a utility application that was filed on April 28, 2000. Jones received a final Office action mailed on June 28, 2001. In response, Jones filed an amendment amending the claims in the first CPA. Jones received an advisory action on September 27, 2001 stating that the proposed amendment to the first CPA would not be entered because it raises new issues that would require further consideration. Additionally, the proposed amendment did not meet the requirements for a complete reply under 37 CFR 1.111. On December 28, 2001, Jones filed a petition for a 3-month extension of time with appropriate petition fee, a request for a second continued prosecution application, a request for suspension of action, and appropriate processing fee for the request for suspension of action. No application filing fee was filed with the request for the second CPA. Which of the following would be a proper communication mailed by the Office based on Jones' actions? A A Notice of Allowability. B A Notice to File Missing Parts. C A first Office action on the merits. D A notice of improper Request for Continued Examination (RCE) and a notice of abandonment. E A letter granting the suspension of action.

D Related MPEP Chapter(s): MPEP 700 - Examination of Applications Answer Reasoning:ANSWER: The most correct answer is (D). See MPEP § 706.07(h), page 700-71, under "IV. IMPROPER CPA TREATED AS RCE." The request for a second CPA filed on December 28, 2001 is improper because the application in which the CPA was filed in has a filing date of December 29, 2000 and is not eligible for the CPA practice. The CPA practice does not apply to applications that have a filing date on or after May 29, 2000. The Office will automatically treat the improper request for a CPA as a Request for Continued Examination under 37 C.F.R. § 1.114. However, the request for a CPA filed on December 28, 2001 does not satisfy the requirements of 37 C.F.R. § 1.114 to be a proper RCE because it lacks the filing fee required by 37 C.F.R. § 1.17, and the required submission under 37 C.F.R. § 1.114. Therefore, the improper CPA will be treated as an improper RCE and the time period set in the last Office action mailed on June 28, 2001will continue to run. Since the time period expired on December 28, 2001, the application is abandoned. Answers (A), (B), and (C) are incorrect because the request for a second CPA filed on December 28, 2001 is improper and the amendment was not entered. Answer (E) is incorrect because a request for a suspension of action will not be granted if the CPA or the RCE is improper (e.g., a filing date was not accorded in the CPA or the RCE was filed without a submission). See 37 C.F.R. § 1.103, MPEP § 709, page 700-113.

n accordance with the USPTO rules and the procedures set forth in the MPEP, which of the following does not constitute probative evidence of commercial success to support a contention of non-obviousness? A In a utility case, gross sales figures accompanied by evidence as to market share. B In a utility case, gross sales figures accompanied by evidence as to the time period during which the product was sold. C In a utility case, gross sales figures accompanied by evidence as to what sales would normally be expected in the market. D In a utility case, gross sales figures accompanied by evidence of brand name recognition. E In a design case, evidence of commercial success clearly attributable to the design, and not to improved performance of the device.

D Related MPEP Chapter(s): MPEP 700 - Examination of Applications MPEP 2100 - Patentability Answer Reasoning:ANSWER: (D) is the most correct answer. Gross sales figures must be measured against a logical standard in order to determine whether or not there is commercial success. The recitations of accompanying evidence in (A), (B), and (C) are logical in that they provide a comparative basis for determining commercial success. (D), on the other hand, recites accompanying evidence which is illogical in that it does not provide a comparative basis for determining commercial success. (E) is wrong because it provides a logical basis for attributing commercial success to the design of the device, rather than the utilitarian function of the device. MPEP § 716.03(b).

A non-final Office action contains, among other things, a restriction requirement between two groups of claims (Group 1 and Group 2). Determine which of the following, if included in a timely reply under 37 CFR 1.111, preserves applicant's right to petition the Commissioner to review the restriction requirement in accordance with the patent laws, rules and procedures as related in the MPEP. A Applicant's entire reply to the restriction requirement is: "The examiner erred in distinguishing between Group 1 and Group 2, and therefore the restriction requirement is respectfully traversed and no election is being made, in order that applicant's right to petition the Commissioner to review the restriction requirement is preserved." B Applicant's entire reply to the restriction requirement is: "Applicant elects Group 1 and respectfully traverses the restriction requirement, because the examiner erred in requiring a restriction between Group 1 and Group 2." C Applicant's reply distinctly points out detailed reasons why applicant believes the examiner erred in requiring a restriction between Group 1 and Group 2, and additionally sets forth, "Applicant therefore respectfully traverses the restriction requirement and no election is being made, in order that applicant's right to petition the Commissioner to review the restriction requirement is preserved." D Applicant's reply distinctly points out detailed reasons why applicant believes the examiner erred in requiring a restriction between Group 1 and Group 2, and additionally sets forth, "Applicant therefore respectfully traverses the restriction requirement and elects Group 2. E None of the above.

D Related MPEP Chapter(s): MPEP 800 - Restriction in Applications Filed Under 35 U.S.C. 111; Double Patenting Answer Reasoning:ANSWER: (D) is the most correct answer. 37 CFR § 1.111(b); MPEP §§ 818.01. MPEP § 818.03(a) states "[a]s shown by the first sentence of 37 CFR 1.143, the traverse to a requirement must be complete as required by 37 CFR 1.111(b) . . . Under this rule, the applicant is required to specifically point out the reasons on which he or she bases his or her conclusions that a requirement to restrict is in error." An election must be made even if the requirement is traversed. MPEP § 818.01. Answer (A) is incorrect since the traversal does not distinctly point out the supposed errors in the examiner's action, and no election is made. 37 CFR § 1.143. MPEP § 818.01 states "[a] mere broad allegation that the requirement is in error does not comply with the requirement of 37 CFR 1.111." Answer (A) is also incorrect because no election is made. MPEP § 818.01 states, "[a]s noted in the second sentence of 37 CFR 1.143, a provisional election must be made even though the requirement is traversed. (B) is incorrect. MPEP § 818.03 since the traversal does not distinctly point out the supposed errors in the examiner's action. (C) is incorrect since no election is made. See MPEP § 818.03(b) (E) is incorrect because (D) is correct.

The MPEP and USPTO rules and procedure provide for ways that a statutory double patenting rejection can be overcome. Which of the following is an effective way to overcome a statutory double patenting rejection? A Filing a 37 CFR .131 affidavit to swear behind the patent on which the rejection is based. B Filing a terminal disclaimer under 37 CFR 1.321(c). C Filing a 37 CFR 1.131 affidavit and arguing that the conflicting claims are coextensive in scope. D Amending the conflicting claims so that they are not coextensive in scope. E All of the above.

D Related MPEP Chapter(s): MPEP 800 - Restriction in Applications Filed Under 35 U.S.C. 111; Double Patenting Answer Reasoning:ANSWER: Choice (D) is the correct answer. MPEP § 804.02, reads, "A rejection based on the statutory type of double patenting can be avoided by amending the conflicting claims so that they are not coextensive in scope." Choices (A) and (C) are each incorrect because MPEP § 804.02, reads, "The use of a 37 C.F.R. § 1.131 affidavit in overcoming a statutory double patenting rejection is inappropriate." Choice (C) is further incorrect since the statutory double patenting rejection is based on the conflicting claims being coextensive in scope. Choice (B) is incorrect because MPEP § 804.02, reads, "A terminal disclaimer is not effective in overcoming a statutory double patenting rejection." Choice (E) is incorrect because choices (A), (B), and (C) are each incorrect.

The MPEP sets forth a procedure whereby an examiner may contact an applicant to discuss election of claims after the examiner determines that a restriction requirement should be made. Assume that a primary examiner contacts a practitioner representing applicant by telephone prior to any Office action on the merits, and the examiner orally makes a restriction requirement. During the telephone conversation, the practitioner orally makes an election of claims without traverse. On examination, the examiner finds the elected claims to be allowable. Which of the following would be improper for the examiner to include in a letter to the practitioner attached to a Notice of Allowability? A A cancellation of the non-elected claims. B A statement that the prosecution is closed. C A statement that a Notice of Allowance will be sent in due course. D A statement that the applicant's election is not upheld because an election must only be made in writing, and cannot be made by telephone. E All of the above.

D Related MPEP Chapter(s): MPEP 800 - Restriction in Applications Filed Under 35 U.S.C. 111; Double Patenting Answer Reasoning:ANSWER: Choice (D) is the correct answer. MPEP § 812.01. Choices (A), (B), and (C) are each incorrect because MPEP § 812.01 reads, "If, on examination, the examiner finds the elected claims to be allowable and no traverse was made, the letter should be attached to the Notice of Allowability form PTOL-37 and should include cancellation of the nonelected claims, a statement that the prosecution is closed, and that a notice of allowance will be sent in due course." Choice (E) is incorrect because only choice (D) is correct.

In accordance with the USPTO rules and the procedures set forth in the MPEP, which of the following statements regarding a proper prior art reference is true? A Canceled matter in the application file of a U.S. patent is a prior art reference as of the filing date under 35 USC 102(a)(2). B Where a patent refers to and relies on the disclosure of a copending subsequently abandoned application, such disclosure is not available as a reference. C Where the reference patent claims the benefit of an earlier filed, copending but subsequently abandoned application which discloses subject matter in common with the patent, and the abandoned application has an enabling disclosure for the common subject matter and the claimed matter in the reference patent, the effective date of the reference patent as to the common subject matter is the filing date of the reference patent. D Matter canceled from the application file wrapper of a U.S. patent may be used as prior art as of the patent date. E All foreign patents are available as prior art as of the date they are translated into English.

D Related MPEP Chapter(s): MPEP 900 - Prior Art, Classification, and Search Answer Reasoning:ANSWER: (D) is the most correct answer. See 35 U.S.C. § 102(a). As explained in MPEP § 901.01, the "matter canceled from the application file wrapper of a U.S. patent may be used as prior art as of the patent date in that it then constitutes prior public knowledge under 35 U.S.C. 102(a), In re Lund, 376 F.2d 982, 153 USPQ 625 (CCPA 1967). See also MPEP 2127 and 2136.02." (A) is incorrect. 35 U.S.C. § 102(e). As stated in MPEP § 901.01, "Canceled matter in the application file of a U.S. patent is not a proper reference as of the filing date under 35 U.S.C. 102(e), see Ex parte Stalego, 154 USPQ 52, 53 (Bd. App. 1966)." (B) is incorrect. As stated in MPEP § 901.02, "In re Heritage, 182 F.2d 639, 86 USPQ 160 (CCPA 1950), holds that where a patent refers to and relies on the disclosure of a copending abandoned application, such disclosure is available as a reference. See also In re Lund, 376 F.2d 982, 153 USPQ 625 (CCPA 1967)." (C) is incorrect. As MPEP § 901.02 indicates, where the reference patent claims the benefit of a copending but abandoned application which discloses subject matter in common with the patent, and the abandoned application has an enabling disclosure of the common subject matter and claimed matter in the reference patent, the effective date of the reference as to the common subject matter is the filing date of the abandoned application. In re Switzer, 77 USPQ 1, 612 O.G. 11 (CCPA 1948); Ex parte Peterson, 63 USPQ 99 (Bd. App. 1944); and Ex parte Clifford, 49 USPQ 152 (Bd. App. 1940)." (E) is incorrect. As stated in MPEP § 901.05, "In general, a foreign patent, the contents of its application, or segments of its content should not be cited as a reference until its date of patenting or publication can be confirmed by an examiner's review of a copy of the document."

Application No. A was published as U.S. Patent Application Publication No. B. A member of the public reviewed the listing of the file contents of the application on the Patent Application Information Retrieval system and determined that the application was still pending, that a final Office action was mailed, and that the application file is in the Technology Center where it is being examined. The member of the public does not have a power to inspect, but would like a copy of the final Office action as well as the other papers in the patent application. In accordance with the USPTO rules and the procedures set forth in the MPEP, can a copy of these papers be obtained by the member of the public, and if so, how can the copy be obtained? A No, a copy cannot be obtained because patent applications are maintained in confidence pursuant to 35 USC 122(a). B No, a copy cannot be obtained because the patent application is still pending. C Yes, a member of the public can go to the Technology Center and ask for the file for copying at a public photocopier. D Yes, the member of the public can complete a "Request for Access to an Application Under 37 CFR 1.14(e)" and, without payment of a fee, order the file from the File Information Unit. Upon the Unit's receipt of the application, the member of the public can use a public photocopier to make a copy. E Yes, the member of the public can order a copy from the Office of Public Records, with a written request and payment of the appropriate fee.

E Related MPEP Chapter(s): MPEP 100 - Secrecy, Access, National Security, and Foreign Filing Answer Reasoning:ANSWER: (E) is the most correct answer. MPEP § 103, under the heading "Published U.S. Patent Applications" states that "If a patent application has been published pursuant to 35 U.S.C. 122(b), then a copy of the specification, drawings, and all papers relating to the file of that published application (whether abandoned or pending) may be provided to any person upon written request and payment of the fee." (A) and (B) are not correct. 37 CFR § 1.14(c)(2). Once an application has been published, a copy is available to the public upon written request and payment of a fee. (C) and (D) are not correct. As stated in MPEP § 103, under the heading "Published U.S. Patent Applications," if the published patent application is pending, the application file itself will not be available to the public for inspection."

Application Number A was published as U.S. Patent Application Publication Number B. A member of the public reviewed the listing of the file contents of the application on the Patent Application Information Retrieval system and determined that the application was still pending, that a final Office action was mailed, and that the application file is in the Technology Center where it is being examined. The member of the public does not have a power to inspect, but would like a copy of the final Office action as well as the other papers in the patent application. In accordance with the patent laws, rules and procedures as related in the MPEP, can a copy of these papers be obtained by the member of the public, and if so, how can the copy be obtained? A No, a copy cannot be obtained because patent applications are maintained in confidence pursuant to 35 USC 122(a). B No, a copy cannot be obtained because the patent application is still pending. C Yes, a member of the public can go to the Technology Center and ask for the file for copying at a public photocopier. D Yes, the member of the public can complete a "Request for Access to an Application Under 37 CFR 1.14(e)" and, without payment of a fee, order the file from the File Information Unit. Upon the Unit's receipt of the application, the member of the public can use a public photocopier to make a copy. E Yes, the member of the public can order a copy from the Office of Public Records, with a written request and payment of the appropriate fee.

E Related MPEP Chapter(s): MPEP 100 - Secrecy, Access, National Security, and Foreign Filing Answer Reasoning:ANSWER: (E) is the most correct answer. MPEP § 103, under the heading "Published U.S. Patent Applications" states that "If a patent application has been published pursuant to 35 U.S.C. 122(b), then a copy of the specification, drawings, and all papers relating to the file of that published application (whether abandoned or pending) may be provided to any person upon written request and payment of the fee." (A), and (B) are not correct. 37 CFR § 1.14(c)(2). Once an application has been published, a copy is available to the public upon written request and payment of a fee. (C) and (D) are not correct. As stated in MPEP § 103, under the heading "Published U.S. Patent Applications," if the published patent application is pending, the application file itself will not be available to the public for inspection."

A registered practitioner filed a first patent application wherein claims 1-10 claims are directed to a widget and claims 11-20 are directed to a method of making a widget. Following a proper restriction requirement, claims 1-10 were elected for prosecution. The primary examiner rejected claims 1-10. The practitioner filed a reply that only consisted of argument. The examiner was unpersuaded by the argument, and entered a final rejection of claims 1-10. In reply, the practitioner filed a continuing application containing claims 1-10 directed to a widget, and claims 11-20 directed to a method of using a widget. In the continuing application, the examiner enters a new written restriction requirement requiring a provisional election between claims 1-10 and claims 11-20. The practitioner believes the new restriction requirement is improper and would like the rejection in the parent application reviewed as well. The new restriction requirement has not been made final. Which of the following best describes whether and why, in accordance with the patent laws, rules, and procedures as related by the MPEP, the reply to the restriction requirement may be by appeal to the Board of Patent Appeals and Interferences? A Yes. An immediate appeal to the Board can be filed to review the restriction requirement if any claims have been twice rejected. B No. An immediate appeal cannot be filed to the Board because the new claims directed to a method of using a widget have not been twice rejected. C Yes. An immediate appeal can be filed for any claims that have been twice rejected because the Board can also review any second restriction requirement made against the same claims. D No. An immediate appeal to the Board cannot be lodged because a provisional election has not been made of either the claims to a widget or claims to a method of use of the widget. E No. An immediate appeal cannot be taken because no claims are currently under rejection. Review of a final restriction requirement is only possible as a petitionable matter before a Technology Center Director. It is not an appealable matter to the Board.

E Related MPEP Chapter(s): MPEP 1000 - Matters Decided by Various U.S. Patent and Trademark Office Officials Answer Reasoning:ANSWER: (E) is the most correct answer. MPEP § 1002.02(c) identifies among the matters petitionable to and decided by the Technology Center Directors "Petitions from a final decision of examiner requiring restriction in patent applications, 37 CFR 1.144, MPEP § 818.03(c)." Hence (A), and (C), which provide for review before the Board of Patent Appeals and Interferences are clearly erroneous. Since the restriction requirement is not yet "final" no review is possible at this juncture. Answers (A), (B), (C), and (D) are also incorrect because no claim is under rejection hence no appeal is possible. See MPEP § 1205, which provides that under 37 CFR 1.191(a), an applicant for a patent dissatisfied with the primary examiner's decision in the second or final rejection of his or her claims may appeal to the Board for review of the examiner's rejection by filing a notice of appeal and the required fee set forth in 37 CFR 1.17(b) within the time period provided under 37 CFR 1.134.and 1.136. A notice of appeal may be filed after any of the claims has been twice rejected, regardless of whether the claim(s) has/have been finally rejected. The limitation of "twice or finally...rejected" does not have to be related to a particular application. For example, if any claim was rejected in a parent application, and the claim is again rejected in a continuing application, then applicant will be entitled to file an appeal in the continuing application, even if the claim was rejected only once in the continuing application.

Following a restriction requirement and election, a registered practitioner received a first Office action dated Friday, December 1, 2016. The primary examiner indicated that claims 1 to 10 were rejected and claims 11 to 20 were withdrawn from consideration. The first Office action set a 3 month shortened statutory period for reply. On February 28, 2017, the practitioner properly filed an express abandonment in the application and at the same time filed a request for continuing application. In a non-final Office action dated May 1, 2017 in the continuing application, the examiner indicated in that claims 1 to 20, all of the pending claims, are rejected. The practitioner filed a notice of appeal on Monday, July 2, 2017. In accordance with USPTO rules and procedures set forth in the MPEP, which of the following most accurately describes the propriety of the practitioner's reply to the May 1st Office action? A The notice of appeal is not a proper response because the claims of the continuing application have not been finally rejected. B The notice of appeal is not a proper reply because all of the claims in the continuing application have not been twice rejected. C The filing of a notice of appeal is not a proper reply because not all the claims in the continuing application have been twice rejected. D A notice of appeal is never a proper response to a non-final rejection. E The reply is proper.

E Related MPEP Chapter(s): MPEP 1200 - Appeal Answer Reasoning:ANSWER: (E) is the most correct answer. MPEP § 1204, under the heading "Appeal By Patent Applicant," states that "[a] notice of appeal may be filed after any of the claims has been twice rejected, regardless of whether the claim(s) has/have been finally rejected. The limitation of 'twice or finally...rejected' does not have to be related to a particular application. For example, if any claim was rejected in a parent application, and the claim is again rejected in a continuing application, then applicant will be entitled to file an appeal in the continuing application, even if the claim was rejected only once in the continuing application." (A), (B), (C), and (D) are not the most correct answer because a notice of appeal can be filed in a continuing application where at least one of the rejected claims was twice rejected, and one of the rejections may occur in the parent application.

Appellant believes that an examiner's answer, mailed on November 1, 2016, contains a new ground of rejection, but it is not labeled as such. If an amendment or new evidence is needed to overcome the new ground of rejection, what is the best course of action the appellant should take in accordance with the USPTO rules and the procedures set forth in the MPEP? A File a reply brief bringing the new ground of rejection to the attention of the Board of Patent Appeals and Interferences and pointing out that 37 CFR 41.39(a)(2) prohibits entry of the new ground of rejection. B Ignore the new ground of rejection. C File a reply brief arguing the merits of the new ground of rejection. D File an amendment or new evidence to overcome the new ground of rejection. E File a timely petition pursuant to 37 CFR 1.181 seeking review such that the new ground of rejection in the answer is so designated, after efforts to persuade the examiner to reopen prosecution or remove the new ground of rejection are unsuccessful.

E Related MPEP Chapter(s): MPEP 1200 - Appeal Answer Reasoning:ANSWER: (E) is the most correct answer. See MPEP 1207.03 (IV). A) is wrong because new grounds of rejection are (rarely) permitted. C) is wrong because the new grounds have not been labeled as such. D) is wrong because this cannot be done until the new ground of rejection has been labeled as such. B) is wrong because any rejection ignored by Appellant is automatically sustained by the Board.

n accordance with the USPTO rules and the procedures set forth in the MPEP, which of the following is true? A Once the issue fee has become due, provided an original application has not been pending more than three years, the applicant may request and the Office may grant a request for deferral of payment of the issue fee. B The time period set for the payment of the issue fee is statutory and cannot be extended. However, if payment is not timely made and the delay in making the payment is unintentional, upon payment of a fee for delayed payment, it may be accepted as though no abandonment had occurred, but there will be a reduction on the patent term adjustment for the period of abandonment. C Upon written request, a person citing patents and printed publications to the Office that the person believes has a bearing on the patentability of a particular patent, may request that his or her name remain confidential. D To obtain benefit of priority based on an earlier filed U.S. patent application, an applicant in a later filed continuation application is not required to meet the conditions and requirements of 35 USC 120. E Each of statements (B) and (C) is true.

E Related MPEP Chapter(s): MPEP 1300 - Allowance and Issue Answer Reasoning:ANSWER: (E) is the most correct answer. As to (B), see 35 U.S.C. §§ 151; 154(b)(2)(ii) and (iii); 37 CFR § 1.704(c)(3); MPEP § 1306. As to (C) see MPEP §§ 2203 and 2212. As to (D), the claim for priority is not required, as a person may not wish to do so in order to increase the term of his or her patent. As to (A) deferral under 37 CFR § 1.103 is not available following the notice of allowance. Since (B) and (C) are correct, (E) is the best answer.

Igor filed a design patent application in the USPTO on January 24, 2000, which issued as a design patent on January 23, 2001. Igor's design patent covered a design that became immediately popular, resulting in numerous inquiries for licenses from various manufacturers. Igor would like to financially exploit his patent by licensing for five years. However, Igor has decided to dedicate five years of his patent term to the public. Which of the following is in accord with the USPTO rules and the procedures set forth in the MPEP, while best allowing Igor to pursue his intentions? A Record in the USPTO an assignment of all right, title, and interest in the patent to the public, conditioned on the receipt by Igor of all royalties from licensing the patent after the first five years of the patent term. B File a disclaimer in the USPTO dedicating to the public the first five years of the patent term. C File a disclaimer in the USPTO dedicating to the public that portion of the term of the patent from January 24, 2015 to January 24, 2020. D File a disclaimer in the USPTO dedicating to the public half of all royalties received from licensing the patent for the terminal part of the term of the patent. E File a disclaimer in the USPTO dedicating to the public that portion of the term of the patent from January 24, 2010 to January 23, 2015.

E Related MPEP Chapter(s): MPEP 1400 - Correction of Patents Answer Reasoning:ANSWER: (E) is correct because 37 CFR § 1.321(a) states, in pertinent part, that "any patentee may disclaim or dedicate to the public...any terminal part of the term, of the patent granted." 35 U.S.C. § 173 states, "Patents for designs shall be granted for the term of fourteen years from the date of grant." (A) is wrong because such action would not permit Igor to financially exploit any portion of the term of his patent, since 37 CFR § 3.56 indicates that the result is a conditional assignment, which is regarded as an absolute assignment for Office purposes. (B) is wrong because 37 CFR § 1.321(a) provides for dedication to the public of "the entire term, or any terminal part of the term" only. "[T]he first five years of the patent term" does not qualify as a terminal part of the term. (C) is wrong because Igor would not achieve his objective of dedicating at least a portion of his patent term to the public, since the term of the design patent would expire on January 23, 2015. 35 U.S.C. § 173. (D) is wrong because 37 CFR § 1.321(a) restricts a disclaimer to "any complete claim or claims" or "the entire term, or any terminal part of the term" of a patent. "Royalties received from licensing" are not addressed by 37 CFR § 1.321(a).

Claims in your client's patent application have been rejected as unpatentable over prior art. In accordance with proper USPTO practice and procedure, which, if any, of the following statements is true? A The prior art is a U.S. patent issued five years before the effective date of your client's application. The claims are properly rejected under 35 USC 102 of the disclosure in the patent that anticipates the claimed invention. Evidence of secondary considerations, such as unexpected results or commercial success, is relevant to the rejection and thus can overcome the rejection. B The prior art is a U.S. patent issued five years before the effective date of your client's application. The claims are properly rejected under 35 USC 102 over the disclosure in the patent that anticipates the claimed invention. The rejection can be overcome by arguing that the alleged anticipatory prior art is "nonanalogous art." C The prior art is a U.S. patent issued five years before the effective date of your client's application. The claims are properly rejected under 35 USC 102 over the disclosure in the patent that anticipates the claimed invention. The rejection can be overcome by arguing that the alleged anticipatory prior art teaches away from the invention. D The prior art is a U.S. patent issued five years before the effective date of your client's application. The claims are properly rejected under 35 USC 102 over the disclosure in the patent that anticipates the claimed invention. The rejection can be overcome by arguing that the alleged anticipatory prior art is not recognized as solving the problem solved by the claimed invention. E None of the above.

E Related MPEP Chapter(s): MPEP 2100 - Patentability Answer Reasoning:ANSWER: The most correct answer is (E). (A), (B), (C) and (D) are not in accordance with proper USPTO practice and procedure. (A) alone is not correct. MPEP § 2131.04 and see In re Wiggins, 179 USPQ 421, 425 (CCPA 1973). (B), (C), and (D) are not correct. MPEP § 2131.05, and see Twin Disc, Inc. v. U. S., 231 USPQ 417, 424 (Cl. Ct. 1986); In re Self, 213 USPQ 1, 7 (CCPA 1982).

In accordance with the USPTO rules and procedures set forth in the MPEP, a Certificate of Correction effectuates correction of an issued patent where: A Through error and without deceptive intent, there is a failure to make reference to a prior copending application according to 37 CFR 1.78, and the failure does not otherwise affect what is claimed, but the prior copending application is referenced in the record of the application, and a petition under 37 CFR 1.323 and appropriate fees were filed. B Through error and without deceptive intent, a preferred embodiment that materially affects the scope of the patent was omitted in the original disclosure in the filed application, and a petition under 37 CFR 1.323 and appropriate fees were filed. C Through error and without deceptive intent, a prior copending application is incorrectly referenced in the application, the incorrect reference does not otherwise affect the claimed subject matter, and the prior copending application is correctly identified elsewhere in the application file, and a petition under 37 CFR 1.323 and appropriate fees were filed. D Through error and without deceptive intent, an inventor's name is omitted from an issued patent, a petition under 37 CFR 1.324 and appropriate fees were filed, and the petition was granted. E (A), (C) and (D).

E Related MPEP Chapter(s): MPEP 1400 - Correction of Patents Answer Reasoning:ANSWER: (E) is the most correct answer. (A) and (C) can be corrected by a certificate of Correction. MPEP § 1481. (D) can be corrected by a Certificate of Correction. 37 CFR § 1.324; MPEP § 1481. (B) is incorrect. Such a mistake, which affects the scope and meaning of the claims in a patent, is not considered to be of the "minor" character required for issuance of a Certificate of Correction. MPEP § 1481.

A U.S. patent was granted on May 8, 2015 to five inventors. The five inventors assigned their entire patent rights to Q Company. Q Company needs to file a reissue application to broaden the claims of the patent. The registered practitioner preparing the application has been unable to locate any of the five inventors to sign the reissue oath or declaration. Today is May 8, 2017. Which of the following should the practitioner do to enable the applicant to broaden the patent claims in accordance with the patent laws, rules and procedures as related in the MPEP? A Wait to file the reissue application until the first day the signatures of all five inventors can be obtained. At that time, pay the filing fee and file a petition seeking May 8, 2017 as the filing date. File with the petition a showing of the unavailability of all inventors until the filing of the application. B Wait to file the reissue application until the signatures of at least three inventors can be obtained. At that time, file a petition seeking May 7, 2017 as the filing date accompanied by a showing of the unavailability of all inventors on May 8th. Payment of the filing fees may be postponed until receipt of a decision on the petition. C File the reissue application on May 8, 2017, presenting only the claims in the patent, and include a listing of inventors, but not pay the filing fee at the time of filing. D Wait to file the reissue application until the signature of one of the inventors has been obtained since at least one inventor is needed to show a lack of deceptive intent on the part of the applicants. E File the complete reissue application complying with 37 CFR 1.173(a) and 1.53(b) with an unexecuted reissue declaration listing the names of all the inventors with at least one broadening claim on May 8, 2017.

E Related MPEP Chapter(s): MPEP 1400 - Correction of Patents Answer Reasoning:ANSWER: (E) is the most correct answer. 35 U.S.C. § 251; 37 CFR § 1.53(f); MPEP § 1403. Filing a broadened reissue application with at least one broadening claim prior to the expiration of the two-year time period set in the statute satisfies the diligence provisions of 35 U.S.C. § 251. The executed reissue oath/declaration and the filing fee may be filed at a later time. According to MPEP § 1403, a reissue application can be granted a filing date without an oath or declaration, or without the filing fee being present. See 37 CFR § 1.53(f). The reissue applicant will be given a period of time to provide the missing parts and to pay the surcharge under 37 CFR § 1.16(e). See MPEP § 1410.01. (A), (B) and (D) are clearly incorrect since the inventors and assignee would be barred from a broadening reissue if filed after the two year period set in the statute. (C) is incorrect since the reissue application was filed without at least one broadening claim prior to the expiration of the two-year time period set in 35 U.S.C. § 251.

Inventor A filed a patent application and assigned the entire interest in the application to his employer, MegaCorp. The application issued as a utility patent on July 9, 2002. In June 2004, MegaCorp's management first learns that a second inventor, Inventor B, should have been named as a co-inventor with respect to at least one claim of the issued patent. There was no deceptive intent in failing to name Inventor B in the original application. Inventor A, who is unfamiliar with patent law and concepts of inventorship, incorrectly believes that he should be the sole named inventor on the patent, and refuses to cooperate with any effort by MegaCorp to change the named inventive entity. The issued patent contains no other error. In accordance with the Manual of Patent Examining Procedure, which of the following procedures is/are available for MegaCorp to seek correction of the named inventive entity without any agreement, cooperation or action from Inventor A? A File, on or before July 9, 2004, a reissue application, made by MegaCorp only, that seeks to add Inventor B. B File, after July 9, 2004, a reissue application, made by MegaCorp only, that seeks to add Inventor B. C Request a Certificate of Correction to add Inventor B as a named inventor. D Submit in the issued patent file: a Request for Correction of Inventorship Under the Provisions of 37 CFR 1.48 that sets forth the desired inventorship change; a statement by Inventor B that the error in inventorship occurred without deceptive intention on her part; an oath or declaration executed by Inventor B; all required fees; and the written consent of MegaCorp. E A and B are each available procedures.

E Related MPEP Chapter(s): MPEP 1400 - Correction of Patents Answer Reasoning:ANSWER: The best choice is (E). See MPEP § 1412.04. Reissue is a proper vehicle for correcting inventorship in a patent. Because correction of inventorship does not enlarge the scope of the patent claims, the reissue application may be filed more than two years after the patent issued. Answers (A) and (B) are therefore both correct, and (E) is the best response. Although a certificate of correction may be used to correct inventorship where all parties are in agreement, the facts of the question show that Inventor A is not in agreement. Choice (C) is thus not an available option for MegaCorp. Choice (D) is incorrect because the provisions of 37 C.F.R. § 1.48 are not available to correct inventorship in an issued patent.

In accordance with MPEP § 1500, relating to design patent applications: A the invention may be properly represented in a single application by both an ink drawing and a black and white photograph. B the invention may be properly represented in a single application by a black and white photograph disclosing environmental structure by broken lines, in lieu of an ink drawing if the invention is shown more clearly in the photograph. C the invention may be properly represented in a single application by both an ink drawing and a color photograph, and the application should be accompanied by the required petition, fee, three sets of color photographs, and an amendment to the specification. D the invention may be properly represented by a color photograph disclosing environmental structure by broken lines, in lieu of an ink drawing if the invention is not capable of being illustrated in an ink drawing. E the invention may be properly represented by a color photograph if the invention is not capable of being illustrated in an ink drawing, and if the application is accompanied by the required petition, fee, and an amendment to the specification is presented to insert required language regarding the color photographs, and three sets of color photographs.

E Related MPEP Chapter(s): MPEP 1500 - Design Patents Answer Reasoning:ANSWER: (E) is correct. The statement finds support in MPEP § 1503.02, V. "Photographs and Color Drawings." (A) and (C) are wrong because 37 C.F.R. § 1.152 states, "Photographs and ink drawings are not permitted to be combined as formal drawings in one application." Reproduced in MPEP § 1503.02. (B) and (D) are wrong because 37 C.F.R. § 1.152 states, "Photographs submitted in lieu of ink drawings in design patent applications must not disclose environmental structure but must be limited to the design claimed for the article." Reproduced in MPEP § 1503.02.

Joan goes to a registered practitioner wanting to know the status of the applications of her competitor Pete. During Joan's previous relationship with Pete she believes she may have been a coinventor on one of the applications filed by Pete. Pete owns Applications A, B, C and D. Application B is a continuation of Application A and a redacted copy of Application A has been published under 35 USC 122(b). Joan is listed as a coinventor on Application C. Pete has an issued patent that claims priority to Application D. Assume only the last six digits of the numerical identifier are available for Application D and Application D is abandoned. Which of the following, in accordance with the USPTO rules and the procedures set forth in the MPEP, is not true? A Joan may obtain status information for Application B that is a continuation of an Application A since Application A has been published under 35 USC 122(b). B Joan may be provided status information for Application D that includes the filing date if the eight-digit numerical identifier is not available and the last six digits of the numerical identifier are available. C Joan may obtain status information for Application D since a U.S. patent includes a specific reference under 35 USC 120 to Application D, an abandoned application. Joan may obtain a copy of that application-as-filed by submitting a written request including the fee set forth in 37 CFR 1.19(b)(1). D Joan may obtain status information as to Application C since a coinventor in a pending application may gain access to the application if his or her name appears as an inventor in the application, even if she did not sign the § 1.63 oath or declaration. E Joan may obtain access to the entire Application A by submitting a written request, since, notwithstanding the fact that only a redacted copy of Application A has been published, a member of the public is entitled to see the entire application upon written request.

E Related MPEP Chapter(s): MPEP 200 - Types, Cross-Noting, and Status of Application Answer Reasoning: Statement (E) is false, and is thus the most correct answer. A redacted copy of the application was used for publication purposes, and 37 CFR § 1.14(a)(1)iii: provides that "If a redacted copy of the application was used for the patent application publication, the copy of the specification, drawings, and papers may be limited to a redacted copy." For (A) and (B), see 37 CFR § 1.14a(2). For (C) see 37 CFR §§ 1.14(a)(1)iv. As to (D), a coinventor is entitled to access to the application independent of whether or not he or she signed the declaration. Note that as stated in 37 CFR § 1.41(a)(2), if a declaration or oath is not filed, the inventorship is that inventorship set forth in the application papers.

20. On January 3, 2016, a registered practitioner filed a continuation application that includes a benefit claim to a prior-filed application. The practitioner simultaneously filed in the prior-filed application an express abandonment in favor of a continuing application. The prior application contained five drawing figures described in the specification. However, the continuation application contains only four of the five drawing figures. The specification of the continuation application did not include a complete description of the missing drawing figure. A postcard from the USPTO, listing the contents of the continuation application, contains a note that only four drawing figures were received. The practitioner inadvertently omitted one of the drawing figures mentioned in the specification when he filed the continuation application. The missing drawing figure shows a claimed feature of the invention. On February 10, 2016, the practitioner received a Notice of Omitted Item(s) properly according a filing date of January 3, 2016 for the continuation application without the missing drawing figure and notifying the applicant that the drawing is missing. Which of the following procedures for filing the missing drawing would comply with the patent laws, rules and procedures as related in the MPEP for according the continuation application a January 3, 2016 filing date with the five drawing figures that were present in the application? A The practitioner files the missing drawing figure in response to the Notice of Omitted Item(s) within the time period set forth in the notice with no change in filing date. B The practitioner files the missing drawing figure and an amendment to the specification to add a complete description of the missing drawing figure in response to the Notice of Omitted Item(s) within the time period set forth in the notice. C The practitioner files an amendment to cancel the description of the missing drawing figure from the specification of the continuation application. D If the continuation application as originally filed includes an incorporation by reference of the prior-filed application to which the benefit is claimed, the practitioner can file the missing drawing figure any time prior to the first Office action. E (A) and (D)

E Related MPEP Chapter(s): MPEP 200 - Types, Cross-Noting, and Status of Application Answer Reasoning:ANSWER: (E) (D) is a correct answer. MPEP § 201.06(c), under the heading "INCORPORATION BY REFERENCE", subheading "B. Application Entitled to a Filing Date," states that "[i]f the application as originally filed includes a proper incorporation by reference of the prior application(s), an omitted specification page(s) and/or drawing figure(s) identified in a "Notice of Omitted Item(s)" may be added by amendment provided the omitted item(s) contains only subject matter in common with such prior application(s). In such case applicant need not respond to the "Notice of Omitted Item(s)." Applicant should submit the amendment adding the omitted material prior to the first Office action to avoid delays in the prosecution of the application." (A) is also correct inasmuch as an assertion of priority in the originally filed paperwork operates as an automatic incorporation by reference for inadvertently omitted subject from the priority application. (B) is incorrect because the application filing date will be the date of the filing of the missing specification section. (C) is incorrect. The continuation purposefully omitted the description of the missing drawing.

Which of the following is not in accordance with the provisions in the MPEP? A A continuation-in-part is an application filed during the lifetime of an earlier nonprovisional application by the same applicant, which repeat either some substantial portion or all of the earlier nonprovisional application, and adds matter not disclosed in the said earlier nonprovisional application. B A continuation-in-part application may only be filed under 37 CFR 1.53(b). C A continuation-in-part application cannot be filed as a continued prosecution application (CPA) under 37 CFR 1.53(d). D An application claiming the benefits of a provisional application under 35 USC 119(e) should not be called a "continuation-in-part" of the provisional application. E One of the formal requirements of 35 USC 120 is that a continuation-in-part application must be "filed before a notice of allowance or abandonment is mailed in the prior application."

E Related MPEP Chapter(s): MPEP 200 - Types, Cross-Noting, and Status of Application Answer Reasoning:ANSWER: (E). As to (E), it is not in accord with MPEP § 201.08 since the application need not be filed before a notice of allowance, but instead before patenting of the first application. (A) through (C) are found in MPEP § 201.08. As to (D), calling the patent application a continuation-in-part will result in the application having its patent term calculated from its filing date. An application filed under 35 U.S.C. §§ 120, 121, or 365(c) will have its patent term calculated from the date on which the earliest application was filed, provided a specific reference is made to the earlier filed application(s). 35 U.S.C. § 154(a)(2) and (a)(3).

Smith's first invention is a new method of fabricating a semiconductor capacitor in a dynamic random access memory (DRAM) cell. Smith filed a first patent application on December 13, 2016 disclosing and claiming the first invention. Smith's later, second invention, is an improved semiconductor capacitor in a DRAM cell and a method of making it. Smith filed a second application on December 16, 2017, claiming the benefit of the filing date of the copending first application. The second application contains claims 1-20, and a specification that provides support for the claimed subject matter in compliance with 35 USC 112(a). In the second application, claims 1-10 are drawn to Smith's first invention, and claims 11-20 are drawn to Smith's second invention. The primary examiner found a non-patent printed publication authored by Jones published on February 4, 2017. The article discloses the both of Smith's inventions. Which of the following courses of action by the examiner would be in accord with the patent laws, rules and procedures as related in the MPEP? A The examiner can reject claims 1-20 in the second application using the article because the publication date of the article is earlier than the filing date of the second application. B The examiner cannot reject any of the claims in the second application using the article because the second application claims the benefit of the filing date of the first application. C The examiner can reject claims 1-20 in the second application using the article because the second application is not entitled to the benefit of the filing date of the first application since the second application was filed more than one year from the filing date of the first application. D The examiner can reject claims 1-10, but cannot reject claims 11-20 in the second application because the first application did not disclose the improved capacitor set forth in claims 11-20. E The examiner cannot reject claims 1-10, but can reject claims 11-20 in the second application because the first application did not disclose an improved capacitor set forth in claims 11-20.

E Related MPEP Chapter(s): MPEP 200 - Types, Cross-Noting, and Status of Application Answer Reasoning:ANSWER: The most correct answer is (E). See MPEP § 201.11, under the heading "VI. When Not Entitled To Benefit Earlier Of Filing Date, " states "[a]ny claim in a continuation-in-part application which is directed solely to subject matter adequately disclosed under 35 U.S.C. 112 in the parent nonprovisional application is entitled to the benefit of the filing date of the parent nonprovisional application. However, if a claim in a continuation-in-part application recites a feature which was not disclosed or adequately supported by a proper disclosure under 35 U.S.C. 112 in the parent nonprovisional application, but which was first introduced or adequately supported in the continuation-in-part application such a claim is entitled only to the filing date of the continuation-in-part application. See In re Chu, 66. F.3d 292, 36 USPQ2d 1089 (Fed. Cir. 1995) and Transco Products, Inc. v. Performance Contracting Inc., 38 F.3d 551, 32 USPQ2d 1077 (Fed. Cir. 1994)." Accordingly, claims 1-10 are entitled to the benefit of the filing date of the first application, but claims 11-20 are not entitled to the benefit of the filing date of the first application because claims 11-20 recite an improved capacitor, which was not disclosed in the first application. Claims 1-10 have an effective filing date earlier than the publication date of the article. Claims 11-20 have a filing date later than the publication date of the article. For 35 U.S.C. 102(a) to apply, the reference must have a publication date earlier in time than the effective filing date of the application. See MPEP 706.02(a), paragraph "III. 35 U.S.C. 102(a)." Thus, answers (A)-(D) are incorrect.

Which of the following is in accordance with the provisions in the MPEP? A In order to correct inventorship in a nonprovisional application where the statement of the lack of deceptive intent is not available from an inventor to be added, a petition under 37 CFR 1.181 may be properly filed. B If a person A learns that a patent application has been filed by person B without naming A as coinventor, A may file in the USPTO a petition that protests inventorship and directs B to add A's name as a coinventor to the patent application. C If the application is involved in an interference pre-AIA, and a petition under 37 CFR 1.48 is filed to correct inventorship, the Board of Patent Appeals and Interferences will remand the case to the primary examiner for consideration of the petition to ensure that a search of the relevant prior art is performed. D When a second conversion under 37 CFR 1.48(a) is attempted by the practitioner, the conversion decision will be decided by the Technology Center Director. E None of the above.

E Related MPEP Chapter(s): MPEP 200 - Types, Cross-Noting, and Status of Application MPEP 1900 - Protest Answer Reasoning:ANSWER: (E) is correct. As to (D), see MPEP § 1002.02(e). Requests under 37 CFR 1.48 are generally decided by the primary examiner. (A) is incorrect because the petition to be filed would be under 37 C.F.R. § 1.183. As to (B), 35 U.S.C. § 116 provides that inventors may apply for a patent jointly. A person not named in the application could not file a petition under 37 C.F.R. § 1.48 as this would not be a joint filing. In accordance with MPEP 1901.05, unless a protestor has been granted access, the protestor is not entitled to any information including the fact that the application exists. The patent application process is ex parte, and inventorship may only be contested inter partes (between opposing parties) in the USPTO through the interference process. MPEP § 1901.07 precludes the protestor from further participation beyond submission of the protest. See also 37 C.F.R. § 1.291(c). (C) is incorrect at least for the reasons that if the application is involved in an interference, the Board will decide the petition.

A registered practitioner files an amendment to the client's claim which inserts language into the claim. The primary examiner improperly rejects the claim under 35 USC 112(a) description requirement. The examiner's rejection states that the amendment inserted new matter which does not have descriptive support in the original specification. The examiner correctly points out that there is no literal support for the amendatory claim language in the original specification, but erroneously concludes that it constitutes new matter. Assume that there is support for the amendment in the original disclosure. In accordance with the patent laws, rules and procedures as related in the MPEP, a proper reply would include which of the following argument(s) to show the examiner is in error? A The original specification would enable one of ordinary skill in the art to practice the invention as now claimed. B Literal support for new claim language is not required. C The original specification reasonably conveys to one of ordinary skill in the art that the inventor had the claimed invention in his/her possession as of the filing date of the application. D The new claim language is described in a related application filed by the inventor that is now a U.S. patent. E (B) and (C).

E Related MPEP Chapter(s): MPEP 2100 - Patentability Answer Reasoning:ANSWER: (E) is the most correct answer because (B) and (C) together are correct. Regarding (B), see MPEP § 2163.02, which states, "Whenever the [written description] issue arises, the fundamental factual inquiry is whether the specification conveys with reasonable clarity to those skilled in the art that, as of the filing date sought, applicant was in possession of the invention as now claimed. See, e.g., Vas-Cath, Inc. v. Mahurkar, 935 F.2d 1555, 1563-64, 19 USPQ2d 1111, 1117 (Fed. Cir. 1991). An applicant shows possession of the claimed invention by describing the claimed invention with all of its limitations using such descriptive means as words, structures, figures, diagrams, and formulas that fully set forth the claimed invention. Lockwood v. American Airlines, Inc., 107 F.3d 1565, 1572, 41 USPQ2d 1961, 1966 (Fed. Cir. 1997)." Regarding (C), see MPEP § 2163.02, which states, "The courts have described the essential question to be addressed in a description requirement issue in a variety of ways. An objective standard for determining compliance with the written description requirement is, 'does the description clearly allow persons of ordinary skill in the art to recognize that he or she invented what is claimed.' In re Gosteli, 872 F.2d 1008, 1012, 10 USPQ2d 1614, 1618 (Fed. Cir. 1989). Under Vas-Cath, Inc. v. Mahurkar, 935 F.2d 1555, 1563-64, 19 USPQ2d 1111, 1117 (Fed. Cir. 1991), to satisfy the written description requirement, an applicant must convey with reasonable clarity to those skilled in the art that, as of the filing date sought, he or she was in possession of the invention, and that the invention, in that context, is whatever is now claimed." (B) alone and (C) alone are incorrect inasmuch as they do not address each of the examiner's rationales for the rejection. (A) is incorrect. MPEP § 2161. The written description requirement is separate and distinct from the enablement requirement of 35 U.S.C. § 112, first paragraph. The argument does not address and otherwise traverse the rejection that was made. (D) is incorrect. MPEP § 2163.03, under the headings "RELIANCE ON FILING DATE OF PARENT APPLICATION UNDER 35 U.S.C. 120," and "RELIANCE ON PRIORITY UNDER 35 U.S.C. 119." The related case must be an application having a filing date to which the instant application is entitled, e.g., a parent or provisional application. The argument does not show the instant application is related to the "related application" under 35 U.S.C. §§ 119 or 120. (B) alone is not correct because (C) is also correct. (C) alone is not correct because (B) is also correct.

In accordance with patent laws, rules and procedures as related in the MPEP, an abandoned U.S. patent application: A is never available as evidence of prior art. B may become prior art only when it is properly incorporated by reference in the disclosure of a U.S. patent. C may become prior art as of its filing date, but only if it is properly incorporated by reference in the disclosure of a U.S. patent. D may become evidence of prior art as of its filing date, but only if it is properly incorporated by reference in the disclosure of a U.S. patent or U.S. application publication. E may become prior art when it is properly incorporated by reference in the disclosure of a U.S. application publication.

E Related MPEP Chapter(s): MPEP 2100 - Patentability Answer Reasoning:ANSWER: (E) is the most correct answer. As set forth in MPEP § 2127, under the heading "Abandoned Applications, Including Provisional Applications," and subheading, "Abandoned Applications Disclosed to the Public Can Be Used as Prior Art," states "the subject matter of an abandoned application, including both provisional and nonprovisional applications, referred to in a prior art U.S. patent may be relied on in a 35 U.S.C. 102(e) rejection based on that patent if the disclosure of the abandoned application is actually included or incorporated by reference in the patent. Compare In re Lund, 376 F.2d 982, 991, 153 USPQ 625, 633 (CCPA 1967) (The court reversed a rejection over a patent which was a continuation-in-part of an abandoned application. Applicant's filing date preceded the issue date of the patent reference. The abandoned application contained subject matter which was essential to the rejection but which was not carried over into the continuation-in-part. The court held that the subject matter of the abandoned application was not available to the public as of either the parent's or the child's filing dates and thus could not be relied on in the 102(e) rejection.)." (A) is incorrect since an abandoned patent application may become evidence of prior art. Answers (B), (C) and (D) are incorrect due to the use of the word "only". Answer (E) does not include the term "only". In addition, Answer (C) and (D) are also incorrect due to the inclusion of the phrase "as of its filing date". As set forth above, "An abandoned patent application becomes available as prior art only as of the date the public gains access to it. See 37 CFR 1.14(e)(2)."

In accordance with patent laws, rules and procedures as related in the MPEP, a rejection under 35 USC 102 can be overcome by demonstrating: A the reference is nonanalogous art. B the reference teaches away from the claimed invention. C the reference disparages the claimed invention. D (A), (B) and (C). E None of the above.

E Related MPEP Chapter(s): MPEP 2100 - Patentability Answer Reasoning:ANSWER: (E) is the most correct answer. As set forth in MPEP § 2131.05, "'Arguments that the alleged anticipatory prior art is 'nonanalogous art' or 'teaches away from the invention' or is not recognized as solving the problem solved by the claimed invention, [are] not 'germane' to a rejection under section 102.' Twin Disc, Inc. v. United States, 231 USPQ 417, 424 (Cl.Ct.1986) (quoting In re Self, 671 F.2d 1344, 213 USPQ 1, 7 (CCPA 1982)). A reference is no less anticipatory if, after disclosing the invention, the reference then disparages it. The question whether a reference 'teaches away' from the invention is inapplicable to an anticipation analysis. Celeritas Technologies Ltd. v. Rockwell International Corp., 150 F.3d 1354, 1361, 47 USPQ2d 1516, 1522-23 (Fed.Cir.1999)." Therefore, answers (A) through (D) are incorrect. See also MPEP § 706.02(b) as to ways to overcome a rejection under 35 U.S.C. § 102.

An examiner has properly established a prima facie showing of no specific and substantial credible utility for the claimed invention in a patent application filed in February 2001. An applicant can sustain the burden of rebutting and overcoming the showing by: A Providing reasoning or arguments rebutting the basis or logic of the prima facie showing. B Amending the claims. C Providing evidence in the form of a declaration under 37 CFR 1.132 rebutting the basis or logic of the prima facie showing. D Providing evidence in the form of a printed publication rebutting the basis or logic of the prima facie showing. E All of the above.

E Related MPEP Chapter(s): MPEP 2100 - Patentability Answer Reasoning:ANSWER: (E) is the most correct answer. For (A) see, "Utility Examination Guidelines" MPEP 2100. "The applicant can do this by... providing reasoning or arguments... ." For (B), see, "Utility Examination Guidelines," Id. "The applicant can do this by amending the claims... ." For (C) see again, "Utility Examination Guidelines," "The applicant can do this by...providing evidence in the form of a declaration under 37 C.F.R. § 1.132...rebutting the basis or logic of the prima facie showing." For (D), see, "Utility Examination Guidelines," "The applicant can do this by...providing evidence in the form of a...printed publication...rebutting the basis or logic of the prima facie showing." (A), (B), (C), and (D) alone are not the most correct answer inasmuch (E), referencing all of the above, is the most correct answer.

In accordance with the patent laws, rules and procedures as related in the MPEP, which of the following is not within the scope of the term "on sale" as it is used in 35 USC 102(a)? A A sale conditioned on buyer satisfaction. B A sale that did not result in a profit. C A single sale of the claimed subject matter. D A commercial offer to sale the claimed subject matter. E An offer to sale the patent rights in the claimed subject matter.

E Related MPEP Chapter(s): MPEP 2100 - Patentability Answer Reasoning:ANSWER: (E) is the most correct answer. As set forth in MPEP § 2133.03(b), under the heading "I The Meaning Of 'Sale'," subheading "A Sale of Rights Is Not a Sale of the Invention and Will Not in Itself Bar a Patent," "[a]n assignment or sale of the rights, such as patent rights, in the invention is not a sale of 'the invention' within the meaning of section 102(b). The sale must involve the delivery of the physical invention itself. Moleculon Research Corp. v. CBS, Inc., 793 F.2d 1261, 1265, 229 USPQ 805, 809 (Fed. Cir. 1986)." (A) is incorrect because it is within the scope of "on sale" as it is used in 35 U.S.C. § 102(b). MPEP § 2133.03(b), under the heading "I. The Meaning Of 'Sale'," subheading "A. Conditional Sale May Bar a Patent," states "[a]n invention may be deemed to be 'on sale' even though the sale was conditional. The fact that the sale is conditioned on buyer satisfaction does not, without more, prove that the sale was for an experimental purpose. Strong v. General Elec. Co., 434 F.2d 1042, 1046, 168 USPQ 8, 12 (5th Cir. 1970)." (B) is incorrect because it is within the scope of "on sale" as it is used in 35 U.S.C. § 102(b). MPEP § 2133.03(b), under the heading "I. The Meaning Of 'Sale'," subheading "Nonprofit Sale May Bar a Patent," states "[a] 'sale' need not be for profit to bar a patent. If the sale was for the commercial exploitation of the invention, it is 'on sale' within the meaning of 35 U.S.C. 102(b). In re Dybel, 524 F.2d 1393, 1401, 187 USPQ 593, 599 (CCPA 1975) ('Although selling the devices for a profit would have demonstrated the purpose of commercial exploitation, the fact that appellant realized no profit from the sales does not demonstrate the contrary.'). (C) is incorrect because it is within the scope of "on sale" as it is used in 35 U.S.C. § 102(b). MPEP § 2133.03(b), under the heading "I. The Meaning Of 'Sale'," subheading "A Single Sale or Offer To Sell May Bar a Patent," states "[e]ven a single sale or offer to sell the invention may bar patentability under 35 U.S.C. 102(b). Consolidated Fruit-Jar Co. v. Wright, 94 U.S. 92, 94 (1876); Atlantic Thermoplastics Co. v. Faytex Corp., 970 F.2d 834, 836-37, 23 USPQ2d 1481, 1483 (Fed. Cir. 1992)." (D) is incorrect because it is within the scope of "on sale" as it is used in 35 U.S.C. § 102(b). MPEP § 2133.03(b) quotes Group One, Ltd. v. Hallmark Cards, Inc., 254 F.2d 1041, 1047, 59 USPQ2d 1121, 1126 (Fed. Cir. 2001) as stating "As a general proposition, we look to the Uniform Commercial Code ('UCC') to define whether ... a communication or series of communications rises to the level of a commercial offer for sale."

In accordance with the patent laws, rules and procedures as related in the MPEP, definiteness of claim language under 35 U.S.C. 112(b) must be analyzed, not in a vacuum, but in light of: A The content of the particular application disclosure. B The teachings of the prior art. C The claim interpretation that would be given by one possessing the ordinary level of skill in the pertinent art at the time the application was filed. D The claim interpretation that would be given by one possessing expert skill in the pertinent art at the time the invention was made.. E (A), (B) and (C).

E Related MPEP Chapter(s): MPEP 2100 - Patentability Answer Reasoning:ANSWER: (E) is the most correct answer. As set forth in MPEP § 2173.02, filtered through the new 103, definiteness of claim language must be analyzed, not in a vacuum, but in light of: (A) The content of the particular application disclosure; (B) The teachings of the prior art; and (C) The claim interpretation that would be given by one possessing the ordinary level of skill in the pertinent art at the time the applicaation was filed. Answers (A), (B) and (C) each identify criteria to be analyzed in considering whether claim language is definite, therefore answer (E) which includes each of these answers is the most correct answer. Answer (D) is incorrect since it does not include criteria (C) and references the time that the invention was made, which is no longer relevant to AIA consideration.

A registered practitioner files a nonprovisional utility application in 2015. In 2017, the practitioner files a continuation-in-part application and claims benefit of the filing date of the 2015 application for the 2017 application. Thereafter, the practitioner amends the 2017 application to include claims that were not present in either the originally filed 2015 application or the originally filed 2017 application. The primary examiner properly concludes that the added claims are not supported by the original disclosure in either application. Which of the following is in accord with the patent laws, rules and procedures as related in the MPEP? A The added claims are rejected for lack of written description under 35 USC 112, first paragraph. B The added claims are rejected as new matter under 35 USC 132. C The added claims are denied benefit of the filing date of the 2015 application. D (A) and (B). E (A) and (C).

E Related MPEP Chapter(s): MPEP 2100 - Patentability Answer Reasoning:ANSWER: (E) is the most correct answer. Both (A) and (C) are correct. MPEP § 2163.01, under the heading "Support For The Claimed Subject Matter In The Disclosure," states that "[I]f the examiner concludes that the claimed subject matter is not supported [described] in an application as filed, this would result in a rejection of the claim on the ground of a lack of written description under 35 U.S.C. 112, first paragraph, or denial of the benefit of filing date of a previously filed application." (B) is incorrect. MPEP § 2163.01 states that unsupported claims "should not be rejected or objected to on the ground of new matter. As framed by the court in In re Rasmussen, 650 F.2d 1212, 211 USPQ 323 (CCPA 1981), the concept of new matter is properly employed as a basis for objection to amendments to the abstract, specification or drawings attempting to add new disclosure to that originally presented." (D) is incorrect because (B) is incorrect.

A registered practitioner files a nonprovisional utility application in 2014. In 2016, the practitioner files a continuation-in-part application and claims benefit of the filing date of the 2014 application for the 2016 application. Thereafter, the practitioner amends the 2016 application to include claims that were not present in the either the originally filed 2014 application or the originally filed 2016 application. The primary examiner properly concludes that the added claims are not supported by the original disclosure in either application. Which of the following is in accord with the USPTO rules and the procedures set forth in the MPEP? A The added claims are rejected for lack of written description under 35 USC 112, first paragraph. B The added claims are rejected as new matter under 35 USC 132. C The added claims are denied benefit of the filing date of the 2014 application. D (A) and (B). E (A) and (C).

E Related MPEP Chapter(s): MPEP 2100 - Patentability Answer Reasoning:ANSWER: (E) is the most correct answer. Both (A) and (C) are correct. MPEP § 2163.01, under the heading "Support For The Claimed Subject Matter In The Disclosure," states that "[I]f the examiner concludes that the claimed subject matter is not supported [described] in an application as filed, this would result in a rejection of the claim on the ground of a lack of written description under 35 U.S.C. 112, first paragraph, or denial of the benefit of filing date of a previously filed application." (B) is incorrect. MPEP § 2163.01 states that unsupported claims "should not be rejected or objected to on the ground of new matter. As framed by the court in In re Rasmussen, 650 F.2d 1212, 211 USPQ 323 (CCPA 1981), the concept of new matter is properly employed as a basis for objection to amendments to the abstract, specification or drawings attempting to add new disclosure to that originally presented." (D) is incorrect because (B) is incorrect.

An examiner has properly established a prima facie showing of no specific and substantial credible utility for the claimed invention in a patent application filed in February 2016. An applicant can sustain the burden of rebutting and overcoming the showing in accordance with the USPTO rules and the procedures set forth in the MPEP by: A Providing reasoning or arguments rebutting the basis or logic of the prima facie showing. B Amending the claims. C Providing evidence in the form of a declaration under 37 CFR 1.132 rebutting the basis or logic of the prima facie showing. D Providing evidence in the form of a printed publication rebutting the basis or logic of the prima facie showing. E All of the above.

E Related MPEP Chapter(s): MPEP 2100 - Patentability Answer Reasoning:ANSWER: (E) is the most correct answer. For (A) see, MPEP § 2107, under the heading "Examination Guidelines For The Utility Requirement," penultimate paragraph, which states "The applicant can do this by... providing reasoning or arguments... ." For (B), see MPEP § 2107, under the heading "Examination Guidelines For The Utility Requirement," penultimate paragraph, which states "The applicant can do this by amending the claims... ." For (C) see, MPEP § 2107, under the heading "Examination Guidelines For The Utility Requirement," penultimate paragraph, which states "The applicant can do this by...providing evidence in the form of a declaration under 37 C.F.R. § 1.132...rebutting the basis or logic of the prima facie showing." For (D), see, MPEP § 2107, under the heading "Examination Guidelines For The Utility Requirement," penultimate paragraph, which states "The applicant can do this by...providing evidence in the form of a...printed publication...rebutting the basis or logic of the prima facie showing." (A), (B), (C), and (D) alone are not the most correct answer inasmuch (E), referencing all of the above, is the most correct answer.

On January 2, 2016, a registered practitioner filed a patent application with the USPTO for inventor Bock. The application includes a specification and a single claim to the invention, which reads as follows: 1. A new string consisting only of material Z that has the ability to stretch to beyond its initial unstretched length. On June 2, 2016, the practitioner received an Office action from the primary examiner rejecting the claim. The claim is solely rejected under 35 USC 102 in view of Patent A, which discloses a string consisting only of material Z. The Office action states, "Patent A discloses a string consisting only of material Z. Patent A does not expressly teach the stretchability property of the string. Nevertheless, the recited stretchability is inherent in the string of patent A. Accordingly, patent A anticipates the claimed string." Mr. Bock believes he is entitled to a patent to his new string and authorizes the practitioner to reply to the Office action by arguing that his string stretches to ten times its initial unstretched length, something that patent A does not teach. Since this is not expressly taught in Patent A, the practitioner argues, Patent A cannot anticipate the claimed string. In accordance with USPTO rules and procedures set forth in the MPEP, is the practitioner's reply persuasive as to error in the rejection? A Yes. B Yes, but the claim should now be rejected again, this time under 35 USC 103 as obvious over Patent A. C Yes, because the stretchability property is expressly taught by Patent A. D Yes, examiner nowhere addresses the claimed limitation of stretching the string beyond its initial unstretched length. E No.

E Related MPEP Chapter(s): MPEP 2100 - Patentability Answer Reasoning:ANSWER: (E) is the most correct answer. MPEP § 2112, under the heading "Something Which Is Old Does Not Become Patentable Upon The Discovery Of A New Property," states that "claiming of a new use, new function or unknown property which is inherently present in the prior art does not necessarily make the claim patentable. In re Best, 562 F.2d 1252, 1254, 195 USPQ 430, 433 (CCPA 1977)." The issue is whether the argument has persuasively rebutted the examiner's prima facie case of anticipation. The argument does not rebut the prima facie case. The claim is directed to a string consisting only of material Z. Patent A teaches each every element of that string; i.e., the string shape, material Z, and the string only consists of material Z. There is nothing different between the string of the claim and that of patent A. Under those circumstances, examiner correctly stipulated that the stretchability of the claimed string; i.e., the ability to stretch the string beyond its initial unstretched length, would be an inherent property of the disclosed string. Whether or not patent A teaches the stretchability of its string is not defeating. Even if Mr. Bock had discovered a new property for the string, it would still not render the claim patentable. In re Best, supra . Here all the product elements are the same and examiner made out a proper prima facie case of anticipation. The burden now shifts to the practitioner to show that the patent string is not the same. The reply, which seeks to establish a difference in properties without showing a concomitant difference in product material and shape is not persuasive as to error in the rejection. All the other answers are wrong. (A) is not the most correct answer. See MPEP § 2112. (B) is not the most correct answer. The anticipation rejection was correctly established and was not rebutted by the argument. (C) is not the most correct answer. See MPEP § 2112, discussing when a reference can anticipate based on an inherent, as opposed to an expressly or implicit, disclosure. (D) is not the most correct answer. In discussing the stretchability property, the recited ability to stretch the string beyond its initial unstretched length was equally addressed.

The claims in an application are rejected under 35 USC 103 as obvious over prior art reference A in view of prior art reference B. All of the following statements are in accord with the patent laws, rules and procedures as related in the MPEP except: A Where the combination of prior art references provides motivation to make the claimed invention to achieve the same advantage or result discovered by the applicant, the references do not have to expressly suggest the combination of references. B The rationale to modify or combine the prior art references may be reasoned from knowledge generally available to one of ordinary skill in the art, established scientific principles, or legal precedent established by prior case law. C In considering the disclosure of the prior art references, it is proper to take into account the specific teachings of the references, as well as the inferences that one skilled in the art could reasonably draw from the specific teachings. D An examiner may take official notice of facts outside the record that are capable of instant and unquestionable demonstration as being "well known" prior art or common knowledge in the art. E To rely on equivalence as a KSR rationale supporting an obviousness rejection under 35 USC 103 an examiner may base the rejection on the mere fact that the components at issue are functional or mechanical equivalents without further explanation of the rationale.

E Related MPEP Chapter(s): MPEP 2100 - Patentability Answer Reasoning:ANSWER: (E) is the most correct answer. See KSR Guidelines and MPEP § 2144.06, under the heading "Substituting Equivalents Known For The Same Purpose," states "[i]n order to rely on equivalence as a rationale supporting an obviousness rejection, the equivalency must be recognized in the prior art, and cannot be based on applicant's disclosure or the mere fact that the components at issue are functional or mechanical equivalents. In re Ruff, 256 F.2d 590, 118 USPQ 340 (CCPA 1958)." (A) is incorrect. MPEP § 2144, under the heading "Rationale Different From Applicant's Is Permissible," states "[t]he reason or motivation to modify the reference may often suggest what the inventor has done, but for a different purpose or to solve a different problem. It is not necessary that the prior art suggest the combination to achieve the same advantage or result discovered by applicant. In re Linter, 458 F.2d 1013, 173 USPQ 560 (CCPA 1972). . . ; In re Dillon, 919 F.2d 688, 16 USPQ2d 1897 (Fed. Cir. 1990), cert. denied, 500 U.S. 904 (1991). . . ." (B) is incorrect. MPEP § 2144, under the heading "Rationale May Be In A Reference, Or Reasoned From Common Knowledge In The Art, Scientific Principles, Art-Recognized Equivalents, Or Legal Precedent," states "[t]he rationale to modify or combine the prior art does not have to be expressly stated in the prior art; the rationale may be expressly or impliedly contained in the prior art or it may be reasoned from knowledge generally available to one of ordinary skill in the art, established scientific principles, or legal precedent established by prior case law. In re Fine, 837 F.2d 1071, 5 USPQ2d 1596 (Fed. Cir. 1988); In re Jones, 958 F.2d 347, 21 USPQ2d 1941 (Fed. Cir. 1992). See also . . . In re Nilssen, 851 F.2d 1401, 1403, 7 USPQ2d 1500, 1502 (Fed. Cir. 1988) (references do not have to explicitly suggest combining teachings)." (C) is incorrect. MPEP § 2144.01, quotes In re Preda, 401 F.2d 825, 159 USPQ 342, 344 (CCPA 1968) as stating "[i]n considering the disclosure of a reference, it is proper to take into account not only specific teachings of the reference but also the inferences which one skilled in the art would reasonably be expected to draw therefrom." (D) is incorrect. MPEP § 2144.03 under the heading "A. Determine When It Is Appropriate To Take Official Notice Without Documentary Evidence To Support The Examiner's Conclusion," states "[o]fficial notice without documentary evidence to support an examiner's conclusion is permissible only in some circumstances. . . . Official notice unsupported by documentary evidence should only be taken by the examiner where the facts asserted to be well-known, or to be common knowledge in the art are capable of instant and unquestionable demonstration as being well-known. As noted by the court in In re Ahlert, 424 F.2d 1088, 1091, 165 USPQ 418, 420 (CCPA 1970), the notice of facts beyond the record which may be taken by the examiner must be 'capable of such instant and unquestionable demonstration as to defy dispute (citing In re Knapp Monarch Co., 296 F.2d 230, 132 USPQ 6 (CCPA 1961)."

A registered practitioner files an application on the client's discovery that adding silica to a known plastic composition containing the flame retardant, X, results in increased flame retardance. The application claims a composition comprising the known plastic composition containing X and also silica. The primary examiner rejects the claim on the basis that applicant admits that X was a known flame retardant and that there is no evidence of improved flame retardance. In accordance with the patent laws, rules and procedures as related in the MPEP, a proper reply could include which of the following argument(s) to rebut and overcome the rejection? A The examiner cannot rely on admitted prior art. B The examiner has not shown that the prior art appreciated applicant's discovery of silica to be a flame retardant. C The examiner has not made out a prima facie case of obviousness due to lack of motivation in the prior art or in the knowledge generally available to one of ordinary skill in the art for adding silica to the known plastic composition. D The applicant does not have to show an improved or unexpected result for the claimed invention. E (C) and (D).

E Related MPEP Chapter(s): MPEP 2100 - Patentability Answer Reasoning:ANSWER: (E) is the most correct answer. See KSR guidelines and MPEP §§ 2141, 2142, 2143 and 2143.01. Regarding (C) MPEP 2142 under the heading "ESTABLISHING A PRIMA FACIE CASE OF OBVIOUSNESS," states "To establish a prima facie case of obviousness, three basic criteria must be met. First, there must be some suggestion or motivation, either in the references themselves or in the knowledge generally available to one of ordinary skill in the art, to modify the reference or to combine reference teachings. Second, there must be a reasonable expectation of success. Finally, the prior art reference (or references when combined) must teach or suggest all the claim limitations. The teaching or suggestion to make the claimed combination and the reasonable expectation of success must both be found in the prior art, and not based on applicant's disclosure. In re Vaeck, 947 F.2d 488, 20 USPQ2d 1438 (Fed. Cir. 1991). See MPEP § 2143 for decisions pertinent to each of these criteria." MPEP § 2143 states the same criteria, and further "The teaching or suggestion to make the claimed combination and the reasonable expectation of success must both be found in the prior art, not in applicant's disclosure. In re Vaeck, 947 F.2d 488, 20 USPQ2d 1438 (Fed. Cir. 1991)." Regarding motivation, MPEP 2143.01 states "Obviousness can only be established by combining or modifying the teachings of the prior art to produce the claimed invention where there is some teaching, suggestion, or motivation to do so found either explicitly or implicitly in the references themselves or in the knowledge generally available to one of ordinary skill in the art. 'The test for an implicit showing is what the combined teachings, knowledge of one of ordinary skill in the art, and the nature of the problem to be solved as a whole would have suggested to those of ordinary skill in the art.' In re Kotzab, 217 F.3d 1365, 1370, 55 USPQ2d 1313, 1317 (Fed. Cir. 2000). See also In re Fine, 837 F.2d 1071, 5 USPQ2d 1596 (Fed. Cir. 1988); In re Jones, 958 F.2d 347, 21 USPQ2d 1941 (Fed. Cir. 1992)." The examiner has not shown any suggestion or motivation, either in the prior art or in the knowledge generally available to one of ordinary skill in the art, to modify the known plastic composition. Regarding (D) MPEP § 2142 states "The examiner bears the initial burden of factually supporting any prima facie conclusion of obviousness. If the examiner does not produce a prima facie case, the applicant is under no obligation to submit evidence of nonobviousness. If, however, the examiner does produce a prima facie case, the burden of coming forward with evidence or arguments shifts to the applicant who may submit additional evidence of nonobviousness, such as comparative test data showing that the claimed invention possesses improved properties not expected by the prior art." (E) is the most correct answer since it addresses both issues raised by the examiner, obviousness and evidence of improved results. (C) alone is incorrect inasmuch as it does not address the examiner's comments regarding improved results. (D) alone incorrect because it does not address the examiner's burden of presenting a prima facie case of obviousness. (A) is not correct. MPEP § 2129. Admissions by applicant can be used as prior art. (B) is not correct. MPEP § 2144. It is not necessary for a finding of obviousness that prior art be combined for the same advantage or result as applicant.

With respect to establishing "reasonable diligence" for under pre-AIA 35 USC 102(g), which of the following statements is or are in accordance with the patent laws, rules and procedures as related in the MPEP? (1) The inventor and his attorney must drop all other work and concentrate on the particular invention involved. (2) The entire period during which diligence is required must be accounted for by either affirmative acts or acceptable excuses. (3) Work relied upon to show reasonable diligence must be directly related to the reduction to practice. A Statement (1) only B Statement (2) only C Statement (3) only D Statements (1) and (3) E Statements (2) and (3)

E Related MPEP Chapter(s): MPEP 2100 - Patentability Answer Reasoning:ANSWER: (E) is the most correct, because statements (2) and (3) are true. The entire period for which diligence is required must be accounted for. MPEP § 2138.06, under the heading "The Entire Period During Which Diligence Is Required Must Be Accounted For By Either Affirmative Acts Or Acceptable Excuses," states "[a]n applicant must account for the entire period during which diligence is required. Gould v. Schawlow, 363 F.2d 908, 919, 150 USPQ 634, 643 (CCPA 1966) (Merely stating that there were no weeks or months that the invention was not worked on is not enough.)." MPEP § 2138.06, under the heading "Work Relied Upon To Show Reasonable Diligence Must Be Directly Related To The Reduction To Practice," states "[t]he work relied upon to show reasonable diligence must be directly related to the reduction to practice of the invention in issue. Naber v.Cricchi, 567 F.2d 382, 384, 196 USPQ 294, 296 (CCPA 1977), cert. denied, 439 U.S. 826 (1978). 'U]nder some circumstances an inventor should also be able to rely on work on closely related inventions as support for diligence toward the reduction to practice on an invention in issue.'" (A) is incorrect because statement (1) is not true - an inventor or his attorney need not drop all other work to establish reasonable diligence. Emery v. Ronden, 188 USPQ 264, 268 (Bd. Pat. Inter. 1974); MPEP § 2138.06. (B) is incorrect because it does not include true statement (3). (C) is incorrect because it does not include true statement (2). (D) is incorrect because it includes false statement (1).

An application as originally filed contains the following Claim 1: Claim 1. A doughnut making machine comprising: (i) an input conveyor that receives dough to be used in making said doughnuts; (ii) means for portioning dough from said input conveyor into a plurality of dough balls, each of said plurality of balls containing dough sufficient to create a single doughnut; (iii) means for forming each of said dough balls into a ring of dough; (iv) a deep fat fryer which receives rings of dough from said forming means and cooks said rings of dough; (v) means for selectively applying a flavored coating on cooked rings of dough to produce doughnuts; and (vi) means for placing a plurality of said doughnuts on a flat sheet. The specification adequately describes the claimed subject matter. Two different "means for selectively applying" are described in the specification: a sprayer and a brush. Which of the following original claims is an improper dependent claim?

E Related MPEP Chapter(s): MPEP 2100 - Patentability Answer Reasoning:ANSWER: The most correct answer is (E). A dependent claim must further limit the claim from which it depends. 35 U.S.C. § 112, ¶4; 37 C.F.R. § 1.75(c). Dependent claim 6 (Answer E) improperly seeks to broaden Claim 1 by omitting an element set forth in the parent claim.

There are many factors to be considered when determining whether there is sufficient evidence to support a determination that a disclosure does not satisfy the enablement requirement and whether any necessary experimentation is "undue." Which of the following are among the factors for determining whether necessary experimentation is "undue"? A The breadth of the claims. B The nature of the invention. C The state of the prior art. D The level of one of ordinary skill. E (A), (B), (C) and (D).

E Related MPEP Chapter(s): MPEP 2100 - Patentability Answer Reasoning:ANSWER: The most correct answer is (E). MPEP § 2164.01(a).

Which of the following is a proper basis for establishing a substantial new question of patentability to obtain reexamination in accordance with proper USPTO rules and the procedures set forth in the MPEP? A An admission per se by the patent owner of record that the claimed invention was on sale, or in public use more than one year before any patent application was filed in the USPTO. B A prior art patent that is solely used as evidence of an alleged prior public use. C A prior art patent that is solely used as evidence of an alleged insufficiency of disclosure. D A printed publication that is solely used as evidence of an alleged prior offer for sale. E None of the above.

E Related MPEP Chapter(s): MPEP 2200 - Citation of Prior Art and Ex Parte Reexamination of Patents Answer Reasoning:ANSWER: (E) is the most correct answer. See 35 U.S.C. § 302; MPEP § 2217. The prior art applied may only consist of prior art patents or printed publications. Substantial new questions of patentability may be based upon 35 U.S.C. §§ 102(a), (b), (d) and (e), new questions of patentability under 35 U.S.C. § 103 that are based on the foregoing indicated portions of 35 U.S.C. § 102, and substantial new questions of patentability may be found under 35 U.S.C. §§ 102(f)/103 or 102(g)/103 based on the prior invention of another disclosed in a patent or printed publication. (A) is incorrect. See MPEP § 2217. An admission, per se, may not be the basis for establishing a substantial new question of patentability. However, an admission by the patent owner of record in the file or in a court record may be utilized in combination with a patent or printed publication. (B), (C), and (D) are incorrect. A prior art patent cannot be properly applied as a ground for reexamination if it is merely used as evidence of alleged prior public use or sale, or insufficiency of disclosure. The prior art patent must be applied directly to claims under 35 U.S.C. § 103 and/or an appropriate portion of 35 U.S.C. § 102 or relate to the application of other prior art patents or printed publications to claims on such grounds.

A patent examiner resigned from the USPTO on June 7, 2001, and returned to Sheboygan, Wisconsin. The next day, on June 8, 2001, the former examiner signed up for a one week seminar entitled, "How to Become Rich Without Really Working." During the seminar, the sponsors offered the former examiner a golden opportunity to purchase a 10% interest in a U.S. patent application that they stated is "guaranteed to produce significant royalties and give her a 1000% return on her investment." Soon after attending the seminar, the former examiner became a registered practitioner. Which of the following accords with proper practice and procedure? A The former examiner may accept the offer, but only if an ownership interest in the application is transferred to the former examiner by an instrument in writing. B The former examiner can accept the offer, but only if an ownership interest in the application is transferred to the former examiner by an instrument in writing, which is made of record in the assignment records of the USPTO. C The former examiner can accept the offer, but only if an ownership interest in the application is transferred to the former examiner by an instrument in writing, which is made of record in the file of the application. D The former examiner should accept the offer, but only if an ownership interest in the application is transferred to the former examiner by an instrument in writing, and the original or a true copy of the original instrument, in writing, is made of record in the assignment records of the USPTO and in the file of the application. E The former examiner cannot accept the offer because she is incapable of acquiring an interest in the application at that time under the circumstances.

E Related MPEP Chapter(s): MPEP 300 - Ownership and Assignment Answer Reasoning:ANSWER: (E) is correct and (A), (B), (C) and (D) are wrong. As a former employee of the USPTO, the former examiner is incapable of acquiring an interest, directly or indirectly, in a patent application in the manner described in the question during the period of appointment as an examiner, and for one year thereafter. 35 U.S.C. § 4; MPEP § 309. Inasmuch as the former examiner resigned from the USPTO on June 7, 2001, the former examiner is incapable of acquiring an interest in the application in said manner until June 8, 2002. Registration as a practitioner does not affect the restrictions on the former examiner.

A registered practitioner's client, Apex Mfg. Corporation, bought the entire assets of Pinnacle Mfg. Corporation. Pinnacle gave Apex a list of its patent applications but did not maintain records of the patent assignments. Apex wishes to know which of the assignment documents, if any, of the pending patent applications in Pinnacle's patent portfolio were never recorded. In accordance with the MPEP, which of the following actions could the practitioner rely upon to most expeditiously answer this question? A Request certified copies of the patent applications as filed. B Request certified copies of the assignment documents of record of the patent applications. C Request certified copies of the patent applications as filed accompanied by the fees set forth in 37 CFR 1.19(b)(1)(i). D Request certified copies of the patent applications as filed accompanied by the fees set forth in 37 CFR 1.19(b)(1)(ii). E Request certified copies of the assignment documents of record of the patent applications accompanied by the fees set forth in 37 CFR 1.19(b)(4).

E Related MPEP Chapter(s): MPEP 300 - Ownership and Assignment Answer Reasoning:ANSWER: (E) is correct see fee in 37 CFR 1.19(b)(4) and (A), (B), (C) and (D) are incorrect. As MPEP § 303 states: "Certified copies of patent applications as filed do not include an indication of assignment documents. Applicants desiring an indication of assignment documents of record should request separately certified copies of assignment documents and submit the fees required by 37 C.F.R. § 1.19." (B) is incorrect because the fee required by 37 C.F.R. § 1.19(b)(4) has not been paid for the requested certified copy of assignment records.

A registered practitioner properly recorded an assignment document for application A identifying XYZ Company as the assignee. The document assigns to XYZ Company the "subject matter claimed in Application A." A proper restriction requirement was made by a primary examiner in application A between two distinct inventions, and the practitioner elected to prosecute one of the inventions. Application A was prosecuted, and later became abandoned. Before the abandonment date of application A, the practitioner filed a complete application B as a proper divisional application of application A. Application B claimed the nonelected invention of Application A, and was published as a U.S. application publication. XYZ Company remains the assignee of application A. What must the practitioner do in accordance with the patent laws, rules and procedures as related in the MPEP to ensure that XYZ Company is listed as the assignee on the face of any patent issuing from application B? A File a proper assignment document in application B identifying XYZ Company as the assignee. B File a proper assignment document in application B identifying XYZ Company as the assignee, and confirm that USPTO's bibliographic data for application B identifies XYZ Company as the assignee by checking the filing receipt for application B, the U.S. application publication of application B, or the USPTO's Patent Application Information Retrieval (PAIR) system data for application B, depending on when the practitioner filed the assignment document in application B. C Confirm that XYZ Company is identified as the assignee on the U.S. application publication of application B. D File a proper assignment document in application B identifying XYZ Company as the assignee, and confirm that XYZ Company is identified as the assignee on the U.S. application publication of application B. E Upon allowance of application B, the practitioner must identify XYZ Company as the assignee in the appropriate space on the Issue Fee Transmittal form for specifying the assignee for application B.

E Related MPEP Chapter(s): MPEP 300 - Ownership and Assignment Answer Reasoning:ANSWER: (E) is the most correct answer. MPEP §§ 306 and 307. MPEP § 306 states, "In the case of a division or continuation application, a prior assignment recorded against the original application is applied to the division or continuation application because the assignment recorded against the original application gives the assignee rights to the subject matter common to both applications." MPEP § 307 states, "Irrespective of whether the assignee participates in the prosecution of the application, the patent issues to the assignee if so indicated on the Issue Fee Transmittal form PTOL-85B. Unless an assignee's name and address are identified in item 3 of the Issue Fee Transmittal form PTOL-85B, the patent will issue to the applicant. Assignment data printed on the patent will be based solely on the information so supplied." A new assignment document need not be recorded for a divisional or continuation application where the assignment recorded in the parent application remains the same. Accordingly, (A), (B) and (D) are incorrect. In addition, (A), (B) and (D) are incorrect because unless an assignee's name and address are identified in item 3 of PTOL-85B, the patent will issued to the applicant and the assignee information, even if recorded, will not appear on the patent. (C) is incorrect for the same reason. (B) is also incorrect. Assignment data is reflected on the filing receipt, PAIR, or a patent application publication when applicant includes assignment information for purposes of publication of the application on the transmittal letter. Assignment data printed on the patent will be based solely on the information supplied on the Issue Fee Transmittal Form PTOL-85B. See MPEP §§ 1309 and 1481. Accordingly, (E) is correct and (C) is incorrect.

A registered practitioner properly recorded an assignment document for application A identifying XYZ Company as the assignee. The document assigns to XYZ Company the "subject matter claimed in Application A." A proper restriction requirement was made by a primary examiner in application A between two distinct inventions, and the practitioner elected to prosecute one of the inventions. Application A was prosecuted, and later became abandoned. Before the abandonment date of application A, the practitioner filed a complete application B as a proper divisional application of application A. Application B claimed the nonelected invention of Application A, and was published as a U.S. application publication. XYZ Company remains the assignee of application A. What must the practitioner do in accordance with the USPTO rules and the procedures set forth in the MPEP to ensure that XYZ Company is listed as the assignee on the face of any patent issuing from application B? A File a proper assignment document in application B identifying XYZ Company as the assignee. B File a proper assignment document in application B identifying XYZ Company as the assignee, and confirm that USPTO's bibliographic data for application B identifies XYZ Company as the assignee by checking the filing receipt for application B, the U.S. application publication of application B, or the USPTO's Patent Application Information Retrieval (PAIR) system data for application B, depending on when the practitioner filed the assignment document in application B. C Confirm that XYZ Company is identified as the assignee on the U.S. application publication of application B. D File a proper assignment document in application B identifying XYZ Company as the assignee, and confirm that XYZ Company is identified as the assignee on the U.S. application publication of application B. E Upon allowance of application B, the practitioner must identify XYZ Company as the assignee in the appropriate space on the Issue Fee Transmittal form for specifying the assignee for application B.

E Related MPEP Chapter(s): MPEP 300 - Ownership and Assignment Answer Reasoning:ANSWER: (E) is the most correct answer. See MPEP §§ 306 and 307. MPEP § 306 states, "In the case of a division or continuation application, a prior assignment recorded against the original application is applied to the division or continuation application because the assignment recorded against the original application gives the assignee rights to the subject matter common to both applications." MPEP § 307 states, "Irrespective of whether the assignee participates in the prosecution of the application, the patent issues to the assignee if so indicated on the Issue Fee Transmittal form PTOL-85B. Unless an assignee's name and address are identified in item 3 of the Issue Fee Transmittal form PTOL-85B, the patent will issue to the applicant. Assignment data printed on the patent will be based solely on the information so supplied." A new assignment document need not be recorded for a divisional or continuation application where the assignment recorded in the parent application remains the same. Accordingly, answers (A), (B) and (D) are incorrect. In addition, (A), (B) and (D) are incorrect because unless an assignee's name and address are identified in item 3 of PTOL-85B, the patent will issued to the application and the assignee information, even if recorded, will not appear on the patent. (C) is incorrect for the same reason. (B) is also incorrect. There is no connection between the filing receipt, PAIR or the patent application publication and the recorded assignment. Assignment data is reflected on the filing receipt, PAIR, or a patent application publication when applicant includes assignment information for purposes of publication of the application on the transmittal letter. Assignment data printed on the patent will be based solely on the information supplied on the Issue Fee Transmittal Form PTOL-85B. See MPEP §§ 1309 and 1481. Accordingly, answer (E) is correct and answer (C) is incorrect.

In accordance with the USPTO rules and the procedures set forth in the MPEP, which of the following documents, if any, must also contain a separate verification statement? A Small entity statements. B A petition to make an application special. C A claim for foreign priority. D An English translation of a non-English language document. E None of the above.

E Related MPEP Chapter(s): MPEP 400 - Representative of Inventor or Owner Answer Reasoning:ANSWER: (E) is the most correct answer. MPEP § 410 makes clear that the certification requirement set forth in 37 CFR § 10.18(b) "has permitted the PTO to eliminate the separate verification requirement previously contained in 37 CFR ...1.27 [small entity statements], ...1.52 [English translations of non-English documents], ...1.55 [claim for foreign priority], [and] ...1.102 [petition to make an application special]."

Determine which of the following documents, if any, must also contain a separate verification statement in accordance with the patent laws, rules and procedures as related in the MPEP. A A request to correct inventorship in a pending application. B A petition to make an application special. C A claim for foreign priority. D A substitute specification. E None of the above.

E Related MPEP Chapter(s): MPEP 400 - Representative of Inventor or Owner Answer Reasoning:ANSWER: (E) is the most correct answer. MPEP § 410 states that the certification requirement set forth in 37 CFR § 10.18(b) "has permitted the USPTO to eliminate the separate verification requirement previously contained in 37 CFR ...1.48 [correction of inventorship in a patent application], ...1.55 [claim for foreign priority], ...1.102 [petition to make an application special], [and] ... 1.125 [substitute specification]."

In accordance with the MPEP and USPTO rules and procedure, an application for patent may be made on behalf of a joint inventor in certain situations. Who may make application on behalf of a joint inventor who has refused to sign the application ("nonsigning inventor"), if the other joint inventor ("signing inventor") executes the application? A A person other than the signing inventor, to whom the nonsigning inventor has assigned the invention. B A person other than the signing inventor, with whom the nonsigning inventor has agreed in writing to assign the invention. C The signing inventor. D A person other than the signing inventor, who shows a strong proprietary interest in the invention. E All of the above.

E Related MPEP Chapter(s): MPEP 400 - Representative of Inventor or Owner Answer Reasoning:ANSWER: Choice (E) is now the correct answer. Under prior practice, the available practice here was limited. But under the AIA (Rules 43, 45 and 46), any of those listed can sign if the true inventor refuses to sign.

Which of the following is not in accordance with provisions of the MPEP? A In return for a patent, the inventor gives a complete disclosure of the invention for which protection is sought. B Amendments filed after the filing date that lack descriptive basis in the original disclosure involve new matter. C If during the course of examination of a patent application, an examiner notes the use of language that could be deemed offensive to any race, religion, sex, ethnic group, or nationality, he or she should object to the use of the language as failing to comply with the Rules of Practice. D The examiner should object to application drawings that include depictions or caricatures that might reasonably be considered offensive to any race, religion, sex, ethnic group or nationality. E Where an amendment is filed with a patent application that has no signed oath or declaration, a subsequently filed oath or declaration must refer to both the application and amendment, but in any case the amendment will not be considered as part of the original disclosure and will be treated as new matter.

E Related MPEP Chapter(s): MPEP 500 - Receipt and Handling of Mail and Papers Answer Reasoning:ANSWER: (E) is incorrect as a preliminary amendment may be filed with the original disclosure and will be treated as part of the original disclosure in accordance with MPEP § 608.04(b) (A) through (D) are all correct. See MPEP § 608 at p. 600-53.

In accordance with the MPEP and USPTO rules and procedure, correspondence transmitted to the USPTO by facsimile is not permitted in certain situations. Which of the following facsimile transmissions to the USPTO will be accorded a date of receipt by the USPTO? A Facsimile transmission of a request for reexamination under 37 CFR 1.510 or 1.913. B Facsimile transmission of drawings submitted under 37 CFR 1.81, 1.83 through 1.85, 1.152, 1.165, 1.174, or 1.437. C Facsimile transmission of a response to a Notice of Incomplete Nonprovisional Application for the purpose of obtaining an application filing date. D Facsimile transmission of a correspondence to be filed in a patent application subject to a secrecy order under 37 CFR 5.1 through 5.5 and directly related to the secrecy order content of the application. E Facsimile transmission of a continued prosecution application under 37 CFR 1.53(d) and an authorization to charge the basic filing fee to a deposit account.

E Related MPEP Chapter(s): MPEP 500 - Receipt and Handling of Mail and Papers Answer Reasoning:ANSWER: Choice (E) is the correct answer. MPEP § 502.01, and 37 C.F.R. § 1.6(d)(3). MPEP § 502.01 reads, "The date of receipt accorded to any correspondence permitted to be sent by facsimile transmission, including a continued prosecution application (CPA) filed under 37 C.F.R. § 1.53(d), is the date the complete transmission is received by an Office facsimile unit...An applicant filing a CPA by facsimile transmission must include an authorization to charge the basic filing fee to a deposit account or to a credit card." Choice (A) is incorrect because 37 C.F.R. § 1.6(d) states, "Facsimile transmissions are not permitted and, if submitted, will not be accorded a date of receipt in the following situations: ...(5) A request for reexamination under §1.510 or § 1.913." Choice (B) is incorrect because 37 C.F.R. § 1.6(d) also states, "Facsimile transmissions are not permitted and, if submitted, will not be accorded a date of receipt in the following situations: ...(4) Drawings submitted under §§ 1.81, 1.83 through 1.85, 1.152, 1.165, 1.174, 1.437, 2.51, 2.52, or 2.72." Choice (C) is incorrect because 37 C.F.R. § 1.6(d) also states, "Facsimile transmissions are not permitted and, if submitted, will not be accorded a date of receipt in the following situations: ...(3) Correspondence which cannot receive the benefit of the certificate of mailing or transmission as specified in 1.8(a)(2)(i)(A)..." 37 C.F.R. § 1.8(a)(2)(i)(A) reads, "The filing of a national patent application specification and drawing or other correspondence for the purpose of obtaining an application filing date..." Choice (D) is incorrect because 37 C.F.R. § 1.6(d) also states, "Facsimile transmissions are not permitted and, if submitted, will not be accorded a date of receipt in the following situations: ...(6) Correspondence to be filed in a patent application subject to a secrecy order under §§ 5.1 through 5.5 of this chapter and directly related to the secrecy order content of the application."

In accordance with the patent laws, rules and procedures as related in the MPEP, which of the following can a third party submit in a pending published application within six months from the publication date where the submission identifies the application to which it is directed by application number and includes the appropriate fee? A A list referencing a videotape and copy of the tape showing that the process claimed in the application was in use more than one year before the filing date of the application. B A U.S. patent issued more than one year before the filing date of the application and a written explanation of the patent made by the third party on the patent. C A publication with a publication date more than one year before the filing date of the application and including underlining made by the third party on the publication. D A protest raising fraud and inequitable conduct issues. E A list of the sole Japanese language publication submitted for consideration, including the publication date of the publication, a copy of the Japanese language publication and a written English language translation of the pertinent parts of the publication.

E Related MPEP Chapter(s): MPEP 600 - Parts, Form, and Content of Application Answer Reasoning: ANSWER: (E) is the most correct answer. 37 CFR § 1.290: MPEP 1100. The Rule allows that a third party may submit in an application a foreign-language publication and an English language translation of pertinent portions of the publication. The submission must "identify the application to which it is directed by application number," . . "include the fee (if more than 3 references are submitted) include "a list of the patents or publications submitted for consideration by the Office, including the date of publication of each patent or publication;". . . a " copy of each listed patent or publication in written form or at least the pertinent portions;" and an "English language translation of all the necessary and pertinent parts of any non-English language patent or publication in written form relied upon." Post 9/16/12, third party submissions are much broader than under old Rule 99. (A) is incorrect. The rule does not authorize a third-party submission of materials or things other than patents or publications. (B) is incorrect. A third-party submission may not include explanations beyond a concise explanation.. (C) is incorrect. A third-party submission may not include markings or highlights on the publications. (D) is incorrect because a protest cannot be filed in a published application. See 37 CFR § 1.291(a)(1); MPEP 1901.06

Assuming that each of the following claims is in a separate application, and there is no preceding multiple dependent claim in any of the applications, which claim is in acceptable multiple dependent claim form? A Claim 8. A machine according to any one of the preceding claims wherein... B Claim 5. A device as in one of claims 1-4, wherein... C Claim 10. A device as in any of claims 1-4 or 6-9, in which... D Claim 4. A machine according to claim 2 or 3, also comprising... E The claim form in (A), (B), (C) and (D) is acceptable.

E Related MPEP Chapter(s): MPEP 600 - Parts, Form, and Content of Application Answer Reasoning:ANSWER: (E) is correct because 35 U.S.C. § 112 authorizes multiple dependent claims as long as they are in the alternative form. MPEP § 608.01(n), subsection I A.

Which of the following is not in accordance with the provisions of 35 USC 115 (Oath of applicant)? A The applicant shall make oath (or declaration) that he believes himself to be the original and first inventor of the process, machine, manufacture, or composition of matter, or improvement thereof, for which he solicits a patent. B In the oath or declaration, the applicant must state that the application is filed with their consent. C An oath may be made before any person within the United States authorized by law to administer oaths. D An oath executed in a foreign country must be properly authenticated. E A declaration which accompanies a patent application must state on the document a warning that willful false statements and the like are punishable by fine or imprisonment or both under 18 USC 1001, and the declaration must be notarized.

E Related MPEP Chapter(s): MPEP 600 - Parts, Form, and Content of Application Answer Reasoning:ANSWER: (E) is the best answer. The declaration need not be notarized. See 37 C.F.R. § 1.63(a). A declaration may be used in lieu of an oath. 37 C.F.R. § 1.68, MPEP 602. As to (A) through (C) see 35 U.S.C.§ 115. As to (D), see MPEP § 602.04 at p. 600-34, left column.

Which, if any, of the following statements is in accord with the patent laws, rules and procedures as related in the MPEP? A Where an inventor's residence is stated correctly in the 37 CFR 1.76 application data sheet and incorrectly in the inventor's 37 CFR 1.63 oath or declaration, the discrepancy must be corrected by filing a supplemental 37 CFR 1.67 oath or declaration giving the correct residence. B Where two inventors file separate 37 CFR 1.63 oaths or declarations which do not identify both inventors, the USPTO will presume they are joint inventors and will not require new oaths or declarations. C A dependent claim which repeats a limitation that appears in the claim on which it depends and adds an additional limitation is properly rejected under 35 USC 112(d). D In a statement under 37 CFR 1.97(e)(1) specifying that "each item of information contained in the information disclosure statement was first cited in any communication from a foreign patent office in a counterpart foreign application not more than three months prior to the filing of the statement," the three-month period begins on the date the communication was first received by either a foreign associate or a U.S. registered practitioner. E None of statements (A) to (D) is correct.

E Related MPEP Chapter(s): MPEP 600 - Parts, Form, and Content of Application Answer Reasoning:ANSWER: (E) is the correct answer. (A) is not in accord the patent laws, rules and procedures as related in the MPEP. Regarding (A), MPEP § 601.05, states ("[i]f an application is filed with an application data sheet correctly setting forth the citizenship of inventor B, and an executed 37 CFR 1.63 declaration setting forth a different incorrect citizenship of inventor B, the Office will capture the citizenship of inventor B found in the application data sheet. Applicant, however, must submit a supplemental oath or declaration under 37 CFR 1.67 by inventor B setting forth the correct citizenship even though it appears correctly in the application data sheet. If, however, the error is one of residence, no change would be required (37 CFR 1.76(d)(2))." (B) is not in accord the patent laws, rules and procedures as related in the MPEP. As for (B), MPEP § 602, under the heading "Sole Or Joint Designation," states "[w]hen joint inventors execute separate oaths or declarations, each oath or declaration should make reference to the fact that the affiant is a joint inventor together with each of the other inventors indicating them by name." The examiner should notify the inventors that their oaths or declarations are defective and that new oaths or declarations are required. (C) is incorrect. MPEP § 608.01(n), under the heading "II. Treatment Of Improper Dependent Claims," states "[c]laims which are in improper dependent form for failing to further limit the subject matter of a previous claim should be objected to under 37 CFR 1.75(c) by using form paragraph 7.36." The claim should not be rejected. Further as for (C), MPEP § 608.01(n), under heading "III. Infringement Test," states "[a] dependent claim does not lack compliance with 35 U.S.C. 112, fourth paragraph, simply because there is a question as to (1) the significance of the further limitation added by the dependent claim, or (2) whether the further limitation in fact changes the scope of the dependent claim from that of the claim from which it depends. The test for a proper dependent claim under the fourth paragraph of 35 U.S.C. 112 is whether the dependent claim includes every limitation of the claim from which it depends. The test is not one of whether the claims differ in scope." (D) is not in accord the patent laws, rules and procedures as related in the MPEP. As for (D), MPEP § 609, under heading "(B)(5) Statement Under 37 CFR 1.97(e)," states "[t]he date on the communication by the foreign patent office begins the 3-month period in the same manner as the mailing of an Office action starts a 3-month shortened statutory period for reply . ... The date which begins the 3-month period is not the date the communication was received by a foreign associate or the date it was received by a U.S. registered practitioner."

patent specification discloses a personal computer comprising a microprocessor and a random access memory. There is no disclosure in the specification of the minimum amount of storage for the random access memory. In the disclosed preferred embodiment, the microprocessor has a clock speed of 100-200 megahertz. Claims 9 and 10, presented below, are original claims in the application. Claim 11, presented below, was added by amendment after an Office action. 9. A personal computer comprising a microprocessor and a random access memory that includes at least 1 gigabyte of storage. 10. The personal computer of Claim 9, wherein the microprocessor has a clock speed of 100-200 megahertz. 11. The personal computer of Claim 10, wherein the random access memory is greater than 1/2 gigabyte of storage. Which of the following statements is or are in accord with the patent laws, rules and procedures as related in the MPEP regarding the respective claims under the fourth paragraph of 35 USC 112? A Claim 9 is a proper independent claim, and Claims 10 and 11 are proper dependent claims. B Claim 9 is a proper independent claim, and Claims 10 and 11 are improper dependent claims. C Claim 9 is an improper independent claim, and Claims 10 and 11 are improper dependent claims. D Claim 9 is an improper independent claim, and Claims 10 and 11 are proper dependent claims. E Claim 9 is a proper independent claim, Claim 10 is a proper dependent claim, and Claim 11 is an improper dependent claim.

E Related MPEP Chapter(s): MPEP 600 - Parts, Form, and Content of Application Answer Reasoning:ANSWER: (E) is the most correct answer. 37 CFR § 1.75(c); MPEP § 608.01(n), under the heading "II. Treatment Of Improper Dependent Claims," 37 CFR § 1.75(c) provides "One or more claims may be presented in dependent form, referring back to and further limiting another claim or claims in the same application." Claim 9, though broad, is supported by the specification. The minimum memory recited in the claim as original disclosure, is self-supporting. 35 U.S.C. § 112, first paragraph. See MPEP §§ 608.01(l) and 2163. Claim 10 is a proper dependent claim because it depends from and further restricts the scope of a preceding claim. 37 CFR § 1.75(c). Claim 11 is an improper dependent claim because it is inconsistent with and does not further limit the scope of claim 10. Claim 10, depending on Claim 9, has a 1 gigabyte memory minimum, whereas Claim 11 redefines the minimum memory by setting a lower minimum of 1/2 gigabyte.

Which of the following is not in accordance with the recommended form for an abstract of the disclosure as provided for in the MPEP? A A patent abstract is a concise statement of the technical disclosure of the patent and should include that which is new in the art to which the invention pertains. B If the patent is of a basic nature, the entire technical disclosure may be new in the art, and the abstract should be directed to the entire disclosure. C If the patent is in the nature of an improvement in an old apparatus, process, product, or composition, the abstract should include the technical disclosure of the improvement. D In certain patents, particularly those for compounds and compositions, wherein the process for making and/or the use thereof are not obvious, the abstract should set forth a process for making and/or a use thereof. E The abstract should compare the invention with the prior art.

E Related MPEP Chapter(s): MPEP 600 - Parts, Form, and Content of Application Answer Reasoning:ANSWER: (E). The abstract should not compare the invention with the prior art. MPEP § 608.01(b). As to (A) through (C), see MPEP § 608.01(b). As to (D), when the process for making is not obvious, the process should be set forth in the abstract. See MPEP § 608.01(b).

Which of the following is not in accordance with the recommended format for a claim set forth in the provisions of the MPEP? A Where a claim sets forth a plurality of elements or steps, each element or step of the claim should be separated by a line indentation. B A claim may include plural indentations to further segregate subcombinations or related steps. C The claim or claims must commence on a separate sheet after the detailed description of the invention. D Each claim should end with a period. E A claim should always begin with "A", "An" or "In."

E Related MPEP Chapter(s): MPEP 600 - Parts, Form, and Content of Application Answer Reasoning:ANSWER: (E). There is no such requirement. As to (C), see 37 C.F.R. § 1.52(b). As to (A) through (D) see MPEP § 608.01(m). 37 C.F.R. § 1.75(i).

Assume that each claim 5 is in a different patent application. Recommend which, if any, of the following wording is in accord with the patent laws, rules and procedures as related in the MPEP for a multiple dependent claim. A Claim 5. A gadget according to claims 1-3, in which ... B Claim 5. A gadget as in claims 1, 2, 3, and/or 4, in which ... C Claim 5. A gadget as in claim 1 or 2, made by the process of claim 3 or 4, in which ... D Claim 5. A gadget as in either claim 6 or claim 8, in which ... E None of the above are proper multiple dependent claims.

E Related MPEP Chapter(s): MPEP 600 - Parts, Form, and Content of Application Answer Reasoning:ANSWER: The correct answer is (E). MPEP § 608.01(n), under the heading "B. Unacceptable Multiple Dependent Claim Wording." Multiple dependent claims in proper form depend on preceding claims and refer to the claims from which they depend in the alternative only. Answer (A) is incorrect. See MPEP § 608.01(n), under the heading "B. Unacceptable Multiple Dependent Claim Wording," and subheading "1. Claim Does Not Refer Back In the Alternative Only," second example. A proper multiple dependent claim must refer back in the alternative only. Answer (B) is incorrect. See MPEP § 608.01(n), under the heading "B. Unacceptable Multiple Dependent Claim Wording," and subheading "1. Claim Does Not Refer Back In the Alternative Only," fifth example. A proper multiple dependent claim refers back in the alternative only. Answer (C) is incorrect. Answer (C) reproduces the example in MPEP § 608.01(n), under the heading "B. Unacceptable Multiple Dependent Claim Wording," and subheading "3. References to Two Sets of Claims to Different Features." A proper multiple dependent claim refers in the alternative to only one set of claims. Answer (D) is incorrect. See MPEP § 608.01(n), under the heading "B. Unacceptable Multiple Dependent Claim Wording," and subheading "2. Claim Does Not Refer to a Preceding Claim," second example. A proper multiple dependent claim depends only from preceding claims.

Applicant filed an application containing a claim directed to a polishing wheel coated with diamond grit particles. The application discloses, but does not claim, a diamond grit particle size of 5-7 microns. The examiner rejected the claim under 35 USC 102 as being anticipated by a U.S. patent which disclosed as its invention a polishing wheel in accordance with the claim of the application but coated with glass grit particles instead of diamond grit particles. The applied patent, which issued more than 1 year prior to the effective filing date of the application, also disclosed that diamond grit particles were known for coating on polishing wheels but were inferior to glass grit particles because they were more expensive and did not adhere as well to the polishing wheel. The applied patent disclosed a grit particle size of 50-100 microns. Which of the following timely taken courses of action would comply with the patent laws, rules and procedures as related in the MPEP for overcoming the rejection? A Argue that the patent teaches away from the use of a diamond grit particle coating on a polishing wheel and thus does not teach the claimed invention. B File a declaration under 37 CFR 1.132 showing unexpected results using diamond grit rather than glass grit. C Antedate the applied patent by filing a declaration under 37 CFR 1.131 showing that applicant invented the claimed subject matter prior to the effective date of the applied patent. D Argue the applied patent is nonanalogous art. E Amend the claim by adding a limitation that the diamond grit particle size is 5-7 microns, and arguing that the claimed invention differs from applied patent by limited the diamond grit particle size to 5-7 microns.

E Related MPEP Chapter(s): MPEP 700 - Examination of Applications Answer Reasoning:ANSWER: (E) is the correct answer. 35 U.S.C. § 102(b); 37 CFR § 1.111(b); MPEP §§ 706.02(b), 2131 and 2131.03. As stated in MPEP 2131, under the heading "To Anticipate A Claim, The Reference Must Teach Every Element Of The Claim," "A claim is anticipated only if each and every element as set forth in the claim is found, either expressly or inherently described, in a single prior art reference." Verdegaal Bros. v. Union Oil Co. of California, 814 F.2d 628, 631, 2 USPQ2d 1051, 1053 (Fed. Cir. 1987). MPEP 2131.03, under the heading, "Prior Art Which Teaches A Range Within, Overlapping, Or Touching The Claimed Range Anticipates If The Prior Art Range Discloses The Claimed Range With 'Sufficient Specificity.'" states "When the prior art discloses a range which touches, overlaps or is within the claimed range, but no specific examples falling within the claimed range are disclosed, a case by case determination must be made as to anticipation. In order to anticipate the claims, the claimed subject matter must be disclosed in the reference with 'sufficient specificity to constitute an anticipation under the statute.'" A claim containing a limitation that the grit particle size is 5-7 microns would not be anticipated by the applied reference, because the applied reference discloses a different grit particle size well outside that range. (A) is incorrect. MPEP § 2123(8th Ed.). Patents are relevant as prior art for all they contain and are not limited to their preferred embodiments. See In re Heck, 699 F.2d 1331, 1332-33, 216 USPQ 1038, 1039 (Fed. Cir. 1983) and Merck & Co. v. Biocraft Laboratories, 874 F.2d 804, 10 USPQ2d 1843 (Fed. Cir. 1989), cert. denied, 493 U.S. 975 (1989). (B) is incorrect. See MPEP § 2131.04. Evidence of secondary considerations such as unexpected results is irrelevant to 35 U.S.C. § 102 rejections and thus cannot overcome a rejection so based. See In re Wiggins, 488 F.2d 538, 543,179 USPQ 421, 425 (CCPA 1973). (C) is incorrect. See MPEP § 715, under the heading "Situations Where 37 CFR 1.131 Affidavits or Declarations Are Inappropriate." An affidavit or declaration under 37 CFR § 1.131 is inappropriate where the reference publication date is more than 1 year prior to applicant's effective filing date. Such a reference is a "statutory bar" under 35 U.S.C. § 102(b) as referenced in 37 CFR § 1.131(a)(2). (D) is also incorrect. See MPEP § 2131.05. Arguments that the alleged anticipatory prior art is "nonanalogous art" are not "germane" to a rejection under 35 U.S.C. § 102. Twin Disc, Inc. v. United States, 231 USPQ 417, 424 (Cl. Ct. 1986) (quoting In re Self, 671 F.2d 1344, 213 USPQ 1, 7 (CCPA 1982).

For purposes of determining whether a request for continued examination is in accordance with proper USPTO rules and procedure, in which of the following situations will prosecution be considered closed? A The last Office action is a final rejection. B The last Office action is an Office action under Ex Parte Quayle. C A notice of allowance has issued following a reply to a first Office action. D The application is under appeal. E All of the above.

E Related MPEP Chapter(s): MPEP 700 - Examination of Applications Answer Reasoning:ANSWER: (E) is the correct answer. 37 C.F.R. § 1.114 (effective August 16, 2000); "Request for Continued Examination Practice and Changes to Provisional Application Practice; Final Rule," MPEP 700. (A) is a final action (§ 1.113). "...an action that otherwise closes prosecution in the application (e.g., an Office action under Ex Parte Quayle, 1935 Comm'r Dec. 11 (1935))." Thus (A), (B), (C) and (D) are individually correct, and (E), being the most inclusive, is the most correct answer.

According to USPTO rules and procedure, which of the following can be overcome by an affidavit under 37 CFR 1.131 in a pre-AIA filing? A A rejection properly based on statutory double patenting. B A rejection properly made under 35 USC 102(d) based on a foreign patent granted in a non-WTO country. C A rejection properly made under 35 USC 102(a) based on a journal article dated one month prior to the effective filing date of the U.S. patent application. Applicant has clearly admitted on the record during the prosecution of the application that subject matter in the journal article relied on by the examiner is prior art. D A rejection properly made under 35 USC 102(b) based on a U.S. patent that issued 18 months before the effective filing date of the application. The patent discloses, but does not claim, the invention. E None of the above.

E Related MPEP Chapter(s): MPEP 700 - Examination of Applications Answer Reasoning:ANSWER: (E) is the correct answer. MPEP § 715. (A) is incorrect because an affidavit under 37 C.F.R. § 1.131 is not appropriate where the reference is a prior U.S. patent to the same entity, claiming the same invention. MPEP § 715. (B) and (D) are each incorrect because an affidavit under 37 C.F.R. § 1.131 is not appropriate where the reference is a statutory bar under 35 U.S.C. § 102(d) as in (B) or a statutory bar under 35 U.S.C. § 102(b) as in (D). MPEP § 715. (C) is incorrect because an affidavit under 37 C.F.R. § 1.131 is not appropriate where applicant has clearly admitted on the record that subject matter relied on in the reference is prior art. MPEP § 715.

In accordance with the USPTO rules and the procedures set forth in the MPEP, which of the following is true? A In rejecting claims the examiner may rely upon facts within his own personal knowledge, unless the examiner qualifies as an expert within the art, in which case he is precluded from doing so, since only evidence of one of ordinary skill in the art is permitted. B If an applicant desires to claim subject matter in a reissue which was the same subject matter waived in the statutory invention registration of another, the applicant is precluded by the waiver from doing so, even though the applicant was not named in the statutory invention registration. C If an applicant, knowing that the subject matter claimed in his patent application was on sale in Michigan and sales activity is a statutory bar under 35 USC 102 to the claims in his application, nevertheless withholds the information from the patent examiner examining the application, and obtains a patent including the claims in question, the applicant may remove any issue of inequitable conduct by filing a request for reexamination based on the sales activity. D An applicant for a patent may overcome a statutory bar under 35 USC 102 based on a patent claiming the same invention by acquiring the rights to the patent pursuant to an assignment and then asserting the assignee's right to determine priority of invention pursuant to 37 CFR 1.602. E None of the above.

E Related MPEP Chapter(s): MPEP 700 - Examination of Applications Answer Reasoning:ANSWER: (E) is the most correct answer. (A) is incorrect since facts within the knowledge of the examiner may be used whether or not the examiner qualifies as an expert. 37 CFR § 1.104(c)(3). (B) is incorrect since the waiver is only effective against those named in the statutory registration. (C) is incorrect since on sale activities is not proper subject matter for reexamination, and inequitable conduct cannot be resolved or absolved by reexamination. (D) is not correct since a statutory bar cannot be overcome by acquiring the patent.

In 1995 Patent Agent filed a U.S. patent application containing five claims (Application 1). All five claims are fully supported under 35 USC 112 by the disclosure of Application 1. In 2000, Patent Agent filed a U.S. patent application (Application 2) that was a continuation-in-part of Application 1. Application 2 adds new subject matter to the disclosure of Application 1, and ten additional claims. Of the fifteen claims in Application 2, claims 1-5 are exactly the same as Application 1, claims 6-10 are fully supported under 35 USC 112 by the disclosure of Application 1, and claims 11-15 are fully supported under 35 USC 112 only by the newly added subject matter of Application 2. The effective filing date for claims in Application 2 is: A 1-15 is 2000. B 1-15 is 1995. C 1-10 is 1995. D 11-15 is 2000. E (C) and (D).

E Related MPEP Chapter(s): MPEP 700 - Examination of Applications Answer Reasoning:ANSWER: (E) is the most correct answer. MPEP § 706.02 page 700-20 (8th ed.), under the heading "DETERMINING THE EFFECTIVE FILING DATE OF THE APPLICATION" states "[t]he effective filing date of a U.S. application may be determined as follows: ... (B) If the application is a continuation-in-part of an earlier U.S. application, any claims in the new application not supported by the specification and claims of the parent application have an effective filing date equal to the filing date of the new application. Any claims which are fully supported under 35 U.S.C. § 112 by the earlier parent application have the effective filing date of that earlier parent application." Accordingly, the effective filing date of claims 1-10 is 1995 and the effective filing date of claims 11-15 is 2000. Answers (A) and (B) are incorrect because they do not account for the two different effective filing dates. Answers (C) and (D) are both correct, therefore answer (E) which includes both (C) and (D) is the most correct answer.

Which of the following practices or procedures may be employed in accordance with the USPTO rules and the procedures set forth in the MPEP to overcome a rejection properly based on 35 USC 102(a)(2)? A Persuasively arguing that the claims are patentably distinguishable from the prior art. B Filing an affidavit or declaration under 37 CFR 1.130 showing that the reference invention is not by "another." C Filing an affidavit or declaration under 37 CFR 1.130 showing disclosure within one year of the filing date ot the application that predates the filing date of the reference. D (A) and (C). E (A), (B) and (C).

E Related MPEP Chapter(s): MPEP 700 - Examination of Applications Answer Reasoning:ANSWER: (E) is the most correct answer. See MPEP § 706.02(b), under the heading "Overcoming A 35 U.S.C. § 102 Rejection Based On A Printed Publication Or Patent." (A), (B), and (C) alone, as well as (D) are not correct because they are not the most inclusive.

A utility application filed in May 2001 has been prosecuted through a second action final rejection. In the final rejection some claims were allowed and other claims were finally rejected. Which of the following accords with the patent laws, rules and the procedures as related in the MPEP for a proper reply to a second action final rejection in the utility application? A All of the answers below. B Only a Notice of Appeal. C The appropriate fee for a request for continued examination (RCE). D A continued prosecution application (CPA) under 37 CFR 1.53(d). E An amendment canceling all rejected claims and complying with 37 CFR 1.116.

E Related MPEP Chapter(s): MPEP 700 - Examination of Applications Answer Reasoning:ANSWER: (E) is the most correct answer. The filing of an amendment complying with 37 CFR § 1.116 is a proper reply under 37 CFR § 1.113 to a final rejection. See MPEP § 714.13, under the heading "Entry Not A Matter of Right," which states, in pertinent part , "A reply under 37 CFR 1.113 is limited to: (A) an amendment complying with 37 CFR 1.116." (B) is not the most correct answer because the Notice of Appeal must be accompanied by the appeal fee required by 37 CFR § 1.17(b). (C) is not the most correct answer because the RCE must be accompanied by a submission (i.e., an amendment that meets the reply requirement of 37 CFR § 1.111). (D) is not the correct answer because CPA practice does not apply to utility or plant applications if the prior application has a filing date on or after May 29, 2000. See MPEP § 706.07(h), paragraphs I and IV. (A) is not the correct answer since (E) is a proper reply.

While traveling through Germany (a WTO member country) in December 1999, Thomas (a Canadian citizen) conceived of binoculars for use in bird watching. The binoculars included a pattern recognition device that recognized birds and would display pertinent information on a display. Upon Thomas' return to Canada (a NAFTA country) in January 2000, he enlisted his brothers Joseph and Roland to help him market the product under the tradename "Birdoculars." On February 1, 2000, without Thomas' knowledge or permission, Joseph anonymously published a promotional article written by Thomas and fully disclosing how the Birdoculars were made and used. The promotional article was published in the Saskatoon Times, a regional Canadian magazine that is also widely distributed in the United States. Thomas first reduced the Birdoculars to practice on March 17, 2000 in Canada. A United States patent application properly naming Thomas as the sole inventor was filed September 17, 2000. That application has now been rejected as being anticipated by the Saskatoon Times article. Which of the following statements is most correct? A Thomas can rely on his activities in Canada in establishing a date of invention prior to publication of the Saskatoon Times article. B In a priority contest against another inventor, Thomas can rely on his activities in Canada in establishing a date of invention. C In a priority contest against another inventor, Thomas can rely on his activities in Germany in establishing a date of invention. D Statements (A) and (B) are correct, but statement (C) is incorrect. E Statements (A), (B), and (C) are each correct.

E Related MPEP Chapter(s): MPEP 700 - Examination of Applications Answer Reasoning:ANSWER: (E) is the most correct answer. Thomas may rely on activities in both Germany (a WTO member country) and Canada (a NAFTA country) in establishing a date of invention prior to publication of the Saskatoon Times article or in establishing priority. 35 U.S.C. § 104; see also MPEP 715.01(c).

Which of the following practices or procedures may be properly employed to overcome a rejection properly based on pre-AIA 35 USC 102(e)? A Persuasively arguing that the claims are patentably distinguishable from the prior art. B Filing an affidavit or declaration under 37 CFR 1.132 showing that the reference invention is not by "another." C Filing an affidavit or declaration under 37 CFR 1.131 showing prior invention, if the reference is not a U.S. patent that either claims the same invention or claims an obvious variation of the subject matter in the rejected claim(s). D (A) and (C). E (A), (B) and (C).

E Related MPEP Chapter(s): MPEP 700 - Examination of Applications Answer Reasoning:ANSWER: (E). See MPEP § 706.02(b) page 700-23 (8th ed.), under the heading "Overcoming a 35 U.S.C. § 102 Rejection Based on a Printed Publication or Patent." (A), (B), and (C) alone, as well as (D) are not correct because they are not the most inclusive.

An application naming X and Y as joint inventors, filed on April 3, 2012, has a single pending claim, and does not claim the benefit of any earlier application. Which, if any, of the following items of prior art that have been relied on in various rejections of the claim may be overcome by a suitable affidavit under 37 CFR 1.131 in accordance with the patent laws, rules and procedures as related in the MPEP? A A U.S. patent to G that issued on March 27, 2011, has an effective U.S. filing date of January 4, 2010, and does not claim the "same patentable invention" (as defined in 37 CFR 1.601(n)) as the rejected claim. B A U.S. patent to P that issued on June 5, 2011, has an effective U.S. filing date of February 1, 2010, and includes a claim that is identical to the rejected claim. C A journal article to H published on December 10, 2011, and characterized in the application as "describ[ing] the prior art." D A foreign patent issued to X and Y on November 7, 2011, which claims the same subject matter as the rejected claim and is based on an application filed on January 3, 2011. E None of the above.

E Related MPEP Chapter(s): MPEP 700 - Examination of Applications Answer Reasoning:ANSWER: The correct answer is (E), "None of the above." (A) is incorrect because the G patent is a 35 U.S.C. § 102(b) statutory bar. See MPEP § 715 under heading "Situations Where 37 CFR 1.131 Affidavits Or Declarations Are Inappropriate," states "(A) Where the reference publication date is more than 1 year prior to applicant's or patent owner's effective filing date. Such a reference is a 'statutory bar' under 35 U.S.C. 102(b) as referenced in 37 CFR 1.131(a)(2)." (B) is incorrect because 37 CFR § 1.131 expressly provides that prior invention may not be established under the rule "if the rejection is based upon a U.S. patent or U.S. patent application publication of a pending or patented application to another or others which claims the same patentable invention as defined in 37 CFR 1.601(n)." 37 CFR § 1.131(a)(1). Regarding (C), see MPEP § 715, under heading " Situations Where 37 CFR 1.131 Affidavits Or Declarations Are Inappropriate," which states "(G) Where applicant has clearly admitted on the record that subject matter relied on in the reference is prior art. In this case, that subject matter may be used as a basis for rejecting his or her claims and may not be overcome by an affidavit or declaration under 37 CFR 1.131. In re Hellsund, 474 F.2d 1307, 177 USPQ 170 (CCPA 1973); In re Garfinkel, 437 F.2d 1000, 168 USPQ 659 (CCPA 1971); In re Blout, 333 F.2d 928, 142 USPQ 173 (CCPA 1964); In re Lopresti, 333 F.2d 932, 142 USPQ 177 (CCPA 1964)." Regarding (D), see MPEP § 715, under heading, " Situations Where 37 CFR 1.131 Affidavits Or Declarations Are Inappropriate," which states "(C) Where the reference is a foreign patent for the same invention to applicant or patent owner or his or her legal representatives or assigns issued prior to the filing date of the domestic application or patent on an application filed more than 12 months prior to the filing date of the domestic application. See 35 U.S.C. 102(d)."

Inventor Jones files an application under 35 USC 111(a) on March 27, 2002. The application is a continuation of an international application, which was filed on December 1, 2000. The international application claims priority to a U.S. provisional application filed December 2, 1999. The international application designated the United States, and was published in English under PCT Article 21(2). All applications contained the exact same disclosure. In accordance with the patent laws, rules and procedures as related in the MPEP, what, if any, is the earliest prior art date under 35 USC 102(e) for the publication of the 35 U.S.C. 111(a) application under 35 USC 122(b)? A None, the publication has no prior art date under 35 U.S.C. 102(e) B 27-Mar-02 C 11-Dec-01 D 1-Dec-00 E 2-Dec-99

E Related MPEP Chapter(s): MPEP 700 - Examination of Applications Answer Reasoning:ANSWER: The correct answer is answer (E). See MPEP § 706.02(f)(1) in general and Example 7 in particular. (A) is not correct as the publication under 35 U.S.C. § 122(b) does have a prior art date under 35 U.S.C. 102(e). (B) and (D) are not correct because March 27, 2002 and December 1, 2000 are not the earliest prior art date under 35 U.S.C. § 102(e). The publication has an earlier prior art date than March 27, 2002 and December 1, 2000 because of its benefit/priority claims to the international application and the provisional application. See MPEP § 706.02(f)(1). (C) is not correct as it is not a filing date for any application in this question.

Which of the following establishes a statutory bar under pre-AIA 35 USC 102 to patentability of Applicant's claimed invention? A To further develop the invention, Applicant's invention was tested and experimented with in the United States more than one year prior to applicant's effective U.S. filing date, but the invention at the time was not fit for its intended purpose and important modifications concerning the claimed features resulted from the experimentation. The first actual reduction to practice occurred after the effective U.S. filing date. B Applicant's invention was sold in a WTO member country outside the United States more than one year prior to applicant's effective U.S. filing date, and the sale was merely market testing of the invention to determine product acceptance. C Applicant's invention is rendered obvious by the combination of two U.S. patents, both of which were patented more than one year prior to applicant's effective filing date. D Applicant's invention was sold outside the United States in a non-WTO member country, more than one year prior to applicant's effective U.S. filing date, but the sale was merely an attempt at market penetration. E None of the above.

E Related MPEP Chapter(s): MPEP 700 - Examination of Applications MPEP 2100 - Patentability Answer Reasoning:ANSWER: (E) is the correct answer. (A) is incorrect because it is permitted experimental testing. MPEP §§ 2133.03(e)(3) and (6). (B) and (D) are each incorrect because the sales occurred outside of the United States. 35 U.S.C. § 102(b); MPEP §§ 706.02(c) and 2133.03(d). (C) is incorrect as it provides the basis for a rejection under 35 U.S.C. § 103, but not 35 U.S.C. § 102(b).

The MPEP and USPTO rules and procedure set out factual inquiries that are employed when making an obviousness-type double patenting analysis. Which of the following is not a factual inquiry that would be properly employed when making an obviousness-type double patenting determination with regard to a pending application vis-a-vis a claim in an issued patent? A Determine the level of ordinary skill in the pertinent art. B Determine the scope and content of a patent claim and the prior art relative to a claim in the application at issue. C Evaluate any objective indicia of nonobviousness of the claim of the application at issue. D Determine the differences between the scope and content of: the patent claim and the prior art determined in choice (B) above and the claim in the application at issue. E None of the above (that is, each of the above factual inquiries is properly employed when making an obviousness-type double patenting determination with regard to an issued patent).

E Related MPEP Chapter(s): MPEP 800 - Restriction in Applications Filed Under 35 U.S.C. 111; Double Patenting Answer Reasoning:ANSWER: Choice (E) is the correct answer. MPEP § 804, subpart (II)(B)(1), reads, "Since the analysis employed in an obviousness-type double patenting determination parallels the guidelines for a 35 U.S.C. § 103(a) rejection, the factual inquiries set forth in Graham v. John Deere Co., 383 U.S. 1, 138 USPQ 459 (1966), that are applied for establishing a background for determining obviousness under 35 U.S.C. § 103 are employed when making an obvious-type double patenting analysis." Each of choices (A), (B), (C), and (D) is incorrect because it is a factual inquiry set forth in Graham v. John Deere Co.

Claim 1 of an application recites "[a]n article comprising: (a) a copper substrate; and (b) an electrically insulating layer on said substrate." The specification defines the term "copper" as being elemental copper or copper alloys. In accordance with the patent laws, rules and procedures as related in the MPEP, for purposes of searching and examining the claim, the examiner should interpret the term "copper" in the claim as reading on: A Elemental copper only, based on the plain meaning of "copper." B Copper alloys only, based on the special definition in the specification. C Elemental copper and copper alloys, based on the special definition in the specification. D Any material that contains copper, including copper compounds. E None of the above.

MPEP 2100 - Patentability Answer Reasoning:ANSWER: (C) is the most correct answer. When the specification expressly provides a special definition for a term used in the claims, the term must be given that special meaning. See MPEP § 2111.01. (A) is incorrect because a term is given its plain meaning only when the specification does not provide a definition for the term. Id. (B) is incorrect because the specification defines the term as being inclusive of elemental copper. See MPEP § 2111.01. (D) is incorrect because it does not take into account the definition of copper found in the specification. See MPEP § 2111.01.

In accordance with the patent laws, rules and procedures as related in the MPEP, which of the following does not constitute prior art upon which a primary examiner could properly rely upon in making an obviousness rejection under 35 USC 103? A A U.S. patent in the applicant's field of endeavor which was issued two years before the filing date of applicant's patent application. B A non-patent printed publication in a field unrelated to the applicant's field of endeavor but relevant to the particular problem with which the inventor-applicant was concerned, which was published the day after the filing date of applicant's application. C A printed publication published more than 1 year before the filing date of applicant's patent application, which publication comes from a field outside the applicant's field of endeavor but concerns the same problem with which the applicant-inventor was concerned. D A printed publication in the applicant's field of endeavor published 3 years before the filing date of applicant's patent application. E A U.S. patent which issued more than 1 year before the filing date of applicant's patent application, which the Office placed in a different class than the applicant's patent application, but which concerns the same problem with which the applicant-inventor was concerned, and which shows the same structure and function as in the applicant's patent application.

Related MPEP Chapter(s): MPEP 2100 - Patentability Answer Reasoning:ANSWER: (B) is the most correct answer. MPEP § 2141.01. Quoting from Panduit Corp. v. Dennison Mfg. Co., 810 F.2d 1561, 1568, 1 USPQ2d 1593, 1597 (Fed. Cir.), cert. denied, 481 U.S. 1052 (1987), MPEP 2141.01, under the heading "Prior Art Available Under 35 U.S.C. 102 Is Available Under 35 U.S.C. 103," states "'[b]efore answering Graham's 'content' inquiry, it must be known whether a patent or publication is in the prior art under 35 U.S.C. § 102.' Subject matter that is prior art under 35 U.S.C. § 102 can be used to support a rejection under section 103. Ex parte Andresen, 212 USPQ 100, 102 (Bd. Pat. App. & Inter. 1981) ('it appears to us that the commentator [of 35 U.S.C.A.] and the [congressional] committee viewed section 103 as including all of the various bars to a patent as set forth in section 102.')." Because the printed publication in (B) was not published until after the filing date of the patent application, it does not constitute prior art. (A) is incorrect because the patent pre-dates the application, therefore qualifying as prior art, and comes from the same field as the application, therefore qualifying as analogous. (C) is incorrect because the printed publication pre-dates the application, therefore qualifying as prior art, and concerns the same particular problem sought to be solved in the patent application, therefore qualifying as analogous. (D) is incorrect because the printed publication pre-dates the application, therefore qualifying as prior art, and comes from the same field as the application, therefore qualifying as analogous. (E) is incorrect because the patent issued before the application, therefore qualifying as prior art, and concerns the same particular problem sought to be solved in the patent application, therefore qualifying as analogous. The USPTO classification in a different class does not render the patent non-analogous. See MPEP § 2141.01(a) ("While Patent Office classification of references . . . are some evidence of 'nonanalogy' or 'analogy' respectively, the court has found 'the similarities and differences in structure and function of the inventions to carry far greater weight.'").

In accordance with patent laws, rules and procedures as related in the MPEP, which of the following transitional phrases exclude additional, unrecited elements or method steps from the scope of a claim? A Comprising; B Containing; C Characterized by; D Including; or E None of the above.

Related MPEP Chapter(s): MPEP 2100 - Patentability Answer Reasoning:ANSWER: (E) is the most correct answer. As set forth in MPEP § 2111.03 states "[t]he transitional term 'comprising' [Answer (A)], which is synonymous with 'including' [Answer (D)], 'containing' [Answer (B)], or 'characterized by' [Answer (C)], is inclusive or open-ended and does not exclude additional, unrecited elements or method steps. Since Answers (A), (B), (C) and (D) are all open-ended transitional phrases they are incorrect answers.


संबंधित स्टडी सेट्स

FA Davis: Med-Surg 1: Preoperative Nursing: Ch 15, 16, 17

View Set

Manufacturing difficult questions

View Set

Psychology Ch. 2 Sections 3-4 Review

View Set

EXAM S2: chemistry (exam & test questions)

View Set

NASM Corrective Exercise Specialist

View Set